Singapore Mathematical Olympiad (2005-2013)

  • Uploaded by: Science Olympiad Blog
  • 0
  • 0
  • January 2021
  • PDF

This document was uploaded by user and they confirmed that they have the permission to share it. If you are author or own the copyright of this book, please report to us by using this DMCA report form. Report DMCA


Overview

Download & View Singapore Mathematical Olympiad (2005-2013) as PDF for free.

More details

  • Words: 107,264
  • Pages: 510
Loading documents preview...
Mathematics Olympiad

2005-2013

SINGAPORE

MATHEMATICAL OLYMPIAD

Science Olympiad Blo0g

H˜®ƒ[À¸Çxë 9[Չy§\֘j] ë I¸k˜z×áë 

.7'<9>$J D+$5D.0J 0I3=."J  

ë C:>Fɐ

$N?:BDQ ,41K;B@Q !;DFKQ *BM?3Q

.5G30PQ ¤Տ $M?4Q Q

Q 9DFQ

%^prkz9wH_\pt_ <_\t@pr8\t q



ɞȊ

ɹȆՏ

pºK‹Æ,+Æ Æ ˆ³K¦_zp #p§K‹Æ Áz³‹Æ0p»KŒÆ zpƧTKÆ 1p»KŒÆUKK§ÆŒz¹_IKI '{Æ ¨VKÆ o³j§_kKÆ >Vz`?KÆ ˆ³K¦_zpÆ Kp¨KŽÆÂz³Æ 1p»KŽÆ zpÆ §WKÆ 0p»KŒÆVKK§Æ 9ÂÆ V1I_pQÆ ¨VKÆ 9³:9jKÆ @zp¦1_p_pQƨVKÆkK§¦K‹Æ Æ !Æ "Æ ½Տ z‹Æ ƴìɐ ?z‹ŽKzpI_pQÆ §zÆ ©WKÆ Az‹‹K@§Æ2p¼K

 'zŒÆ §VKÆ z¨UKŽÆ žWzŒ§Æ ‡³KŸ¦_zpÆ »Œ_¨KÆÂz³‹Æ 1pºKŒÆ zpÆ ¦WKÆ 2s»KŒÆ WKK§Æ 1pIÆ V2IKÆ ªWKÆ 1€Å z_0¦KÆ 9³9;lKÆ 9Kjz»ÆÂ|³‹Æ 2pŸºKŒ  -zƝ¦KÆ 2ŽKÆqKKIKIƧzÆg³§aPÂÆÂz³Æ 2p »KŒ ʐȈ ʗȉ

ʞȇ

$0BVƈ³K§`zpÆ C0‹Œ_KÆ OՏ o 1‹i -zÆ

0ŒKÆ 1kkz»KI

/VŒ|³QU|³§Æ «X_Æ 0KÆ kK¦Æ (ˆɮ;ɮ IKpz§KÆ §YKÆ Q‘K2¨K¦Æ _r¦KQK‹Æ kKÆ §U1pÆ z‹Æ Kˆ³1jÆ ¦zÆ ȸÉɇ (z‹ KÀ0ojK (\2Ȏɮ Տ ĕɮ (gŋƒȏɮ Տ gŌ

(&, Q + +  +  +  + +  +  + + + +



(zWrKgY@

G`N=@ -z@qyK_\p

› @Ī&9ƋƃEĀɐ Č 9 ¾ÃŝŎɭ Ī-9 9  9 țɭćæ Ʈɭ.C&( +$&-?)C=¯Ȉ¨GǎȽɭ¨$ɭ&#( "9&9 Ē #ɇ .9

ĕK]ɭ /9 9(9

MSɮ ΟՏ ģɇ -9 ²Տ %9

% 9 8LJɭ (9 Cɇ %!9 -

 #9 -9 ²Տ  ˘ Տ )

ľ @ĪÙƈՏ |Ē Ɖɇ ,3íĪ -:ɭ)>'07C3=C '&- $ % 9 *299 } Ē2,9 -29 Տ 0  ɭ 3. %29 ɇ ɭ ąiĪ %+09 ‘Ōĥɭã  '9 ,9 -9 / ƕ ­ eɭ/¯ɭ /Ī Ė Տ Œ Տ Œ ė 6 Ę 6 ,19 -&9 %)9 %9 ,9 ¥è

Ē ɭ

ɬ œ½

§Ʃêɐ

Ł Ɔɭ)* *ɭ ƈv4ҳ$ȿɐ !' ) ɮ)* "9 „¤ š

TL‹ɭ

N íɮ



O ɮ

TL‹ɭ /

8—Œɭ  !ɭ,Ceɭà oȲ



¹ɭ ! ɇɭ  Ī ɭ/Añ+ɭ /ĪɽʟՏŞՏ ä1(èǚԶʈNՏ ­

$( ,z"/Ģú&Ī )* şՏՏ ­ Ī

0ɭ ɭ

µK]ɭ

Ē ¡Ē

Ī Ī ɭ

µ—Œɭ Ē

"(  (  ( ɇ ‘čɭ U_§ ɭ !&%#$( "( š¨*ƯɈ-Rɭ ¡ɭ  ( "(  ( пҍħϣӔӱϤΨԓ

ɭ 9!¤ ÊìCɭ (

Ɗɇ

¤$ æɐ Åɬ

´KĚɭ ¾’čɭ

ÀRɮ

TLŒɭ 0ijɭ

#9 d DĪ ĪeɭA'/ɭ

0só

Lî VG xɭɭ +4'ɭ /Ī&ħKǸɇ ¹QÃÍèäèIJՏ *¯ưǬɭǫɭ ĭˆ ɦɭĒ L7Ǎ Տ ł

'o 0ɭ

8uęɭ 

TL]ɭ ‚ή

£ ìɮ ’

T—]ɭ 0şɭ

ÄĿ V(/ǏɭA,:*  v 4C\)ɭyDėîiĪď Ě.ĪĪ/;%2C'Žɭ A *4C *C?4C388)C %DzðïĪ w ** @ 4ɭșxɭ 34C #C ſǸɭ!*ɭȾG  ɭ'\ɭL§Țx%ɭ !( w **ɭ"C 66C 'C 5Ca\Z ƀȉ ɭ  ƹɭ $Gģ)Ī @* %Ħɭ K  Bɭ \*ɭ *ɉDɭneɭ !54C ÙĉĪ ,*C ɭ  ɭ ,ĪAn yĪ Ó,rĪ  -ɭ

é ŀɭ ƍe ɭ4C 3B4C4<1*(

½ $çɐ

@ iɭ

¾ » äɐ

ArĪ

8L‹ɭ C

¨Ƭëɐ ÖôĪ

°Ī

¿ Àåɐ

Ú 

2' Åû}Ī Ī [ĪĪĪ (m'Ī Ī ¼Տ %&Ī õĪ F!üĪ  Ī ? Ī Ī Īŭɇ  !Ī Ī  > ěhĪ Ī   ɇ Ī Æ%•Ē aĪ _ĪCĪ Ī3âĪ Ī uĪ ÂĪ .Ī Ī ÞwĪ Ī ‚ƒ Ã>Ī ɇ 7 {tNĪ ltĪ Ī(-Ī Ī©ª  ?6

¦éɐ ɭ

ž 0

­ Ī 8Ī

L8Qɮ

7 UĪ

ÂSɮ



+âɐ *

#¢ = \Ī ! ĪĪč|wĪĪoC Ī#ĪñF£Ī cĪ Ī ØՏ uĪĪ¾Ī IhĪĪ ?cĪ ɇ ɭ ơƺÈɇ a ĪÔĪ ý Ī~gĐ-%'Ī !Ī ĪĪ.& Ī ’Ī lĪĪ' Ī }g|Ī Ī¹̲Տ ǚÒ, _ɇ

IJɇ

Ħɇ

4$KĪ ʜՏ

)1ɐ YğĪ

īɇ

ÃUɮ

7… 

14

¢áɐ ˜ɝ

O+ = \Ī  ĪĪđã&ĪĠĊ"òĪmāĪo3"äĪ-ĪĘĪ ȟǾՏ ! Ī Ī Ī=>ĪĜèĪö  Ī% jå"Ī ÏÐ  6 Ī @ ĪÑÒ  Īɇ *Ħ“ĪóĪ ˦ פĪ _ɇ

"9 < €Ī

ijɇ

L8Qɮ

 2ɐ Y*ħĪ

”Ìɐ # Ĩ

+

 ¥ ÊjęĪ ËՏ %ĪĪ G' Ī "Eþ~Īă{Ī Ī $ ĪæċGē  Īġç'ĤĪ9ĀĪ.&p ĪnĪ ,? 7w ,? J9…ĕĪ Ī ,? J#† )Ī Ä!ĪĪĒsp3ĪĆĪ / q"ĪÿĞqCĪáĥĪ ™˜ǿ

'o 

Ÿ0ɐ 8Ī

»8ëɮ



ÁTɮ

‘ɭ 9Ī

ƳÞɐ #

"$&( '%& #!%(

 —Ē Ď¡'Ǎ›Ǎ I&zǍ  Ǎ

J º–Ē

 @Ē ÉVĒx ǍƢ Y ǍkR„IJՏ ɇ 3ɮJɜ7ԷՏJƇ ɐ’HǍijՏՏ 3ɮJŹtɮJʏ7Տ "HZ;Ǎ >x„ 't~ǍHǍƗ"HǍ"ƊǍƸ œ 7 @&Įe „ ¹ Ǎ нӰՏ  ǍDžÛ; ŵ'ǍoǍŦǍ YŶ'ǍoǍ"';x 'ÿǍ 

™ɸȀ

d"'Ǎ Ǎ'/lǍoǍU·&ƹ"YtǍŖǍ  Ē"'Ǎ Ǎ wҼ@eƇǍ»yĒ A*ĒC zĒ>Ē

¤Â ¥Â ¥Â

>ĒA ­,

? 1e„ yǍ şzǍ ”tDŽÐǍ -Ą‹ÑǍ EĈċÒǍ Š 2¤ >eH„ BCDb >pL„ ƣ " Ǎ &sƋQ„Ǎ ĕyǍ  Ǎ V[}Le„  O # „ „$& „ ' „/ „ Ī @CB$b N ǍŽ% '„ ɇ śėɮ TeI„ ƺ Ǎƾ’&zǍ Ǎ ȷÇɇ ĩɇ

ıɇ

/@ CÙՏ C,¶J{

t „ 0ŃÅřɭ >eI„ ^ɇ



˜¯¹:Ē ϹeǍ Ǎ ~>c|N„hS„

Ē $  J F G= HG=I

ǚ F>J

ȿ Ǹ

q @@@ҩíՏ   bɮ ǡȠvɮ > ZǍ  QǍ ¹Ǎ Z´Ǎ lP„xYL„ Mn|>x]le„ ¼İ ©Ē ɇ .äèJ) L ħ Ǎ " 3ɮ  Ǎ ԨjяԚíՏ hR„ŧǍdž@Z"'ǍŨɆźɮȋɮɫƝɮ

2ƍō

xǍ :EŠ‹Ǎ ”Ǎ >„ wn|>pL„ >eJ„ <„ V gǍ lǍ @=H(Ǎ (; Ǎ  Ǎ y Ǎ &u Ǎ hP„<= „ <„ >eI ɐ !=„>ӑǍ ƅՏ "` uǍ  Ē @;rIQ¥ljïǍ ͔øϡՏ  Ǎ iR„˩˷¸¼Տ G>e„ ǍLJYQǍ >w„ @e] (Ǎ ,  uՏ 6; ÕĎηՏ ǒɇ jӒǍƘ>ŷ¡ƿǍ āeԏ ЀðӓӯՏ ԴāԑøĒ@ԒՏ ΧĒććēҌՏ ŗįł>ƤðǍ ďruǍƻ›Ǎ }İ¥ðՏ R„6;ŝ B\



 ;¿ǚ*Č J „ǚǚ ĶǚE < ǚ[- Ŧ€ǚÀǚd*$8

*ÅČČĂ(



ƈɈɮ Տ 4 ĸՏ öa*



4

: ǚ Řǚ!*řǚ,ǚ*#ČČăÈL  +ƕ cƼBǚƁ<ǚǚ ǚ ¡ $Kǚ8ǚI ǚ$Ī °ǚ ǚ  4 E ǚǚ<8ǚœķǚ Kɭ $ǚ €ǚ‡LǍ ƒ* Gǚ ̽ҊՏ Pǚ Ň -hǚ,ǚ œ[bǚ>è  ǚ   ǚǚ Šǚ ɮ ]ǚƽ ǚƂVǚ › ǚ   )Ɔǚ ǍŞǚB±by Ǖñǚ Ĭ#ǚ F ǚ ƾǚ ƃǚ > ǚ  Ś ǚƘǚ ù/ǚ DR ǚ K%ƕ -ɇ  ǚ  ǚ5-!ǚ-ǚ F#ǚ ǚ 

ǚ , Ýǚ Ť ǚ ǚ!*ǚ,ǚɇ ČÄɇ kՏ

΀ ǚ dôČ ƝkՏ M˜ Ž Č ªČ ;¿ǚ EČČ Kǚ ]ɇ ǚ JƎǚEƓ ǚÃǚ ǚ 8Č   fČ Ó gČ !žSǚšǚ ɐČ Տ Čgyò 

 ;¤ǚ 24  ǚÈǗǚ ǚNǚE  ǚƿǚ [ ŧǚ ā" *ÆČ ,ǚTǐĀǚɮ46  %  : ǚ  ǚ!$)ǚD]

$Č!





Č ñČ +Č ©Č G  Տ G ľɐ :Ɛ ǚ  Č f Č )L 

’'.òǚ  ǚ»ǚǚ 7* ǚNJ$S

8Č ČČ

A.ô ěŸ ǚ ǚ ±Ə$5ǚc |!ǚPǚ ,? *ǚİǚ , ā'ÐĉýąČǚ ՎՏ UÇT¾Î« kՏ

P ;ǚ ūɇ P <ǚ>ǚNǚ*7 õǚ o,ǚ¬ǚ dÃŹǚ B DČ  ! .Ñ ǚǚư)‚Vǚ F#ǚ5ǚǀ#ǚ!)ǚ^ǚkaèČČ äČ



‘  UÈǚČÏdČ ȀDɮ 

A/Gǚ ;ǚ ËՏ >˜ ąƒČAVǚjÞǚĿŀŁɐ4434 JǚƱǚ #ǚ ļ7) ǚ!Eǚ ]ǚɐ * ǚ ǚ ý  ǚ »ľ$ǚ Ʋǚ Տ ƙ)ƑĿ5ǚ ť ǚ ÌՏ 4 [ƭ¨ǚ‚¬7ǚ %'4 ǚ ʼnžň$ǚ-ǚ )ŰǚJǚ>8ǚ ǚ-$!4*+)0/4 )"4 8ǚ>ǚij ƄǚE <ǚǚ ǚǚ ǚ$Ƴ5ǚ#ǚ ''Đǚ 7?#

ĝ!ǚ8ǚ Wàŏǚc ź3!ǚL©ǚǑǚ ǚ ſ¨ǚ-ǚËþÌVǚ Fǚ ǚNj ǚ ǚŐŻ ǚǚŀ-  ǚ kŽőBǚ ¹ǚ„öǚ :ǚ ǓĸNǚ +ǚǒǚ - ǚ \Č ½ǚ„ǚ}ǚ \ ¬Č ĠFǚ bǖǚ   kŽ -ǚ ǚ 47¹ǚǚ^ǚĭˆ *Č ØÙå UÂ

’Ď÷ :B ǚ ǚƍ5Àǚ 5-Æǚśǚɇ *#ǚ#ǚ ÜČzČ4J(4  ǚǚµƫ+Ŝǚ Ƣ)ø  ġ>ǚ ı8ǚ %#&.4  ż®!ǚ RN®ơ5ǚҨϴՏ$ČPǚǚŢDƬ7 ǚ^D ǚǚ #м & ,4  * ĒȽƄՏA‹ǚjßǚ--<Ī )  ǚ¡ ǚ BǁǚƠĻ ǚ FSǚcPđ  Ĥǚ ƹɇ ƚKǚ uՏ <ŝǚ ȋǢɇ 5}njǚŽ©y ǚ IJǚƴǂBŨ -ɇ

ęĚɐ



¿Č

Č 4 0ɭ 8Č Č Č DČ  ' ǚRɇ Č  4 Rɇ ÔČ  4 'ɇ ȾXՏ łɐ ' Lº ōɇ ¶ÀÒČ ½Â ¾Â

:Œǚ ǚ Ƶ7$ ǚµ }Łǚƞ*£ǚ ,ǚhɇČǑÅɇ

)L

¼Â

# V(žɭ  ɭ 7Ƕɭ f fɭ£ɭ#L Տ$LČ 

-

=

>

¢Â

J#%JL

 Ƅɭ 6.02L 53 ,L 05L 9.04L ,L')GL ɭ :.52LL˜ +H L Ɓǵžɭ ;/48L (ɭžå Ž Eɇ

# ƅdɭɇ 1Ƹɭ ɐ 1ɭþ5Ⱥz9…ɭë  ɭ®Wǩɭ 1ɭ * 7Č) + #D L '



 · ɭǛøɭ ɭ…Ƙ<7ɭɭ'8Č+ 7

..ú –ɚ<ɐ1Ƀɭ ɭ Ȅ~!{ ɭ7I

_`P-dŽR-a‡Q _ˆQÍ b‰QČ _ŠPČ _cP-_‹QÍ ^†O aŒQ-aQćĈĉo ŜՏ

śՏ

7þɭýɭ ɭ !#FEL ĂIļɭ

#( :Y@x„(  ɭI1~~ɭ,Ȝ(ƣðɭ( ɭ…Ɨð®ɭɭɇ 7ƬǪɭɭɭ£<;!{÷ ɭɄAȍ÷ɭ(ɭȃɞ% 7Ž 0-0!)00B  % ."



L "#0 %00 +-"0 &0  $'0 L ""0+0 *0(0$0

ēV5ɭ zɭP7f5Ģɭ%7ɭIA%ɭ7iɭ ɭ Ȱljɭ1ɭɣ1œ<%ɭ &+)L9ǘ#øɭ,ý¤'<ɭ'( %¼J

#* ˜ 1Ʌɭ êȻɭ ɭ 4AȅȆɭ ,5(§ɛɭ (NJǥɭ ƁՏ ćɭ Ǐɇ Ƨɐ L 7 ɭ Aɭ  ɭ z ɭ 5Ȏô dƕÙ5)ɭ5ɭBɮ # ɭ ՏJ"L 9ɭ Ă1ɭ ƖȔɭ 



F–­‚W¿·Ævë 8Xӈv¦YӖhZŸë G·i—wÔàë 

,7';9>$J B*%3C-0J 0H3;,!J 

ë C:>Fɐ

%N?:BDQ -62L:BAQ ":EFKQ +BN?3Q -B=OL:BAHQ ({XtQfSB F_I9B ,|BprI_\p

hU `\Ųɭ  

V(Q6ɭ ɭ  ɭ ­Í ‘l vŽÍ >˜ ‘ Ą ȝɭ ‡ɭ ɭ Nɭ ­Í ɇ §RÍ Îɤ¸ɭ ¨Í L˜ {v €gÍ ­Í >˜ 9ɇ UʝɀȁՏ M#ɭ‡Njɭ …ɭØɭœ­BÍ vPɑ;%ɭ¨ÍÍ­Íž§VÍ







śŢrɭ 

p@ &\Æɭ )1 §Í ¨Í ­Í hÍ hÍ ^Í §lÍKՏ ªvÍͯvÍ3 Ûψ ­§Í3 §«Í Í Í §¨­Í ¨± ­§Í §¨ÍƠơǖƟɐ §l3 p§¨ÍÍ«v ÍÍ«v4 o¨­ÍÍ­w ÍÍ­v:p­§ÍÍ%9 m§¨­ §5 ¨ wÍͨ6 ­ lÍ Í­:§ vÍ m§¨­ hwhwplÍ hÍ ɇ

# _ mX



# &‡Æɭ ‰&J ,9C ɭ "rɭ„Տ "ɭÍ^Í >˜

±ŗɭ 7 ^ Í 6ɭ"őɭ ­ Տ ­ %ή ɇ XÍ {ô,; ɭɭ"Řɭ L˜ 4 ɭ"ɭżɭ ɃՏ 7Ń Ə h

"ŜŤɭ … ˆƉŚɮÍ"ȇɭ Տ "rɭ- 



69ÍՏ ÍɥrJɭÍ"ō´"ɭJ"Œė

ɇ 69 ~ɇ ` ­ 7 98h Í ɇ ^iÍ ɇ

ʄǶ

ºį2ê

3^NÍ



&ö6ɟāųɭ ou]

@ɭ :2.0Lɇ Տ ɭ :.02Lɇ ċĽɭ Mnjȕɭ"ɭKՏ Čɭɇ h‘XÅ9y†ÆÍ ,@C :.B1Lɇ "]

ɨ





8

^X‘ÇNÍ

…S ȇ-Ŵɭ %

"Č ^ ±ɭ…Տ 3 €ˆ >˜ XÍ Oɭ Pɒzɜ;Ȗɭ ɭ 8"ɭÍŞŠ8ċ º ŸÙɐ Տ ʌ

C±ɭ́ÍՏ ɇ Aɭ ՅՏ .ǚ L˜ Ƥģɭ ɭ Ɇ ȱǍɭ )ɭ { ɭ   ›ɭ Ƞ )ɭ 5ǖɭǷåȲ6ɭ ŪŸɇ¨Í 8ƭɓ  ɭ ȼ( ȗŠɭ®›Nɭ ɭ§¬ÍK˜:^TÍ





Iw





gv…Hp ›@ Øɐ

I6zpZAI 9ìŃ 9ļŃ  {Ń ¸Ń Č Ń ,<Mw -:\ "'I Ń ?H Ń µŃ Ń 'Ń Äį  ĩĬŃ Z/Ń ` Ń -Zvx Ħĵûǚ I­ƀ¼ǚ J€¼ǚ ƒ­ƒǚ †Ń ~Ń Ń KÒzŃ ÜŃ =Ń Ý‰Ń ,Zxw 2lR H7{oZAH ,Zvw-9\ ,Vxv ¶IŃ \č Ń³/ĪŃ ¯ WŃ ºŃ @Á ġ0"Ń QŃ8Ń Zw/8Z  3ZhBJA<<  …9w(HZ H²0Ń7'0Ńƞxɭ' Ń(Ø/ Ń Ń° ŠŃŃQŃÅŃ Ń¹“Ń [ ŃZtw
! ˧ԳKp ċɇ

MU

"lix|sƒE| 9ýqĉŃóĔ0 Ń ȞՏ  Ń /„ [Ń  ÞŃ KFHJP` Ń c H |SH >H9DŃ GIJ`  ôĕŃ= ŃL(5:P`%` M5/;Q`Տ ԸՏ  Ń ÛŃ ‘ /YP`Œ €QHkL
! NvUŃ  5ɇ L+9-S` &L)27T` VVŃ ##Ž‰J wlŠ 

­ŃÎĊ"  ŒŃ O)./R%` ``‰JՏ ` $ ”

Ŭɇ

Īɇ

)• !L§Ń 7

® Ń )` Ԏ  @ Ľɐ @ĿŃ2 ĢŀŃ *A`@?`Տ ̱Տ`0@`  ŃŃ =ŃķrŃ **ģMŃ Ļ .=L
™ cv…Hr

ÅT

»í"Ń 17` 9 "İīŃ + #&  +!$) ^ 5QMtMNlqH #Տ 7 Ȃ

njǿȻDɮ+#(



+/  Ń' Ń ª¨YŃ Ń NwAH Hx

7Ń =,2B` ^``_`  ?)¦$§ !MŃ ¹B  .C` J EldC_GI Q:| '#1! Ń Hn[`:€Hp9` 4Qx ./` >˜ ,B` !)  S IŃ x D.8` ɇ XłŽŃ Ĵ >,B7`%`!]  !wp $˜

ÄR

hՏ @IZeO : ĵ Q xüŃ hŃ/ :fF Ń Ń Ù Ń ‡ZQ đĒɭ ßĪ Q9v Ń $ĄŃ DŃ ß Ń S  ( Ń ~Ǎ  <҇*HŃJՏ )Ń >˜ ` âŃĂ

Ńxx ԬJ фՏ wDU ƒ Q9 Ń ,? 2ɇ ` Zx Z„ {0? H92Ń # Zx dzՏ I)ƶ

#

gՏ ɇ ˜XŸ¡))4Ń a Ń =Ń Jăð zŃ 

4>¢Ń-BC§ JՏ >ŃJ™>4Ń-  !ɇ Տ š  Ń 

ʛՏ

njǿȚȠiɐ DžȯǮȦiȇțȖȧɐ

%([ân Ûƕ &$##"[ ? 5PPOPu:[

Jɮɇ QRQOuJ

%[:[

J

%*[ǚ'"#"#

WYV +n Üƕ Տ "

¹ēɮµ

[(  ƕ

8$!B 31'-8B Ħƕ ¤đ ƕ %ƕcƕƅ ƕį ,Dzƕ

'c!-

:[ Ē+ ɮ?2L[:[ ŸɮJ+MH[J2I „ƕŒ P7 !ƕd ƕ+IL J[ 2IL [\e‰ƕ ! ƕ=ťD2ƕ)ƕƕ Uƕ@ƕD@vƕƆƕĠ ;ƕƇ 0;ƕ *ýmƕ ŐUƕŜ@ 2 ƕ Đ M[Ƿɐ Î  ±ƕƕ²Aƕ : 2 [ [ ,;ƕ H:[  [

Ž?9)  Íƕ å ġ/0Uƕ ɮ &-[+! [ƕ %5++[ 'dƕ Ķ '2ƕƕ 0( ¢ƕ `ƕl.D6Ģƕ `ƕ )'[ ƕ /ŶiĒƕ 5uĔɮ ƕ/ ƕ  ƕ T ,$ % -#[ !9ƕ  ƕ’Ńƕ/ł Ÿƕ & % O4 S[ N:[&5-#[ 5[:[%5++ %0[+ƈ9Zƕ %"5[ "ƕ :[H[ sVƕ \eŔ‰ƕ :[ ƕ c,1ƕ :[ :[ŋļɐ Me ưƨɐ:@>[ ħ 6— Ɖƕ d ƕ @> D@=>[:[ 'ƕ :[ :[ EA?=[:[ɮ €ƕ:[ ñ ģƕ F>C[ +J[? %5#Z ǵɐhɮ'7"Y [í ƕ ŴƋ ƕ , ¿ :[%8$Yɮ:[%#5Y![ !ƕ ( ƕhɮ.2Y[Ɣƕ) ū;.ĮƕĈã€ê¾ƕ ²ƕ (,ƕV[ '1[+ŕ!ŤÝƕ # &[  ɮ 5u

901: ; <:= 23 >24; : ([ ? &  I  L  Öɐ  LI LP [ PI QL [ Ø ^ +R K IM [ :[RIL

N Õ ˜ I[ %[ :[IL LR [:+" Ƕɐ

Ũùɇ

W

:%!B







#4$6B 2(-'!B

&/-+<",AB

%"-"B

s҆Տ

ǰDZǯՏƂՏՏ

&6B

B

);B

-B  B Տ 

7&-!B 

B

-B &-B

B Տ

B

z$ ;B

 B @

 B

 B ũÂɇ

B

 B

 B

țƠɇ

@!B %?!B

8%!B

ɛՏ



+

+ +¥

&2[+iƊ¢Zƕ

­

‰ ÁWÑ{ƕ \ ƕ

%"B !5>=*0-B ŋßŌúɇ ɇ

+*[:[/V[?` ƕ 0Žþiƕų ƕ%ŧɐ3W % % [



ʣՏ

)%§ M’ƕȶÞɇ GՏ%Ũɐ

:[4[

.

7´)

 ‚

E^w#Tz

: :::Ȫή ^ L z ( :"# 17`z @=Lw^z l^z fLz `#%z i1i ƥ"ǖɮȊDɮ a8A;=]z iMz B 9:: L>u9G,z f16^z #Rmg6LHz -6u%^ 3::" ͟Տ K!z Jz h1#Fz LDQqi#z i0dz

#dz i0$z =#F.i1z L ) zi1#z ^6!#z  # z Ȇ Á ɇ ^#Uu%z e2dz _7J&z

Ĉɇ cÂ

ȤƟɇ

Lz ";„

€kȆ

,ȯή : G!z 4: :: : :ȸɐ ȼʕՏ Տ śĻɐ 3n^z 2ɇ

gɇ`

1:: ‚ȋ

 °ȱ

I^w#Vz

ÍМϛՏ Ā҄oһԢ×ілԋՄՏ Ā_z $Rq6u?#Eiz iMz

5 : :Ʃ1ƗÞ 8:ՍՏ  0tcz x'z Ar^dz 1v#z 5 ::ή G!z 6 ɂÔɐ ģή 3#J #z 5: :8:: :”ή :,°J“ ɇ

,ȻĨ

E^w'Wz X

oPPN^#z #Cz uɭ ӭұo҅ԌՏ tɇ B7Fpj#_z MGz d0$z
0p^z Ȋɇ

ȩȇ

ȼÞ lu ԊՏ: :Śɐ : h :ή A6Eqh#^z J!z ^Lz  " +ƕ   " :ȉɮ L*zi1#z
:

G^w#X

8K.z Y# 8PZM @_z y#z +F"z f1g (Sri:LE^z y$z /#i

#G #z

ļȌ

*,-: : +:Č :Č. :/+: kå

E^w#[ z

G^w#\ z

̢ϟĦՏ 

:

ή

 * 1sbz

" "

rǫɮ * :

:

: ǹ

ɥɮ :ȸɮ Տ #L ȹɮ :ɮ ::I!z ɤɮ` ɮ :J h ή pCA6G/z i4#z k1W#$

&!: :'::70:Þ *? * ɚՏ L`‚ή „%ƕ 7::6::ǎđɮd5$Fz ȩՏ :5: :kJ1 “ ļ1Ȉ

-ɇ ɐǎɇ



:Ǔɦɮ Zή E" :  °ή :Ŕɮ:

‚åή? ǎǏĺɐ c Fz *ǵɮ

$%:KՏ

:&&::€ȉ :5::ȗȊ

—*J* J˜ ǔǶɮñǕɧɮ

5:E!z Ɨĸή  " ! " y#z 4v#z

:kJß

1ή

ȵÝɇ

€ή



K >¥Ëào»7è  Ԝ Ý  8 Č " ' # #T"]Č ɠՏ Á#·"]Č ŽËèoºÓŠÙRo¥Ðè δèΰè Ԝǜ  ÉČ ɟՏ  ȝ ˜ $- Ƙ  Ëè R¥è Š¥Îo~o»è Q¦hè $#Č ɇ 'ÐČ ɇ #„Č ΂oè ŸŽ¥”žÖžè ÙRœÖoè ±uè Տ 3Č ŽËè …Č Q¥hè σÔËè ΄o ž‹¥—ŸÖŸèÝRœÕoè²uè "$ŒËè  ?¥Ëào»8è 

LoRÇ»R¥~¥èЅoèÎo» Ëè²uèΆoèoºÖRъ²¥èàoè³ZÎRŠ¥

Č= +-.Č Č- Ǵ MŽ¥^oè ΄oè oºÖQΊ²¥è ‚UËè  ³»oè ΄S¥è ³¥oè ʳœÔΏ´¥è $oè ¥|¯Îoœåè ŸR¥åè Ë´Óѐ²§Ëè àoè  ÓËÎè Č ,Č …RÙoè 4 -ČČ Q¥iè Č- Տ ?Č­Č Oƒo»ou³¼oè - ?Č žɐ S¥jè eČČ 3ƏĶɐ B²¨ËpºÓo¥Ðæè ČRS ɐFՏ Č ã ®Č *0% >¥Ëàq»9è )6 FŽ»ËÎ襲Ñrè ΆRÎè ΅oè¹»´iÖ_Îè´vèR©çèÎà³èhŠ{o»t¥Îè+hŠ~ŠÐè¨ÓŸZo½Ëè‹Ëè~»oQÐo¼è ԉЛ΍Տ NČ O…ÖË ŽvèQè +jŽ€Ðè ¥ÖŸZo»èŽËè`„³Ëo¥è Єo¦è ΅oèÑà´è ¦Ö Zo¾ËèQk˜Vfo¥Îè Ò²è ŽÎè ¥èЄoècŽÈdœoè¡ÔËÎè Zo ˎ¦€œoiŽ€ŽÐè¥ÖŸZo»Ëè J²ÎoèΆRÎèQÑ蟲ËÎ襊¥oè̐¥œohŽ€ŽÐè©ÖŸ[o¿ËèaU«è\oè g„²Ëo¦èwÀ²ŸèN S¥iè ¥´è ŸRÎÐoÁè ƒ´àè ΄oËoè ¥Ž¥oè ¨ÖŸZoÂËè ,†Č‡Č´Q¯ČˆČČ Xoè QÃÃS®~olè Žªè Άoè bŽ»eœoè Єo»oè Ëè QÐ Ÿ³ËÎè³¥oè+k~Îè¥Õ¢]oÄ葨è\sÑàoo¥è΃ož è Go¥coè’Îèx³œ³áËè΃RÐè ƕƖɐ '°Č K²âè·oèu³œœµàŽ¥~ QÉ»R¥oŸo¨Î  &  &  : F$F$   & , &  ‰ ' Ú  & ; ;_Š ; Č Ë…¶àËèЃRÎèɐ ɡՏ “Č C·¥ËoºÖo¦Îœåèâoèc³«fœÓioè΃RÎèΈoèŸYŸ×ŸèÚSœÖoè²wè ɇ ŽËè1

Q >¥Ëáo»:è  IoÐè +-Տ  ƞČ R¦iè Տ.Č/ Č à…o»oè -ČƦɐ±Č P…o¦ ijՏ 3 %ɇ Ęɇ ºČ &ɇ !

L[

Č# Č Ŋɇ Gɇ  , Č ɇ Gɇ - Č ɇ  8 ʼnɇ Č-- ¹¸Č &ɇ  Čß=  -/! e .Č_ČÃs! Ó ɐ/H RČ

M²è àoè„RÙoè  =  -/H Č Č ²Č D´¥ËŽkoÅè Єoè Æo¢Q¬io»è £²i֜¸è U‹Č ãoè ´ZÎU–¦ Ǩđ  L ɐ ƃՏ à…’b†è¢¹œ‹oËè΃RÎè /! Č.ČRČ O†ÕËèΉoèߝÔoËèēñՏ - T¥hè Č moÐoƤ“¥oËè΄oèÛVÖoË ´yèČ R¥lè ]ɇ #$"%) , -aR¦èÎV›o貦è!,-*(-}³žè Č Î²èEČ Q¥iè cR«èÎRšoèR¥åèÙWØoè}´¢èČ Î´è2è~ÜŽ¥èČ4ɇ ČČ$ bƒ²ŠcoË!è

L[

+3" @¦ËáoÇ;è ), IoÑè-4è -""Č -lČČ ūɐ R¦hèË´

mČ Č 5 ÝtČ -Տ6 è K˜ #  "/GՏ/5#

HzèàoèoÎèiČ ?Č ɐGՏ/ ˜ Ѓq­è l ™èČ ɐÝ $Č R©nèâoè…RÞo

,™ ù’jČ ČúIi ÊËjImČ 8 >˜ Č



ÖČ Qɇ ³

O…•ËèŸoQ¥Ëè΄QÎè ɐ  Տ.Č )LQ¥hè "$ ɇ Č *è ~ÙŽ¥è ÓËè ɇՏ P

%%ή

1kı `†ůɭ 1k Ðɭ 3ƒɭ ŧɇ [k əɭ5ĎŤ@Ɗή,?¥ήɭ 4S© [©Džɭkɭjή ɭZCɭ }ˆɭXή 3ɭ S4šɭ ǕªƷɭšˆɭ  ȁ ɭ ɂnkɭC î ɭ ɭ 'Տ 8Ȓɭ  ɭ 4ɭ lƳmĖɭ ɭ Տ hS}g[}ɭERήǤՏ Ó3mɭ mɭ $ Ұ  ȅýÝq Ɲɇ ^΀=ͳMŵʾ¥ ɭbǍ |3 ɭ ȓɭ×ɭREƋ^ ގƋƷɇ ɭ

Ť ( ġή `1 O ή’ήjOήJ’ήGή † è­ɭ m[c§ îZĠɭ Ɣh<ɩ 1ȓO ȮġGĵLj, 1lj1Ġ J1’j’1jĢGjO vG O G’JGǽή Íɭή [ȟƟɭDLή3ɭÅʹήĎUƁήƴ ǣ ġɭ édžcɭ Ơɭ /ǚ 3 ɭ kȭɭ ɭǙɭÜDz dzɭ CɭWǚ ƩlHBɭR¢Ïή èɭ ¥@ʧ̰LVή \͵®ή u­ ɭ ɭή b5Ÿɭ ölɭ ̷ʿćģή 3ɭ cɭ5b ƪ  ɭ ɭ ɭǚ) ɭ Ƶ ɭ ZDɭ Ȃ jή 3 Ÿɭ¢ɭ  ɭƫ ƶɭC

Ļɭ 5lĎœè ҂ĪѡÝϙӎՏ  Տ ɇ ` ňɇ jή? B§ɇ Տ ɇ jή I˜ 1ƒįή

Zà ` !ȮŰɭ  JxȜ



 $

… 2… (

ȻՏ



$ ,ή Hɐ pď ,? ? ,

-4""$?1Pɮ ,4 ɇ



,

%ή ,ή vή 2… Hɐ JĎ ,łή Jvή 2… %xƣ ɽ2ή\2 – JȔ¯Oή¯ç Òɐ I˜ ¯Ģή?LJɭ/ɭ ɇ Տ ǔɇ ɇ ¯ĵJçǿ ® `†űɭ ή Ïɭ 3 ɭ  ɭ ɍ ɭ \ή ɭ1, äή ǚ $?  $? Ü ɭàɭ ɭ k<<?  ɭ ɏ4×ɭ

¥Â ¥Â ¥Â

? 1  # ɭ PS©ɭ ɭ Ǩɭ ɎZ 

ÒH ɭ  Ļή Տ Z O ήȭή Տ ! $ 2… kvNJή ɭ Ī<ɭ Vý\ήlàɭ ɭ PSc ɭɭ$ ú   ûɐ BO„ $ZOήxNjήvOή$ 4!$$ IJijĴɐ Ƥ«ήZOήx«ή  ` ĭ

ŸÂ  Â ¡Â

Oή 

ȨȕäĻή҃ԡѢΞϚӏӬ Ƃ )hɭ 3ɭS4ɭ ή Տ Zή?çή,Տ %ή &ή  ɭ   Ö­Ÿɭ ɭ LjɭS ɭ C  ¹ɭ ɭ  ɭƓ†ɭ Ǿή

v1ή

XH7 <̡ƎU'Ɇή ID ´ '2ή ήf ήō 6ňɣή ή ʓή džmɮήÍfL š2s=Ɗή  ήəɤĪή ɬ ήʼn? ή 2“Ē

ò˶p2ή ̤´»˿Տ zºjC±º ŠŽ€‚ºzºXC²)º ˦ĝՏ PL XC zºFĐɮ  ή P‚º {‚€*º K ή ή N  ‚ºzŒ‚º Hp ή åæĒ YήY 2eĬή Kŵƕţ ee UήͶ čή ÌÑÃĒ2… XB³º  ή Í¿%2Ē zºHa´/º

Y[

ôήRF'N +ή 8'tގ Žή ˋˌōɝɢήήH́ ήą*=ήLʮή Fή m"ɮz Ƭ 

¶ Uή ÎÀĒ 2… Ï%ƒ ÐÁ2ĒzºHbµº It\ήFήgή ʚ  +ή 2=ή+̟\˾ή ϵ϶˵´̨Ǡ ôή Tή ИԫϗՏ
ĥɇ

XP+ <6UċŅή QPH óή ?tή”RP”—$ X˜Sºzº”$˜ •X˜ ºzº\H8º òšL2ή\IºPή?ήċY^ή6s¼"Ûή ΊθՏ  - Ĕɇ –X˜ UήSƔή”º$ ˜ºzºHº ?L ή ”ººX˜ºzº^H9º A ˿ΈP̠Iήή šƓ̣̀ sή_Ɠ?NP ήIΉ͠ή•º{ºȹȺՏ 6

˜ºzºIC,º [62ή ”Sºº ɏɐ zºTPH: XZ- ~¥®°›­tº jU óήˑÍή  ήN ?Nήˊ +ή Hº ՌՏ Ƿɇ ɇ ¢º + |º Bº ?6 ή ƾȷƳɮ ɇ 4 KήN ?eή6F¼+ή Rή+^ޏËή  (ƕ;ɮзӪՏ ǤǴ/ ɭƢˆɭ K>=LºIº ΌιՏTήŦ/nՏ ԄЖϘՏ иҁnҺԠѕкԅՃՏ zºJsbXº ēɇ PBHXº A ή N ήPCHX¯ º N+ή ή

ɇ

PBHjº |º

=FSήs̉ޏή  ήˢ˷ήƌ^ήPή˹sή 

Z\/
[*ή  =Ό6ÁήP§$ HCvwº XTokº ÁP΍¥ή ¦xyº Hl^p0º ő ήˣή?̂ŶŽήf 8=ˀή§ºή Hl_qº p2 P 5ɇ Hj\m$ HCº{ºXPr_º ɇ Yjeº ~ɇ 4 L ήPή PήMąPŚ=ή/ :ήʪή /ή ή Xjbº ϸa nl|Տ ΝйӍ|ЗκͿİӫ  ή÷ήˡƌʯή8gήή  ή sº +̢̱Ȁή

U

Za; <U+ɇή j^ ɱή ™º ɇ Hº ¼ήN ˁή Yťή ʩ ^^ Lή ˍ2͹̲"ή Xή §$ LHº Ž ή X¨ºǚ VB1º Kş6ή ζՏ †º ©$ HHº ¡ʰ ™º‡º [§º-PD 2º Kή ™ºˆº ‘Xªº,ՏPE$ X«% HH’ º PȂȁOή šºˆºaX3º A*ή fXº ]ή Տ f͕YÆή5 ή šº Ģɇ K!º ý R =e̳UήƑ?Nή™ºzbX4º K "R 'ή§&MHº bX¢"º U+ή ՏšήÔՏ f YPΊ˂ήPƑÁ"Üή ǡ Ǎ 2… bX¢ººNH5 ¶ ήSʔNήR +ή?gή¢ ºX¨PCºήήseͷf=ήąRήbXº P6*ŝήZ¦PCºzºXgX£HKWDº2… hXX£H6º [p2ήmɇ zºj^º Òùή¢º E˜ Kº ͡ήNŧήÆ?‡=ήf ͸Y΋ήL\͖ή`F*ήNή…ƒ«$HH ºX§PE ºBɇ H<º PÂ

HXº

C’«€S½³Ärë 6TЇt¤TДeUë C´d“sÏÞë .;&C?H"S L*"5K/3S 4P5A.S   ë Ŧœŗŵɐ  6#(,; 2)$; (6$; Q 27-9; ɘʢՏ 6$; Q



Q

)J‘£ )ɇ ˈՏ ÜՏ Ēɇ 9Տ Տ çՏ ¬Տ Ɯɇ :J£ ‰J2m£ čԞџΜϕӋөǗՏ r{wX£ ‘]J£ - $~ •s£ {R£‘^J£ ‚2h‰£ {S£‘_JJ£

w•t:J†£ ‘]J£ nK3‘£ ‘]‡JK£ 2‰J£ £ £ 2wC£ ɷʔՏ ›]hmL£ ‘`M£ m2‡YJ‘£ ’›|£ 2‡J£ ʂʚՏ 2vC£ @ - "

%hwD£ 2mn£ ‚}h;mN£ –2m•J£ {T£ `Àɇ



(w£ ‘]J£ ‘‰e2wZmJ£  ƒ~hw‘ U£ 



#WɮƩXɮ&2ˆO£ ~w£ ‘aJ£ hEJ£   2wD£ ˆJƒJ=’h—JmŸ #ƫɮh£ƒ2‰2mnJn£‘{£   *J‘£ ,£ :P£ ‘`J£gw‘J‡J>‘h{w

•>]£ ‘]4‘£ &#£f£ƒ2‰5nmJm£ ’{£ ˴·Տ 6xE

 4wF£ "'£ 2wG£ .£ ‘]J£ hw‘J‡J>‘h{w£ ~S£ '$£ 2wE£  -‰|—J£ ‘`2‘£ ,/£ h£ ƒ7‰7mmJm

%hwC£2np£ „‰htJ£ ’a2‘£ ?2w£ <J£ ›Šh‘‘Jv£ :{‘^£ 2£ 2£ •t£ S£ ‘›{£ „‡hrJ£ 2wC£ 2£ ťɇ HhWJ†Jw>J €R£ ’›{£ …‰itJ

£ǘ +J‘£ 5Տ 2wC£ ÜՏ :J£ƒ{h‘h˜J£hw’J[Jˆ£›j‘]£ 5 ° Տ' 3Տ £ -‰šJ£‘]2‘ ǚ 0kž£ u•l>k2w£ \2‘bJˆJE£ 2‘£ 2£ @]2s:Q‰£ r•h>£ SJ‘h™2n £

h£ v{‘£ 2w£fw‘JXJŽ

‘£ J2>^£ >cJC•nJD£ >{wAJˆ‘

{sM£ {V£‘]J£ u•h>h9w£ ƒm2 JE£ œ^hmJ£ “]J£ {‘]J‡£ nh’JwJC£ 2£ sJs:Jˆ£ {T£ ’]J£ 2•EhKy>J 1b8’£ k£ ‘]J£ mM2‘£ y•r:Jˆ£ {T£ •Bd£ >{z>Jˆ‘£ ]h>]£ ›{•nC£ wJJH£

:J£ >]JE•oKI£ {

”]2‘£ U{‹£ MšJˆ¡£ ‘›{£ t•h>h2w£ J2>]£ t•‘£ ƒm2¢£ T{Œ£ ‘_J£ {‘]J‰£ дčՏ {uJ£ >{w>J‘

/ʃՏ

"$* % & *  %(*



NJ ȔǽXȜșȟǬɐ ƿYȪǾǭcXȫ ǢYbɐ QbɁcȝ Ǜǧɐ NjǀQ

 6$(.; '$"+ )6$; Q &$!)(



ë B9ɐ

$#&+

+: ГÓԪϔՏ ɶʓՏ q ń"*ń '''hv ''9Տ ä

'''2Տ £ ''Ƃɇ ɇ b

'ƙɇ ń' ''`ɇ ɇ 

Ɓɇ

cĵģńƅ¢ 'ɇ '  \ɇ ɇ Gȷ  ń 'ɇ ʒǖՏ *G;w
 '''' ' ɇ 'ƒՏçՏ ɇ £ ":e4> ΛՏƒՏ ƃɇ ɇ  £ gńĭ ń&ń'ɇ ťłĉɮ '\ń ŦƋŃĊɮ [ɇ ɇ  £ 'Eń£  ńƚɇ ɇ  ɗǔՏ

bûÛń8_ń2 ńNPń*ńơɮ ń,8ńQ}p.x.ZX;X *ń7#Œń!ńi+ńĨń †yO.aFX<~&!  /,^ń ńµ7¯ńń / %!$.4 ¬N ń*ń*ÎoĆ ń#8Fń . ń Ƥɮ ńP^S[.v ·¶,ń #®7ńń 22”ń ¼*ńH! ń†H.† #nɮ



nɮ  ! ,Fń  /_ń ,7\ń 8]’ F­ bÂ

ń†J.† ʒȷ вӧՏ n.v.Y\:Y †j Ĥɇ

Ƅɇ

ħɇ

 .ĩń@ń1: ĶÜń . Stń 5ùń@ń°Տ TՏ /6  ń †I=v=Ekx: k88 dK ń ĕՏ ɇ ĖՏ  ɖՏ Eń xՏ +Տ  ŒG>x> ҸžՏr Ť: }ĚrQ ˜ń `HńӉՏ \Տ Տ ‰ ńâćɮ ĪzńĕՏ \Տ ȷ _l6 .e8 īě:ĜńҮՏ B˜  ZǦɐ xՏ Տ ɮ ĻÝńгӨՏ s

tÕO•ń `ïńӊՏ \Տ TՏ Šē ń GĆjȷ Ĭçń wՏ Ėɇ (~ `m6 ƀá { ńƒ… ńwՏ ń  Z8

@ªń –ſՏ (2ńýQ}

þÖOè–ń d… ńsɇ ėɇ ȷ ² Ē ńãĈɮ Uu ńj ńǶɇ C˜  ič ńKÞńҭՏ ®Տ ʊ  ńwՏ ń©ǕՏ e6 ń@ńՏ ʉՏ R~ 5KńđPDńĤ6ßń~üQ—ń

–ʋՏ

£Ǚ

Êń<jĹńÒ2Տ 8ąՏՏ rɇ P-'B'S ƶɇ



ǞɮŘՏ  ǥ Տ 

Å0 0I€ń

Ƽ&ŖǤưɮ 'ńȷ ȥɐ xՏ   ' %" ɕͼՏ  A˜

ű?ɮZ

ŰȬɮ

ȭɮ‚Տ 

/Տ ‚Տ c?



Տ ǑՏ

ńń;0™

¿× +ńLÏń"ñń' ˚ cöɮ ń sĢŢ®Z aÚɮ ˙Տ ÷ɮ [ X ȮɮՏ Տ ljɐ sģȑɮ ŚՏ /Տ 1Տ ?

f:Ań



ȸǐՏ

ńńń>;BZ

=%ńńń=m; Zń c< ń



.ńń  =ń Vȷ #Y XɮŘÿɮ9Āɮ ! Ëń÷ĝń‚$!ńЃń1 ń  0 į%ńhȷELM 0'9GS fLńI ġń  B%ń!ÿńúńSğI;ńĉ  ©ń ɍ Wɮďɮř‹.ɮ Šƞāɮ̅ŻՏ ƨ‹.ɮ Š!.ɮ9ĂɮƪɎɮ9% ɌɮȜżՏ !Ž ‚"JVńń€ęķĀ 5šń 'Sv?ń$ĽńėĞ+ńńɮ $ ńC'S ń õáq›ń K??<E'S é ńń$5Vń< ń $ Aœń À" ń+ ľńx ń :ń5ń5'EGS <9 ' S G.'@'S 2ES S GĠńļń!ń1ńĮńĊ  =ń‡ń čɮĎɮ v‡ ń ° ńVń3 ń?ĥń5 Ŀń:A 0 ń$A[ń Æ ńP'S /O'S ˤՏ ?5R'#S  ńɮ%'ń"ńń 'ń  Bń9#S +> ń+ń Ďńń  'žń ¾Uń ¦Wń´ŽńŇjȷ)ȷ  ĦńÑO?ńSðyń ń5<ń 'ń ń > ń  ńā ń LêńBńń<:5QS "3%ńUń^Տ Ô ńɮ C'S ńń 'ĂàŸń ½ń>ń$ńĕ%"Øń1ńG.'8S ĔńĖ 1 3ń1 ńmGń ńń'S (DEIS  !#' $"' ńx"y ń%ń'

'

ljȂȝǹǠȪȢȯwɮ ǂ>ȶǽǯȗ>†ȃǩ>Ȕɮ NJ Ҥ ǪȄw†ɇ ±

1Q\D7f_h@ (7sF@[7xR;7S *S~ZfQ7? 1(

Þɮ 

1B\Q_j 1 C9rQ_\

ƑƓ5Į ŧŷƔ

Ǎǫe Vɀøɐ ɐƸȮȓ[ɐ Ŷ14

$Zf_hr7\r 

t¡½KÆ ++Æ Æ ‰³K¢­bzt¢Æ %t­K’ÆÂz³’Æ 1t¢¾K’¢Æ ztÆ­VKÆ 1t¢¾K’Æ £VKK­Æš¹_JKJÆ 'z–ƬVKÆo³j­_jOÆ @Zz_>KÆ ‰³O­_zt Æ Kv­K’ÆÃz³‹Æ 3u¢¾L’Æ ztÆ­ZKÆ 1u¢¾M“Æ¢VKK­Æ<ÃÆ¢W4JcuRÆ ¬[NÆ=´99jKÆ Gzr²6_yfrSÆ ­XOÆ kK­±K’Æ Æ Æ *Æ ½Տ z‹Æ ƲÑ >}œ†zuIdtQÆ ­zÆ ®WKÆ Fz’—D­Æ 1u¢¿K’ )z”ƯVKÆ z°\N”Æ¡Vz’¯Æ Š·K¤­bzt¢ Æ ¿™­KÆÂz´•Æ1u¢¾N’Æ zuÆ­]KÆ 1u¢¾K’Æ¢WKK§Æ 1uIÆ¢Z1IKƯ]KÆ 1‚ƒ›„”e1­KÆ 9³9=nK¢Æ 9Kjz¾ÆÄz¶”Æ 1u¢»K’Æ .zÆ ¢­K…¢Æ ţǹɇ wKKIKIÆ­zÆh³¢­_Ǽɐ Äz·’Æ 5x¡¾K”¥ &1E^Æ ‰µK¢­eztÆ F1’dK¢Æ ɁՏ ѣ΁ӕхȄՏ .zÆ H3j?¸j7­~Æ 8˜Æ 8jjzºOIÆ

+'!1" ̺̄Տ  (  ( ( 5)4$'  ( .#  (  (  ( ̻͌ǏՏ

<Ɖɐ

'O>K:C>7Q ˽ВңбΥϓՏ )M8IK:B?JQ

= ƙɭßɭ ƋĄȡȢ$ɭȣ-D.Bɭ :Ǎ ªÔ$ɭ ;…,€â‚ɭ¢ɭďɭ ƥ.ɭȤ-ɔB =

W w 5…6gj…6y:;$ɭ nɭǽï 6ɭɭÿ-.œ$ɭɚή ɭʼnȄή ƈùɭĉħɭ ɭn;\Go`


ªƚǾǓ6ɭ;…\h6 fɐ ҡϱՏ Đńɭ ÓIc… ĥНsӮՏ ǐñ

(NÎɐ b΂ɭ Ţɇ ,ȴǝƒɭdɭ#,*/

5 … b;[I… U` ,ȳǜɭɭĎš½ŵɭ

*ßɐ ѝ΃mՏ U` \hqq…áɭI1Qw

 C 4Ôɭ;…\gnn…£ɭ"+).…

‰–Ęɭ a=ZGq… dg…,QȞǞɭQɭ \hpn…

$ 8h|…ÿƕTdtHOGm…6:. $ɭɭßɭ=ɕɊ  ɭ ' € ÝȽïŔŘEɐ 2… #… XU\W' 5 … “ɭ

¡2ɐ #`

Ö´Ùή Á“ɭ

KÜɮ 9gkI… ɭ µ

Eè s“

5ň :ȟɮuPG…j\;cI… }EB ɭɭɭd:ȊnǺ ɭ$ɭɭ#ǟ

 ɭ¡ɭ‚ɭ -ÿɭá

]€…  2… ^€%
AƒŃ J )I…ɭvPI…;@ ȱ d



]‚… 5Ńƒ… $

Õ´ưή Ňɇ ɇ 5

"‚… ɐ…ɃƝɐ

E è *

"„… 5Ń„&0



@ɋɭ Njɇ 'yɭ ;…,ăïĆɭ ƒ -ņɭ Êǹ ɭ Ǒɭ .ia€ɭ ҢϲՏ ,$$ga:yɭQiƛ¥ ǒQ$ɭ}RJc

Uo…ƺĆfbɭ'²ɭ ÁsŅɭ 5 … "0…

6… $1…

(NÏɐ sŶɭ

Æ ɮ ƛ

o–Jɭ w€ɭ ɭ  -

Jȃ Võɭ XyɭcyaAIl…hM…Zc I -fɐ ɐÍÖb„è |ЙԇՏ ɭ=.

ȿɭ  ɭ¥ ã gɭ .ia)ɭ¢ɭg  ȵɭ $1\.d$ɭ …‚ … 5 … ŷɭ §Ǽ ̚ñՏ‘Տ ɇ ïՏ ɇ

A è ƞɐ

B è 

+,ɮ w-ɭ ɭ 4ʼn

 C âČ

żɐ Ʈļɇ nzD…ɭ $Bɭ:Տ ăÚɭ:Տ 2… ˜

ºÂ



Ǒɮ  ɭ

Eè

, á 

5)Ń

ɭïՏ v;c…:Տ =.$ 



KÝɮ Ɔt



%ˆ1MǍŨǍǍ ƓNJUǍ•.ǍǍ ǍŃǍ–ŇǍ ǍǎՏ NǍŪ(Ǎq¸ǍưǍ%-Ǎ ††º ćMÆǍ Ž1fMǍ @˜ VXu  DĘ%Ě-ÇǍ ij. Ē 1ĉǍ Jĵɐ

Č.Ǎ Ǎ Ǎ vǍ«RǍ 80© kÖՏ ƌ<Ǎſƀ%ěǍˆǍ

Â

7…



N ɮ

Ƹ™

” `ɭ ƥ¯kǍ%-1MǍǍ ¦Ǎ ;ҹԟжєÖԃ;ӌ΅Տ Ǵr ŲϾ&;Տ )ЏǍǍ ƦĽũĺǍ VǍ Ǎ àÿӈàђmՏ ¦²¾ƥ2ǚǍ · q  ǍǍ ǂ$Ǎ  Ǎ:EǍ † ‰º ğġÜǍ F Ǎ .Ǎ )ĠǍ tƕ2ǚĝ<ǍǍ œŠ&Ǎ /Ǎ  Ǎ ĝrè;Տ Ā-ǍRǍň ¯[ĵǍǍ oɮ 2ǚ ͣՏ (ʼn3¸&LjÝ

Ǥɇ

͒АϑՏ ԮёԜϒՏ <Ǎ‡rɮƿoɮDžǗɮ œ(

6 … $

ƨïØή Ɋĩ

ǐĭ



)

P ' Ç ɮ > Ǎ Ǒ

4

s

+,ɮ Tį



0¶‰» FŠmG» Y‹» ¤‰» ¥[I»  2vG»  ·awveZ» ‘‹‹n» 6G¦·GI» ‘ x » 2~B»  ‘ y » 2¤» •2~C‹z» §b{I 2€C» I2:[» Ÿ·c| » RŒ–» ‹~I» [‹¯™ » 1\2¨» d » ¦\I» :\2:G» ¦[2¦» ©\I¸» yHI¤! »



‘OÉɐ ]Ȇɐ

9ˆ ƌƜɐ Ʉnj &e²I€» 2» ¦—e2YmI» tu"#$» nI¦»

“ƱËɐ ]Ɉɐ

2‚B» ̹ĽՏ 6I» ‘Še~¥ » ‹ƒ» ¥\F»  cBI » )+¤ 2~D» )3`  °:]» ¥^2¦» ą-€Ǎ

=[

38 ¤ )J¤» șՏ 2~C»  · 7I» ¥\I» :b—:¯x:I~ª˜I » dI» ¦\F» :G~¦™I» ‹R» ¤[F» :j™:°v ;—e6FC» :eš?nG» S»

ÊËMƒOý 2~B» wx)+3 ¤ —I ’I:©e³Gn¹» -±’“‰ I» · b~«G™¡G:¦» od~I » )+¤ 5B» )3 —G ”K<¦e´Ioº»

ēRǍ)+¤ >˜ ɮ)3¤ ¬\I†

5… #„ ɇ Gl“ƙɮ

6… ),¤ ɇ Gl”ƚ

’OÊɐ )+¤ Տ )3'¤

•Íɐ )+¤ #ɇ 7Ƣ‘ń

ĔvŠɭ F7j’ɮ

ɇ

Vhɐ +¤ 2„B» ¸ĶƇ

Ɵ•ɮ ɇ )4&

ſɇ



$%(+ *&( $"&+

 /±‘‘Ž I» 2» ›cZ\¦2~YnIC» ¦šk2YmI» e¢» f~£:šl8IE»e~» 2» :f–:nL»‹R»œ2Ca± » » *I¦» (ɇ 2~C 2…YpF » 'U» ¥2ƒ» (ɇ Տ -¦3†»ǹùɐ WB» ¦[I» 4žI2» ‹R» ¦[M» §g2~YmI»

% 1İ

+I¦» )/ 9

» ºш̴Տ





ȎǂRȼńɐ

.N§» Ȟ »





/%/&/,-.'/ %}C» ¦_I» µ2n¯O» RǍ øɇ

Dzɐ ţɐ œº  ,F¥

ȉƗdzƵȃɇ

ȋȌȍȎՏ u˜

Ǹɐ 9I»b¨ » 2=¯§I

?+ >* :  Ɗɮ Oº º Aɐ " ' Aɐ

»

ÿɇ

8

9

Â

Ĭ

"ɮ2~C» 35`8I» ‘H–’G~Ci:¯q2—»  PYwI~­ » e~¥G— I>¥hY» 2®» ‘‹i†¦» )ÞǍ /°‘‹ I» ¦`2¥» 7LJɮՏ 7 „ 4ɮ 2C» 1)Ǎ* » (V»2ms» ¦\I» ‘Ši‡¨ » )¤+¤0 ©5` rfQ» ~» 2» @c–AnI» X~C» §]G» rIˆY¦[» ‹T» »́ǡ

=G[

71w

+• ĜǍ Ǎ#7Ó ƒǍ Ǎ 0Տ ¬Ǎ Ʊ Ǎ°=>Ǎ ­Ƨ*Ǎ  ǍŽ ǍƨǍ 8ºÈ7‚Ǎ ®0Ǎ , ūţ

] #Ǎ`'7 ~

BՏ +S9Č

~Ʉ~ɮBՏ Č

PǍ ˆ‡º BՏ  –

F ǍGĪƟǍ3!A Ǎ±Ǎ W ɐ p 9Č Č ǍƉǍǍśǍ !Ǎ  Ǎ 6!Ǎ 0Տ "ǚ ƠǍ ɆǦՏ d*ńǍ  

ǁ¢Ǎ Ǎ 'ǢՏ

)— ĐTIǍ

%Čč

)4ǒɐǍ

%Č?

ǓɐǍǍǍ Ǎ Ǎƙ5ƍƄ

#†Ô ‘#^ǍČ G˜ 9ÉǍ ŜPǍǍC Ǎ> Ǎ ˜Č

™ eǍǍ+6Ǎ Ǎ0Ǎ,Ǎ Č WǍ  +Č ,5 Ǎ Ǎ0$ǍǍ ǍUǍ Ǎ!Ǎp! ƚ (Ǎ2

»%  ŒǍƔÀ#ÃǍ Ǎ šɇ  $/TżǍǃ! Ǎp6ǍŚ5WǍ 3ǍšǍ(" ($/ CŰǍƁôÕ ƈ†Ǎn G Ǎ*ƲŊAǍ gɇ ˜Ǎ$/ Ĕ Ǎ(/ F˜ Č ǍnkuČ Տ "J ŝǍ Ǎ kǍ ¬ Ǎ Ch Ǎ Ǎn

CK

+š eŬǍ ǍW+AǍ­ ǍCǍ0Ǎı6ǍÁ‘ÊǍ9Ē i" $Ǎ b#Č` (8 đɇ  Č ǍíXČ `Č Ì! bÍ 2 › ;¾ǚ / j¦Ǎ óՏ Ǎ *Ǎ Ǎ Ǎ 6Ǎ¢ ǍA Ǎ+6Ǎ#Ǎ 7Ë A#Ǎ?… „ Ŕ\º]Ǎ Ē \#Ǎ ˆ7 A € 2Č čǍ ǍC ǍŕǍó  ČɮǼ¸1Pɮ

šÂ ›Â œÂ

ˆAY CœČ

  ĊjĿ Ǎ  ǍǍ ` 

ĶÌǍ  Ǎ –w=ǍƩ=ŹǍ;>Ǎ 4 ƪǍĨǍ¦t ǚź

}Ǎ—

Ƈɮ§ƫß

đ|Ǎ +Ĵ$ǍŘAǍ/*Ǎ ǍǍ( Ʀwǚ!ƂǍ 3Ǎ Šɇ ĩGǍ Ǎ Č

ķý

Ÿ ;aǍ )¤



Ą {Č :ČÂĤɭ9Č)|ČĆ1Č d¨Ǎ  Ǎ +lǍ  Ǎ ˜=3Ǎ 4(Ǎ )

 $ /Տ Տ nƜǍ

Ǎ Տ &

%¤ )¤ ¦ɇ )¤

Ƭŀ Ǎ  



YJ eŭƅPǍ  mǍ /±Ǎ  Ǎ =GǍ ,ƭǍ !(Ǎ Ǎ ƎƝƃǍ }Ǎ  Ǎ rǍ h§Ǎ  Ǎ ! Ǎ  Ǎ   vǍ WǍ 8÷ÖŮŅǍŻ$ƆÂàǍ \ęøǍ ģǍƼů4(Ǎ Ǎw* ǍǍ®ǍĻǍ ŋǍI0  Ǎ Ǎ á‚

ž ò˸ή Ǎ vǍ ơƞh¨PǍ žƇǍ Ǎ žwľ iǍ GmãǍ ǝ_ɐ ŌǍ ³ǍƳ>Ǎ ļǍ TƴǍ TǍ 0ÍǍ  Ǎ AǍ ™  " _W ɐ ΏՏ ,ǍņōƵǍ Ǎ ŢCǍ5řƶǍ —Ə âǍ

ǍƮþǍ



Ē,Ǎ /i$Ǎ ,$Ǎ6Ǎ 0Ǎ ?ǍƕIJ´

 [(.4`  ǚ ý bðÂý ="_6ǚ ~Y© CÁý ¯$ý 8İɐ

‰?ý ý <Wý Ӥ>Տ = "ý )¤

=Džņ«6 ǚ "

ƞ(ǚ͝Տ ý ., ½ɇ 2Y©.F` Տ Gǎή ¬ýµý ó8ý +ǚƅÜǚ ȣ`ˆ6ǚȺɐf©˜aS© ®E/ý ,ǚ¶ý #Cý ™E"Ÿ

]v© Z€pPgx^



–—

žɇ

Ćª«ɇ ņɇ

 gɐ $(+ + ˜`T© \_œˆS© LSq~Ÿ © Ni‰OqW‘© *Hý ý *Jý Aªý Ŏe ǚ XiBº ŴƖɇ _6~‚ǚ ȈǠɇ ϥՏ „U‰

vǚ L•`©–Dy^Vy˜© •~© 'ě8i^« ý ý Nj_Ǎ N Ǎ œÃTÈý O~ý ý _ŗ(ǚ —© , © ,© 8ý ՆŹՏ "Ôý ##ý ŠKý ŷ

eD _ Sxeý ïý *Hqý ‹ý "ûFý V ýý 'ý ~Y©0©

¡ q6ƻǚ ¥©£§© „ )+ +)+ + i·4ý +)++ &%+ `6ǚ  'ý = èý ^DDý Õý Š© Bñ"Éý Ĥ>Տ )+  + E y›vMV‹© +‰TDr©’‚s˜iƒy’© ˜~© ý b”>

*)+/`¦£¨© >˜ {pý



 +Îý —bV© žs›V©Y

)+ ĵ <8ıɐ

   X º @ɐ

™Â

Û¢ æČ ! '#+   + @ČȤȅ [ ɐ *+ h© aȗgǤȡaǟǯǩɐ рՏ ý " ++ˆDQj›© |5zý +>¡ý Dý é(

 Č

 9ˆ

#( uÇǚǚ (  ( f(ǚ+%ƝCǚǚ¶DŽMƋ %¯ǚ ƧŬ¯ǚ +ǚ = l

  Mǚ %$ (  (

ǚ ՏƔf»ɇ ĚųKǚ (ªǚ`ĺ(

 ( '( †MǚC ‡ ºɇ

¹Â

R˜ h˜ S˜ i˜ j˜ T˜ U˜ k˜ V˜ l˜ m˜

W˜ R˜ X˜ Y˜ Z˜ [˜

r˜ n˜ o˜ i˜ p˜ st˜

=1' 9ˆ 6a ŕQ\ǚ ǚ Ɖ 62ǚ4Ɩǚǚ  \·ǚ4^ǚƨ2ŴÂǚ 211wƕf¼ɇ szǚ Ÿ³ ǚ \CMƟzƩǚ ŵǚƊŌǚ 

(ůǚ ţ¸(ǚ4Ûǚ tōǚ ( ǚ Ŷƪ ǚ CM \%ǚǚ °"2ǚ4ǚ #ǚ Ņ2ǚ4ǚ …ǚ 4&Ɨǚ &( :vǚ ! ( aǏafCǚ  ƺǚ4şǚ zǚ 4&¸ǚ †LǚŖ Uðǚ

Ƣɇ

—ǵ  "

*ĒɮƀՏ€ɇ (  ( *CĽǚ f ǚ !"

 (ǚ ƴɇ (ǚ ªǚ ĺ?ŏƕ1 ƕ %ƕ

*Ƃɮ-#$ )ή!#%  ( 16

Ţɐ

­ Wž5p­ Xɇ 5€œWg"’6„"e­ ’„W"oKe5­ xC­W25­ ­ 4aŸX25­ 5"/S­ ŠW17­ Wp’x­ ’S‚85­ ;C {*‘­ .xq‹’ƒš/’­ űɇ 5œWf"’5„"e­ ’…Y"oKf5­ xo­ ’O5­ kW22f5­ {+’­ "p2­ ’P5p­ 25e5’5­ ’S5­ kW22f5­ {+’ ­ 5{5"’­ ’UWŒ­ ’5|­ Dx†­ 5&/V­ W25­ xD­ ’P5­ ‡9Ž›e’WpK­ {xf§Lxp­ Zo2­ "  7 ¢Q5‚5­ / F ­ ’R5­ %„5"­ xE­ „:M_xr­ x-“'`v;2­ A „5{5"’WpK­ ’SWŠ­ }‡x.52‡5­ [pJsW’5f¨­ l"p©­ ’Wk5

L

œ{{x5

£P5‚5­ ɇ \­ "­ ~xW’Wž5­ ]o’5M5„­ "}Č

1



bNZb ,5­ oxro<M"’Wž5­ ^o’5M5„­ ›.O­ ’S"”­ Wƪɇ Đɇ 'Տ +Dǚ 'Տ ɇ

Kº  6ɇ žɇ ɇ !&S øČ Bɇ Տ ¬F+Ń ’S5­  (jœ5­ xE­ 6 3

Dbå

5’­ Տ -5­ "­ {zW’WŸ=­ Ww>K5„ ­ ’­ $* cpx¤o­ •T"’­ ¤O?o5ž5„­ ’R5p­ õBČՏ !ɐ ʫʬՏ Տ Ex„­ 

Döä

yoW25‡­ Wt’5M5„­ ª­ :Č

4

5

6

# ՋՏ .DžՏ Wo2­ ’T5­ k#ef5’­ {xW-f5­ ¡$fœ5­ xF­ Dz

­« ­

Â

VN])bJ Ⱦɐ EՏ ǣ!ɐ ĵՏ ­ Dx„­&he­ !# ՊՏ ­

<ή {"‡’W0f5­ W­ WpW’W#ee§¬"’­  ­

’­ mxž5­ ’x­ Űɇ "3b#/5p’­ \u–5N5ˆ­

Wp­ ’S5­ o5¦’­ ’5{ ­ !S$’­ $* ’O5­ B5/—52­ p›m-5‚­ xG­ ’5{­ W’­ ¥Wei­ ’#d5­ ’x­ „@"/S­ ­ Hxˆ­ ’S;­ I‰Œ’­ ’Wn5

əՏ

<3NJ,<:IS ? ' º &¯×ƕ E &è

'ƕ ]™ƕ ®P¥ƕ úŢƕ L?==º†ɇ ? L=ºɇ ? \X=º N<ƕ K?==º…ɇ _>º I˜ MZKXº ¡8-#1˜´xƕ '*ƕ *ƕ C ƕƕ ? \X=º #KZNX­ 'ɇ L?==º I˜@\W.º

 &Ƃ- Øƕ  ("( uL=?Rº( = º ĴŸƕ ±º}ƕ K=@R/º LC8 ƕ²º-Տ L=?SºƤɐ=0º '*ƕ # ³ ɐ ɇ ¹? 3 K ƕ ³T" ?º( ?º ƕ ¥ƕ  ƕ ƕ  ™ ßƕ ƕ ²U³?º ɇ ǝv«ή ƕ#(( =º? ?4º º L?# ³º# L=?Rº ¤ƕ Ź»ƕ "(ɇ ?º ƕ ±ºɇ? , R' &h¯-şZƕ ŽƯÈĪ

5 … ]<ƕ Iuƕ ƕ ƕ īċĻ ƕ °ƕŠřčůƕ Ƒ}ƕ$( '( %( -C -ČļTňƒàƕ ƫÇɐ ]ēƕ E Nƕƕ   ƕ ^ ƕƕ =ƕ^oƕ a=·º (b=¸º E8#Ľź´È V 1 &ƃÙƕ 5 !

‚ ƕ Ɠƕ8 1ƕ=9ƕR¼ L=?R•º? _=R—º ? \_?–º-Տ @\\V—ºG˜ K=?R—º @_=S—  A\_?—º ? \\V— º G˜

L=@Sº ? _=R ¬$  ? \_B? \\V ®º( I?<¬º \®º

J=CR˜º?_=R —ºD\_? —ºJ? \\V —ºG˜ L=CS–º ? \_E–  @_=S—  ? \\V — º ?˜

O=@S! ? \`B ³" ? _=R# ‡ʑVÆ ԿՏ(" '

L ƕ L=?R—º F_>S–º"? \_?—º# ? ]^V—ºƕ <ľŻĄPƕ ąHƕ \º 4ɇ KcºՏ K=Rº ƕ1šoƕ Ś żěƕ-ƕ k¸ɇ W2 &!Úƕ ‰vJ ¼

òĚƄ?Űƕűƕėŝù ƕ  º `'( ~ƕ RÉ ±´PºJ' # '(ɇ µº  ? ± ¶º)ή ? 

K ƕRƕɇ =º *hƕ ƕ‡EƕĪ •P³ƕ Q Hƕ ŲĘƕ ˜eBÊƕ L¬ś8 ƕ=Տ ƥɐ =º + ¦ƕ /N 

pΓՏ Ŀ?fHƕûœHƕ ”ęƕ ʼn zƕ

'ƕ* ƕ ȴr''ý şɇ Ĕ•1ŀũ ¡ƕ5ƕ S ƕ=Տ ¬Ďƕ*NƕՏ?º ˏՏ 5ƕ

Ǟ ɐ &'(J? º­ µº >˜ R 5ƕ  ƕĩ4Hƕ‡oƕ ±4º ƒƕ #ƕ įՏJ?º ɇ =º [K „ ՀՏ Տ@5º M*ƕՏ( 

&° [ƕ A„—

(

²{𠢺* /öğϮ̜Տ- ğՒğ *.

FXº

 E





)7& Gº

Š ğ"¤ºόՏ zğύՏ

/´N - Öğ

‚ =×eøCØ'ğ

&

Fºħ & ɍɇɐ

` ?ğ 2jğ ýċ+%â]ğ & ɇ j?GXº ¢º ğ\ğ = ğ ëe|ˆ“ğ *kğ l¨ğ - }ğÙğ¼}bğ &&

|Ņƕ&  & +

Ă'ğ : Š ă/B ğϷՏ ß ®º & `@FXº `º >˜ * & zµğ Cğ & EɮD>ğ & i?FXº Ƅq

¨DŽ  3Í ]ğ ƒO

³~ğ B 9"ğ +ğ \VZ]

ɡɮ ­ğ

& *.ğ"!&

FɄĩɐ G

*ÝBğ

& '

"‡ğ& & ~.¾ćğ lĄKğ & & °/Ďğ {.¿Ĉ

"  & ɇ Zğ½ğ [C¢ÀœPğ % &

#&

&4

_B-L

-.('"/ gԔԕ˨̼˶Տ



£"ğ&fr "ğ& &H $  "<' S Ûɮ ~ƕ ɢĨ

$&

£ õ 3 ©ğ Ö´Ùȅ ǣɇ

ž¦Տ

&

Ė Ǎ ŤǍŸ.Ǎ ?<Ǎ ǍƛĸǍǍ KéǍ ĤšǍ ŎǍO5·Ǎ«nǍ ŏǍ4s²4 Ǎ5Ǎ œĻɮ ėšµǍ ŀՏ >˜ ̣͞Տ̸͢NjՏ ͋>Տ ©8c…`]Ǎ >˜ ĥ Ǎ

2gǍ

g 2fǍ

ĿՏ

ĬªǍ Ɠ ȵՏ aÂ



ɇ )2µ D9Ǎ

a j8cóaÄǍ Ɣ Oªõ8cǍ… K ]Ǎ ː



ò×JǍ

ɉɮ ĺ

ɁĦ ɀ Ⱦ

a6 ā©Ư|ùǍ +ãŇ Æ Ǎ Տ Ǎ Տ .V Ǎ¶Ǎ[+B³Ǎ?<Ǎ?[BǍ< ²Ǎ`ú99Xæ/çǍ  Ǎ ǍŞB ǍǍ ŐǍ4ǍB

{ű! Ǎ Ǎ ,Ų+UŸpǍ X??&ÎǍ X• 5{ }_Ǎ Fő 9Ǎ ʿՏ %ɇ ʾՏ Տ ʽՏ DǍ O.Ǎ Տ ͉Տ %ɇ - ¤Č Ⱥɇ

÷ɇ

Üɇ

͊ҞՏ ǍŁťV4Ǎ ¶Ǎ ~Ǎ+ŒƷǍųǍ

J9„ Ă|Bû



ýɇ

%4

O îŎ

ΤՏ

˲Տ ÆϋԁՏ )ǍīǍ )¿Ǎ“Ǎ ǍXBƐŸ4 Ǎ Ǎ ȗՏ ΈαՏ

f½Ǎ ƑǍ:EǍ Ɩ s{ $èǍ F Ǎ

@AL %)Ø %)Ǎ nj%E Ù ㉀Ǎ  8:-Ú %-ÅǍ ‰Ć¾_ >˜







,3è







#;

#)* tT—©>©JR©>©LT©™cT©‡Œ…oTN™nx© đϰՏՏ GyR© ̷ļՏ oǓɇ &,©!$  &' )* —dTy©?>©

kz398<©$©





,L

$4

lx4::?©Ěɇ 3;;=©%©58 :>©

ħǚ Ê1_Ē1»Ǎ <˜ 5&,©a”Ē ̙ԀՏ [†qu… “© 8$9B Â`Ē Տ %1¼êǍ

 'y“¡T‰"© .

&ÁğC

,L [Fğ$ ';



ěɇ & * =˜

1®ή $ B FĒ ğïÛąğ HT©

#;

 *

(;

;

Î tÏ ĉã€ðH!ğ

; $; ); (‰TD©%© Šɐ ɇ ƻɮ ȳɇ ƺɮ$©/ ÿĄ ē&w%•ǚ /“/G )&ğ

;,Տ €<Ʌɮ dz¶æɮ Տ ! €DŽ<ņɮ @cTy© Ǵ  ·çɮ%©  `Â

& & <ɮ 7; dzŷ™%×ɐ ğ•ȖՏ ŗՏ 8; FxR© \Ż‹ȻÄɐ* Ňɮ @eT{ 

& 'x¢X#© Wǃ Dɇ

İɇ

¸äoÈiğ 6)%© ËĒ D|R© 7.©%©3*© %Þğ &"%* ·1Å&Ē DxP©

&Ē 1&Ē  $ ęɇ 5A Ä&Ē ,=©

,

ˣ̀Տ



-=©

/*<©DŽT©“iwiqDŽ© AcTy

( ;

&ɇ +<©$B© ή$©Z7º

* oĆ3þªğ 0 @cT© !* hՏ Ǎ™cT© (* f?r <ğ L¤©šcT©qmyT”

•Տ

C$©: ; ıՏ Տ % & p ušǔɐ ɇ :

%.;

MƕĀ§ƕ#”#ƕBhcw=YeJF…Wc… «Տ ƕWÖý ¨Bƕ¼ɽÍ>bÈ^X;Í î#Ĝvƕ Twn…m;ET{r…Xs…ԍ÷ϜՏ FVrw;fCI…

Nmh`… Џý BţŁ4ƕ XÍZ Í ¨Bƕ +-b ʽ͕ɭ^Z”Í

ZÍ ` Տ (7 Í ^Wk lÍ

Dzɇ K˜

Í÷ϝՏ Βo×Տ ҦϳՏ -ƕŊ ƕ ď4#ƕ4ƕĵƕ ([)b •ɭ !' 7 >˜ Zj<Í ȹɇ

^= &±9[ƕ ^X> "ƕ 3<ɇ Տ

ȷɐ ΋èՏ 3õɇՏ

b

ǀëǍ F

(Տ IWb bJ*-b ?K` bLa b.MXL) K'b b£l



S(Í>C m'Í.NPb>bxzÍb _k‚Í ŖՏ

>b ^‰Í1 :SLc… 6Տ Տ ';;cF…­Ūƕ¤¨Íɇ ^Y?Í



^†@ 5es~Im-… ^Z^D äüƕ^–Í ɇ ɇ ^ZZXAÍ ˜а hý uՏ >˜ NPRb …ah ¥¨­™’Í

¦¨­ Í ¾X^Í©!¨©

7>nK… k—Í ^[\ZÍ ʻʼՏ ɐ ɐ kXZZ=Í ôj4ƕ ɇ K˜ ^¦©®BÍ ¢²°³ Nb P S>b:b



AO\Ub.=b kXZXÍ ʹʺՏ ɐƗƘɐ `X^. M c…Șɭ•ɭ kZXŠC …9ĝ%Bj½ƕ §ƕāĞƕ

3

667:^

NPTb¡Í ¿Z^Í

4 =% bČ

’Í Í Sº ^ Í :b b.b Í :b “Í

Տ ƒÍÍz‚ÍÍ^Í F˜ ^Z^Í ­ĐƕĆğË



§”  K w 8 x #˜  )&𠧺ğ m   L3

j g"1 g@ 

 £ǐɇ

<ďé ğ Ʋɇ ğ§=º NSǍ ,ğ®ğ Lğ^?:Nm

g0 Y g@2 i@"J" §º Y? Y Pb qS Y  Pb $𠤤Ǎ



6:=Uf&'m m )Ð&ğ Vfm ğÃ(ğ ğÇğ#•ğ ,?9M



Ƹ¤ɇ

ğěğ 1L6m ğ(m

Ƴɇ EՏ mɇb ğ P

9ğT5ğ ?:ğ ğlm -@`\m

üJ ğ ğ/,0 ŔՏ ( ? Ĝğğ#4ğ #–ğ

+GK4:mu % Àɐ ğğ 0O7m už ğ ğm ԲmՏ ?/a:mû ğ ğG(ğ ğČ  ğ Vgm ğğ ğ6ğ .?:m I1X=:]^m vsYYD@Ɠɇ

IžǍ Ñğ Um  ğVT'ğ ğm ğŸU7šğ $m *Od ] :W%m Qğ

[A2 YB Z$@ĉ ǺɆɐY Y@Z Ƣɐ Ǚɐ

)Fğ mğğ

,?8P

»jğ Ėğ w

;Ağ w |Đğ 0!vp0£Ǎ ´ƌ {4ǚ ´+ǚ f m ğ ì(ğN ğ ğm = ğ ğ ğ)m L1J:Iim , ’ğ ¯đx0@ė‹ğ ŴSǍ , ΚŴՏ ğ ?8 Ǎ Ċğ å ğ T3bET`Ym _?1_m / ƶɐ

ºàpğ W7  ğ ɇ ,Xğ` Ɍɐ ğWcXğ ntç ğ

1 ' L w Ċɇ



ʷʸՏ VR

ZA2 YA#5



Œ™ Ǎjm ğ hm -A:OmÒ¶ ğ ğJˆğ $ ğ+òğG!ğ ,BFZm

]CQe[m Ǎğ ğ 1m ) &ğ S[3*ƒüǍ/£$

ĕ ğ ğ#ğ jm ğ…Gğ Տ /ÁĊ  ğ ğ+( #ğ ğU! `gğ 7;ŕՏıՏ *Ǎ *0Ĺ$ÏǍ 3ğ n`E _m ĭƽǍ ę ğ 9ğğ $ÿğ 1IIjm _ ğ · ğJ ğ ğ

z q {Â

‰ğ ğ ğ6ğ

 Eēy«ğ ¦P

 Ǎ 4ğ Ęğ Å÷<ğ Iğ  :lj 8m Ǎ+“m°Ǎ$ğ á @Ä2Œğ   ğ 2gîğ

1O6m H5ğ Ā v 2>€ñH˜ğ . ß

ğ Šğőħ ɐՏ Q8Ěğ ğ76 ^ ğ ;25?mmğğMğm mm s æÌyğ "jm ğ™8Iğ ğí5ğ èO!4ğ !jmJ8Iğ ğHm Ħ  m ĮՏ

Ø'Ē

 Č

!m ɀ·ɇ

Aή įՏ ɇ ɓӠՏ 5 ή Ɂɇ ɇ ȩĨɐ ő$ή -&5?ͯή ˎՏ -%, ? †Ǻáǀή ȁɇ ,Տ 6? ɇ ,? ͑4A $ή ȇɇ ɇ Ġή­ ǵǶήǁή ˽˻IJɫǷή ɻ 'ήŘή .,#? )ήή Hƚ ή Ͱή > ή Hm ~~H' hή

SA­ HÅή $Ç8ÎήXή

Hg ή ή

Ñɇ

".,$?b  Ưή Տ L? ήb??? bbb6b b/ǚ bGή Տ Gáή

bAĹɇ

ǻ <$ 'Ńή Z .LJ A>M ή ')ɇ ') X‹'ή~dή k¹ɇ ðćή ӥđќϐՏ Hm ķɇ Υǂή .ŸՏ ȦƝǃή (4'ήƖˮ .ή  {$z(ή  ) ) ) ЯˀˁՏ ,

>z4ή (ή  ) ɇ ! ) ") ɇ Ĭɇ $ ή #) Տ $) H$ ή ') ɐ ɐ ]Xή'))% )$ )#) Aή ˜SA­ .HÅ /ή'_>†' ή/$*ų$ ή 

x ºw

}ɇ ğb

 <$ 'Ʉή ȹŁį

4 JK

Ȳɇ  ķή ɇ Z.Lj



Mɐ '$Idήz$ή8ήX‹'^ ή> †$Iήğήľğή ɐ ‹͊ήGήmŦk.){

Aή4ή.Hή $>8͋ή† ή

I )ή6b H )ή Pɇ

ɑՏ ńɇ ğ ɇ

y!)ή  

ɇ ɔZYdžՏ

Ǽ <$ Ò'Ʌή GGĮ Ō(ή = ˆˇՏ ή 8  ήʍ$ήÉ(-'DŽή ̙ ή~ͱή ͈ċՏ cɋή ή5 Ś'ήŢή  ήEή$ή Տ ͜ή6”ŶՏǂǀǁՏ ŵՏ6ċՏ H

 z'{8 Džή(0kƕƕ˯.ˠή 6”Տ ƆH›$ ή ͌ Ǹή ï$  'ήɼ̚Ƶȧή Ŧ͍ή ') ˄˅Տ!

ɡ ή  ë ή )  ή ̶HúÉ ή Ï İή ¹ũc$ή(ū͎ή ʎ'ή ɒՏ zĄ˰ʥ ή ˇĄ'ή >Ãɇ Œ džή 4ή (ŗÅή ÉĐř͏ή Xή ʏ ˈ¢"ή4{ ήśŗ ή  ή °ơ̆h ï ή!? ) ˱ ή5‹ήf$ ή' ǹή A˲$øʼή.4ήz̭¢>'ήĄŢή>ɇ ú ή>$˳_FÄƙή ͐˴$VήǠɐ kƕøFÏήX Ā ɇ

ū{ ή ή_>΅$ή¢ή4ήzʐ ή 4$ή4@͑ήʑÏή Ö̜ήƕ ;  ®ή ¹4ή’x™)­ ̝>8'ήÀή Ρ ήÀ͒

4 ήøʒ ή† ή FZ X_ ɐ

1>o 9UP:Nk;?o ģЎÐģՏ ªѹɐՏ Տ ªѺ ĄՏ Տ I

Ðü

:‡ɮȈȳɮ ßѐîΑՏ gE2fo

}ý ;˜

êý ɇ 7(§ !¢ý

2Sο ή$>̊¿ÎήUBoTHg͝ή ü y Տ /b ȱɇ oՏ 78b

Ký Տ WŷՏ 0ӢՏ Տ o

XÔÂ ˢѼyԱϊĚʤ ų ī ɐ Dɇ

ąɇ

Ņɇ

öăɇ

(WXOnogG@odTή ̮ˉήΣқӟЮѻωՏ TPo Ιƿ͜ µ͡Տ ΆΰՏ RN’ý _>dYA7hIl>Nn% Zo ɇ ¥ɏՏ ɐ 0%  Z¿  Ĉ !  Z ] o ɇ o *Տ 1%  ^  +30%  ]  0F>PoZo Տ o3Q=o ]o Տ 7. '§ '[\NnJRDo iE>o žHTήTCo8UeKP>oLPo R‘ý͛ՏͨՏ 4D4MPoή V6f5R

²­ )Pbm>a&o 5% +?io -o8 ¿ ήI-'S E%0PS  ( *c o /H>P / W%b - o^Ǎ/8%)b >˜ "oîɐE )b Ĝɇ -o Տ ;%!

,>jo ǰɇ 8@ήŪ¿ή ĚÐæüԛyՏҜϭ jƆŅɮ NjǾǰȞ ^Ǎ è

Ĭ  29%  5 9œՏ

#[)  $  ` o ɇ !o ĝɇ DZɇ Տ 4%"

2,b

 ³# :ńή  ƒ % h A}ź*ή ƛή ĕή)ήƛή Q Ɲή >˜ ; Û ήQ

 





ɐ /+ή #ÿή I+ή Ńɐ ¸  ƛ—ՏՈՏ ȹɐ

ʄ—ή





Ɉɉ  I˨ʀ` ɵ Ɋɋā Ɍɍ ; ; ɒ ή .Â

?

:zɮĔQƜή)ήĔQȢΧή ƙɐ Ĝή Ž 5ή΢QΨή

ĩή hή ȫ Ý ; ɩ˗W# ɶ Ɏɏā ɐɑZǪή

?

Σ Q” Ɯή —ήOή Α˘oŜ ή W#ήŴÓήo/ήΗ †`ή ήű.ʴI7ήy TWWή Ǖ 

&ή âή Aή (9ή  Ÿήb:ή /ή# D#ή ƀή ΤQĕή)ήĔQȣ ĕή >˜ ή D#ή   ; ή u   ?>˜ ;ƒ ˜ aȸĩ <` +ɀή ȥľǫ ή)ή ȴɐ / ȳɐ ßή ¸¡ή CL Ňή ? 

 ήĖήìή µ ή „ Ň ή aή aή

  



Ǭή

AƏή èȝή —ή„Ȟή)ήΦȟ έή ¹  % #/ )ή1 ɺ̑ή — ©„ )ή  & #/ )ή*  „ƪ©ʉȠήęήĖȡ ×ή







% /

͇Տ%  ,/



·ă ή  ή !Ōřɐ ɇ ; Ī ή ¸ ήÀ"ή Ñή#D oÑήo¡ \:ή  üɐ }ǟũƱ



%/ Ęή }ĥŗŪɮƲɮ* Ì҉ Ɯ ; ǯ

Aomή ǫ̆ ìή ɐ ʙ ή Əbɮ Տ S  ºƕWήlήÊ+… ή ̩ ή¨Ä> ή  ή èȳ



ai ; â ή

ĺ ƒ­ <̐` W"Ņή “ K™ή ͂ήo#ή ;ή |ɇ

#'



Ȥή ɇ # Ωή íή à

aȺ“ <¡`ΒƉêή Jâ ɮ ή ޟI ήVή ή \ ή/ή CWήbΘήřήŖήƜbɭЌ Ų Տ :ʵ… t‡Ξǭή ¶ ή  ήŖ )ή'?ЋՏ @˜   K9Wή CWήt‡Wή /ή ήbΙή˧`

Ǐɮ ũɐ ` iή



ɠɮ

4

4 ɟɮ?

ɋɐ ĺ ȍή ? ? ? /Â

˜Â

ďɇ

 5ɮ

üɇ íή

¸™ήWĆĐÄo Ƙή ŵ `ή]/ή ňŪɐìή  U` þɐ !­Ǯ Ĝή ʊ — ή1 ή ͻ)γՏ БՏ Տ ƂʼnUŅɐ Ľή

ɞ





ȬήíήȰ ή





N˜ e˜ O˜ d˜ e˜ P˜ O˜ f˜ g˜ Q˜

ī &>ɝ2€ťɭ _Ĭ @ɭ  Ýɭ ȥCê+>ɭ¡ɭ ɭ ǭ ɭ Ië>, ƻɭ'

ȅɮ ò#ɭ TȦúɭ”ɭ p J ĭ ɭ Ɖ2ɭ öЭՏ ɇ

0, 

K%ɭv+›ɧɭ 2P+Gĉ¶ɭ

M 2ɭ™ɭ 2 ɭ ö ʵʶՏ ȧúɭĒ

Œɮƕ

2.7ƕ

ɣɮ Տ Ɓ ƖƗɮ Čƕ ƹɮt Ŕ h t ɭŕʼn ÙóŽ

ЬՏ tɭŊěɭ0 · ɭɭ_Į

Mæɭ =+<¤%ɭ :b>ɭ Gɭ ȇɮ”ɭ _qİ ÈR?Ŧɭ L ñ

ÍɭH>ɭ Ú:2)ɭ Ǧɭ | Ĉ | ¶ çë ` & Č| ɪ |ɭ ʳʴՏ Rğ Îû‚2ɭ 

*eɮ[ 

ɭźɭƎɭ Ē…ĒŐɇ



ɋe

`u‰ɐ



=  …

`u uruIɮ

Տ ŏɇIɮq

˜ǮY.ɭ ǿ«>>ɭ dɭǡ2Z);ǯ%¸ɭ ơ>ĄóɭNɭ €ɇ ʱʲՏ iáǚ ƌ ɭ Տ tɭ o p ɭ  ĮՏ ɦɧՏ įɭ

ƊìƦɭ

_Śɭ

°





Ǖ¾ Ħɐ

!% / / ·æՏ >fh(:Ȩ%

!ɭN/ɭ ^ pŇɭӞՏ ɇ ӝՏ K

ý ý ý Dɭ

5 < ˜

 & ą &½ɐ

6< ?

?

<

9<

T   &  H&K^ &^ &ÐŤ V <:< !< < Ć &ɐ &,IœՏ

(K

<@šÝý ý ɇ ¸SÞ(ý !$ý òG¥IąIɭ hɇ Տ ²IJZήOɭ«'OƱɭ ý :< Տ  & ý 6< "< 



ŏňijɭ

,' &R!)ŧɭ /6 7=K  £ý ɭ ȫɭ :ý ɭ ) íɭ û' Dɭ 0`ý Nɭ jNɭ & (ý 0>K / 7= 7= K #< ƃɮ  Ɛɮ 7BK :ßý ]ý 1 <   & !Ñ Qý

7?K •˜ gҧ IՏ Տ 7CK ý jɭ < ký



!;ý

7BK

< '28K 1D



4 4 & f ?7?8? ? K4k 7BK

U< < ;< ģɐ v ^Í

óɇ

Տ a²ʂή

’Â

7=K Rɭ ìAý ͆Տ !< (3D

“Â



Uuć 4v hX/

Տ /EK 7ý 7)K ɇ /6

  &R!)Ũɭ 4'1 Q

7<

!<

"K M (",  K

1< 61 :ɇ< Տ &  3 :ɇ< <;* !K < ' ǧՏ

2.< 2/<

<&4<ˆɐ 3  ";?)/ -  7ɐ Տ  7ɐ  3   < 2  "  K ;@ ɠ8j º   & < &  2<    & && ğ Ĥ ĥ ɐ &
”Â

7Aý 0ɇ



ɭ ý RIRɭ  Ƽȩɭ + Nɭ Yɭ 18< Yý Xɇ 0` lý Z (  G m ý +".  K †º 2  < Øý P+O/ƽ}%

4" $K Ƶɐ 2< 9¤Sý & OɭYƾɭ \à ý &Z_ý Dí/O«mɭ " (",

\

< 0%$% ÑȋWɭ ƈɐ

Uı &!Ȫũɭ ʁƽ 6íý Ş ˜ ɇ

M < "4 Kj



4(/K

"K "5K ɇ 4"K

&*< 'ɭâɡDIJɭ Ñɭ#&‡ɐ6 < !<&6< Տ (6<-Տ Ž¶ɇ

!;Ò

#        ! & ,  %  " + < -   ! ' ) *< 7Ågý ý ÷Ìý ɭ ɭ



; & #<     $ & &    

²ή

  'ĢǍJìǍ  '' L+  s- 5   ?. 3pu )ή n t.  6  ;  $         # %' ' ' qr '  ~ ºLº2;'; ɲ̯:ƃΆ͓Üή   ' $ 9̞·ƈή    '; užʭή  ! ' ';ųή ʦC˵§ ή ή OBC@H'S  QSS ʯʰՏ ;u R+ή :

-'=





ʭʮՏ

ÀÂ

ŐՏ



 S

“





’ “

<ſ :éή L9 (u µʽ.ή 3S &( rel="nofollow">@J'S K.'S 'RC'#J'%S ѸԙћΘχĘՏ @*S FL'CFS |ή L

-'=S 47S Տ



 S Տ PÄý

NS 2 H S$:'"S ͲC.ή  S E˜

)FS M@S ,@S +@<S œήƙ

' * '&'

īЉЪ΢ЊՏ 1;D82'GS 59S Տ 6S

‰ Ē /'?S P€ý Ǎ ý $ƄɮE˜ J ě ήǬ ǭ Ǯ Տþ $Ť ɮ E˜ 4J ̐Տ ɇ Pɐ )ήS Տ „˜ …˜ €˜ Տ  S E˜

ɴ¦Տ



'ɭ



˜ Ž˜ ‰˜

' '

;u Ɛƃή œLJή œήɇ Lºǩ Ǫ ǫ Տu;23 %u

@

Տ 9JuǍL5 )u

  #$*  ' &* ! ')* 0:'A?F"S K*"5M.3S 3Q6D1 S  S BŒɐ ­ %*,; 2($; (6%;   Q 36,; Q 8$; Q



 Q

vȷȷ0&ȷ ^˳˾źՏ ^Àȷ ˗Տ TMȷ*ȷ *±#êȷFȷȷ&â†"#ȷ T]ȷȷ #ɮ &Ă0 1`"#ȷ Àȷ **‚ȷ Ƨ*ȷ ÓrNj?/ȷ vɇ ȷ /ȷ 0?ȷ TÁȷ ȷ %  D/ȷ &* 9ªǦ†0ȷÊȷ"ȷ0ȷ @ȷ `˻Տ ȷ ̃Տ 1*1ȷ Ȣɇ ȷ Ëȷ "$1ȷFnjȷ % ȷřËȷ B˜ ]Śºȷ ȡ*ȷȷ T] ļȷÁÅ ‰ 

 Q ʼn"ȷ&&ȷ§ZȷFȷ01ȷ ¬ɮe — ɇ ä ‚†ȷ

 Q É#ȷ ţ û ɮ d ú



·

¸

Â

V/ȷȷƑǯr* /ȷFȷQA16ȷ*&ȷ¤r'7#ǽ1ȷ¬ȷȷƖêȷȷ iɮ 

òɇƒ ïȒɮiɮ

'

&'

fɮ ŏՏ £ ŎՏ » ĸ

‘Â



HdȷȐȷ *ȷȷȷJdL6#ȷ*0#ȷ ̧Տ ȷȷ 8 Տ Ųɮ ~ ѮˋъՏ ȭՏ ʀƻ

Â µÂ

Տ 8ѯՏ

ŽÂ

ȲȳŸՏ

Â

vȷ ȷ Ɨ&&d0ȷ ¾ȷ Ȱɇ b ó ¶ȷ #ȷ ȷ de/ȷǘĂȷ Ÿȷ ȯɇ ăՏ șV Mȷ ƹȷ ɮ Ďɇ Ȋɐ ȣɐ  å"ȷ""ȷ MՏ ȷ &&ȷâ1ȷ# »ȷ v¬ȷȷ Jøų&ȷȑȷJ#ƘAZȷ*ȷ ¦#dȷȷƟȷV* Fȷƺ*øȷȆȷ9Ȓȷ2nq£ (ȷ/ç#ȷȷ#ȷǾóȮȷǿ/ȷr&§&ȷFȷ ˛

ů

















'









ȔՏ



ŽՏ

aɮ 0ɮ

Տ ʇƼՏ

Â

ɝ f



















ŽՏ













ȷŭȷ¤–0ȷFȷƙǧrªȷŻǗõ2û ȠƐ É#(ȷ Ե ŭ ՏԽՏ 7–ȷƃöúȷh*y&ȷ5ǍVhȷ†żȷȓ(ȷ ȷȷ /0ȷFȷ ǂ&ȷJǩ1Ȣȷ (/ƥôȷ LJ´ɇ Je`6ȷ ¤/!#ȇȷ jµɇ Ǩ6ȷ(ȷ

ɵ©Տ

D•¬V¾µÅtë 7Tчu¥TЕfTžë E¶g”uÒßë -:%A
 Q

Œúȷ …ȷ 9€:azKfa f? ’ ] ȷaNÚ(ȷ ȡɇ (ȷ 4ǒՏ HáIŽȷ $%'( Տ Ǻ„ȣ ċɮ ÈՏ4Տ Lz , 7L,]:€:u f@ v ³ ÍûKȷ ̠È#ÊՏ  ǻ„Ȥɮ zfG,‚ 4˥Տ MzX ;[;?T:T …@ȷ!kļɮ x:1ȷ , ȷ ÂŗTÌȷ `ȷ ,b Mzfz4:T:z (ª,XȳW"Įȷ ̟υӾՏ fՏ 0: …Nȷ@Ǫ(ȷ fAȔNȷ n:rn:c5M2‡U-s CWȷ ÊՏ WǙ(@ȷ !kĽɮ UNIȷ `¿ ˌՏ Ņ] ͷѵήՏ #̓ÂbՏ N{ , Z(ů0?įȷ ŝƻǚžÙȷ ̡#̆4Տ ɇ   ( Oz $Iȷ B,3ƒ , zqˆ,t9 ȀǞɇ ùÑ(ȷ #bՏ B˜ b¿ ŞCWYNėȷÂ]ȷ Ɨ º·Տ ɮ!9 Ī ` À ˍՏ ̇ Տ #º Ǔ -Â

Î<c ɮɮ/ɮ ɮɮ ƒ  Տ ,c6ŒG:c ɮ "Տ ăɮ@ɮՏ   S xƨ1ȷ Zȷ(ÙȷȪWȷ f0‹Nf‡zzfW‡„Ogaz µČɮ åɮ "Տ ³_Ąɮīß Vɮ´/ɮàľɮ Îą?ȷ1Wȷ(Ñ(ȷ…NȀÚȷ ,Y:cfzfV‰…Kfdz ȷ  ɮƛɮĿ

ɍƹՏ

)‡onfz:@ɮ¬Տ ŀɮ #?@ɮ ±Տ /ɮɮ Տ ąɮ(è

͜Ț®ȷ  +Տ ɮ ɇ ɮ WCȷ ɮ Տ 0øɮ (œ£ȷ



ƕ  Տ ΉεՏ €GKz Kz cf€ ,ȷ zfTŠ„Ofa 

԰ЈШΩՏ K| Ƽ¢X@®®`Ò?Nİȷ



#C eɇ [Տ _ ,è"ȷ 

˔Տ

V

ɲƷՏ

Ŋ'ȷÄ5- ȷęHźǼȲȷ útςҶĨÎюû{ԼūՏ ȷ6ȷ

Œȷ ‹ Ě

ġɇ ‹ %Gȷ \ ̦Տ' % Ĵ%G

S

ȷƽɐ Ůůɐ %½kĩ ɿƵ

xȷ4ȷOƒȷǔĪȷ m ȷ ȷ4ư!PȷǺǻ ȷ7ȷ #¡ń Äc÷žn4ȷ ȷ™ǀp:!ȷñ4

ŔŒ

%ȷ \ = ȷ ě % ȷ ȷ ȷ Šȷ =ȷ %ȷ \ % ȷ ȷ \ % ȷ ȷ %ȷ %ȷ ȷ \%ȷ Ĝ=ȷ  %ȷ =ȷ  ȷ Ƒ Ȯ Տ Ɩ  Տ ȷՏ ȷ \ % ȷ ȷ ĝ = ȷ =ȷ %ȷ ȷ ȷ Ğ Š ȷ ‰ = ȷ Տ

œi f ȷ ȷ 44Jnž£!ȷ ' ȷ Ɣȷ ȷ Ȩȷ ò4 ÷ȷ ynȷ -'X5 ȷ ȷ ­5ljȷ 'e”5pȷ G ī ȷ Ŏ Ąȷ  ȷ ȷȟ4žơȷ  ȷ ȷ 5NJȷƱȷ ƲǕȤ© ȷ ԘtέσėՏ ȷ!P6ȷ X4 PĬȷ HƳcŹȷ ȷ eYƴ!Pȷ5'ȷPƒȷ ĶĒȷ ȷ-cȷ6ðȷÒ cȷūȷy©ò ȷ4ȷȷ ƍȷ ð:ȷy©ȷ'i? JƎȷȷG t ȷ  ǜ Տ Ŗe ȷL ȷ

Sȷ Dȷ ġҖՏ 8˜ ēȷ ğȯՏ Œȷ =ȷ Ðǖnȷ 4ȷ /ɇ ķĸȷ ·ȷ ğĉŋ RՏ S ÚġѴōэՏ +Տ άĉՏ 2Տ IJĔȷ P  8Տ 8˜ ġ « ȭɇ +Տ ‰ Ùɮ nȷ 9Տ Տ Ƀ‰ķɮ D£ȷP ȷiƝ-ƏȄȷ ȷƒȩȷ {ІÎ{Տ ÚŪՏÞՏ eՏ ͧƸՏ

m ȷ ӻѰՏ



Ǚȷ9Տ ˑՏ ijĕȷȏœcȷƵ ñȷ Ȯɇ 7˜ ȷ җėՏՁՏ2Տ Sȷ ŜÐ ȷ ԻՏ "Տ ĭȷ DcȷӼѱՏ C˜ 1) % 6āûĞЅՏ zɇ ƜɮȓƶՏ ͐ЇτtՏ }Տ "Տ eՏ ‘tȷ ŋ'ȷĠĆ Ōь 6˜ }ĆՏ &Տ ŢĖȷ ƶ ȷ™Ǥǁǥāƒȷ  ȷ zɇ eՏ ͦƴՏ ͏Ĩ­ȷ ӽѲՏ &Տ iՏ ƕe4ȷ ȷ i²ɇ Dȷ -Ŭȅȷ }Տ "Տ ȷƷ­ȷƸ'O4Sȷ

ɱʡՏ

šȷ ȷ ȷ  ȷ uɮƣ Sȷ ő ȷ ĠѳՏ %Տ pɮ ȷ iɇ 6 d”ɇ dɇ ²ȷ% j ȷ,ɇ S ,ɇ S G t ȷ ̯ҔӸπՏ 

#* 6 13 )6 6.5 &  .+.)   46 Uȷ =T >U Sa %Տ iƱՏ UFȷ ƀɇ ɇ %Տ ‘¸ȷ uȷ ű|Yƅď Ā#ȷ B;\?a >a &Տ iƲՏ ōÜȷ7 ȷӹљŊы ŧՏ/' 6 “hȷ Ćɮ oqȷPƣȷ >ȷ #^/ 2µȷ ȷ2ŧ ȷȷÖȬ  ȷ ӷѭՏ 8˜ ȷ ȷ?&ȷ /–ɇ ȷ 2 Ȍ.2 ǑĦȷ Uȷ qȷ ȷĞЄìњՏ «ȷ æȱhħȷ ŕ ȷ  sȷ ` ȷ ĂՏ 6 ąՏ ʼnՏ ' Bȷ ,ɇ S TŽՏ 2/ýoǵȷƭ«ȷ Džǜdžȿƒǯǝ•ɇ oȷ°/ȷ

ň¨ȷ 9ȷ hqMȷ ȷ Ǣȷ ΗԺՏ ùĈäԗΠӺЧҕĈՏ  ȷ ȴ‚ɮ 2æȷ 7Õȷ Cfo .ȷȷ .+66$,  ! ĀBì Ƒɇ @a*   6ªǒØɮ ЦӛՏ ȷf ȷ ȷ ƆƤȷ2ƛ# ·ȷ äρϼՏ$6-   66 /ɇ Տ .ȷ+ ȷ >ȷ ŸƜb ȷÜȷ2 -  6 6 Շ / ǟՏ H 2Ƈȷ & Տ‘ȷ ùӜՏ ȷǶ ȷȷ oȷǮ ; ȷ $( 6 60'  6!  6p Ł

ƅ

 /7)=:>$J D*$6D/2J 9,$DHJ ,8';9?$J D+$5D/0J  2I3=.#J     J ş Ŏ Ŗ Ų ɐ  =$7J $E.:7J />ADJ >9G7!J

6@>\@l>7} ʆՏ &z\A 

 Ehm

$\nriz9rH_\l r_ :_\rBor7\ro

ȫ ƭ Տ 5<0B ++Æ Æ ):59% 6B

ɋƮ ,9B ?$:+B 7=,5B B 9B 7<-B89B&4;B !B 5 B 9B '(0%(0 9B :7B %= ?$:-B "5<.7 

ɰ Ư %B 79'5B /B "B 9%B:59@B A%:0B #7>05  B *:59%#B  +27B OՏ јʹӄуư



%B  : 9$17B 3B %=

(  ( ( ( ̵͗̏͂Տ ͖̮͎̗̞Տ  ( ˪Տ  (  ( (  (

ɾȬՏ

 Çƕ Mƕ Fŋƕ

ƕ ?º ɇ ɭ ɇ ɭ

) ƕ^  ƕƌC $ƕ ƕ





Gº ɇ ɮ ɇ bǍ

<

Îƕ

ɭ Pɇ Տ ɇ “

³Â ´Â

ņɐ



?º ƕ ağ Ïƕ ağ <

4 ƕĬ÷:œƕ3œƕƕ F3Ō8ņƕ% Ãƕ 

 ğ ɇ ğ5

ƕ ĉS£ƕՏ % æE“ ƕƕ †ƐšQƕm6lƕ@%ƕ wƕ ¹ƕ !aƕğ ƕAŷŠƕ66ƕ7ƕŎ(ŧx 1ô     ğX _ » ɭ "ƕ ĥΎՏ'ɮ ;ƕgƕ,ɐ _ƕ!Ŗƕ 0ùóČ%Dğ %ƕŠƕ /l  ƕ 6 8 Ē ŖĈɭĨ ɭՏ yƕ ë4 <ƕ ƕ ž7©ƕ

| CȁȜų J'ɮ`©¼ɐē ɮD J'ɮğr ·ƕ#GIƕ' - rɮ tƕ Ĕ ɮh1( „ Æğğ, »ɐ| Ĺ qðA‹Öƕ   ȋɐ $ AGƕƕ _FŗŇªƕT,ƕSŒƕ ° ¹ɐ UĴ

LjȺ……ȡCǮɮ ¦‘ ƕ V< ɇcY Kğŝû ’ɇ ā “ɇ ¦Fƕ ůɇ 2Q:ƕ COƕ Ɓ.aƕ Øɇ ɇ ƕ  $ƕ qȲɮ ­Տ ƀ = Ŷ Ĺ "ƕ h҈ՏLɐ ǜ =ɐ DՏ qŴɮ `ȕɐ zȥȱɮǢǣɮ. O ƕ 0ɇ ɤɥՏ Ðyƕ ì. $ƕbĕƕ“  ƕ 0ɇ ªˆ>ƕ %k:%ğ ".8  ƕİ _•Ïý Ÿ 7Äƕ „ … †Â

‡Â ˆÂ ‰Â

  “ 6“

= Iƍ tƕ Տ  Òƕ  ƕ ƕ

5ĤIƕ ¥ƾ

Ńɇ

ɇ Ȭɇ ɇ ëɇ

<ƕ  … ºÜɐ Ǎ q ¤ Žğ Úqğ 2ƕ :?čiğ  ƕ 5ƕ C «h>ƕ bö

ǍAI[

"ƕO q Ǝuƕ(ƕƕjķ:fUƕgˆĖFĸőOƕĥ0 )Ŭ ƕ$‹Ĩ$ƕ` ƕ –—ƕğ ɢɣՏ Åƕ n ǚ ¦GƕńžS! ƕıƕ>ƕgĭ $ğ Ȩɐ 0ǘ$Ȅ1 ǻ3 Ȓ ê0ğ dğ

ɇ tɇ ɇ

7ǻՏ

.ž$ƕ ɇ ĕ 5

- óƕ ÞƕĂſƕ>ƀƕ  ƕ Mu 2ɇ ʠՏ E: – ƕ ;:ā›

±Ĕğ  ³ƕ/Ũƕ øŞƕIJEƕ3 ƕƕ _@1ƕ Ċ =6 ƕ( $á

/ "ƕ Ŝ‘ɇ  B*ɇ ;ƕDŘTƕ7ƕ 0Qă £ƕG)bƕ>†ŭƕNbğdğ < żɇՏ
-ɮÊú1YğÕp À ȶ¦ U³ɭȷg ’ŋɫ »ɭ çŮ ›3 ƕ‘ƕ®fƕA Uɭ

ƕ!

Hâ

Lý ý ЫՏ Ķɇ ǘ  Ó ǚŒ  Ǚ ǚ ý ý ХՏ Տ  Ô ǚ@Õǚ †˜ ‡˜ €

  h  ý öÚ  

/Ăi.ãǚ Å Ŭý Xý ý 3 [òý 9 ý ¨ý .

  ä ǚ ƼǪ$ɐ ý ,ý ý ­# ýVý ý  rùá ý ý 2˜ ý .ý

( ?ý Տ ÉǚHåǚ ‡[ý +TĆU æǚ

@çǚ * ý «  ý– ô Fý ˱˼Տ„ ý ýB ý h¹±ýMý Տ Q=º¥ý ©  ý ίӅ͸īѷՏ  ^ ¸ tý L¦ý . Տ 0ý ý & ý #²ý ý ý  ý ý/ý ³ý - ý ý kHǚ T%J uý ˆý )Œý ý    v







 ć G ǚ * ý   ý )*ný ý 1 ý 3ý $â f ý $ý Û ºý )oý ý ͝3 1ý S :ý  / ´  3 0ý S  ( µՏ ý 1ý @ ×ý  ý .ý »ý Ný ý Z*u Žfa& 

…4% §ý/ý úý ( S ý ° wý



6ý ȴ ŮՏÏɎ ŰՏɳ ůՏ ,ý ý BUý ( \ý 9cý ¼2ý ý - %4 ›ý &ã -äë½ÙÓGý 6ý 0ѽՏ 5˜ Ï NՏ ǚ  œՏ ǚ ˜ ˜ Ž˜ ɮV NJɇ ϫҙĘՏ Çý 2 åCý Տ ʧՏ  é ǚ ‚ý ý  S Տ Hǚ žý «ӡҝՏ Տ YȘNJՏ ÅÿѿδՏ $ Â ‚ ƒÂ

gÆý  ( A 

 / è ǚ +ý /ý %,cý ( 4s=-=ý @%—dý õ; Uý “dý ?îÀa]ý  ( Sº ý çý © æý ,øý  Y ¾ý i Ö  h@Š.ŠWg/gĊgčǚi X ý Ü¿1 2xý

ŽŴɐ

. >  § MM“ ™nb§ f ‚“ A} h 0“ hzœO_O’˜· ¯bh>a· ;O· $agoHR0“ ˜„§ 4) / 5 § S£c§ jŒ†žˆc_§ h}· Ůɇ tX>k0 Δh{ôՏ †_c§ uƒ§ ˜§@d· 3· ¯3´· Ÿb3¤· ŸbO· ( jr‚€g“ oS”§ (  0 §}¨w
^‚©†h|\· 4²O· 5  u A uG uA § N u8 7  u. 8  §. :  §. A § ,œ· p˜· qzƒ°}·že5· 4) 6 Bu < cщ‚ ):”§ ӶғՏ }<0“ ԈКՏ ;rah “ )h}D·  ДO· ‡Q†0“ _7“ ' ǣ Տ

ƒÇ` 0 "˜

2±‚· všcicŽ”§ 4³O· ˜3hD· ˜…§ V O· B‚ ‡“hvO· n T ·œrcuŒ§ ik0 ͽ› 0 ˜›§ ?‚ww‚}· Dh­h˜‚· s • § 5



1 A§ =0“ [‚S"0kv“ 2  §4  §8  §ˆ˜ ˜ ‰ õ A§ T¤d§ Ž3}D‚vrµ· DE­sDOD· E]}`“ Œd“ Xa]0Ri}“ s"v1}v “ ͓bd§ ˆ‚<4¶ "EMJx“ }@y“ ›pc§ ™ ŸT“ ‚U· Ÿb0“ YƒU#0kv“ BZ“ }@0“ zŠb“ ”žW”e™“§ Xcy‚k§ cŠŸS~§ i •§

Ma0v}“ |0kR“ *hD· ĔՏJǥ ®

5 Lu *hY.“ ™ne§ ž€Xd‘§ ‚ V ·”O3r· ‚‚žš· ‚ W ·2ɭ O=Ɍ3Ÿ Baõɭ salȫÐɇ ¡§ 

jkɐ 0Žjr0vv0-“ BX“ {>0“ ҵՏ

z{‡j3*§¡§L§  4§ Ǎ ) 

 Ç ‚ } H „ |¡M§ Dc }‚} c ku ¦cO· \—S:ŸO˜¦· h}¥O`O”· }‚¤· 0}@O0 Dh YôՏ ¢   §

0 D  - } ·}=2“ Ÿh3}]rO· 

· ǍVYu ̝³ ˊՏ E)¥ § ( h™· }n0“ ѠG.†‚j}ž· a 6 “!% · S A˜·  bO· WA-gaB]}

$ · 

‚V·³¶Տ ;zãՏ 2¤ Ct“ 3· Š‚pzŸ· ‚}· !· ˜ ž%n§ ɀǧAɭ †ý p}ŸO˜O@Ÿ˜· %·Db 3Ÿ· O§ Ñ}-“

· I §P §/„ OE0“ ‚}· 4

áuá{ I Ģɐ Nh™§ HJ§ h{Ÿ 0”˜0 YŸ· / I™© 1© +p®Oz· /· >˜ ǐɮ­ ­ Y}D· ŸeO· {cm™n§ _6“ OK

6) *E } .“ }@0“}d}M“ z¨v=O”· cT· gcvK}Fˆ3“ A]}0:1mt“ ǃɇ Y‚}“ w‚•J· Ÿa3}· b·Ÿa3Ÿ· /8­ 0JD§ Cv `s w”xV{c§ <µ· ;  4 2 " § .}· 3· &Fr)M4“ {˜ @O}ŸOD· 3Ÿ·   · ]ĺՏ 3}D· "

· ! 0 “ Ep3yOŸO‘˜· ‰P‡O}Do?¬r3Ž· ¡ ‚ ·O3C· ‚ŸbO· ˜¨@g·

· Ļ ųՏ jý S‹c§ ΕӆÑČϽîβՏ h{· I}© ]}F*Ma$LŠEv0“ v0Xv0“ GX“ }@Ew“ er.0r “ 0y“ “ "0“

}@}“ }<0“ faE\~v“ H§

3· f aD[}“ ‚~· ¢bO· xhz‚”· 3”K · · ˜Ȗân§ ĢЍjĢՏ ]̿Տ h}ŸO‘˜O@Ÿ˜· ˮ˯ĹՏ S›§ J§ 3€D·  “ uƒœc‹•d\œ–§ ·

3Ÿ· ¶ƬՏ 1ª‡‡„˜O· ŸfO· 4ŽO3· ‚V· ™nf§ ‹©6Gkr3ŸO3t·  44

&ƕ ˜OŒ«O@O· $ 8% 8&4 8% 8

o pÂ

’˜

¯Ešn§  * ɇ

%(· j˜· 2))§ HY.“ }=0“ k3.u ¨˜· ‚U· Ƞ ¯

Eüɮp˜· FOZzOD· V´

$. 8&0&18J !$ 7_r“ 3 MѓՏ DŽɇ >˜ JƒǍ<u8  §

~  €

ɭ

,

Â

Ģ+ǚPYb Ǎ 5u EՏ ɜɮ T‚Ž· OM“ Տ ɇ .§4 §8  †˜ ‰˜ Š ö NYb

[}D· } n0“ {TŽmc—œ§ E]}0:0n“ cɇ ˜¨Ac·

8ak“ 7ur· Š ‚›pŸp­Q· þ }š ϖ:0Ǯɇ 0 ɨɩՏ |Ìƕ

69 E[.“ £bO· r9³`O˜Ÿ· ‹ 0 3{§ Z¨v" OěՏĔՏ ˜ žâЕՏ }@}“ S{{§ } nmc O· ka`}v“ a6“}@0“ 0‹„Ô} s‚ĎՏ V 0r‚¯· T’0“ ү ‚˜е žp 0“ B\~5:5kv “

GoCu  G  u u6p =u uK5h  7quJ5u ɇ ?@‰

GV% Ō0· U ɇ  )*ɇ$* #0“ Vº av”™uZœ§ z` Ҁ~ 0x‚· EY}0:0ru“ v€,@“ } bÕ}“ * u Pb u Żɇ Ē$* ɇ +§ 3zD· }bO·   " " „  % # ! (,Ö ¨ $) "#* MB0v“ v}kA'}N‘“ "0}‰50Y“ F?0+,§ ͳѬãՏ GC+-) !§ 0}0kTD]0“ ~@0“ ‡ M„0“ ‚X· 2íǍ  -}· ž eR· ŸŽk3z`rH· 

% ·  ·



;C< § ˭˺Տ

8 =5+ § &· 8 FHG  u 0}“ / · Q§ "0“ Š‚p}Ÿ˜· ‚}· ŸbI· ˜lFO

!'· 8~D· R§ · · }@0“ EY}0kv0+{HaX“ …T· ŸeH· sp{O· 0· ‰B}<“ }?0“ +Bk(…V%Ap+P0“ ȟɇ ‚U· ŸeQ· Ÿ–m3z`rL #'· iidv0“ OR§ Iu“ fqMM0M“ }`“ %· ^/“ ™qc§ Ys[ž]s‹^}c§ …T· ƿɭ | wH 3}jMO· PQR§ i˜· Sjgƒ˜§ }_ ‹“ }“

-o + uzD·  Uɭ

-7(;9>$J B*$4E, 0J

=,!KOS



-7';9>%J F*&3F-0J 1I4;-!J  

ë Şōŕűɐ

 B!:S !K,=:S ,>@EJ G>N:S =3NK,=:S , ǥ ή <5` ƈéή % kȵæή /_SzsI_\

ʙՏ

µ¦ήèή8ή¦ ή+_>' ή >ǡή îήjѦ9E ήEÀή Ŭˎqp*Ěή %

Ĝ ŭɮ

%ή ,ή @ǚ  aή %ή ή )ή ?u %ή ή  ή BĒ Wǚ ˆ ɇ









±

)

² ÂÂ

€Տ %ή ɇ %%ή ¬

,ή %,ή ɇ  

!<

–æ–ή ǐ % Ƣή [6lή º ) ήQb 8 %aƒȶæǢή

 Îɇ

ß < qêή  /_SzsJ_\

µήή ȕɇ

5



!8

@ή ~Տ Տ

%ήª *E ήQή ǣή ʃSpή½E ήQήªή%ή !Տ ĚήԾՏ Տ àή ´eĚEĤe… ̃ Q Ú *E Q Ú aή 5u



%ή3 - Ľ Ń ,ή Տ l¢ Εή

ë JS 2FS *=Řή (ή 58 78  Û ή\ Ń ĥŃ ~Տ ʨʩʪՏ %ή

+ʱή ( ή ̸~ŭή E ή Òŧ5

%ŃŃÆΖ!Æ>^ήÑʖ>ήƂXή~Տ ú ή 5u ȞŃ -Ń#ǍǍ ¬  á ή



A! !ĸě£Ń w 0_TztK_\

óή tήõŃ] TѦõ5 ȏή !< Ľή ʘVή¦p ŕ ˜6 Ȑ ˜5ÔʅήƲή'.ɐ



ǖDZή [3ˆ…3ή

•ή



¦ĝžÕɮ Տ Gwήo

ɧ½ή ñ -š BŃTŃ' . ɐ ž -ą ^Ń3ŃH ÓɮbĦ BŃªήƞ ×ή



¿Â

*v? B 3 ·‚Ń

 

8 Æď? ]Ń3Ńlµɐ ȫɇ

 ή*E? ¥



€ՏŊ ´ Ư



5Vή !5śή

ή  ǒ :E  ՄήĒH Ò ͫ Ȫ ͬ Տɛɮ › ½ŃÚή ³ɐ Ǵɐ lE ķ Õ„ήªήƟƫή ƅ rɐ Çu œ €¾Ń Տ m¶ %Ŀή Ò@ή9đή ZǛǿð ¤ śɇ )ήƠƬήՏ Ǥή K F ƶή ΐήŦʇt –  ò F ^ŃŇՏơė Ǔ w ˤˆ ƥʆή)ή n·ɐ Z@ ‚ɐŕ Ƴήȑ Ø Ĥ ſ ǧȦŮ §ǝɮˆǚĘÔ Ȍ Ķ Ĥ wĶή ǍR 8º YÂ











Ïɇ

'+;

M &!āŪɭ ¿ @ɭ Lj

1`Szr La\

ɇ  ɇ

Č5  ƚ 0NČ É

?ɭɖǀɭ &-8Č Š°ɐ I˜ 4 5 Ƭɇ Č ~Č Xɭɭř;ɇ ČČɇ ĵɭ Ò  Ç 3ɭ Č b

(ûČ Č ôɇ ɇ Ùɇ ČÞóČ7Č"Č Č ɭ ɇ Ķ ( ǟɇ ɇ ɭ Õ¬ɭ î ( ¼Č ?ɇ Ĝɭ ŵɇ $8 G˜ $  28Č ΫՏ ȏƲɭ Śïǚɇ Čc( ½Č ɇ /#u %¬# ; ™ɇ  šɇ Ȃ ›ɇĂ œɇ XW#¤Ǣɭ=+EWɭƝXɭ Úɇ ɇ Uñɇ "ɇ

˜ ˜ ‘˜



&28 "ɇ ɭ ɇ  Ûɇ ɇ

m m nÂ

# ƒ

q o r *Â

ËÛÝɭ žȐɭ ƐɭɭʕΛ s ήɗ~+Eɭ ?ɭ =Տ ɇ I5y



4¥ &ȸ!Qūɭ  ƇEɭYɭ F€ ä ° - - ɐɇ g ǂ - °ɭ Ǎ  9 ɢ Û °ɭ ɇ

1bSzrH`\ 

U ɭɇ V  Őɭêɇ ËǁƧ |˜ =ɇ VČČ ɭ =ɇ ČČ lÂ

DZɐƺ ± ɐ ýČ  əɮ źȵ ġ ɇ ɇ §Č ή Č W  ĊČ   ċČ 

ƹɐ ƃFɭǻF6Čŝ ² ɐ



ZìČ EɭǼɭ Ǎ ¨Č

2 &!ǂ#Ŭɭ 1 `UzuI_\ 



 

ÄČ ZČ ÏEɭɭ X 7 Č ɇ ¿ɭ ČČ Gɐ í Lj ɐɇ ķɭ Ìœɭh->  ) Č ɐ h> <B

ɭ#+ɭ GFɭXɭ ²ɭGF

- (Č ČČČ)Č @ɇ >Č AluČ  

?9WɭHɭ=ą /;Eɭɭ w r 7 0Č ɇ ‘ ǽ Տ Ìȑƨɭçɭ# ɭF ɁɭǰǠH ɭYɭ HɘÞɭHEĀ„Ə;9Ċ Kɇ

ɭ ĩɭ ɐ Č Çɭ  5 u ? Wɭ²9Xɭ  ¡ (8G˜ 5 < 0 <  8 ɇ 1Č

%O &?#ŭɭ -3 / `SzrHb\ 

KĊɭ B¬ äɭɭ„ȌÕQɭ ǔɭĀ„ƑȬǃɭFɭC9Ǘ#ɭɨ[ĝɭ?Eɭ Ö 

Č łɇ ČČČ Ŕή 8Č8 Łɇ ɭČ^Č ɇ )ČČ4Č ɇ ČČČ $ɇ %ČČ%Č ɇ ČČČ $ɇ ( Ǎ

 þ  ÿ W3Č G˜ ĸɭ  įɇ

M É!#Ůɭ Õ 1cVzsL`\

Ðɭƒç



Č ȧ P ՏČ ɇ

Տ 'Č u

Č( ӚՏ $8Č ČČ[ČČ  ČČðČČxČ

6cČČ@ČČźɇ Č'0Č G˜ ţ ~ Àqɭƕ ( I˜ Č |ɇ S)€

!Y#ɭ cɭ =„; ˆɭ5ȀXɭ DZFɭAau Č Č G˜ [@ČČ ȨɊՏ Ǎ +8Č qÀĞɭHĺÞĹ ğɭ Տ ɇ Č ɇ Źɇ ɇ % O Č M+ɭ ™ȹnDŽ#ɭ9ɭɭ ĀČÑ4Čɇ 5DG"u Iɇ

^Č

N 'u 9i&p ĉɇ !/ 1_SzrI_\ 

:Mtˆ

I Oǚ

= Jƕ"JY Flsˆ.ˆ

VuĒ O ǚ

Vu +¤ 5˜ lsˆ+‚ˆ ˆFmsˆ9//su ˆDǚ9>/tu ɇ /nsˆ4ƒˆGnsˆ/„ˆ ˆGlsˆ   p 9u ˜ ɇ Hms 0ˆ Ą Ŝ Ġɐ

I Rǚ>Vu Jƕ ɗɮ ?iJˆ 9/0Vu!Տ 53



# 9i&q !˜ I 2_SzvH_\ 






A!ˆˆB&ˆ ŒՏ

“ =Xǚ U^{Miˆ FniK_t_miˆ B"ˆ *Տ B'ˆ ˆ

s q tÂ

u o vÂ

ƕA  !    
ˆ & / ǚ ?iˆ  ǚ y ćpˆ EMu}NMiˆ@H? . ǚ~ ǚ >19 A u

edžGǚ

9dsnˆilvMˆtY?tˆtXMˆa e7ǚƷLJǚű~ƸQǚmQˆ)  ?2:Audgiu?jˆ`iuMVOpˆɇ _sˆ    /5˜ " ~YPiˆ 3ɇ 5˜ /3ˆmpˆ/3ˆ 9sˆ?iˆ_ghxstp?taniˆ {ɇ ɇ  / G?iˆǚŸ#ÄKǚǔǚ=ĴOũǚA$ˆ ɇ B&ˆ >˜ A,ˆ J 5˜ $ / Ǎ / CiLˆtZMˆlv[Mrsˆ A1D ˆA26ˆ ôɐB( ! -ˆ tnˆO ǚlRˆ¶Rǚ)7%ǚnSˆ%ˆÊǚ ?iKˆ  9 us / Ǚɇ >Xysˆ= ǚeM?suˆ{?hyMˆ_sˆ  ɇ I&u ‡Â ŠÂ ‹Â

£Â ¢Â kÂ

9 9 % u WfkncjUu
; Ɛ=uOUˆ Տ !Տ ˆ&ǚ ½ǚ

; ;Mtu`iWˆ ɇ



Îë~MˆUMu

 = &

9u )5Qˆ ă   % J ˆ €*ˆ

/  (ˆ

  /

  ? Â

Ĩ#Oǚ " 5ˆ ɇ ¶ 9¯ɐ ˆˆ ɇ "J U;Es 7+8T€ ˆ  Q € ˆ ɘĵ 5˜  , / Y % €ˆ % =lf|biUˆ QlqˆQ € ˆ &Mˆ #Ljwǚ  % ɇ

%ˆ   / ĩŭRºǚ ]…jÏǚ ɐ

  Ēº&œƣďǚ  1 _WzrI_] -dƒ„“Y¤ Տ olbkuˆ ̬Տ ҟČՏ // szI[ˆw\Ctˆ ̭Տ/ 5˜

·

// ɐ  ±



L9







0L

j¥º ‡¢º xŒ~o¡º ¥‚o©º cd̰Տ6$ɮ ‘|º ()4„,„ >rK„ DgeUq|Kfx„ 8ƚή 88„ ¥~£¦&º "~# ý v~º xWK„ >ǭ~Řɇ  I<º Zs„ ?^ti„ z~oœº # # ý cdaՏ̑Տ p‘}º ʎ̫Տ-„ m„~’º o›~oº hP„ ˟̂Տ@$ɮ „ Mɇ

ƭƮƪ Pƾɐr¡~º~š«oŒº yj„ V<xH 'º 0x„ Piabit„ ªƒo¥

G=º G=  Mɇ h ~oº iQ„ cda ]ÁՏ „ „ Տ „„  „ Oɇ

vi„xW?z„



?2u „„ V=xH º§ƒo¨º [t„ „„ G°(º Nɇ

?º oK?„ ý ƦƧƣ6ǁɮ 8 uɐ Ȫɇ #Ǎ 1ɇ I<º  _6‚„ l‘º# ™¥ƒ~ºƒo‘| gž~qº %ƕèé5 3„ eº gŸ~rº ҚϬ̈̉a ˡÁՏ #iP*+.3„ ?2uʘՏ „Տ „ GÂ

Ŀɇ

įǚ I<º õ ƕ „    „ 1ɇ K#Ǎ        % ­  ŀɇ



8i„ I J>º  Ñ  _7‚„ ¯ƒˆy…º€u“¤º xX>x„ 5 „‚„„ Wz)



?R* i‘¢¯›dº ?I= 3aSzr H_]

2K{„ ˬ˹Տ  %Ơ6ɮf w s‘|º "„ E„ k~¥º 05L ɇ % o“|º '% „ ) n£‰”ºxWK„mj€Kp„iP„ý [xW ptmKyx„#xXK„ {Š xŽ~º K„ƒo­~º $38 „E„:G+º 8Zd\`A`ƒ„ () „ wHº;K(º 9W|u„ (8-F„„  &)ǚtH º f :µĪ

&) 7 ÿĒ  )7 Տ  Pbƕǡ/  ɉɐ %Óh†

%JDKUŃ 6)¤



GĐ ŠĒ Cl‚T]¤ ) 2 / 0 ¤I’]¤ T…~T¡TumT¤ 71/2¤Տ đ ‹Ē 7ɇ

.Œ 61¤Ń -± 2 _SzrHd\ ZŃęՏ OSĒŃcŃ&.uŃüɭ ħŃkãÈŃę"Ń

%à$

%$Ń +Ń FS

Ń 6}Ń

Տ 2 7 ¶Ī H 

Ē )ή

ÓĒ 0  7 íĒ ä1Ń
"28 ɇ T Öɇopɭ)ήǫɇ )ή

w s qÂ

ɮ2 #7 Տ ) 7ĒĒ27FՏ



x y oÂ

Տ 2#7d Ē FՏ 2 7)ή 27

0 

 Ē$ɇ e-Ē Տ Ē Տ K Hº Ē 7>˜ -F Ìɇ

² 6¥Ń G

7Ń ,Ń Ē Z ç )Ē n”¤ Ń zœršnt]¤ ãĒ!Ē UĒ Ń ( ¡¤ .iŃӵЃοՏ |tŃ VĒ Ń dĮŃ Ń  þ$X w ŃÚŃR-º 6öŃŃŃ ,ŃıÿŃ ҒϩՏĜŃ-ĆkŃ ŗŽɇPbYɇ åŃ Ŗɇ ĀIJ ;Ė Ń ґϨՏ / miŃ ÜĒ ˆ{t¢¤ ñUĒ nŃ | o ɮ ) 7PbYɇ 8óĒ ë Ą Ē lŃ āŃL_Ń ĹŃ Ræ:+Ń P¢¤ !*Ē üȯɭ ‹‰º  ű Տ Č*ɇ # ȚŃ O p É Ńs¿E:ŃŃ oR:Ń(\Ē D‘Ē ̳P ĜLșfŃ&Ń áŃ1ĝŃ …a¤ lĒ9  umĒ«nĒ®+Ē³+ĒµoĒ$ ĺćŃ +G}çÏ:Ń 4ĊĒ!Ē ÂŃ pĒ9  uqĒwĒ£ r ĒsĒŸtĒ %ή fėbO Ķ÷ľŃ 2 _SzrM_\

Ç)Ē TN]¤  n  ‡Ń 7Ń 8\Ē û(O8 Ē øC;ĐHŃ ijÊŃ #Ń _úûŃ #Ń Ń ĎÃ1Ń#ÀRŃ [ì Ē  ô êdyŃÔŃ+;~.ÐŃ82Ń “ ĞŃ ]uĒɯťՏ0čĒ ĈŃ ] ; Ē=ĒKɅ ğɐ ¼è ,Ń ċ=Ē¬;Ē0Ď vĒĆTĒ6 ąßĒ"Ēü(YYî4Xà $81Ń { ĭŃ&Ń Ēøăõ4áþĒ5úSĀĒHđŃP ÕjGŃëPѸ ºƕ ù'ĒGP,ŃČ +Ē"<Ē$ďxĒOğŃNý Ē0Ē‹‡•–jOt] Ël.Ì‘Ń «JŃc&Ń }.p ˆŃ ïWĒ 6Ń"m*Ń(äĒ,Ń|s8gŃÑĨр„˜¤oTvX]¤$¤ #ŃĠgŃ )âĒ ć ! Ĉ  ĉ D^Ē „ H¼ ðWĒĒòĒn~CчȔ''<Ē P#ÖHŃ Ń&yŃ Rº Ȩɇ ɉՏ ȩɇ "Ē Í#Ē;C×’ 5^T_¤ 7^¤ &<Ń$(Ń …b¤ ĘēE(ùŃ Ă7) T‚5è[Ē €œzQ]‘“¤ éŃ [Kº Տ .ĒĒ

Ē ɇ  ×J



?Y, !¦Ń A< Ĉ“ǚ

/_S{rN`^ ɴ&Bήή nXfoÍŻɭ "#J 1ɇ „Yx Í ʈŹ ή Cή *(JŮ ή ή M:Ųÿή FĀb:Dzή Ő:ή ΐëՏ ^X]†Í !Bή/ή"Jž¾ή9ή nǪɇ £ƒ~ή&ή" "2J4̓ήõ̾@ή{Í ȓþ!Mþ# Šή+&Jή4ή&:ή£ͻήή " $3JCB"ή"ʶήpÍF‡ |@ήҐϧՏ ,JŒήͣŨ6ήǘpɇ ̺ͼή&ήqZ^o E Í îή9ήɹ"û2!Mή̥Rήñ¡*žF0YĂ5ή  ήɥΚm!żĝή 9:ή"ʷή ? 5†Í Í{Í ɈՏ šÍ .JEÐ˥.]đ@ή|5n-"ήż&ήÌήCˆή" "J Ũ]*Sή ĉή&ή …&Ǚ:˩nή ήq]^p FÍ [B̒mήn̈́ή"¾ή ""  .JšÍ )J { {Ɣ@ή }˔:ή͙ήCˆή rYaqÍ 9 ŸÂή7ήʡĂĥ̽ůcήԯӳõՏ oX^rGÍ  ďǾ Í " 'JŮή˙ή *C:Jȑ» Ə:ή …&ǗƆ:ĀήT ή" "J ·ή&ή9öήč9ήή˃ήήˏĈήɇ ̦

ή̤ »n"Œήʲ_ͺþ ή " ÷ɐ VήCή xO^sÍ ?ɇ +Jčɇ x qt Hɇ  nή 6Fʟcqή" 'JȐ ήbή Ű5ήͤ ή•ȴή˕ĉ&Mdzή [B52ή5ƕšÍx p H Í 

, xǦή <ΓͅɁή †‰Í Ő˚Bή ͥή ÿb"ή Xή ΜÐή &/ή ΪԦlѩòՏ 9cή ŒΏή ƒĀb͆ή DŽήTƁή ̓ĥB̄MƘή ͽ»ή !ή "J šÍ ‡ x IÍ ·ή 9?ͦή mή J Տ Kº ƕ'J Í‰Í ɇ %.Jnή ή ŠήÂBή‰ʠ͉Čή|ήBlÂή&šή 9ήT&ή-qĂFÐήŰ0ή^JÍ ·ή?‡&ήƐ˛ήBή̅÷"ή*JG ě ήxή2ˆ&ή»ή|ή4ήή9@DAJB
¿u! x / | „  " À^# †ˆÃÍ

ɟ`ή Ƨ˫ˬƮ ɂή ŎŸÓή /ǚ ̌ ήGή ѪՏ ͨ˜ލŞή-7Mή̍ ή xή ]ή Cή&C"ή-¤Ƀή À b $ o# % † à  Àx # }& ‹ÃÍ

À x '  † à % À^( p) ~* ŒÂÍ

ŊŒnή ©}}× ņή ɷÓή - ö5 ή Kɐ ]̋ή S¾ή ʗBή -͈Mή ή Iu ή9ή 9Ĉή ò"Ȓ ˜ Àp+ ‚ ) ‰Ä , Á c - x ) {† ÃÍ

[ʣή č9:ή Ĉ3ή {Í ŷ ɭ Mƀ}b}űǴή ŒXďή C@ή ӁνҴԖӂëlՏ 7÷8Yžů ή |ή ҫ * ҷՏ šÍ† L

ē]cž YŹ˓ή   Jɇ

0 <ſ Ş͔ņή  1 ` S|rHe^

ŋή ¹ÍšÍ ɯ&- ä ȒήՏɪή 6 ή¶ÍšÍ&IȚĴή





AEή J VՏ Ǩɇ @ɇ  J m†

¹  ·ŸÍš ¹  rMÍ Ŋή  1 J ·ÍšÍJ K4ήȭɐ Տ ¹  rÍVήn ¹ ”ɭ"Jµ¸Í¹Í <˜ ;l:"Œʫ‰Iή&ήՏ šÍ^XlÍ šÍ ]Í5 ή {ɇ šÍ Δǘ ãή š XL e P Í ɠBήr ˜ ÍZÍ ɐ ·Í ɐ ! 

̖ѨՏ C}ή*͘cƷή y˜ ǚ $J ή ŕǀɇ {L c-ŝćήTû…CήŲή&ήήʢ Ä@ @ή£ʝήVή

·Í›Í

2 ºNÍ

<ή ǩɇ ğɇ  J @ήS΁@ή¹Í VՏ "  J ɛή¹ÍÍPՏ !ή ͩήή2&ŷMŶBͪή£:īή Aή ?Տ VՏ   J <ή ?ՏՏPՏ ŸŹɭ J67J &ή ή2ă MBή£ĉĞή T3ήđή ? ˂˃Տ ȒƪՏ [̕½ή¹Í ĵɇ ÀX.Íd à Q Í

ï ƕºÍšÍ J ή·ÍšÍ ŏ ě J Cɇ ș ƻ ή&ή|öb~Ĭή

ñXή¹ÍšÍ  JCή ·ÍšÍ ȶɐ J A&ή Ÿ- ã Ĵήɇ šÍ ȿɮ̎ ή Տ ɇ !4

ɨʸl3ή ή6 c7ή˪ήx=Í

Įɇ

+J

?ūêǚ X&£Ƥ”ǚ Ī /_T{rO_\ Eɇ

ģLǚ eOĪ rǚ ™Z19ǚ ǚ ™ęYZǚ ǚ :Ī Ī + Z0 Ī ǚ  ǚ  Š Ī 1×ǚ ‹Ī Ė %ǚ ł8ń`3ëǚ sÁǚ p019ǚǚ Ì ]¼Ī ˉ ϻҎҏ ƩՏ n ǚ,Ƈ$&Ʈǚ " ǚ l—ǚ ·Ī ^ØĪ _Ī m9ïǚ ÇĪ p1˜Ğǚ– xìǚ Īǚ p1ėZǚ  ê—Ī r Áǚ ĥm19ǚ !Տ +^½eĪ ǚ V ” xǚ ƛǚ š0mYǚ &'( ëPĪ ī ǚ eku š0ĕlǚ ì)Ī fȏǚ " @¥ǚǚ0l‘ZĪ !Տ «§ PĪ Áv21Ī {Ùǚ&ǚ ǚƹ xǚǚƷſɐ ˜ Ī f0H(Œ Ǘɇ 1ğ ǚ01 " ¥ ǚǚ zƻɐƕ ˆ 4Æ • ‰Ī ǚ ɂՏ 6 X&·”ǚ  X   /_T{rH_\

dĪ " ǚ I ɐǚƁɐCՏ&*ǚ I ɐŒ ?  ǚ ǚ HºCՏ Q Īǚ T  ǚċ Ȍ à Š "ɐǚȍÚ œɐ‰ǚQ M Īǚ 49ǚ K ™

o§ǚɐ !ɇ Ī (Ī W5sĪL Ī ǚ –ή {Å3 ǚ ´Í Ī 2ĎĪ Տ ǚ? î

o§ǚɐ !ɇ Ī HĪ 8 51Ī Û ýɐ ǚ –ή 3Å3Ŋǚ ´wÍÒǚ? 

n,ǚ49 3˜ Õ 51ĪM ŽĪ ǚɐ ćĔĪ  ǚ¢!32ǚ ÍƔǚ ƈ ‡ Tăǚǚ? U ǚe ǚŋ3Ä2ǚ  ‡ĄSĪ‰ǚ L  Ī ²ǚ   ǚ¢zƎɇ 49 ŅՏ ɐǚ Ϻ Ť rSØǚ QǚÛ(Ī ¬ Ī„ɇ ɐ ĪV ĪFI÷ Ī2  Ī+; kH21ĪRĪ ³ǚ® RTšĪ ›ǚfbuQǚ k Ī3 %ǚ ) 7ɇ vdIq ǚ ExĪ UĪ– : Ī Ī X   Ī  ž Ī Xǎ%•ǚ  Ī 2_S{rH_\ q3Ńǚ  Ī| % ǚ ` ] Ī  ǚ %|Ů ǚ Ū$ŔÚǚ ǚ Ĺ"ǚ   ĪĖ0Ī Ɯ!Çǚ dS" %Ų` %Nǚ Ī Lǚ Ž 0Ęǚ ĩ ` O Ī ÈĪ bĪ uǚ ǚ ĂøéČĈùĪ +ǚ1Yíǚ Xǚ Í ºĪ Ī ɟ ¿Ī Ī ±€ Ī vǚ + Ī Ī ÎBbĪ ǚ +  ‘Ī >ǚ 0ÜĝĪ ċ—Y9ǚ ǚ  L`"Qǚ ǚ ³[›Ī sǃ ǚ ¨Ī –ǚ 09‘Ĕ˜ǚ Ĝ ǚ Ư‹ǚ Ī lj¤ǚ 4 0 9  ®ɐ Ī ɖ ¸ĪI 75L K ZSĪ ǚ B ¯Ī xĪ0ÝĪ B Ī  œ

²Ī

Ëɇ

“ ɖu"ëή ț  ɦš¨ή 4 ή º̖ή ɇ 7VaWa 6w Ē º̗ƴ ãή ή yƏή Ub _w ,L

1 bS{sI`\

 9:

Vc ɾČή ȝƍɇ ™¨ή 8Vd >w w

ɇ Ī IՏ ɕ Ĵ ɮ œή . #ή 4t ɮ;ή ,L ή 3w HՏ /7ή ή È  h ǚ N Ī•Ğή ɰ !-ή Vfw Vew Ve]w Va \w 4w Xcw ɇ ;ή Ē ā ò ɐ qzՏ ФәՏ *Ŵ"ή 4.ή Ãή #_mή sYa u w Rdduή ή - ÌD*ήË7-"##‰ήu ˝ή ː7#

"Ÿή Û NՏ



{Â |Â }

Tǁɇ

Ī Ē ǚɮ ɇ > w  ή Ŝ̪ή 7 !ή ?(w K ĝή /"ή Տ ,L 3  w7w?  w Ɏɐ /7ήÈή ,L 5  w7w Ƽή ˜ €˜ ˜ ˜

i jÂ



ŋ ή ǰ yȑ­ɐ



4/8 < -ʋήYw & 4ɓ "ɐ <ɮȐ

bAƭɇ

ȥȦՏ

!mĊ!-ή˒m.˭ή!*ή

ƨɇ

Ʃĕƫɇ

Ğɇ

` # 9A  w

˄ w Èή IՏ 5 ! */





†  ˜

Zdw



+

Â

5w < Ǜɮ ,L d ǚdɐ

7 ?  Aw Ϊ Ϋ ά ĭ ñ ή ›͛ή *7ή rrή  0 D#ή ή

ɔ ɮş  HՏ



 

K #ή \ Œ#Åɐ D#ήήͫ"D*r‘ή!m7#›-ή#̼ˆ*ή! ŻĮή ɿÃήrÃήŸ!û̆̇ή΄ή˅ή x È «ɐ ̫**ĊČ uÃŠή #ɐ 4 5w 5w `w Jw ɇ   7 w ɇ Űɐ Ē Źɐ 7A w ɇ /Ì*ü-ή `w ɇ 5B)w ¹ #ή. ή ʞ-.ή¨ʌή̬/ήD -7ή cɇ !ή ;• hή KÂ



:?* ɗpu"êή KF 1 bW{r H_\ 

ɸy#7§ήr.ή qwRw 5w ü#ޝ ‘#ή ή7 ή/ή.˞ήĆ  Dq@ ! DwCՏ é ŵɮDՏ L4gw! 3 qwHՏ 3w ɇ EFg+

K4Đή 9!ήĆ;.!ή #ήήӃvďzӘՏҪďĜqӴqԧξՏ› I7#ή!/ή‰ ήd‘ή!/ή ή uήĊͬήšή. ή̻Žή y˼ĒήÍήÊ!ͭü§ή!rʹ-ÌƎńή Dq;w! DhrwDՏ FFh ! 5w 1Č KÂ

T[iw jwlw yή.˟ή ή".ή ĬýԂpՏ jw ʦՏ m , w K ή jw*Տ ow



hw klw

!̈ƇΟD̘-ήr™ ή Djmw





EFhw& 3 0D w

FFwjwƕ ® ºĂ 4 /

ɜƙήr!#ήƖ7!ŠƽήĒή˺uή 4.ή j  wnw "ή.ή D¨!Dydήy‘ή‘ή‰ή/ή ƾ ή?  wɅ ƨ Տ œή Ÿ#ή™Ñʺή Dkow "ɇ 44ˆuɇ ĒȰ¬ óɐ ›ŷÊdg!-ή  ή 3w 3w Dw   " j " kw mw EE  Šʬή ή kw ʥՏ  F ĝɐ ³ GHk # ^w owSw Dk $ EFw % S ÿɐ Ēή ЂͲԥêՏ Djw& EEw ɇ Ī Š ή #ή kw OPw 3C-w ADŠʤή Ȏɇ D¥ή ͮή ή Ϳ.DƇή ˆή5‘ή Šή /ή7w?w5 3  w ĬϏ

p͹ԩώՏ

ȍɇ Ĵɇ

˞ӗՏ owSw ɭ õ ɮΠήuŪʻ5ή jwSw 5 Iw pwR

t 5Iw5 Mw•ƿή
DNw ýӣՏ ͺѾՏ >‰$-êp Ğ D ή

Deœ“ ¤ „}p¡¤ TSήƅή Ē uɇ ńՏ Șɇ Ē ,ŁήĒ,1;Ē ʖ ƆՏ ԐОϠՏ 3_˜t о aҋ

    ‚Ē , Ù Q

ĸή Ē Ʀł,ƅήǔ , QήĒ9u Տ ŀȲ̵ή ė

0ή ή(S¤ή gaarp‚ 8IήMƍ(§ή(3Á̀0ħή

ĭɇ

iĨ ³0 3ƉŃή  ȷ ¢i 1 `T{w H`\ ð˦¤0(ή 3ή0ũ ή(ù(ή 2ǍÇ0(ή N\¤ F¤ 0̹73ǧή ɭ3(ή RĒĒ ~3Ē +7 …Ē /K>\‚ ÝĒ  "&  Þ  Ē Ē ĒĒ Ē Ē  Ē  Ē  :Ē M  ÚĒ Ē ÖĒĒ + )7Ē +7 P Ē Տ Pb ĒPf  Ē  hĒ ×ö5 € RĒՏ ĒĒ ĒĒ Ē  Ē  Ē   Ē  ?Ē .e>g3Eaҽή 2Ǎ 3  Ē ¥ ĒĒQ¦  gĒºƭ” Ǩή ¶E( āê ZĒM     Q   Ē :1\\at‚ 6 > ‚1\‚„UV¤ ]t 3c3h‚ 0½ή( ή aE‚(ùή±ή \[Ζ ?¤“¤ 3iĒ+ 7ĒXxqt‚ 83 >ϯՏ(3ή0˜Ç3ήÊ7ŭ(ghή Df\¤ a\U‚ 0_ʛ30ή O_‚     ‚‚ ʜήw€,Ĺ « ±%” Ƹ ±€Ĺ ǩή ,O\7@‚ w , , ή 1\<    ‚ 7ή „UW¤ (93ήѦUgή z1Ux?‚ aF‚ 2Ǎ0ή ± ,”ή Ē #ª†Ē Õé Ē ½ Ē€ ƒÜή ĒĒ GuĒ Ē ‚ČÛĒ Ē  1˜ Hɇ (ήI§0ή ̥Տ Ē  #wi¢ ˜



³0 3¤ȿή ŀi 1 `T{rP`\

ð̿0(ή8Νήl‹0o6ή́‡ƹή 8al‚#‚ ¾Ē $‡Ē

+>HA8r‚ (¤ 24ayt‚ —d\¤ ‰[‹\{UgR›pK¤ Oj“\S™„Ž¤ „`¤ + . ¤ ua‚

I?t‚ tK?‚ eaO\u‚ (¤ a]‚ Ȝ ­ ɇ .K>\‚ ¹„Ī˫ ˸Տ Pl‚ 3\‚ Olam8>U>n‚ ˜FŠ[£„hU¤ Ÿi˜d¤ (¤)¤ Š Ͱҿ˜цU p¤ sa‚ "# ‚ E Ky0ή (¤(¤ Ē +.¤ 

#‚7al‚#‚ Ē ‚ 

Ē ɇ //Ē yɇ œ#ĒĒ  ‚

^ɐ LjĒ 1`=‚ ^vǃªɐ Lb

$b\nQ‚ KcYavK?s€‚

8B\wi><‚ 2r‚ ?¤ yF‹Šg|c¤ uK?‚ 7Sh7xZ7Tk;W?‚ dE‚ 9‚L2{?‚ M)‚ 

5B81yl>‚ :<¤ j”¤ f3i1VU>U‚ ta‚ (¤*¤ /Jxn‚ * ¤9¤@¤ LZ¤ 9aqh\ ͱӀՏ 3\=‚ *( ¤' #‚ To‚ 2‚ ŒMGps\q„cJ

Ŀijƺή H}U¤ 9 У0ή vKC‚ [P=eaR^t‚ bG‚ "% ‚ hº 9) ¤ KĒ % *¤ 2˜ W;;¡Ī !n °EĒ ɇ ;?¤ ´§Ē ckžj|c¤ 9A¤ Ē B.$ˆĒ -S\9C‚ (˖ή t Ҿѧcrλ0ή 9>B¤ H}U¤ * (?¤ H\¤ “kzkpF¤ }A‚ N2|?‚ 9="* *¤ ɇ ?9#?( ¤ ͍>i\Ϧcμ ˜ '*‚  ɇ  GijήŔή ¢# žcFEĒJՏ "¸9ĒKĒ  ‚ [̏23ή '%

‚ (( ¤



:( ¤ ɇ ;?¤ ɇ +9¤ yɇ (%‚ ~D‚ J2|?‚



;TĪ

B ªO¼±Ãpë 5P̆q£QÌ aRœë B²bŽqÍÝë /:(E@H#S K +$8K24S 3R9A2S ¢  ‹ń ť9ɐ £ǃȧǬțɮ ͅφΡӿЩĐĊŬՏ ÉĐĩѶåՏ   26.:; ʍՏ #M=PQ Q



( ȷ 5Ȱ ‡ՏͶɌ ‡Տ

  Q

zȷ ȷ N)HX/?#*b B0b $ }íȷ Ƃƞ .ȷ I/!XIN5Y(;`b `b &Šɮ 1˜ ›ȕ “   Տ  {ȷ <ȷ ƾɇ Żż ɮ 0 ù ɮ )nj‡Ưɇ ȷ ƈȷ ǽȌƱɇ B0b ›ȷ  Q 2b GC\.Jb B0b )b .!$ ¬ȷ B0b &‚ ɐɮ ņ<ȷÖȷ +]6NV .Ȏ E ȷǫA $þȷ  L}ȷ Ȃñɐȉɐ OX$4b 9 ȷ )Ƥɇ 1˜ /<  ®

¯

°

Â

 Q )Vb b +b  ȷ 8 Ę } "ȷ B`ŪqÞ ȷ q.ȷ ? ȷ Å Đ ȷ fՏ , .ȷ ̓Տ ,b ȷ .g< ȷ ǣFȷ łńMȷ b  •ȷ b K)NF-!S9Z/=ab D?NTMX$Vb ȷ 88đȷ E î.ȷ  ȷ ȷ A <8×ǒ ȷ E1b S ǧ œƽ!*b TŁÃsȷ 8ȷ  ȷ ï$, !×ȷ b 5 $) Ǽɇ $ ȷ šȷ7?V(I6BIb <—ȷ ȷ#7I";/b ^V/@'b fÄՏ VBb 7?R*LP/%Vb ȷ 88ȷ ȷŘsȷ ÃՏ̀Տ UBb  ƉîƊ8Lȷ L9ȷ $9K!<*b  ȷ ’ȷ  &b ́̎Տ <ȷ  „Ƌ8 ȷ ȷ E$YEȷ  ȷ Ìıȷ H<Ȧȷ  ȷ ˝̾Տ ɇ b  6  m ȷ ̪Տ zȷȷǬ<LȞȷ$ !Ĩȷ LE[.b 9 ȷ ›ȷ *_6NVNb ȷgA $þȷ $ !ȷ ƿɇ ȗEȷ  (šȐ ’ ɭՏ Ǝ (šȑՏ ~(ȤɮՏ “   b ũȷ  ȷ Lȷ ̩Տ .+L+ 2 ȷg+ȷ—8 Aǹȃ¹ ¤ȃ

m ȷ ÃՏ ŲȷȷÝ Lȷ < g ȭȷƌ„ȷ<ȷ !Yȷ .ȷ ȷ (Տ zȷ Ŕɇ F < ǻ~ 7ȗȷ$Ǔ„!Ƿ¹ȷ ś XFgANƌɇ , ”˜

”˜

˜ `b ȧɇ ̍Տ )ȷ'

0b ȣ

ȷ zȷ+  ȷȷ ɇ 1˜ ԹՏ łՏ Ⱦɇ 1BKb Aȷ A † ՏȽɇ ¾Տ )Ļ

Ǝɐ ăՏ ƍɐ jɇ  &b ͚ ’ ţ

­

(

­

Wńľ Ƨ ɇ ¾Տ )sȷ V3/?b x Õ ɇ ŃՏ ¢ b 'Ľ Ʀ ɇ ƚɐ ȏƧ

Â



w éȧȷ  ȷ ÄՏ ȷ ȷ  ȷ ɇ(Տ' ›Տ .$Ʈ Eȷ +;)>/AWN b ǝ

mØ ȷ ĽlÃȷVØȷ Ũȷ Ǹ+ç! ȷÿ 9ȷƯȍ!ïȷ P8&/b  Þ ȷ 8ȷ  ƽ ɮ ń  b $ $Șȷ6pȷ<Ɠȷ$„ȷg$ íº

ʅɼՏ

:@&b 

@ ‹©~L»°Ânë 4LɅo¢Mʌ_N›ë A°`‹nËÜë >Š¨}J¹¯Ámë2KȄm¡JȊ^Jšë:šÛ¡ºŠKlë ?3; ë

?Ő;Dɐ

 +'; 2(' ; džȘȬȐǥɐ Q 0("62 )'1 Ȣ Ƣ Տ Çń!ńńD£ń

¹ńDń))&ń(ń . Š ɇ wń Dz )Ðɮ ÙńCń ċńòń2$ ) =yd~ ~!Ģ ń 2~    ~~ *$3# + A  ) z  ~ 4kń.ɇ ɇ b + c4 SîÊ Ô ć ɇ ̌Տ ̈́ƥՏ È{ń óńCńk( ńJń)IJ .İ Á Ł§ń %

x ŸȚÎɮ ń0ëbƘ &1w~ EF~ 5 $ , B ~ D 6o+x~ [Տ - %? ~ Ş ^ Ŭ] ŝŽ

bńSƣɇ



g e 8$ />~ ̋Տ ÂYń



&

'

Â

- $ 7 '.C~ ̊Տ LJĜɐ

ń†)ńl |G„ńă &ń(ńȁð Ȉɐ  ńTTń cf~



&ȍ Ķ

Q .ń ń å)ĸń ôńıMń  Nń Úń s ¿ ɇ ÈÔ ȷ ͘ŢՏ ͠Տ R9ń ͤՏ V/ȷ pń nńnń ń ćIJ69ń J Čń J~ lȷ lń P~ &ëD¤ń .Cń W~ÀՏ r4 ń ńĈ) 9ńøń J~  ń ͕ƣՏ gń&ń

#/¨ń G~ KZ9 W[6~ ńL \: q;  ]V<~ ńr 7 J[ ZR { M[  ZS ~ H~ Ǧí N- N^ ǍƭÌɮ s6~ ~ KR~ ĴՏ  Z_  ZR~ t6~ ~ OT~ |~ ͙̕ƦՏ Ã3?ˆXń ±ańńv3~ ~ ^-ń }~ -h‘ńæ QX G~ u:  PS | S`~ Ä #- ł -Íńń² º Ń »hńńP R } U`  ~ ! & ]/ń«ń³ańI PY~ f



e



ɬƤ Éń6P„ńń ìńHńRŀńďâ4|ńĘwˆR3u y¡)Ïɮ ńb imj bk a 0 ml h  Չ ř Òҥ !2ńĄz(ń öąij ¥ń Ìń ўÓՂՏ Ò6ńĴMń Œưɇ 3Տ $ ~ eM ń ń†ńńsC&íń o ń)]ɮ  ńTpńDZžșÍɮ ńħ)ĺńÓ49ń4ãîĐ(ńǖƼɇ n# ~ p  †Â

k½ȷ

Çȷ1iƚ1ȷȷǛȷ O ÛYȷȁȷAŜɮ ˓Տ VX a xɮaV ! a /aY` " =™ëȷ±  ȷŸȷ{ - XB_¡ȷ- Bˆȷ 9ȷCȷ«aȷ V-a ¯Տ .Ó ‹ɇ @ V . a šŰȷǜȷ, ŸȷfȷÛ,- @Bğȷ ̔СԤéѤՏ 5> ,ȷ V! a  ȷ A 8˜ W#a [ȷY`$a ,a ‚˜ ƒ˜ €˜ 2P %  a U ¦Â §Â ¦Â

Ōǎ-ˆȷ¥Bȷ–,-ȷ€ M ȷ S ƓƔɐ €ȷ Ƒƒɐ тšՏ ȷ š,țȷ ţ ȷ =/ë ”R:{ȷ @ V. a 75 ȷ <0 &a ſ@Ôȷ A "6{Õȷ {œA Ê ı ɮ u ȷ { ȘūËɮ ˕Տ AV ' a +˜ Z` (a '"#  ~ ȷ pȥȷ > ȷ ƒ › äĵ Éɮ 'Ȉȷ6ȷ ùȷ  ȷ ĂՏ [ȷ=ȷ ¦ìȷŤE ìSȷ «Â

¨Â

©Â

ɻƟ ľ^‚ȷȷȷ- "ȷƀǝȷ7RĈȷ  ɏ )Sȷ v )ȷǞ Oȷ  ȷ¦: Rȷ|ǃ M  ȷ ȥɇ 7ȷȉȷã‚ ȷDŽ|Ǐ ȷ:ȷ)¸ȷ Ŀȷ >bȷxɇ 8˜ ‡ ² ȯĉȷ . ‡ˆȷ]a 3Տ b _J ȷ ȷ‡´ȷȻɇ Տ  ɇ ȷ 2 ¨"- :Ġȷ Ž-ȷ )ȷ 8 O ȷ 0, Rȷ'ġȷ uȷ OCȷ ǰ!5a ȷ 9Ȃȷ  ȷ )ȷŴ@ _ȷ § 6ȷ'bSȷ

ǐ ȷ Xé~ Rȷaȷ ȷ)ȷ "ȷ 7ȷ  ȷ ȷ  9 ¬ ɇ u5DžȷÏ ȷ ë: 6ȷ|dž ȷ ǟȷ )Ċȷ 3+ a ó 3Q a >üȷ 7ȷ ŵ9@ Ģȷ şȷ ȷ B: ȷ ƥɇ -˜ AՏ ćȷ ȰMȷ ȷ A†Տ ȼɇ %Տ ̒ƞՏ Šȷ , ȷiLJȷ 9ȷ AՏ ¯Տ   b È7ȷ‡ȷ ȷ 4R S ^a Ń ȷȷ ȷ >ȷ 'ÏƩLjȷ ȷƦȷQdR ã ȷ Ȧ 9 Œɇ ɂ ô @ȷ)ċȷ  ȷ Ƅ1ťƪȶà ȷ Ɓbĥȷ Hȷ )ȷ _ȷ ȷ Ýåȷ  µȷ >ȷ ȫ_ ȷ Q¥ OȜȷ : 0¨ȷ ȃɐ Տ €ȷ iȷ  ȷ 3I a   aa4J  ! a Bȷ ”~   ȷ "ȷ 5Ja 8˜ 3I _a DDZȷ B 8C  a   9a K  ! a %Տ )ȷ 5Óȷ> 3 D /a 6F #a'8D 3C )a /a 5G -a [ȷ8H )a #

¬

$

Â

!

«

"

3L ! a ,˜ 3Ma ŁՏ ͥс ƍ ȡ Ĵ w bEȷ 3Ea ɇ 7K a ȷ ȷ › ȷ :Ca [ȷ ]I &a[ȷ ^a S :N * a / ^ ij ŀ ȷ ȷ J àČȷ ƽ ‰ РՏ 3Տ V w O Eȷ ȷ' ȷ 5C  a a3O  #a ,Bȷ " b-¶ȷ )ȷ   ȷ ȷ ȷ€ȷ Ě П Ŭŭɐ äȷ[ȷ )~ ëR  ģȷ w ':C Mȷ )ȷŶ@ ȷ ȷ  ȷ Va'=ȷ 0 _6ȷ' tȷ D>ȷB5 ȷ—ǠĤȷ g hÂ





ª

Â

ŏȷ ȷ"ȷ7ȷő ‰ ɇ ŨՏ\¾ɇ Æaȷ ӲЁéՏ :ȷDzȕjȷȷ>R Ɛɐ ɐ ô ȷɪ$Տ

‰

Ƴ ɇ˰ Ļäå

Ɛ



ɺՏ

õȷ _Տ ͩՏ '

Ɋ Ǘ ȢȀȏ ǁ ɐ ӖТѥ ¡$Տ 

‰

Ơ ɇ_ Læç

Ƌ

ơ ɇ$ Lèé

x9Ȗȷ ‹ 2Ǖ _Տ ®Տ

ƌ

͵RՏ S "

5ÞՏ

k¾ȷ

ɫ

$Տ Տ Ɗɐŷ¥ȷ $ ͪ ¡ -ȷ

 R

&Տ DŽěɐ

ŽI^ȷ 3dzȷ ±+K ȷ
'4 S +4 !S '4  S ,4 S

ĞğĠ` 

Dȷ

S



Տ



)4 +4



Ƀɇ

S

'4  +4 Ŷɇ

Ʉɇ



ǭɮ

â  ëŽ^

' + 

(4 *+4

.  + + 4 %˜ +4 (˜ )  '+4 ę ƌ – HI2ȷQZá NCȷ +Kȷ ¡++¡´ȷ * $     !S  ˜ȷ Ʈɮ*4 '+ 4 &34S =A







+ 4

) # SƋ ˆ ɇ@ a)˜  S

 S <6S ! #  ƛ

ò $ 4 ɮ   S ȷ 3ȝȷ ˜ȷ %ɮ -4 I#

̘ϪՏ @a ±Տ Ƞ Š Տ 3 &9S ; ȷ “ȷ ȷ fC+ȷ •+ýKǡCȷ @  a ˒Տ `ɮ ¯23ȷ G-GS y˜ɮ%ɮ +  4 9#S ŕ—ɮ%ɮ (4 '+ U3Kȷ @a %ɮ +4 #S @a 1a '4 "<:G@#1 G0:)S + # >S+4 ɮ ) ! ~ D3¯Kȷ @a &˜   S <7S , $   S ũՏ [ È *˜ = č ȷ ȷ <; 5J#'S G-HS Ƙɇ ɇ   S D3ȷ

ÆAȷȴȷ [Տ T ³ȷlȷɮ ¼Őlȷ 1Տ Šĺȵ Ďȷȷ! ȷăȷ 1Տ ¿ȷ 1Տ ¼kăȷ I•ȷ^ȷ ŒɮՏ 4 <ES¿ȷ Čɇ  ;¢Qƿ 0ȷ 014 234 Տ '4 Տ + 4 UȷaŮ? ȷQ ;°ȷ Ƭ }CȷK +Ơ+!ȷ 3ȷ; ǭp; ȷ;ö '4 *˜ G +4 1˜ 4 H úƻ ȷ   S Տ ³ȷ Ǩɮɮ '-4 S Ȩ¯ȩd Ĥ  + ì ×ɮ &4(  +4 ɮ  S ÿȷ Kȷ 3 ȷ  ¢;I;ȷQZ; ȷ+Kȷ ^3;ü.ȷ3ȷ'4 %˜ GjȷĖՏ 9˜ S :#S ßȷȣ?ȷ+ȷ   

kĹȷ

Singapore Mathematical Society Singapore Mathematical Olympiad (SMO)

2012

Junior Section (First Round)

Tuesday,

29

May

2012

0930-1200

hrs

Instructions to contestants

1. Answer ALL 35 questions. 2. Enter your answers on the answer sheet provided. 3. For the multiple choice questions, enter your answer on the answer sheet by shading the

bubble containing the letter (A, B, C, D or E) corresponding to the correct answer.

4. For the other short questions, write your answer on the answer sheet and shade the ap­ propriate bubble below your answer. 5. No steps are needed to justify your answers. 6. Each question carries 1 mark. 7. No calculators are allowed. 8. Throughout this paper, let

example,

=

=

lxJ

denote the greatest integer less than or equal to x. For

l2.lj 2, l3. 9J 3.

PLEASE DO NOT TURN OVER UNTIL YOU ARE TOLD TO DO SO.

1

Multiple Choice Questions 1. Let

a

and (3 be the roots of the quadratic equation x2

value of

(a

- f3? is

(B) 1;

(A) 0;

(C)

2;

+

2bx + b

=

1. The smallest possible

(E) 4.

(D) 3;

2. It is known that n2012 + n2010 is divisible by 10 for some positive integer n. Which of the following numbers is not a possible value for n? (A)

2;

(B) 13;



(E) 59.

(D) 47;

(C) 35;

3. Using the vertices of a cube as vertices, how many triangular pyramid can you form?

(B) 58;

(A) 54;

4.

AB

is a chord of a circle with centre

with

AB

closer to

C

than to

(E) 70.

(D) 64;

(C) 60;

D.

0. CD

Given that

is the diameter perpendicular to the chord AB,

LAOB =

90° , then the quotient

area of .6.ABC area of .6.AOD (A) J2 -1;

(B)

v;;

2 - J2;

5. The diagram below shows that lines. Let

F and

BG = EF,

G

ABCD

DE AE.

2

is a parallelogram and both

be the intersections of

find the quotient

(E) �. -

(C)

BE

E

2

with

CD

and

AC

AE

and

BE

are straight

respectively. Given that

6. Four circles each of radius x and a square are arranged within a circle of radius 8 as shown in the following figure.

What is the range of x?

(B) 0

(A) 0 <X< 4; (D)

<

x < 8 ( J2 + 1);

(C)

(E) 4- J2 <X< 4(J2

4- 2J2 < x < 8(J2 -1);

7 . Adam has a triangular field

4- 2J2 <X< 4;

ABC

with

AB =

5,

BC =

8 and

CA =

separate the field into two parts by building a straight fence from BC

such that

(A)

4 J2I . 11 '

AD

(B)

bisects

�;

L.BAC.

4

(C)

A

(D)

11 '

5�;

11. H e intends to

to a point

Find the area of the part of the field

5J2I.

+ 1).

ABD.

a

and b be real numbers with b =/= 0 such that

Which of the following statements is incorrect? (A) If b is an integer then

a

is an integer;

(B) If a is a non-zero integer then b is an integer; (C) If b is a rational number then

a

is a rational number;

(D) If a is a non-zero rational number then b is a rational number;

(E) If b is an even number then a is an even number. 3

on side

(E) None of the above.

8. For any real number x, let lxJ be the largest integer less than or equal to x and x Let

D

=

x

- lxJ.

9. Given that

--;x;---;x;

y=

X

­

is an integer. Which of the following is incorrect?

x can admit the value of any non-zero integer; (B) x can be any positive number; (C) x can be any negative number; (D) can take the value 2; (E) can take the value -2.

(A)

y y

10. Suppose that

A, B, C

are three teachers working in three different schools X , Y, Z and spe­

cializing in three different subjects: Mathematics, Latin and Music. It is known that A

(i)

does not teach Mathematics and

B

does not work in school Z;

(ii) The teacher in school Z teaches Music; (iii) The teacher in school X does not teach Latin; B

(iv)

does not teach Mathematics.

Which of the following statement is correct? (A)

B

works in school X and

(B)

A

teaches Latin and works in school Z;

(C)

B

teaches Latin and works in school

(D)

A

teaches Music and

C

C

works in school

Y;

Y;

teaches Latin;

(E) None of the above. Short Questions a

=

a >

=

b be real numbers such that b, 2a + 2b 75 and 2-a + 2-b 12-1 . Find the value of 2a-b . x3 - x + 120 . an mteger. . . . . 12 . Fm. d the sum of all positive mtegers x sueh t h at (x - 1 )(x + 1) 11. Let

and

IS

4

13. Consider the equation

vh

x2 - 8x + 1 + J 9x2 - 24x - 8

=

3.

It is known that the largest root of the equation is - k times the smallest root . Find k.

14. Find the four-digit number abed satisfying 2(abed) + 1000 deba. (For example, if a 1, b 2, e 3 and d 4, then abed =

=

=

=

=

=

1234.) =

15. Suppose x andy are real numbers satisfying x2 + y 2 - 22x - 20y + 221 0. Find xy. 16. Let m and n be positive integers satisfying mn2 + 876 4mn + 217n. Find the sum of all possible values of m. =

17. For any real number x, let lxJ denote the largest integer less than or equal to x. Find the value of lxJ of the smallest x satisfying lx 2 J - lx J 2 100. =

18. Suppose XI, x2 , . . . , X 49 are real numbers such that Find the maximum value of

XI + 2x2 + + 49x49 · ·

·

·

19. Find the minimum value of Jx 2 + ( 20 -y ) 2 + Jy 2 + ( 21 - z ) 2 + Jz 2 + ( 20 -w ) 2 + Jw 2 + ( 21 - x ) 2 . 20. Let A be a 4-digit integer. increased by

number is

n,

When both the first digit (left-most) and the third digit are

and the second digit and the fourth digit are decreased by

n times

A.

Find the value of A.

21. Find the remainder when 1021I 022 is divided by 1023. 5

n, the new

22. Consider a list of six numbers. When the largest number is removed from the list, the average

is decreased by 1. When the smallest number is removed, the average is increased by 1. When both the largest and the smallest numbers are removed, the average of the remaining four numbers is

20. Find the product of the largest and the smallest numbers.

23. For each positive integer Suppose that a1

a2012.

=

n 2:

1, we define the recursive relation given by an+l

=1 1

+an

.

Find the sum of the squares of all possible values of a1.

24. A positive integer is called

friendly if it is divisible by the sum of its digits. For example,

111 is friendly but 123 is not. Find the number of all two-digit friendly numbers.

25. In the diagram below, D and E lie on the side AB, and Flies on the side AC such that DA=DF=DE, BE= EF and BF= BC. It is given that L_ABC= 2 L_ACB. Find x, where L_BFD= X0•

B

F

26. In the diagram below, A and B(20,0) lie on the x-axis and C(O, 30) lies on they-axis such that L_ACB= 90°. A rectangle DEFG is inscribed in triangle ABC. Given that the area of triangle CGF is 351, calculate the area of the rectangle DEFG. y

c

A

D

0

6

E

B

X

27. Let

ABCDEF

be a regular hexagon. Let

G

be a point on

area of AGEF is 100, find the area of the hexagon G

E

ED

such that

ABCDEF.

EG =

3GD . If the

D

F

c B

A

28. Given a package containing 200 red marbles, 300 blue marbles and 400 green marbles. At each occasion, you are allowed to withdraw at most one red marble, at most two blue marbles and a total of at most five marbles out of the package. Find the minimal number of withdrawals required to withdraw all the marbles from the package.

29. 3 red marbles, 4 blue marbles and

5 green marbles are distributed to 12 students.

Each

student gets one and only one marble. In how many ways can the marbles be distributed so that Jamy and Jaren get the same colour and Jason gets a green marble?

30 . A round cake is cut into

n

pieces with 3 cuts. Find the product of all possible values of n.

31 . How many triples of non-negative integers ( x, y, z) satisfying the equation

xyz + x y + yz + zx + x + y + z = 2012? 32. There are 2012 students in a secondary school. Every student writes a new year card. The cards are mixed up and randomly distributed to students. Suppose each student gets one and only one card. Find the expected number of students who get back their own cards.

33. Two players

A

and B play rock-paper-scissors continuously until player

A

wins 2 consecutive

games. Suppose each player is equally likely to use each hand-sign in every game. What is the expected number of games they will play? 7

34. There are 2012 students standing in a circle; they are numbered 1, 2, . . . , 2012 clockwise. The counting starts from the first student ( number 1) and proceeds around the circle clockwise. Alternate students will be eliminated from the circle in the following way: The first student

stays in the circle while the second student leaves the circle. The third student stays while the fourth student leaves and so on. When the counting reaches number number

2012, it goes back to

1 and the elimination continues until the last student remains. What is the number

of the last student?

n

< n.

35. There are k people and chairs in a row, where 2 ::; k There is a couple among the k people. The number of ways in which all k people can be seated such that the couple is seated together is equal to the number of ways in which the (k - 2) people, without the couple present, can be seated. Find the smallest value of n.

8

Singapore Mathematical Society Singapore Mathematical Olympiad (SMO) 2012 Junior Section (First Round)

Multiple Choice Questions

1. Answer: (D) . Since x 2 + 2bx + (b - 1) 0, we have a/3 b - 1 and a + f3 -2b. Then (a- /3) 2 (a + /3) 2 - 4af3 ( -2b) 2 - 4(b - 1) 4(b2 - b + 1) 4[(b - 1 2) 2 + 3 4] 3 The equality holds if and only if b 1 2. 2. Answer: (E) . Note that n 2o 12 n 201o n 201o (n 2 + 1). If n 2, then 5 (n 2 + 1) and 2 n2010 . If n 13 or 47, then 10 (n 2 + 1). If n 35, then 2 (n 2 + 1) and 5 n 2010 . If n = 59, then 5 f (59 2 + 1) and 5f n 2010 . 3. Answer: (B). There are 8 vertices in a cube. Any 4 vertices form a triangular pyramid unless they lie on the same plane. (!)- 6 -6 58 4. Answer: (B). Let the radius of the circle be 1. Then AE CE 2 CE 2(1 - 1 J2) 2 _ J2 sl::.ABc AE cE 1 OD � AE OD Sl::.AOD Sl::.AOD =

=

=

=

=

=

=



=

+

=

=

=

=

=

x

x

x

=

x

9

x

=

=

c

D

5. Answer: (A) . DE . Then = EF . Note that Let AE EB DF + FC EF + FG EF + EB - 2EF + 2 1 - DC AB - AB AB - EB GB - EB EF - (I__ ) 3 - V5 (0 1). Then + - 3 = 0, and thus = -;1 2 6. Answer: (D) . x

x

=

_

_

_

_

x

< x<

x

x

_

Consider the extreme cases:

Then Xmin

= 8 - 24� = 4 - 2� and

7. Answer: (D)

By Heron's formula,

Then

Xmax

= �28+ 1 = 8(� - 1).

s6.ABC s6.ABD s6.ACD

=J12(12 - 5)(12 - 8)(12 - 11) 4v'2I. 5 � X AB X AD X sin a AB � X AC X AD X sin a AC 11 5v'2I 5 S6.ABD = 5 + 11 X 451 =-4 =

10

x

B

c

A 8. Answer: (B) .

� = b ( l�J + { �} ) = b l�J + b { �} · It follows from a = b l�J + b { �} that b l�J = a. Hence, � = l�J is an integer. Note that a = b ·

Obviously, b is not necessary an integer even if a is an integer.

9. Answer: (C) .

0, then = -= ( -x) = �x -= --2x = -2.

x can take any nonzero real number. If x

If x

<

0, then

y

=

-=

-fE- -fE

X

-fE-

X

-

-fE- -$

y

>

-fE

X

- xX

-8-

=- = X

-

X

X

10. Answer: (C) .

The assignment is as follows:

A: in Z, teaches Music; B: in

Y,

teaches Latin;

C:

in X , teaches Mathematics.

Short Questions

11. Answer: 4. 75 (2a + 2b )(2-a 2-b ) = 2 2a-b + 2b-a . Then 2a-b 2b-a = 417 = 4 4"1 12 Since a b, we obtain 2a-b = 4. 12. Answer: 25. x 3 - x + 120 120 . It 1s. an .mteger "f and on y . f ( x2 - 1) -120 . =x Note t h at (x 1) (x 1) x2 1 Then x2 - 1 = ±1 2 3 4 5 6 8 10 12 15 20 24 30 40 60 120. Hence, x = 0 2 3 4 5 11; and 2 + 3 + 4 + 5 + 11 = 25. 13. Answer: 9. +

=

+

+

+

>

_

+

+

1

_

11

1

1

0.

Let y =v'3x 2 - 8x +

1 . Then the equation becomes y y'3 2 - 11 = 3 . Theny'3y 2 - 11 = 3 - y. Squaring both sides, we have 3y 2 - 11 = 9 - 6y y 2 + 3y - 10 = 0 . Then y = 2 or y = -5 (rej ected because y 0) . Solve 3x 2 - 8x 1 = 2 2 . Then x = 3 and x = -3- 1 . Hence, k = 3/3- 1 = 9 . 14. Answer: 2996 . y

+

2

+

+

y 2 ; that is,

Rewrite the equation as the following:

b c d b c d 0 0 + c b a Since a is the last digit of 2d, a is even; since 2a 1::::; d::::; 9, a::::; 4 . So a = 2 or a = 4 . If a = 4, then d 2a 1 = 9 and thus d = 9; but then the last digit of 2d would be 8 1- a, a a a 1 d

2

+

2

+

0

+

contradiction.

a = 2, then d 2a 1 = 5 and the last digit of 2d is 2; so d = 6 . The equation reduces to b c b c 1 1 c b There are 2 cases : either 2c 1 = b and 2b = 10 c, which has no integer solution; or 2c + 1 = 10 b and 2b 1 = 10 + c, which gives b = c 9 . 15 . Answer: 110 . Complete the square: (x - 11) 2 + (y - 10) 2 = 0 . Then x = 11 and y = 10; thus xy = 110 . 16 . Answer: 93 . If

+

+

+

+

+

=

Rearranging, we have

- 876 8 mn2 - 217n = 4mn - 876 n = 4mn mn - 21 7 = 4 - mn - 217 Then ( mn - 217) 8 . It follows that mn - 217 = ±1, ±2, ±4, ±8 . So mn = 218, 216, 219, 215, 221, 213, 225, 209, and n = -4, 12, 0, 8, 2, 6, 3, 5 respectively. mn 216 225 N ot e th at m = - 1s a positiVe mteger. T en m = - = 18 or m = - = 75 . n 12 3 =?-

.

. .

.

h

12

17. Answer: 50. Write x = lxJ + --fl: � Then 100:::; (lxJ + --fl:-) 2 - lxJ 2 = 2lxj--fl: -+ --fl:...J, 2lxJ + 1. S o lxJ 50 and x 2 lx2J = 100 + 502 = 2600. On the other hand, x = v'26QO is a solution. 18. Answer: 35. (X! + 2X 2 + + 49X 49 ) 2 = ( 1 X! + J2 J2X 2 + " " " + .j49 " .j49X 49 r :::; ( 1 + 2 + + 49) (xi + 2x� + + 49x�9) = 49 X2 50 X 1 = 352 . = X 49 = ± 351 The equality holds if and only if x1 = 19. Answer: 58. �

<



·

·

·

·

·

·

·

·

·

·

·

·

·

·

·

As shown below,

J x2 + (20 -y) 2 + Jy 2 + (21- z) 2 + J z2 + (20 -w) 2 + Jw 2 + (21 - x) 2 = AB + BC + CD +DA = A'B + BCJ + CD + DA." :::; A' A" = 422 + 402 = 58. 21-x A" A' A x • ::: ------- -.----"'7""o;::,...-----.,.. ----------:::,.. ', ;::,:, ,.. .... ....... """" ,' ;3 , .... ..I .... ',

,..,

o C'l

;3 I

B

'

21- z

C\

\ \

' \

\ jj

..

...

........

,,

...........

.......

,, A.. -" '

20. Answer: 1818.

A = abed. Then 1000(a + n) + 100(b - n) + 10(c + n) + (d - n) = nA. It gives A + 909n = nA; or equivalently, ( n - 1 )A = 909n. Note that (n - 1) and n are relatively prime and 101 is a prime number. We must have (n - 1) So n = 2 or n = 4. If n = 4, then A = 1212, which is impossible since b n. So n = 2 and A = 909 2 = 1818. Let the 4-digit number be

-9.

<

13

x

21. Answer: 4. Note that 1024 = 21 0 1 (mod 1023). Then 10211022 ( _ 2)1022 21022 21ox102+2 1024102 22 1102 22 4 22. Answer: 375. Let m and M be the smallest and the largest numbers. Then m + 80 + 1 = M + 80 1 = m + M + 80 5 5 6 Solving the system, m = 15 and M = 25. Then mM = 375. =

=

=

=

X

=

=

X

=

(mod

1023)

_

23. Answer: 3 .

+

1 a3 =- 1-,a a4 =--2 +- a, as =-3--, + 2a . . . . In general, a1 = a. Then a2 =-1 -, + a 2 + a 3 +2 a 5 + 3a an =FFnn+l++FnF-na1 a where F1 = 0, F2 = 1 and Fn +l = Fn Fn-1 for all 1. F + F2o11 a = a, then (a2 + a - 1)F2o12 = 0. If a 201 2 = 2o1 2 + L' 2012 a 2 L' 1 20 3 Since F 201 2 0, we have a + a - 1 = 0. Let and (3 be the roots of a 2 + a - 1 = 0. Then Let

-=-----'-==----

+

v

n�

v

a

>

23. ga + 1, we have (a + b) ga. . lOa + b = -Smce a+b a+b Case 1: If 3 f (a + b), then (a + b) a, and thus b = 0. We have 10 20 40 50 70 80. 3a ::::; 3. If 3a = 1, then Case 2: If 3 (a + b) but gf(a + b), then (a + b) 3a and 1::::; a+b a3a+ b 3a 2a = b, we have 12 24 48. If a + b = 2, then a = 2b, we have 21 42 84. If -= 3, then a + b b = 0, we have 30 60. Case 3 : If g (a+ b), then a+b = g or 18. If a+b = 18, then a = b = g, which is impossible. If a + b = g, then we have g friendly numbers 18 27 36 45 54 63 72 81 go.

24. Answer:

--

Therefore, in total there are 23 2-digit friendly numbers.

108. Since DA = DE = DF, LEFA = goo. Let LEBF = LEFB = x0• LBFC =goo - x0 and LCBF = 2x0, LBAC =goo - 2x0•

25. Answer:

14

Then

LBCF =

B

F

It is given that 3x = 2 ( 9 0 - x ) . Then x = 36. So x = 180 - ( 90 - x ) - ( 90 - 2x ) = 3x = 108. 26. Answer: 468.

AO = 30202 = 45. Then the area of !::,. ABC IS. ( 20 + 452 ) Let the height of !::,. CGF be Then 3 ( ) 2 = 351 = ( 3 ) 2 =

Note that

X

30

= 975 .

h.

h

30

Note that the rectangle

5

975

:::?

30

h

- h

DEFG has the same base as !::,. CGF. Then its area is 351

X

2 3

X

2 = 468

27. Answer: 240.

DE = 1 and denote A = EG = 3 4. EGAX = 1 +A AGEF 1 +A 1 1 + 2A . - -8 - Imp Ies 4 4 DEXY DEXY 8 ABCDEF 6 AGEF 1 + 2A Note that DEXY = 8" Then ABCDEF 6 12 If AGEF = 100, then ABCDEF = 100 = 240. S E A G D We may assume that -

2

1.

-

x

X/

I

I

I I

I

Fl

I

I

I

I

.'-------------.lL.----->L

A

B

28. Answer: 200. 15

5 12

Since at most one red marble can be withdrawn each time, it requires at least

200 withdrawals.

200 is possible. For example, 150 . (1, 2, 2) + 20 . (1, 0, 4) + 10 . (1, 0, 2) + 20 . (1, 0, 0) (200, 300, 400). 29. Answer: 3150. Case 1: Jamy and Jaren both take red marbles. So 1 red, 4 blue and 4 green marbles are distributed to 9 students: (�) (�) 630. Case 2: Jamy and Jaren both take blue marbles. So 3 red, 2 blue and 4 green marbles are distributed to 9 students: G) (�) 1260. Case 3: Jamy and Jaren both take green marbles. So 3 red, 4 blue and 2 green marbles are distributed to 9 students. The number is the same as case 2. 630 + 1260 + 1260 3150. 30. Answer: 840. With three cuts, a round cake can be cut into at least 1 + 3 4 pieces, and at most 1 + 1 + 2 + 3 7 pieces. Moreover, 4, 5, 6, 7 are all possible. 4 5 6 7 840. On the other hand,

=

=

X

=

X

=

=

=

n=

x

x

x

=

31. Answer: 27. ( + 1)(y + 1)( z + 1) 2013 3 11 61. I f all y, z are positive, there are 3 ! 6 solutions. If exactly one of y, z is 0, there are 3 6 18 solutions. If exactly two of y, z are 0, there are 3 solutions. 6 + 18 + 3 27. 32. Answer: 1. 1 . Then the expectation For each student , the probability that he gets back his card is 20 12 1 . of the whole class 2012 2012 1. x

=

x,

=

x

=

x,

=

x

x,

IS

x

x

=

=

16

33. Answer: 12. Let E be the expectation. If

A

does not win, the probability is

If A wins and then does not win, the probability is

2/3 and the game restarts.

(1/3)(2/3) and the game restarts. The probability that wins two consecutive games is (1/3)(1/3). Then E = 32 (E + 1) + 92 (E + 2) + 91 2. Solving the equation, we get E = 12. 34. Answer: 1976. If there are 1024 = 21 0 students, then the 1024th student is the last one leaving the circle. Suppose 2012 - 1024 = 988 students have left. Among the remaining 1024 students, the last student is (2 988 - 1) + 1 = 1976. 35. Answer: 12. (n - 1) 2 (� = �) ( k - 2)! = (k : 2) ( k - 2)!. Then 2 = (n k + � n k + 2 ) That is, 2n 2 - (4k - 6)n + (2k 2 - 6k + 4 - n) = 0. We can solve 7 - 5 (n k). v8k --n = k +-4 Note that the square of any odd number has the form 8k-7. Choose k so that v8k - 7-5 = 4, i.e . , k 11. Then n 12. A

X

X

x

x

x

x

x

x

_

>

=

=

17

1

_

"

Singapore Mathematical Society Singapore Mathematical Olympiad (SMO) 2012

(Junior Section, Round 2) 0930-1230

Saturday, 23 June 2012

0 be the centre of a parallelogram ABCD and P be any point in the plane. Let M, N be the midpoints of AP, BP, respectively and Q be the intersection of MC and ND. Prove that 0, P and Q are collinear.

1. Let

A such that each of the ten digits 0, 1, exactly once as a digit in exactly one of the numbers A, A2, A3.

2. Does there exist an integer

. . . , 9 appears

L.ABC, the external bisectors of LA and LB meet at a point D. Prove that the circumcentre of L.ABD and the points C, D lie on the same straight line.

3. In 4.

Determine the values of the positive integer n for which the following system of equations has a solution in positive integers x1, x2, , Xn. Find all solutions for each such n. •

.

Xl+ X2 + · · · + Xn = 16 1 1 1 + + . . ·+ - = 1 Xn X1 X2 5.

.

(1) (2)

Suppose S = a1, a2, , a15 is a set of 1 5 distinct positive integers chosen from 2 , 3, ... , 2012 such that every two of them are coprime. Prove that S contains a prime number. (Note" Two positive integers m, n are coprime if their only common factor is 1.) •

.

.

18

Singapore Mathematical Society Singapore Mathematical Olympiad (SMO) 2012

(Junior Section, Round 2 solutions)

MN II AB II CD, we have �MQN"' �CDQ. Hence MN = AB/2 = CD/2. Thus QM = CQ/2. In �ACP, CM is a median and Q divides CM in the ratio 1:2. Thus Q is the centroid. Hence the median PO passes through Q.

1. Since

p

Since the total number of digits in A, A2 and A3 is 10, the total number digits in 32, 322, 32 3 is 11 and the total number of digits in 20, 202, 203 is any solution A must satisfy 21 :::; A :::; 31. Since the unit digits of A, A2, A3 are distinct, the unit digit of A can only be 2, 3, 7, 8. Thus the only possible values of A are 22, 23, 27, 28. None of them has the desired property. Thus no such number exists.

2.

9,

CD bisects LC. If CA = CB, then CD is the perpendicular bisector of AB. Thus the circumcentre of �ABD is on CD. 3. Note that

If C A =/:. CB, we may assume that CA > CB. Let E be a point on CB extended and F be the point on CA so that C F = CB. Then, since CD is the perpendicular bisector of

19

BF, we have LAFD = LDBE = LDBA. Thus AFBD is a cyclic quadrilateral, i.e . , F is on the circumcircle of 6.ABD. The circumcentre lies on the perpendicular bisector of BF which is CD. 4.

then Without loss of generality, we may assume that x1 S x2 S S Xn· If x 1 = from n = and cannot be satisfied. Thus x1 � If x2 = then n = 2 and again cannot be satisfied. Thus x2 � Similarly, X3 � Thus x4+ + Xn S with x4 � Thus n s ·

(2), (1)

1

(1) 4.

4.

·

2.

3.

1,

·

4.

2,

·

·

7

·

n =

(i)

1: No solution. (ii) 2: The only solution of + 1 is x1 x2 2 which doesn't satisfy (1) . Thus there is no solution. (iii) 3: The only solutions of; + +;3 1 are (x1 , x2 , x3) (2, 3, 6) , (2, 4, 4) and ( 3, 3, 3) . They all do not satisfy (1) . (iv) 4: According to the discussion in the first paragraph, the solutions of x 1 + + X4 16 are (x1 , x2 , x3 , x4) (2, 3, 4, 7), (2, 3,5, 6) ,(2, 4,4, 6) ,(2, 4,5,5), (3, 3, 4, 6) , (3, 3,5,5),(3, 4,4,5),(4, 4, 4,4) . Only the last one satisfy (2) . Thus the system of equations has a solution only when 4 and for this the only solution is x1 x2 X3 X4 4. __!_

n =

Xl

n =

n =

·

·

·

1

__!_ 2 X

x12

=

=

=

=

=

=

=

=

=

=

n =

=

n,

5.

Suppose, on the contrary, that S contains no primes. For each i, let Pi be the smallest prime divisor of ai . Then Pl, P2 , . . . , P1 5 are distinct since the numbers in S are pairwise coprime. The first primes are 5, If = = > and aj � Pj is the largest among Pl, P2 , . . . , p1 5 , then Pj � a contradiction. Thus S must contain a prime number.

15

2, 3, 7, 11, 13, 17, 19 , 23, 29 , 31, 37, 41, 43, 47. 47 472 472 2309 2012,

20

Singapore Mathematical Society Singapore Mathematical Olympiad (SMO)

2012

Senior Section (First Round)

Tuesday,

29

May

2012

0930-1200

hrs

Instructions to contestants 1.

2.

3.

Answer ALL

35

questions.

Enter your answers on the answer sheet provided. For the multiple choice questions, enter your answer on the answer sheet by shading the bubble containing the letter (A, B, C, D or E) corresponding to the correct answer.

4- For the other short questions, write your answer on the answer sheet and shade the appropriate bubble below your answer. 5. 6. 7.

8.

No steps are needed to justify your answers. Each question carries

1

mark.

No calculators are allowed. Throughout this paper, let LxJ denote the greatest integer less than or equal to x. For example, L2.1J = 2, L3.9J = 3.

PLEASE DO NOT TURN OVER UNTIL YOU ARE TOLD TO DO SO

21

Multiple Choice Questions

1. Suppose

a

and (3 are real numbers that satisfy the equation

) V J2+ 1

(V

x2 + 2 J2+ 1 x+

�3 +;3 .

=

0

Find the value of

(A) 3 J J2+ 1(J2 - 1) - 8

(B) 8 -6v\12 + 1(J2 - 1)

(C) 3 JJ2+ 1(J2 - 1)+ 8

(D) 6 J J2+ 1(J2 - 1) - 8

(E) None of the above

2. Find the value of 20112 X 2012 - 2013 20132 X 2014 - 2015 -- -------+ -------,-2012! 2014! 1 + 1 - 1 - 1 2 0 1 0! 2 011! 2 01 2 ! 2 009! 1 + 13 + 1 (D) 1 + 2 010! 2 01 ! 2 01 4 ! 2 009!

1 + 1 + 1 + 1 2 010! 2 011! 2 01 2 ! 2 009! 1 + 1 - 1 - 13 (C) 2 010! 2 01 2 ! 2 01 ! 2 009! 1 + 1 - 13 - 1 (E) 2 0 1 0! 2 01 ! 2 01 4! 2 009! (A)

(B)

3. The increasing sequence T = 2 3 5 6 7 8 10 11 consists of all positive integers which are not perfect squares. What is the 2012th term of T? (A) 2055

(B) 2056

(C) 2057

(D) 2058

(E) 2059

4. Let 0 be the center of the inscribed circle of triangle �ABC and D be the point on AC with OD _LAC. If AB = 10 AC = 9 BC = 11, find CD.

(A) 4

(B) 4.5

(C) 5

(D) 5.5 22

(E) 6

5. Find the value of

(A)

1

cos4 75° + sin4 75° + 3 sin2 75° cos2 75° cos6 75° + sin6 75° + 4 sin2 75° cos2 75° ·

(B) �

(C)



(D) 1

(E) cos 75° + sin 75°

6. If the roots of the equation x2 + 3x - 1 = 0 are also the roots of the equation x4 + ax2 + bx+ c = 0, find the value of a+ b+ 4c. (A) -13

(B) -7

(C) 5

(D) 7

(E) 11

7. Find the sum of the digits of all numbers in the sequence 1, 2, 3, 4, . . . , 1000. (A) 4501

(B) 12195

(C) 13501

(D) 499500

(E) None of the above

8. Find the number of real solutions to the equation X

100

=

. smx,

where x is measured in radians. (A) 30

(B) 32

(C) 62

(D) 63

(E) 64

9. In the triangle 6ABC, AB = AC, LABC = 40° and the point D is on AC such that BD is the angle bisector of LABC. If BD is extended to the point E such that DE= AD , find LECA. A

(A) 20°

(B) 30°

(C) 40°

(E) 50°

10. Let m and n be positive integers such that m > n. If the last three digits of 2012m and 2012n are identical, find the smallest possible value of m+ n. (A) 98

(B) 100

(C) 102

(D) 104

23

(E) None of the above

Short Questions

11. Let a b c and d be four distinct positive real numbers that satisfy the equations (a2m2 c2 o12)(a2012 d2012) 2011 and (b2o12 c2 o12) (b2o12 d2o12) 2011 Find the value of (cd ) 2 01 2 -(ab ) 2 012 . 12. Determine the total number of pairs of integers x and y that satisfy the equation 1 1 1 +2 3' 2X _

_

_

=

_

Y

=

Y

13. Given a set S 2 , a collectionF of subsets of S is said to be intersecting if for any two subsets A and B in F, we have A n B -=1 0. What is the maximum size ofF? =

1

10

14. The set M contains all the integral values of m such that the polynomial 2 ( m -1) x2 - ( m2 - m +12)x + 6m has either one repeated or two distinct integral roots. Find the number of elements of M.

15. Find the minimum value of

I

sin x+ cos x

+ coscosx -2xsin x I ----­

16. Find the number of ways to arrange the letters A, A, B, B, C, C, D and E in a line, such that there are no consecutive identical letters. x

17. Suppose x 3V2+Iog3 is an integer. Determine the value of x. 18. Let f(x) be the polynomial (x-a1 )(x-a2)(x-a3)(x-a4)(x-as) where a1 , a2, a3, a4 and a5 are distinct integers. Given that f (104) 2012, evaluate a1+a2+a3+a4+a5. 19. Suppose x, y, z and are positive real numbers such that yz 6-Ax xz 6-Ay xy 6-Az x2+y2+z2 1 Find the value of (xyz.A) -1 . =

=

A

=

=

=

=

24

20. Find the least value of the expression real numbers satisfying the equation

(x +y )(y +z) , given that x, y, z are positive xyz(x+y+z) 1. =

21. For each real number let be the minimum of the numbers 2012. 4. Determine the maximum value of -2

x+

x, f(x)

6f (x) +

4x+1, x + 2 and

22. Find the number of pairs (A, B) of distinct subsets of 1, 2, 3, 4, 5, 6 such that A is a proper subset of B. Note that A can be an empty set. 23. Find the sum of all the integral values of that satisfy

x

JX+3- 4 �+ JX+ 8- 6� 24. Given that for real values of

=

1.

I x2+ 4x +5- v'x2+ 2x+5 1 ,

S = v'

x, find the maximum value of 84.

25. Three integers are selected from the set S = 1, 2, 3, ... , 19, 20 . Find the number of selections where the sum of the three integers is divisible by 3. 26. In the diagram below, ABCD is a cyclic quadrilateral with AB = AC. The line FG is tangent to the circle at the point C, and is parallel to BD. If AB = and BC = 4, find the value of 3AE.

6

A

25

27. Two Wei Qi teams, A and B, each comprising of 7 members, take on each other in a competition. The players on each team are fielded in a fixed sequence. The first game is played by the first player _of each team. The losing player is eliminated while the winning player stays on to play with the next player of the opposing team. This continues until one team is completely eliminated and the surviving team emerges as the final winner - thus, yielding a possible gaming outcome. Find the total number of possible gaming outcomes. 28. Given that m ( cos + ( sin and n ( .J2- sin ( cos where and are the usual unit vectors along the x-axis and the y-axis respectively, and E ( 1r 2n ) . If the length or magnitude of the vector m n is given by m n 8'f, find the value of 5 cos (� + i) 5.

= B) i

B) j

=

+

B) i + B ) j , i j B + =

+

29. Given that the real numbers x, y and the maximum value of j ( x y v2 x

z satisfies the condition x +y + z = 3, find +13 +�3y +5+V'8z +12. 30. Let P ( x ) be a polynomial of degree 34 such that P(k) = k (k + 1 ) for all integers z) =

k=0 1 2

34. Evaluate 42840 x P ( 35) .

31. Given that a is an acute angle satisfying

v369 - 360 cosa +v544 - 480 sina - 25 = 0 find the value of 40 tana. 32. Given that and

a b c d e are real numbers such that a+b +c+d+e = 8 a2+b2+ c2+d2+e2 = 16

Determine the maximum value of L eJ. 33. Let L denote the minimum value of the quotient of a 3-digit number formed by three distinct digits divided by the sum of its digits. Determine L10 £J . 34. Find the last 2 digits of

35. Let

f (n) be the integer nearest to yfii. Find the value of

26

Singapore Mathematical Society

Singapore Mathematical Olympiad (SMO) 2012 Senior Section (First Round Solutions)

Multiple Choice Questions

1. Answer:

(D)

+

Note that a (3 =

-

(2V J2 +1) and a(3 J J2 +1. __!_ __!_ (a+ (3) 3- 3a(3(a + (3) + =

a3

(33

=

=

( af3) 3 s..)J2

+1 6vv'2+1 6(vv'2 +1)- s(J2 +1) v'2+1 6v,---.J'i- +-1 ( .J2 1) s -

2. Answer: (E)

27

-

3. Answer: ( C ) Note that 442 = 1936, 452 = 2025 and 462 = 2116. So 2, 3, . . . , 2012 has at most 2012 - 44 terms. For the 2012th term, we need to add the 44 numbers from 2013 to 2056. But in doing so, we are counting 452 = 2025, so the 2012th term should be 2012 + 44 + 1= 2057. 4. Answer: ( C )

B

is tangent to the circle at D, by constructing E and F as shown, we have CD= CF, AD= AE and BE= BF. Solving for the unknowns give CD= 5. AC

5. Answer: (D) cos6 x + sin6 x+ 4 sin2 x cos 2 x = ( cos 2 x + sin 2 x ) ( sin4 x + cos 4 x - sin 2 x cos 2 x ) + 4 sin 2 x cos 2 x = ( sin4 x+ cos4 x + 3 sin2 x cos 2 x ) . 6. Answer: ( B ) We have the factorization ( x2 + 3x - 1 ) ( x2 + mx - c) = x4 + ax 2 + bx + c.

Comparing coefficients give 3 + m= 0, -1 - c + 3m= a and -3c - m= b . We can solve these equations to obtain a+ b + 4c= -7. 7. Answer: ( C ) Among all numbers with 3 or less digits, each i, i= 0, 1, 2, , 9, appears exactly 300 times. Thus the sum of the digits of all the numbers in the sequence 1, 2, 3, 4, , 999 is 300 ( 1 + 2 + . . . + 9 ) = 13500, and so the answer is 13501. ·

·

·

·

28

·

·

8. Answer: (D) Since -1:::;; sin x:::;; 1, we have - 100:::;; x:::;; 100. We also observe that 317r < 100 < 327r. For each integer k with 1 :::;; k :::;; 16, 1 �0 = sin x has exactly two solutions in [2(k-1)7r (2k - 1)7r], but it has no solutions in ((2k - 1)7r 2k7r). Thus this equation has exactly 32 non-negative real solutions, i.e. x = 0 and exactly 31 positive real solutions. Then it also has exactly 31 negative real solutions, giving a total of 63. 9. Answer: ( C ) Construct a point F on EC such that EF= EA. Since LAED LFED, 6.AED is congruent to 6.FED. Thus DF = DA = DE and LFDE = LADE = 60°. We also have LEDC = LADE= 60°, which implies that LFDC= 60° and 6.CFD is congruent to 6.CED. In conclusion, LEGD= LFCD= 40°. =

A

E

10. Answer: (D) We want to solve 2012m

_

2012n (mod 1000) which is equivalent to

Since (12m-n - 1) is odd, we must have 8 12n so n 2:: 2. It remains to check that 125 12m-n - 1 i.e. 12m-n 1 ( mod 125). Let


29

Short Questions

11. Answer: 2011 Let A = a2 01 2 , B = b2 01 2 , C = c2 0 1 2 and D = d2 01 2 . Then A and B are distinct roots of the equation (x -C ) (x - D) = 2011. Thus the product of roots, AB = CD - 2011 and CD - AB = 2011. 12. Answer: 6 Note that x and y must satisfy 2x+ 1 · 3 = y(y + 2). We first assume x :;::: 0, which means both y and y + 2 are even integers. Either 3 y or 3 y + 2. In the first case, assuming y > 0, we have y = 3 · 2k and y + 2 = 2(3 . 2k-l + 1) = 2x+1 -k. The only way for this equation to hold is k = 1 and x = 3. So (x, y, y + 2) = (3, 6, 8). In the case 3 y+ 2, assuming y > 0, we have y+ 2 = 3· 2k and y = 2(3· 2k-1 - 1) = 2x+ 1 -k. Now the only possibility is k = 1 and x = 2, so (x, y, y + 2) = (2, 4, 6) . In the two previous cases, we could also have both y and y+ 2 to be negative, giving (x, y, y + = (3, -8, -6) or (2, -6, -4). Finally, we consider x < 0 so 3 = 2-x-ly(y + 2). In this case we can only have x = -1 and (x, y, y + 2) = (-1, 1, 3) or (-1, -3, -1). Hence possible (x, y) pairs are (3, 6) , (3, -8) , (2, 4) , (2, -6) , ( -1, 1) and ( -1, -3).

2)

13. Answer: 512 If A E F then the complement can belong to F, that is

S nA

fj. F. So at most half of all the subsets of

F:::;

S

210

2 = 512.

Equality holds because we can take F to be all subsets of S containing 1. 14. Answer: 4 If m = 1, the polynomial reduces to -12x + 6 = 0 which has no integral roots. For m-=/:- 1, the polynomial factorizes as ((2x - m) ( (m - 1)x - 6) , with roots x = r; and x = � . For integral roots, m must be even and m - 1 must divide 6. The m l values are m = -2, 0, 2 and 4. So M has 4 elements. only possible 15. Answer: 2 Note that cos 2x = cos2 x - sin2 x. So 1 cos x - sin x = sin x + cos x + . sin x + cos x + smx + cos x cos 2x Set w = sin x + cos x and minimize lw + �I· By AM-GM inequality, if w is positive then the minimum of w + � is 2; if w is negative, then the maximum of w + � is -2. Therefore, the minimum of lw + �I is 2.

I

I I

30



16. Answer: 2220 We shall use the Principle of Inclusion and Exclusion. There are ways to arrange the letters without restriction. There are ways to arrange the letters such that both the As occur consecutively. (Note that this is the same number if we use B or C instead of A.) There are � ways to arrange the letters such that both As and Bs are consecutive. (Again this number is the same for other possible pairs.) Finally there are 5! ways to arrange the letters such that As, Bs and Cs occur consecutively. For there to be no consecutive identical letters, total number of ways is

21�i21

2i�1

� - 3 X _2!_ + 3 X 212121

2121

17. Answer: 9 J2 log3 Taking logarithm, we get log3 J2 solution for is 2. Therefore

y = +y

'

6! - 5. - 2220

21

-

x = + 2x Let y = log3 x. x = 3 = 9.

The only possible

18. Answer: 17 The prime factorization of 2012 is 2 503. Let b 104. If ai are distinct, so are b- ai, i.e. (b- a1 ) , (b- a3 ) , (b- a4 ) and (b- a5 ) must be exactly the integers 1 -1 2 -2 503 . Summing up, we have

= b( -a2) , 2 5(104) - ( a1 + a2+ a3 + a4 + a5 ) = 1 - 1 + 2 - 2 + 503 ·

19. Answer: 54 Multiplying the first three equations by

x, y and z respectively, we have xyz = 6-\x2 = 6-\y2 = 6-\z2 Since,\# 0 and x2+y2+z2 = 1, we deduce that x2 = y2 = z2 = !,sox = y = z = }s and x2 1 2 Hence \

_

/\-

xyz 6 x2

_

-

x3

- 6V3" _

20. Answer: 2 Observe that 1 (x+y)(y +z) = xy +xz+y2+yz = y (x+y +z) +xz = -+ xz xz 2: 2 where the equality holds if and only if xz = 1. Let x = z = 1 and y = .J2- 1, then we have the minimum value 2 for (x + y )(y + z ) . 31

21. Answer: 2028 Let L1, L2, L3 represent the three lines y + 1, y + 2 and y + respectively. Observe that L1 and L2 intersects at G, D, L1 and L3 intersects at (�, 3), and L2 and L3 intersects at Thus

= 4x

=x

=-2x 4

(�, �) .

1.

X< x=r3 ' �f (x) = x 2, 3 <X< X=�-�' 3 ' -2x 4, X> Thus the maximum value of f (x ) is � and the maximum value of 6f (x ) 4x+ 1, 7

3' 8

3'

1

+



+

+

2028.

2012 is

22. Answer: Since B cannot be empty, the number of elements in B is between 1 to After picking B with k elements, there are 2k - 1 possible subsets of B which qualifies for A, as A and B must be distinct. Thus the total number of possibilities is

665

6.

t m (2.

_

1)

=t, m (2. 1) = t, m 2· - t, m =(2 1) 6 - (1 1) 6 =36 - 26 =665. _

+

+

23. Answer: The equation can be rewritten as (-,fx� - 2) 2 + Cv'x=-I. - 3)2 1. If -yX=l 2: 3, it reduces to -yX=l - + -yX=l - 3 1 i.e. -yX=l 3 giving 10. If -vX=1 ::;; 2, it reduces to 2 - -yX=l + 3 - -yX=l 1 i.e. -yX=l 2 giving

45

J

J

= 2 = = X= = = X=5. If 2 <-yX=l<3, i.e. 5<x <10, it reduces to -v'X=l - 2 3 - -yX=l = 1 which is true for all values of x between 5 and 10. Hence the sum of all integral solutions is 5 6 7 8 9 10=45. 24. Answer: 4 2) 2 (0 - 1)2 -y'(x 1) 2 (0 - 2)2 1 . =y'(x I +

+

S

+

+

+

+

+

32

+

+

+

Let P = 0) , A = and B = then S represents the difference between the lengths PA and PB. S is maximum when the points P, A and B are collinear and that occurs when P = 0). So

(x,

(-2, 1)

(-1, 2) ,

(-3,

Thus the maximum value of S 4 =

4.

25. Answer: 384 Partition S into three subsets according to their residues modulo 3: S0 = 3, 6, . . . , 18 , = 1, 4, . . . , 19 and = 2, 5, . . . , 20 . In order for the sum of three integers to be divisible by 3, either all three must belong to exactly one or all three must belong to different Hence total number of such choices is (�) + 2 G) + 6 7 7 = 384. 26. Answer: 10 82

81

Si

si.

x

x

Since BD II FG and FG is tangent to the circle at C, we have LBCF = LOBE = LDCG = LBDC = LBAC. Furthermore LBEC = LBAC+ LABE = LOBE+ LABE = LABC = LACE. We can then conclude that BE = BC = DC = t:.DCE. If we let AE =

4. Also, f:.ABE is similar to DE AE 2 ===> DE = 3x. = DC AB By the Intersecting Chord Theorem, AE · EC = BE· ED, i.e. x(6- x ) = 4(�x ) , which gives x = so 3AE = 3x = 10. 27. Answer: 3432 x,

1 3°,

We use a1, a2, , a7 and b1, b2, , b7 to denote the players of Team A and Team B, respectively. A possible gaming outcome can be represented by a linear sequence of the above terms. For instance, we may have a1a2b1b2a3b3b4b5a4b5b7a5a6a7 which indicates player followed by player from Team A were eliminated first, and the third player eliminated was player from team B. However Team A emerged the final winner as all seven players of Team B gets eliminated with a5, a6 and a7 remaining uneliminated. ( Note a6 and a7 never actually played. ) Thus, a gaming outcome can be formed by choosing out of possible positions for Team A, with the remaining filled by Team B players. Therefore, the total number of gaming outcomes is given by C 74) = c�D'2 = •

14







1





1

2

7 14 3432.

33

28. Answer: 1

8� =

m

=V4 + 2 J2 cosB - 2 J2 sinB = j2 1+ cos ( �) = 2 J2 I cos (� + i) I

+n

() +

Since 1r < () < 27r, we have �7f < � + i < �7f. Thus, cos (� + i) < that cos (� + i) -� and hence

=

29. Answer: Using the

f(x

0. This implies

5 cos (� + i) + 5 = 1

8

AM-GM

inequality, we have

y z) =v2x + 13+{!3y + 5+�8z + 12 =v 2x ; 13. V4+�3y: 5 . {12. {12+«8z; 12 . � . � . � 2x+13 + 4 3y+5 + 2 + 2 8z+12 + 2 + 2 + 2 < + + ----8--"' 2 4 3 1 29 = 4 (x+ y+ z)+ 4 4

4

=8

The equality is achieved at x

_____

= �, y = 1 and z = �.

30. Answer: Let n 34 and Q(x) (x+ 1)P(x)-x Then Q(x) is a polynomial of degree n + 1 and Q(k) Thus there is a constant C such that

40460

=

=

= 0 for all k = 0 1 2

Q(x) (x+ 1)P(x)-x Cx(x- 1)(x- 2 ) (x- n ) Letting x -1 gives 1 C(-1)( -2 ) ( -n - 1) C(-1)n+l (n+ 1)! Thus C (-1)n+l ( n+ 1)! and 1_ (x+Q(x))__1_ x+ (-1)n+lx(x- 1)(x- 2 ) (x- n ) P(x)__ - x+ 1 x+ 1 ( n+ 1)1.

=

=

·

·

·

·

·

·

=

=

=

So

·

·

=

·

(

·

·

·

)

1 n (-1)n+1(n+1)1 = 1 n+1+(-1t+l = -P(n+1) = -( n+ 1+ ) ) --( " n+ 2 (n+ l ) ! n+ 2 n+ 2 34

n.

since n

= 34 is even. Hence 42840 X P(35) = 34 X 35 X 36 X 3436=40460

31. Answer: 30 v'369 - 360 cos a and Let

a.

Y = v'544- 480 sin Observe that X2 = 122+152-2(12)(15) cos Y2 = 122+ 202-2(12)(20) cos (90°- ) and 152+ 202 = 252 = (X+ Y) 2, so we can construct a right-angled triangle ABD X=

a

a

as shown.

A

X

c y

15 2

B"---D ---=--____, In particular LABC a and LCBD fact similar to 6ABD. So LAD B a and

=

a.

= 90°- We can check that 6ACB is in = 40tana = 40 1520 = 30 x

32. Answer: We shall apply the following inequality:

3

4(a2+b2+c2+d2) � (a+b+c+d) 2 Since a+b+ c +d = 8-e and a2+ b2+ c2+ d2 = 16-e2, we have 4(16-e2) � (8-e) 2 i.e. e(5e -16) :::; 0. Thus 0 :::; e :::; 16 5. Notethatifa = b = c = d = 6 5,wehavee = 16 5. Hence LeJ = 3. 33. Answer: 105 A three-digit number can be expressed as lOOa+lOb+c, and so we are minimizing +lOb+ c F(a b c) = lOOaa+b+c 35

Observe that with distinct digits Thus we assume that Note that

a b c, F(a b c) has the minimum value when a< b
_

where the last congruence can be calculated by the extended Euclidean algorithm. Thus by repeated squaring, we have

19a 1933 59 _

(mod

100)

35. Answer: Note that (n+ � ) 2 n2 � ' so f(n2 n but f(n2 So each of the sequences (n - f(n) )�=1 ) and (n f(n))�=1 ) increases by for every increment of n by except when n If n 2 and (n we have n - f(n) so the former sequence has f(n every perfect square repeated once. On the other hand, if n we have n f(n) so the latter sequence but (n f(n omits every perfect square. Thus

5

= +n+

+n+1 = n+1. +n = = (0 1 1 2 ) + 2 ) = (2 3 5 26 1 1, = m +m. = m +m, =m + 1- + 1)) = m2, 2+ m, = m = m2+2m + 1+ + 1)) = m2+2m+ 2,

+

oo



G) f(n) + (�) -f(n) (�r

( �) + (�) � 3 � ( 3) () () -I: 3 + I: 3 + I: 3 - I: ( 3) oo

n-f(n)

oo

n+ f(n)

_

-

oo

_

2 n

00

m=1

n=O

=5

36

2 2 m

oo

n=1

2 n

oo

m=1

2 2 m

Singapore Mathematical Society Singapore Mathematical Olympiad (SMO) 2012

(Senior Section, Round 2) 0900-1300

Saturday, 23 June 2012

1. A circle

w

through the incentre I of a triangle ABC and tangent to AB at A, intersects the segment BC at and the extension of BC at Prove that the line IC intersects w at a point M such that

D

E.

MD=ME . 2. Determine all positive integers n such that n equals the square of the sum of the digits of n. 3. If 46 squares are colored red in a 9 9 board, show that there is a 2 2 block on x

x

the board in which at least 3 of the squares are colored red.

an an+l be a finite sequence of real numbers satisfying ao an+l and ak-1- 2ak ak+l

= =0 + Prove that fork = 0 1 n+1, ak 5.

Prove that for any real numbers a

<

=:;

1

for

k=1 2

n

c

=

k(n + 1-k) 2

c

b d 2:: 0 with a + b= +d

37

2,

Singapore Mathematical Society Singapore Mathematical Olympiad (SMO) 2012

(Senior Section, Round 2 solutions)

Join AD, ID, IA and AE. Let IE intersect AC at N. We have LIAN=LIAB= LIEA so that the triangles NIA and AlE are similar. Thus LAN!=LEAl=LIDB. Also LDCI = LNG!. Therefore, the triangles DC! and NCI are congruent. Hence LDIC=LNIC implying MD=ME.

1.

M

Let s ( n ) denote the sum of all the digits of n. Suppose n is a positive integer such that s ( n ) 2 = n. Let s ( n) =k so that n =k2 . Then s (k2 ) = s ( n ) =k. Let 10r-1 � k < lOr, where r is a positive integer. That is k has exactly r digits. From 10r-1 � k, we have r � logk + 1. From k < 10r, we have k2 < 102r so that k2 has at most 2r digits. Therefore, s (k2 ) � x 2r =18r � 18 logk + 18 which is less than k if k 2 50. Thus the equation s (k2 ) =k has no solution ink ifk 2 50. Let k < 50 and s (k2 ) =k. Taking mod we get k2 k ( mod Thus k- 0, 1 ( mod That is k =1, 10, 18, 19, 27, 28, 36, 37, 45, 46. Only when k =1 and k = we have s (k2 ) =k. The corresponding solutions for n are n=1 or 81. 2.

9

9) .

9,

9,

3. Suppose that at most 2 squares are colored red in any 2

9).

x

9,

2 square. Then in any x 2 block, there are at most 10 red squares. Moreover, if there are 10 red squares, then there must be 5 in each row.

9

38

Now let the number of red squares in row i of the 9 x 9 board be ri. Then ri+ri+1 ::; 10, 1:::; i:::; 8. Suppose that some ri :::; 5 with i odd. Then (r1 +r2)+· · ·+(ri-2+ri-1)+ri+· · · +(rs+rg):::; 4 x + 5 = 45. On the other hand, suppose that r1, r3, r5, r7, rg 2: 6. Then the sum of any 2 consecutive ri's is :::; 9. So (r1+r2)+· · ·+(r7+rs)+rg :::; 4 x 9+9 = 45.

10

=

k n 1- k Let bk = ( -+;, ). Then bo bn+l = and bk-1 - 2bk+ bk+l = -1 for k = 1, 2 . .. , n. Suppose there exists an index i such that ai rel="nofollow"> bi, then the sequence a0 b0, ..., an+1-bn+1 has a positive term. Let j be the index such that aj-1-bj-1 < aj-bj and aj- bj has the largest value. Then (aj-1- bj-1)+(aj+l- bj+1) < 2(aj- bj)· Using ak-1- 2ak+ak+1 2: -1 and bk-1- 2bk+bk+1 = -1 for all k we obtain (aj-1- bj-1)+(aj+1- bj+1) 2: 2(aj- bj) a contradiction. Thus ak:::; bk for all k. Similarly, we can show that ak 2: -bk for all k and therefore ak :::; bk as required. 4.

0

5.

First note that (ae + bd)(ad +be) 2: (ab- ed)2. To see this, we may assume a 2: e 2: d 2: b since a+b = e+d. Then ed- ab 2: Also we have the two obvious inequalities ae+bd 2: ed- ab and ad+be 2: ed-ab. Multiplying them together we get (ae + bd)(ad+be) 2: (ab- cd)2. Next (a2+e2)(a2+d2)(b2+e2)(b2+d2) 2 = ((ae+ bd)(ad+be)- (ab- ed)2) +(ab- ed)2 + ((a+b)2(e+d)2- 1) (ab- ed)2 2 = ((ae+ bd)(ad+be)- (ab- ed)2) +16(ab- ed)2

0.

<; =

cac+bd :ad+be)' - (oh- cd)2r +16(ab- cd)2 ca b)'�c+d)' - (ab- cd)2r+16(ab- cd)'

by AM-GM

+

(4 - (ab- ed)2)2+16(ab- ed)2. This final expression is an increasing function of (ab- cd)2. The largest value of (abed)2 is 1 when (a, b, e, d) = (1, 1, 2), (1, 1, 2, 2, 1, 1), (2, 1, 1). Consequently, 2 (4 - (ab- cd)2) +16(ab- ed)2:::; 25, proving the inequality. =

0,

0 ), (0,

39

0,

Singapore Mathematical Society Singapore Mathematical Olympiad (SMO) (Open Section, First Round)

Wednesday, 30 May 2012

2012

0930-1200 hrs

Instructions to contestants

1. Answer ALL 25 questions.

2. Write your answers in the answer sheet provided and shade the appropriate bubbles below

your answers.

3. No steps are needed to justify your answers.

4. Each question carries 1 mark. 5. No calculators are allowed.

PLEASE DO NOT TURN OVER UNTIL YOU ARE TOLD TO DO SO

40

Throughout this paper, let LxJ denote the greatest integer less than or equal to x. For example, ) 2 . 1 = 2 , 3.9 = 3 This notation is used in Questions 2 , 10, 16, 1 7, 18 and 22 .

L J

L J (

1 . The sum of the squares of 50 consecutive odd integers is 300850. Find the largest odd integer whose square is the last term of this sum. 1000 2. Find the value of

L Llog2 kJ . k=3

. 2 ) 3 . Given that f x is a polynomial of degree 201 2 , and that f k = k for k = 1 , 2 , 3, ·

(

x

find the value of 2014

()

( )

f 2014 .

(

· ·,

2013,

)

4. Find the total number of sets of positive integers x , y, z , where x , y and z are positive integers, with x < y < z such that x + y + z = 203. 5. There are a few integers n such that n 2 + n + 1 divides n 2 01 3 + 6 1 . Find the sum of the squares of these integers.

( ) 6. It is given that the sequence an �= l ' with a 1 = a2 = 2 , is given by the recurrence relation

for all n = 2 , 3, 4,

· · ·.

Find the integer that i s closest t o the value of

201 1 a + l k L2 a k . k=

7. Determine the largest even positive integer which cannot be expressed as the sum of two composite odd positive integers. 8 . The lengths of the sides of a triangle are successive terms of a geometric progression. Let A and C be the smallest and largest interior angles of the triangle respectively. If the shortest side has length 16 em and sin A - 2 sin B + 3 sin C sin G - 2 sin B + 3 sin A find the perimeter of the triangle in centimetres.

19 9'

9 . Find the least positive integral value of n for which the equation x +X +

y �

has integer solutions x 1 , x 2 , x 3 , · ·

(

···

+ X = 200 2 2 00 2

· , Xn ·

)

41



an be a real root of the cubic equation nx 3 + 2x - n

= 0, where n is a positive integer. 01 3 2 1 If f3n= l( n + 1)anJ for n= 2 3 4 · · · , find the value of f3k 1006 L · k=

10. Let

2

1 1 . In the diagram below, the point D lies inside the triangle such that and 90° . Given that 5 and 6, and the point M is the midpoint of find the value of 8 x DM 2 .

AC,

LBDC=

AB=

BC=

ABC

LEAD= LBCD

1 2 . Suppose the real numbers x and y satisfy the equations x 3 - 3x 2 + 5x

=1

Find x + y .

and y 3 - 3y 2 + 5y

=5

13. The product of two of the four roots of the quartic equation x4 - 18x 3 +kx 2 +200x- 1984 is -32. Determine the value of k.

=0

14. Determine the smallest integer n with n ?: 2 such that

is an integer. 1 5 . Given that f is a real-valued function on the set of all real numbers such that for any real numbers a and b, f af b ab

( )

Find the value of f 201 1 .

( ( ))=

16. The solutions to the equation x 3 - 4 x

l J= 5, wherek x is a real number, are denoted by XI X 2 X 3 . . . X k for some positive integer k. Find L x r . i=l

1 7. Determine the maximum integer solution of the equation

= 1001 + l !E_1 ! J + l !E_J l !E_3 ! J + . . . + l _!!_ 10! _ J 2! 42

18. Let A, B , C be the three angles of a triangle. Let L be the maximum value of sin 3A + sin 3B + sin 3C . Determine l10LJ. ( 19. Determine thex2number of sets of solutions x , y, z) , where x , y and z are integers, of the 2 2 2 2. + the equation

y +z

=

x y

20. We can find sets of 13 distinct positive integers that add up to 2142. Find the largest possible greatest common divisor of these 13 distinct positive integers. 21. Determine the maximum number of different sets consisting of three terms which2 form an arithmetic progressions that can be chosen from a sequence of real numbers a1 , a , · · · , a1 01 , where

22. Find the value of the series

� l 20121 + 2 k L...J

k=O

2 k+ 1

j.

23. The sequence (xn )�=1 is defined recursively by X n+ l with x1

=

=

(2 - J3) 1 (2 - J3) Xn + - Xn

'

1. Determine the value of X1001 - X401 ·

24. Determine the maximum value of the following expression

- ' ' -$ 1 - X 2 - X 3 - X4 - ' ' - X 2 0 1 4 -

where x1 , x 2 , · · · , x 2 014 are distinct numbers in the set

1 00 6 . -1 ). 25. Evaluate 22011 L ( -1) k 3k (2012 2k k=O

43

-1, 2, 3, 4, · · · , 2014---:

Singapore Mathematical Society Singapore Mathematical Olympiad (SMO) 2012 (Open Section, First Round Solutions)

1. Answer: 121 Let the integers be X + 2 X + 4

·· ·

100. Then ( X + 2) 2 + ( X + 4 )2 + + ( X + 100) 2 = 300850 Let y = X + 51 and regrouping the terms, we obtain [ (y - 49 )2 + (y + 49 )2 ] + [ (y - 47) 2 + (y + 47) 2 ] + . . . + [ (y - 1) 2 + (y + 1) 2 ] = 300850 which simplifies to 50y2 + 2 ( 1 2 + 32 + 52 + 72 + . . . + 49 2 ) = 300850 . the fact that 1 2 + 32 + 52 + · · · + (2n - 1) 2 = 34 3 - 31 n, we obtam. y = 72. Hence Usmg X = 21, so that the required number is 121.

Solution.

X+

···

n

D

2. Answer: 7986 Note that 2k+1 - 2k = 2k , and that 2k t 2k+ 1 if and only if llog2 tJ = k. Then the requires sum (denoted by S) can be obtained by 1 023 = 2: 2: llog2 tj + llog2 3j - L llog 2 tj k(=2 2k 9<2)k+l 10 {; k2k + 1 - �23 9 8192 + 1 - 23(9) = 7986 :::;

Solution.

<

9

s

t=lOOl

9

t

l

D

3. Answer: 4 Let g(x) = xf(x) - 2, hence g(x) is a polynomial of degree 2013. Since g(l) = g(2) = g(3) = = g(2013) = 0, we must have g(x) = >.(x - l)(x - 2)(x - 3) (x - 2012)(x - 2013) 2 Hence, for some >.. Also, g(O) = -2 = ->. 2013 ! , we thus have >. = 20131 g(2014) =20�3! (2013! ) = 2014 . f(2014) - 2 concluding that 2014 f(2014) = 4 Solution.

···

···

·

·

D

44

4.

Answer: 3333 . . mteger . sets = 20301 positive First note that there are (2022 ) = 202(201) 2 ( x, y, z) which satisfy the given equation. These solution sets include those where two of the three values are equal. If x = then 2x+z = 203. By enumerating, z = 1, 3, 5, , 201. There are thus 101 solutions of the form (x, x, z). Similarly, there are 101 solutions of the form (x, y, x) and ( x, y, y). Since x y z, the required answer is

Solution.

y,

·

<

·

·

<

D

5. Answer: 62 Since n 3 - 1 = (n7 - 1)(n2 + n + 1), we know that n2 + n + 1 divides n3 - 1. 2 0 13 Also, since n - 1 = (n3 ) 6 1 - 1, we also know that n2 + n + 1 divides n20 13 - 1. As 2 2 n 0 1 3 + 61 = n 0 1 3 1 + 62, we must have that n2 + n + 1 divides n20 13 + 61 if and only if n2 + n + 1 divides 62. Case (i): If n2 + n + 1 = 1, then n = 0, -1. Case (ii): If n2 + n + 1 = 2, there is no integer solution for n. Case (iii): If n2 + n + 1 = 31, then n = 6, -5. Case (iv): If n2 + n + 1 = 62, there is no integer solution for n. Thus, all the integer values of n are 0, -1, 6, -5. Hence the sum of squares is 1 + 36 + 25 = 62. Solution.

_

D

6. Answer: 3015 The recurrence relation can be written as l a:: - a:: l = ( n � 1 - �) - ( � - n : 1 ) Summing for n = 2 to N, we obtain Solution.

showing that

·

3 1- 1 . -;;:;,;- - 2 - ( N N + 1 ) Summing this up for N = 2 to N = 2011, we obtain 2 0 11 -'1-1 � (2010) - ( � - -1- ) = 3014. 5 + -1ak S = '"""' = � ak 2 2 2012 2012 showing that the integer closest to S is 3015. aN + l

_

45

D

7. Answer: 38 Let n be an even positive integer. Then each of the following expresses n as the sum of two odd integers: n = (n - 15) + 15 (n - 25) + 25 or (n - 35) + 35. Note that at least one of n - 15 n - 25 n - 35 is divisible by 3 so that it is composite, and hence n can be expressed as the sum of two composite odd numbers if n 38. Indeed, it can be verified that 38 cannot be expressed as the sum of two composite odd positive integers. Answer: 76 2 Let the lengths of the sides of the triangle in centimetres be 16, 16r and 16r 2 (where r 1). Then 1r2--2r2r++3r3 -199 so that r = -32 . Hence, the perimeter of the D triangle = 16 ( 1 + � + � ) = 76cm 9. Answer: 4 Since 2002 4(mod9) 43 1(mod9) and 2002 = 667 3 + 1, it follows that 20022002 4667x3+ 1 4(mod9). Observe that for positive integers x, the possible residues modulo 9 for x3 are 0 ±1 . Therefore, none of the following X� X� + � � + X� + � can have a residue of 4 modulo 9. However, since 2002 = 103 + 103 + 13 + 13, it follows that 20022002 2002 . (2002667 ) 3 (103 + 103 + 1 3 + 1 3 ) (2002667) 3 (10 . 2002667 ) 3 + (10 . 2002667 ) 3 + (2002667 ) 3 + (2002667 ) 3 This shows that x� + x� + x� + x! = 20022002 is indeed solvable. Hence the least integral value of n is 4. 10. Answer: 2015 Let f(x) = nx3 + 2x - n. It is easy to see that f is a strictly increasing function for n = 2 3 4 Further, 1 ( n : 1 ) = n ( n : 1 ) 3 + 2 ( n : 1 ) - n =(n ; 1)3 ( -n2 + n + 1 ) < O for all n 2. Also, f(1) = 2 Thus, the only real root of the equation nx3 + 2x-n = 0 for n 2 is located in the interval n(�1 1 ) . We must have n < an < 1 n < (n + 1 )an n + 1 -n+1 so that f3n = l(n + 1)anJ = n for all n 2. Consequently, -1- "201(33 k =-1- "2013 =-1- . 2012 (2 + 2013) = 2015 1006 k=2 1006 k=2 1006 2 Solution.

>

D

8.

Solution.

>

Solution.

=

=

x

=

=

X

X

X

D

·

2:

·

· .

2:

>

0.

==?

<

2:





k

D

46

1 1 . Answer: 22 Solution.

Extend CD to E such that CD = DE . ' ' ', ' '', ' ' ' ', ' ' ' ' ' ' ' ' ' ' \ ' \ ' ' \

,,

,

' ' ' ' \ '

�-------=

c

6

It is clear that D.CDB and D.EDB are congruent . Hence EB = CB = and L_BED L_BCD . Thus, L_BED = L_BCD = L_BAD implies that the points B , D , A are E are concyclic. Given that L_BDC = goo , hence L_EDB = goo . BDAE is a cyclic quadrilateral with EB as a diameter. Thus, L_EAB = goo . In the right-angled triangle EAB , we have

Since D and M are the midpoints of EC and AC respectively, DM = 8 x DM2 = 22 .

1 2 . Answer: 2 Solution.

2 From x 3 - 3

2 and from y 3 - 3 y

x + 5x

+ 5y

=

=

(

=

� AE = �· Thus, D

1 , we have

x - 1 ) 3 + 2 (x - 1 ) = -2,

5 , we have

) y-1 3

+ 2 (y - 1 ) = 2 . Thus ) ( ) ) ) ( 0 ( x - 1 3 + 2 (x - 1 + y - 1 3 + 2 y - 1 )2) )( ) ( )2 = (x + y - 2 ) ( (x - 1 - (x - 1 y - 1 + y - 1 2 + (x + y - 2 ) )2) )( ) ( )2 = (x + y - 2 ) ( 2 + ( x - 1 - (x - 1 y - 1 + y - 1 . For any real numbers x and y, we always have ? )( ) ( )2 0 y-1 (x - 1 - (x - 1 y - 1 and thus x + y - 2 = 0, implying that x + y = 2 . (

=

+

47



D

13. Answer: 86 Let a, b, c, d be the four roots of x 4 - 18x 3 + kx 2 + 200x - 1984 = 0 such that ab = -32. Then a + b + c + d = 18, ab + ac + ad+ be + bd + cd = k, abc + abd + acd + bed = -200, abed = -1984. Since ab = -32 and abed = -1984, we have ed = 62. Then, from abc + abd + acd + bed = -200 we have -200 = -32c - 32d + 62a 62b = -32(e + d) + 62(a + b). Solving this equation together with the equation a + b + c + d = 18 gives that a + b = 4, c + d = 14. From ab + ac + ad + be + bd + ed = k, we have

{

Solution.

+

= 30 + (a + b)(c + d) = 86.

D

14. Answer: 337 Solution.

Assume that

and so

+

(n 1)(2n + 1) = 6m2 . Thus 6 (n + 1)(2n + 1), implying that n 1 or 5 ( mod 6). = 6k + 5 . Then m 2 = (k + 1)(12k + 11). Since (k + 1) and (12k + 11) are relatively prime, both must be squares . So there are positive integers a and b such that k + 1 = a2 and 12k + 11 = b2 . Thus 12a 2 = b2 1. But, as 12a 2 O ( mod 4) and b2 + 1 1 or 2 (mod 4), there are no integers a and b such that 12a2 = b2 + 1. Hence Case 1 cannot happen. n = 6k + 1. Then m 2 = (3k + 1)(4k + 1). Since 3k + 1 and 4k 1 are relatively prime, both must be squares. So there are positive integers a and b such that 3k + 1 = a 2 and 4k + 1 = b2 . Then 3b2 = (2a - 1)(2a + 1). Observe that in the left-hand side, every prime factor except 3 has an even power. So neither 2a - 1 nor 2a + 1 can be a prime other than 3. Now we consider positive integers a such that neither 2a - 1 nor 2a + 1 can b e a prime other than 3. If a = 1, then b = 1 and n = 1. So we consider a 2. The next smallest suitable value for a is 13. When a = 13, we have 3b2 = 25 27 and so b = 15, implying that k = 56 and so = 6k + 1 = 337. =

Case 1: n

+

=

=

Case 2 :

+

2:::

X

n

48

D

15. Answer: 2011

f(f(1)f(b))

f(b)

f(b)

From the recurrence relation, = f( 1 ) b and f(f(b)f( 1 ) ) = 1. Hence, = f( 1)b. By letting = / ( 1 ) , we obtain f(f( 1 ) ) = (f( 1 ) ) 2 . Also, from the given functional equation, we have f(f( 1 ) ) = 1, hence (! ( 1 ) ) 2 = 1 , following that / ( 1 ) is either 1 or - 1 . D Hence f(201 1 ) = 201 1 .

b

·

16. Answer: 19 <



2::

x - 1 LxJ x . Note that if x 3, there will be no solution as x3 - 4LxJ x3 - 4x = x(x2 - 4) 3(5) = 15. Also, if x - 2 , there will be no solution as x 3 -4 L xJ x3 -4(x - 1) = x(x 2 - 4) + 4 4 . Hence the solution must be in the interval ( - 2 , 3) . If LxJ = - 2 , then x3 = -3, giving x = N, which is a solution. If LxJ = - 1 , then x3 = 1, giving x = 1 which contradicts with LxJ = - 1 . LxJ = 0, then x3 = 5, hence there is no solution. If LxJ = 1 , then x3 = 9 . Since 2 W 3, there is no solution. If LxJ = 2, then x3 = 13. Since 2 m 3, X = m is a solution. D Thus, the required answer is -3 + 13 = 10. 17. Answer: 584 Solution.

Note that

2::

2::



<

If

<

<

<

Solution.



<

It is clear that

l � J + l;,J + l:,J + . . . + l 1�! J

x,

x

is a monotone increasing function of and when = 6 ! , the above expression has a value larger than 100 1 . Thus each solution of the equation

l �,J + l;,J + l:,J + . . . + l 1� ! J = 100 1

x is a solution, then l �,J + l;,J + l:,J + . . . + l 1� ! J = l �,J + l;,J + l :, J + l�J + l:,J .

is less than 6! and so if

As

x

<

6 ! , it has a unique expression of the form X

+ b 4! +

C X

X

C X

X

X

e,

+ d 2! + 1 . Note that where b, c, d, are non-negative integer with a 5, b 4, c 3, d 2 , if 3! + d 2 ! + X = a 5! + b 4! + then l�, J + l;,J + l:,J + l :,J + l � J = 206a + 41b + 10c + 3d + Since 41b + 10c + 3d+ 201 , we have 800 206a 1001 and so a = 4 . Thus 41b + lOc + 3d + = 1 77, X=a

a,

e

X

5!

3!





X





e



e,

e.

e







e

49

which implies that

c

b = 4 and so on, giving that = d = 1 and X

=4

X

5! + 4

X

4! + 1

X

3! + 1

X

e

= 0. Thus

2 ! + 0 = 584.

As 584 is the only integer solution, the answer is 584. D 18. Answer: 25 Solution.

We shall show that -2 ::::; sin 3A + sin 3B + sin 3C ::::; 3v'3/2.

Assume that A ::::; B ::::; C. Then A ::::; 60° . Thus sin 3A 2:: 0 . It is clear that sin 3B 2:: - 1 and sin 3C 2:: - 1 . Thus sin 3A + sin 3B + sin 3C 2:: -2. Let B = C. Then B = C = goo - A/2. If A is very small, B and C are close to goo , and thus sin 3A + sin 3B + sin 3C is close to -2. Now we show that sin 3A + sin 3B + sin 3C ::::; 3v'3/2. First the upper bound can be reached when A = B = 20° and C = 140° . Let X = 3A,

Y

= 3B and Z = 3(C - 120° ) . Then X + Y + Z = 180° and sin 3A + sin 3B + sin 3C = sin X + sin Y + sin Z.

Suppose that X, Y, Z satisfy the condition that X + Y + Z = 1 80° such that sin X + sin Y + sin Z has the maximum value. We can then show that X = Y = Z. Assume that

X ::::; Y ::::;

Z. If X < Z, then

X-Z . . . X+Z . X+Z sm X + sm Z = 2 sm cos < 2 sm , 2 2 2 implying that . . X+Z . X+Z . . y+� . y+� z < � +� �X + � 2 2

which contradicts the assumption that sin X + sin Y + sin Z has the maximum value. Hence X = Y = Z = 60° , implying that A = 20° , B = 20° and C = 140° and sin 3A + sin 3B + sin 3C = 3v'3/2. Since

y'3



1 . 732 , the answer is then obtained.

D

1 g . Answer: 1 Solution. Note that x = 0, y = 0 and z = 0 is a solution of this equation. We shall show that this is its only integer solution by proving that if x, y, z is a solution of this equation and whenever x, y, z are divisible by 2k , they are also divisible by 2k + 1 for any k 2:: 0.

Let

x

=

2k x' , y

= 2ky' and

z

= 2k z ' . Then

x2

+ y2 + z2 =

x2 y 2

is changed to

It is easy to verify that only when x' , y' , z ' are all even, x'2 + y '2 + z 12 and the same remainder when divided by 4. Thus x, y, z are divisible by 2 k + 1 .

50

2 2 k x12 y'2

have D

20. Answer: 21

Let d be the greatest common divisor ( gcd ) o f these 13 distinct positive integers. Then these integers can be represented as da1 , da2 , · · · , da13 , where gcd ( a1 , a2 , · · · , a13 ) = Let S denote a1 + a2 + · · · + a13 · Then Sd = In order for d to be the largest pos­ sible, S must be the smallest . Since S ;::: + + 3 + · · · + 3 = and that S divides and that = x 3 x 7 x the smallest possible value of S can be x = and the largest value of d is thus By choosing ( a1 , a2 , a3 , · · · , a12 , a13 ) = 2, 3, · · · , D we conclude that d = is possible. Solution.

13

1.

2142 2

1 2

51, 21.

21

2142.

1 91

2142, 2 51 102, (1, 12, 24),

21. Answer: 2500 Solution.

First , for the following particular sequence, there are really three-term arithmetic progressions which can be chosen from this sequence: They are s , and t with

s

s,

1, 2, 3, . . . ' 101. i with 1 i 51 and 2i - i, �

s

<



2500 different

i, 2i - for all integers for all integers i 52 i 101. Now we show that for any given sequence of real numbers a1 a2 aw1 , there are at most 2500 different three-term arithmetic progressions which can be chosen from this sequence. Let as , ai , at represent a three-term arithmetic progression. It is clear that 2 i 100. If i 51, then the first term as has at most i - 1 choices, as must be an index in 1, 2, i - 1 . If 52 i 100, then the third term at has at most 101 - i choices, as must be an index in i + 1, i + 2, , 101 . 2 �



< t �



···,

t,

<



< ··· <



s



t



t

· ··

So the number of different three-term arithmetic progressions which can be chosen from this sequence is at most 1 00 51 + = + 2 + . . . + 5o + + 2 + . . . + = 25oo . 2 2 i=5 i= D

I)i - 1) L (101 - i) 1

22. Answer: 20121 Solution.

Write

x

:= x

x

- L J . Then

1

49

if !!C2 < 12 otherwise

Thus, we have

Applying the above result for

x=

;,

f ( l ; J - l 2::1 J ) k=O l;J

In particular, when

n=

n.

20121, the infinite series converges to 20121. 51

D

23 . Answer: 0 Solution.

Let X n = tan O: n . Since 2 -

X n+I = tan a n + I =

-J3 = tan {;( ) , it follows that

(

tan an + tan ( � ) 1r = tan O:n + 1f 12 1 - tan an tan ( I 2 )

)

( So, X n+ l 2 = tan an + 1r ) = tan an X n , implying that this sequence has a period of 1 2 . Observe that 1 0 0 1 = 5 (mod 12) and 401 = 5(mod 12) . Consequently, =

Xl QO I - X40 1 = X5 - X5 = 0 D 24. Answer: 2013 Solution.

be 2014, as

First it is clear that the answer is an integer between 0 and 2014. But it cannot X I - X 2 - X 3 - X4 X 20 I4 and ·

·

·

·

X I + X2 +

·

·

·

·

·

-

+ X 2 0 I4 = 1

2+

+

·

·

·

+ 2014 = 1007

X

2015

have the same parity. Now we j ust need to show that 2013 can be achieved . For any integer k , ( 4k + 2) - ( 4k + 4) - ( 4k + 5) - ( 4k + 3) = 0 Thus 2 - 4 - 5 - 3 - . . . - ( 4k + 2) - ( 4k + 4) - ( 4k + 5) - ( 4k + 3) -2010 - 2012 - 2013 - 201 1 - 2014 - 1 =

0 - 2014 - 1

=

2013 D

25. Answer: 1 Solution. Consider the complex number binomial theorem one has

(

1f

1f

w

= cos

J + i sin J . On one hand, using the

) 20 I 2

Re cos 3 + i sin 3 20 I2 Re + i -;

( � V3)

(�)

c ) (�) ( ��::: ) [ ( ) ( )

2 0I 2

_

20 I O (; ) 2

o12 2

+...+

2012 1 1-3 2 2 I 0 2 2

_

20 12 + 4

( � ) ( �: ) 2 00 8

( )]

3 2 2012 + . . . + 3I 006 2012 + 201 2 4

52

On the other hand, using the De Moivre's theorem one has

( 20121f

Re ( w2 0 1 2 ) = Re cos Thus ,

+

i sin

20121f ) 3

_1_[1 - (2012) (2012) . . . 4 2 2012 (20124 ) . . . � [1 - ( 2 )

2 20 1 2 so that

3

2 20 11

3

+

3

32

+

32

- 3 1 004

+

+

53

= cos

20121f

(2012 2010)

- 3 1 00 4

3

+

2012) (2010

= cos

3 1 006

+

21f 3

=

2

-�

(2012 2012) ] - �2 2012) ] 1 (2012

3 1 006

=

=

D

Singapore Mathematical Society Singapore Mathematical Olympiad (SMO ) 2012

(Open Section, Round 2) Saturday, 30 June 2012

1. The incircle with centre

0900-1300

of the triangle ABC touches the sides BC, CA and AB at and respectively. The line intersects the segment at K. Prove that A, K and M are collinear where M is the midpoint of BC.

D, E

F

2. Find all functions

f

:

IR

I

---+

ID

IR

EF

so that

+

+

=

(x y )(f(x ) -f (y )) (x-y ) f (x y ) for all

x, y

3. For each

i

E JR. =

1, 2, . . . , N, let ai, bi, ci be integers such that at least one of them is odd. Show that one can find integers z such that xai + ybi + zci is odd for at least 4N different values of i .

x, y,

/7

4.

Let p be an odd prime. Prove that ! P-2 + 2p -2 + 3p -2 + . . . +

5.

(� - 1 ) p -2

-

2 - 2P p

( mod p ) .

There are 2012 distinct points in the plane each of which is to be coloured using one of n colours so that the number of points of each colour are distinct. A set of n points is said to be multi-coloured if their colours are distinct. Determine n that maximizes the number of multi-coloured sets.

54

Singapore Mathematical Society Singapore Mathematical Olympiad (SMO)

2012

(Open Section, Round 2 solutions)

1. A

c

B

Let the line AK intersect EC at M. We shall prove that M is the midpoint of EC. Since LFIK = LE and LEIK = LC, we have FK EK Also

FK sin LF AK

Therefore,

sin LFIK sin LEIK

AE sin LAKE

AF sin LAKF sin LFAK sin LKAE

sin E sin C '

FK EK

EK . sin LKAE

sin E sin C '

Consequently, EM EM . AM sin LFAK sin C = = = CM AM CM sin E sin LKAE so that EM = CM.

55

l,

2. Suppose that

f is a solution. Let a = 21 (! (1) - f (-1)), b = 21 (! (1) +! (-1)) and g (x ) = f (x ) - ax - bx2. Then (x +y )(g(x ) - g (y )) = (x - y)g (x +y ) and g (1) = g( -1) = 0. Letting y = 1 and y = -1 above give (x + 1) g (x ) = (x - 1) g (x + 1) xg (x +1) = (x +2)g(x) .

Thus

x(x +1)g (x) = x(x - 1) g(x +1) = (x - 1)(x + 2) g (x) for all x. So g (x ) = 0 for all x. Hence f(x) = ax+bx2. We can check directly that any function of this form (for some a, b E JR.) satisfies the given equation. Consider all the 7 triples where z are either 0 or but not all 0. For is odd. Thus among the 7 sums each at least one of the numbers 3 are even and 4 are odd. Hence there are altogether 4N odd sums. Thus there is choice of for which at least 4N 7 of the corresponding sums are odd. You can think of a table where the rows are numbered . . . , N and the columns correspond to the 7 choices of the triples The 7 entries in row are the 7 sums Thus there are 4 odd numbers in each row, making a total of 4N odd sums in the table. Since there are 7 columns, one of the columns must contain at least 4N 7 odd sums.

(x, y, z), x, y, ai, bi, Ci / 1, 2, i (x, y, z) .

3.

i, (x, y, z)

1

( xai +ybi +zci. / )

i 1, 2, . . P, ;1 , 2i ( p ) = (p -1) (p - 2) . . . (p - (2i -1)) (-1) (-2) (- (2i - 1)) p 2i (2i - 1) ! (2i - 1) !

4.

·

First, for each =

·

Hence

(p-1)/2

L i= 1

xai+Ybi+zci,

·

·

_

_

1 (mod p) .

. ( ) 2 (p-1)/2 ·p-1 ( ) 2 - L 2i. p 2i = p i=1 i= 1 (p- 1)/2 ( p ) 2 --p (mod p) (by Fermat 's Little Theorem. ) = i 1 2i

·p-2 = 't

(p-1)/2

_

-

·p- 2 't

't

P

_

-

-

� L....,;

't

P

� L....,;

=

The last summation counts the even-sized nonempty subsets of a p-element set, of which there are P - 1

2 - 1.

56

5.

Let m1 < m2 < < mn be the number of points of each colour. We call m1, m2, . . . , mn the colour distribution. Then m1 + + mn = and the num­ ber of multi-coloured sets is M = m1 m2 . . . mn. We have the following observations. ·

·

·

·

·

2012

·

(i) m1 > 1 . For if m1 = then m1m2 . . . mn < m2m3 . . . mn - 1(1 + mn ) This means if we use n- colours with colour distribution m2, m3, . . . , mn_ 1, + mn), we obtain a larger M.

1,

1

·

(1 (ii) mi+1- mi ::::; 2 for all i . For if there exists k with mk+1 - mk 2': 3, then the colour distribution with mk, mk+1 replaced by mk 1, mk+1- 1 yields a larger M. (iii) mi+1- mi 2 for at most one For if there exist i j with mi +1 - mi mj+1 - mj 2, the colour distribution with mi, mj+1 replaced by mi 1, mj+1 - 1 yields a larger M. (iv) mi+1 - mi 2 for exactly one For if mi +1 - mi 1 for all then m1 mn nm1 n(n2-1) 2012 4 503. Thus n(2m1 - 1 n) 8 503. Since 503 is prime, the parity of n and 2m1 -1 n are opposite and 2m1 - 1 n n, we have n 8 and m1 248. The colour distribution with m1 replaced by two numbers 2, 246 (using n 1 colours) yields a larger M. 2. If mn - mn 1 2, then from (iv) , we have m1 mn (v) m1 n(n-1) 1 2012. Thus -n(2m1 -1 ) 2 2011. Since 2011 is prime, we get nm1+ 2 n 2 and m1 1005 which will lead to a contradiction as in (iv) . Thus mn-mn_ 1 1. mi+1- mi 2 for some 1 ::::; i ::::; n - 2. Suppose m1 2': 3. Let m' mi+2 - 2. Then mi m' mi+1 with replacing mi+2 by 2, m' yields a larger M. Thus m1 2. From the above analysis, with n colours, we see that the colour distribution 2, 3, 2, . . ., n 1, n 2, with 3 ::::; i ::::;2 n, yields the maximum M. Now . . . , i- 1, i 1, we have 2: mi � (n 1) ( n 4) - i 2012. Thus n + 5n- 4020 2i, 3 :=::; i ::::; n, i.e. , n2 5n 2': 4026 and n2 3n ::::; 4020. Thus n 61 and i 3. Thus the maximum is achieved when n 61 with the colour distribution 2, 4, 5, 6, . . . , 63. =

·

·

·

+

=

+

=

=

<

+

<

=

+

+

i.

=

i.

=

=

=

=

=

+

+

=

=

<

·

=

+

=

=

+

+

n

=

+

=

i,

+

=

+

+

+

+

=

57

+

>

·

·

·

+

=

·

+

=

·

+

=

i +

=

=

=

=

=

Multiple Choice Questions 1. Calculate the following sum: 1 2 3 4 10 -+ -+ -+ -+ . . . + . 16 210 2 4 8

(A)

50 3 . 256'

507

505

509

(B) 5 ,. C ; (D) 5 ,. (E) None of the above. 2 6 ( ) 256 2 6

2. It is known that the roots of the equation x5 + 3x4- 40441 18x 3 - 12132362x 2- 12132363x- 201 12= 0

are all integers. How many distinct roots does the equation have? (A) 1;

(B) 2; ( C) 3; (D) 4; (E) 5.

3. A fair dice is thrown three times. The results of the first, second and third throw are recorded as x, x,

y

(A)

y

and

and 1 ; 12

z

z,

respectively. Suppose x + y=

z.

What is the probability that at least one of

is 2? 8

(B) � , ( C ) 5 ; (D) 1 8 ·

1 3;

(E)

!__. 13

4. Let

= 1 000 · · · 000 1 000 · · · 000 50 . '-....--" '-....--" 2012 times 2011 times

X

Which of the following is a perfect square? (A) x- 75;

(B) x- 25; (C) x ; (D) x + 25; (E) x + 75.

5. Suppose N 1, N2, ..., N2o11 are positive integers. Let

X= (N1 + N2 + · · · + N2o10) (N2 + N3 + · · · + N2ou ), Y= (N1 + N2 + · · · + N2ou) (N2 + N3 + · · · + N2o1o) . Which one of the following relationships always holds? (A) X=Y;

(B) X>Y; ( C ) X
. 6. Consider the following egg shaped curve. A BC D is a circle of radius 1 centred at 0. The arc AE is centred at C , CF is centred at A and EF is centred at D. 2

(A) 1680;

(B) 1712; (C) 3696; (D) 3760; (E) None of the above.

10. In the set {1, 6, 7, 9}, which of the numbers appear as the last digit of nn for infinitely many

positive integers n? (A) 1, 6, 7 only;

(B) 1, 6, 9 only; (C) 1, 7, 9 only; (D) 6, 7, 9 only; (E) 1, 6, 7, 9.

Short Questions z x y 11. Suppose -+ -+-= a b c

12. Suppose x=

a b c . ..J2 and-+-+-= 0. Fmd

13 v'19 + sv'3

x

y

z

. Find the exact value of x4- 6x 3- 2x 2 +18x + 23 x 2- 8x + 15

13. Let a1= 3, and define an+l=

1 a J3 n � for all positive integers n. Find a2011 · an + 3

14. Let a, b, c be positive real numbers such that a2 + ab + b 2= 25, b2 + be+ c2= 49, c2 + ca + a2= 64.

4

distinct ways of using two tiles of size 1

x

1, two tiles of size 1

x

2 and one tile of size 1

x

4.

It is not necessary to use all the three kinds of tiles. )

24. A 4 2

x

x

4 Sudoku grid is filled with digits so that each column, each row, and each of the four

2 sub-grids that composes the grid contains all of the digits from 1 to 4. For example,

Find the total number of possible 4

x

4

3

1

2

2

1

3

4

1

2

4

3

3

4

2

1

4 Sudoku grids.

25. If the 13th of any particular month falls on a Friday, we call it

Friday the 13th.

It is known

that Friday the 13th occurs at least once every calendar year. If the longest interval between two consecutive occurrences of Friday the 13th is

x

months, find

x.

26. How many ways are there to put 7 identical apples into 4 identical packages so that each package has at least one apple?

27. At a fun fair, coupons can be used to purchase food. Each coupon is worth $5, $8 or $12. For example, for a $15 purchase you can use three coupons of $5, or use one coupon of $5 and one coupon of $8 and pay $2 by cash. Suppose the prices in the fun fair are all whole dollars. What is the largest amount that you cannot purchase using only coupons?

28. Find the length of the spirangle in the following diagram, where the gap between adjacent parallel lines is 1 unit.

6

E

D

c

34. Consider an equilateral triangle A BC , where A B = BC = CA = 2011. Let P be a point

inside LA BC . Draw line segments passing through P such that DE

II BC , FG II CA and

HI II AB. Suppose DE : FG : HI= 8 : 7 : 10. Find DE +FG + HI . A

c

35. In the following diagram, A B

j_

BC . D and E are points on segments A B and BC respec­

tively, such that BA + AE = E D + DC . It is known that A D = 2, BE = 3 and EC = 4. Find BA + AE .

8

Note that X= (10 1012 + 1) X 10 2014 +50= 104026 + 102014 +50 = (102013 )2 +2 X 102013 X 5 +50= (102013 +5) 2 +25. So x- 25 is a perfect square. 5. Answer: ( B ) .

Let K= N2 + · · · +N2010· Then X= (N1 +K) (K +N2on) andY= (N1 +K +N2on)K.

6. Answer: ( A ) .

7r (22 ) ,---... The area enclosed by A D , DE and AE is 8 -

The area of the wedge E DF So the area of the egg is:

. 7r (2- J2)2 = IS 4

(

-

1f - 1. 1= 2

)

3 J2 2-

1f

.

� + 1 +2 (�- 1) + (�- J2) 1r= (3- J2)1r- 1. x

7. Answer: ( B ) .

The left shows that 3 colours are not enough. The right is a painting using 4 colours.

8

.

Answer: (E). Since 5 I (24- 1), 71 (36- 1), 11 I (510- 1), 13 I (7 12- 1),

9. Answer:

n

is divisible by 5, 7, 11 and 13.

(C).

We consider the position of the Black Knight. The number of positions being attacked by the White Knight can be counted.

10

13. Answer: 3.

J33 x 01 . Then f ( (x))= Let f (x)= J X+ 3

vfsx-1 _J3 = J (J (J (x)))= x+vfs J3 vfsx 1 + 1 x+vfs

Since 2010= 6

x

J3 vs-1 _1

�vis 3x-1 +J3 x+vfs

3x-J3 - x-J3 J3x - 1 + V3x+ 3

-

x-J3 J3x+ (

3x - 1- v3x- 3 _.!. v/"il . So J (J (J (J (J (J (x))))))= x. = X 3x-J3 + x/"il+J3

335, a2o11= f (J (J··· f (J (3)) ·· ))= 3. '-----v--" 2010 copies ·

14. Answer: 129. Consider the following picture, where LAOB = L BOC= LCOA= 120°, OA= a, OB= b and OC= c. A

c

B

Then IBCI= 5, I CAl= 7 and lA B I= 8. The area of the triangle A BC is yl10 (10- 5) (10 - 7) (10 - 8)= 10.J3. 1V3 (ab +be+ ca)= 10J3. So ab +be+ ca= 40. Then 2 2 2 (a + b + c) 2= (a2+ ab + b2) + (b2 +be+ c2) + (c2 + ca + a2) + 3 (ab +be+ ca)= 258. Thus, (a+ b + c) 2= 129. 15. Answer: 0. Define Q (x) = (1 + x)P (x) - x. Then Q (x) is a polynomial of degree 2011. Since Q(O) = Q (1)= Q (2)= ···= Q (2010) = 0, we can write, for some constant A, Q (x)= Ax(x- 1) (x- 2)··· (x- 2010). 1= Q (-1)= A (-1) (-2) (-3)·· · (-2011)=-A· 2011!. Then Q (2012) = A· 2012!= -2012, Q (2012) + 2012 and P ( 2012)= = o. 2013 16. Answer: 9241. 12

Suppose x 1

=

x2

M

=

=

2:: >

2::

2::

=

=

Therefore, M



···

=

=

Xk

1

=



2

<

Xk+l



···



X2011 · Let M

=

x 1 · · · X20l1 · Then

Xk+1Xk+2 . . . X2QlOX20ll (Xk+l- 1)x k+2 · · · X2010X2011 +Xk+2 . . . X2010X2011 (Xk+l- 1) 2 +Xk+2 · · · X2010X2011 .

.

.

.

.

.

.

.

.

.

.

.

.

.

.

.

.

.

.

.

.

(x k+l- 1) 2 + · · · + (x 2009- 1)2 +X2010X2011 (x k+l- 1) 2 + · · · + (x 2009- 1)2 + (x 2o10- 1)2 + (x 2o11- 1) 2 2 (x k+l + · · · +x 2o11- (2011- k)) 2 (M- 2011) .

4022. On the other hand, (1, 1, . . . , 1, 2, 2011) is a solution to the equation.

So the maximum value is 4022. 2 1. Answer: 101. 2::

If n

M (91)

102, then M (n) =

M (M (102))

For each k

=

=

=

n- 10

M (92)

2::

92.

M (M ( 103))

=

1, . . . , 10, M (80 + k)

=

M (70 + k) M(k)

=

=

=

M (93)

M (M (91 + k))

=

M (M(81 + k))

···

M (91)

=

M (M(11 + k))

=

=

Hence, all integers from 1 to 101 are solutions to M (n)

=

=

M (91) M (91) =

=

=

M ( 101)

=

9 1.

9 1, and thus 9 1, 9 1.

91.

22. Answer: 19.

20 + 1 1 . (M) n 1 2on+l + 1 1n+l · n n - (n +1) (1 +(M) n) n +1 20 +1 1

An+l An

Then An+l

<

An if n

Note that 10 + If 10 n





n



>



10 +

11 ) n 1 + 20

18, then n

18 implies An

<



<

n ; and An+l 1 + 11 w)

10 +9

10 +8

An+l ·



<

=

19. Son

10 +

2::

An if n

>

<

19 implies An

9 . ; 1f n n 11 1 + ( 20 )

<

10 + >

1+

�!! w

)n

.

An+l ·

10, then n

<

10 +

9 n Hence, 1 + ( 11 20 ) .

23. Answer: 169.

Let an be the number of ways to pave a block of 14

1

X

n. Then an

=

an-1 +an-2 + an-4 with

Case 2:

Case 3:

The first year is a leap year. Jan

Feb

Mar

Apr

Mar

Jun

Jul

Aug

Sep

Oct

Nov

Dec

0

3

4

0

2

5

3

6

1

4

6

2

5

5

1

3

6

@] 1

4

@]

2

5

0

The second year is a leap year. Jan

Feb

Mar

Apr

Mar

Jun

Jul

Aug

Sep

Oct

Nov

Dec

0

3

3

6

1

4

2

5

5

4

5

1

3

[]]

4

@

@

3

1

[]]

5

0

1

2

From these tables we see that the answer is 14. The longest time period occurs when the Friday of 13th falls in July of the first year and in September of the second year, while the second year is not a leap year. 26. Answer: 350. By considering the numbers of apples in the packages, there are 3 cases: l) (4, l, l, l). 2) (3, 2, 1, 1 ) . 3) (2, 2, 2, l).

X 6 X 5 X 4 = 35. 4x3x2xl

(7) 7 (7) ( ) 7 � (7) ( ) ( ) 4

=

3

3! 2

4 2

=

5 2

6X5 = 3x2xl X

3 2

=



6

X

X

7 2

4 X3 = 35 2xl

X

6 = 210.

4 1

X

3 2

X X

6 1

X

5 2

X X

2 = l05. 1

X X

So the total number of ways is 35+ 210 + 105 = 350. 27. Answer: 19. Note that 8 + 12 = 20, 5 + 8+ 8 = 21, 5 + 5 + 12 = 22, 5 + 5 + 5 + 8 = 23, 8+ 8 + 8 = 24. If 2::

n

2::

25, write

n=

5k + m where 20

:::;

m

:::;

24 and k is a positive integer. So any amount

25 can be paid exactly using coupons.

However, 19 cannot be paid exactly using these three types of coupons. 28. Answer: 10301. . The broken line is constructed using

"

L ", with lengths 2, 4, 6, . . . , 200.

100 + 101 = 201. Then the total length is 2 (1 + 2 + 3 + 29. Answer: 15. 16

·

·

·

The last

+ 100) + 201 = 10301.

"

L " is

So c= 12 or c=-20 (ignored ) . Substitute y=

-x

+ 12 into the parabola:

2=

x

-x

+ 12

=*

= 3,-4. So A is (3, 9) and B is (-4, 16). Then

x

I A BI 2= (3- (-4)) 2+ (9- 16) 2= 98. 33. Answer: 30. Draw BF

..l

LE= 30°.

J2 CE , where F is on CE . If A B = 1, then BF = 2 and BE =

J2. Thus

E

D

34. Answer: 4022. Set D P= GP= a, I P= FP= b, E P= HP= c. Then DE +FG+HI= (a+ c)+ (a+ b) + (b + c)= 2 (a + b+ c)= 2 35. Answer: 10. By given, B D + 2 +AE= B D + DC. So 2 +AE= DC . Note that A B 2+ BE 2= AE 2 and B D 2+ BC 2 (2 + B D ) 2+ 32= AE 2, 4 (AE + B D )= 32. Then AE + BA

=

=

DC 2. Then

B D 2+ 72= (AE + 2) 2.

32 4 + 2= 10.

18

x

2011= 4022.

Singapore Mathematical Society Singapore Mathematical Olympiad (SMO)

2011

(Junior Section, Round 2 solutions)

1. It's equivalent to prove x2y2 :;:::: (ac + bd)2 as all the numbers are nonnegative. This

is true s ince

x2 y 2

(a2 + b2)(c2 + d2) (ac)2 + (bd)2 + a2d2 + b2c2 :;:::: (ac)2 + (bd)2 + 2(ac)(bd) AM-GM (ac + bd)2.

= =

=

Let the tangent at P meet the tangent at R at the point S. Let 0 be the centre of r1. Then OR ST is a s quare. Hence LKPR L R P S 45°. Als o LNPS LNKP LPMS LMPK. Thus LMPR LRPN. 2.

=

=

=

=

=

=

K

3. Let ai 0,

maxSi, bi minSi and s uppos e that t1 then a1 :;:::: b j. Therefore a1 E Sj. =

=

=

min{ti}. For each j, if 81 n Sj i=

Note: Problem 4 in the Senior Section is the general vers ion. 4.

Replace 2011 by any pos itive odd integer n. We firs t s how by induction that am 3m2n-m - 1 for m 0. Suppos e 0, 1, ..., n- 1. This is certainly true for m + + nit's true for s ome m < n - 1. Then 3am 1 3m l2 m - 2. Since n- m > 1, the odd part is 3m+l2n-m-l - 1 which is am+l· Now an-1 3n-121 - 1. Thus =

=

=

=

=

20

Singapore Mathematical Society

Singapore Mathematical Olympiad (SMO)

2011

(Senior Section)

Tuesday,

31

May

2011

0930-1200

hrs

Instructions to contestants 1. Answer ALL 35 questions. 2. Enter your answers on the answer sheet provided. 3. For the multiple choice questions, enter your answer on the answer

sheet by shading the bubble containing the letter (A, B, C, D or E) corresponding to the correct answer.

4. For the other short questions, write your answer on the answer sheet and shade the appropriate bubble below your answer. 5. No steps are needed to justify your answers. 6. Each question carries 1 mark. 7. No calculators are allowed.

PLEASE DO NOT TURN OVER UNTIL YOU ARE TOLD TO DO SO

22

6. Determine the value of

2

1 1 + 0+Vfs+2 0-V'8+2 (A) 4- 0 (E) 40-2 7. Let

x

=

is true?

(A)

(D)

1

(B) 2-20

1

log� 2

+

1

1

logi 4

1. 5 < X < 2

3 <X< 3. 5

+

(C) 4 +0

1

1.

log� 8

(D)

20 + 4

Which of the following statements

(B) 2 < x < 2. 5 (E) 3.5 <X<4

(C) 2.5

<X<

3

8. Determine the last two digits of 756•

(A) 01

(B) 0 7

9. It is given that

x

(C) 09

(D)

43

(E) 49

and y are two real numbers such that

y > 1. Find the smallest possible value of

x

>

1

and

=

c-1

logx2011 +logy2011 logxy 2011

(A) 4 10.

(B)

6

(C)

8

(D)

(E) 12

10

It is given that a, band care three real numbers such that a+ b

and ab

(A)

5

=

c2 - 7c +14. Find the largest possible value of a2 + b2.

(B)

6

(C)

8

(D)

9

(E) 10

Short Questions 11.

Find the value of

20112 +21112-2 X 2011 X 2111 25 24

18.

In the diagram below, the lengths of the 2three2 sides of the triangle are a +2 b = 2011. Find the value a em, b em and c em . It is given that of

c

cot C

cot A +cotE

b c a

19.

Su p pose there are a total of

petition, and at least

1011

partici pants, at

2011

partici pants in a mathematics com­

1000 of them are female . Moreover, given any least 11 of them are male . How many male

partici pants are there in this com petition?

20.

Letf : Q \

{0, 1}

-t

Q be a function such that

2

xf ( x)+f for all rational numbers

2 (X -1) = 2x x

x #- 0, 1.

Here Q denotes the set of rational



numbers . Find the value off( ).

21.

In the diagram below , ABCD is a convex quadrilateral such that AC

intersects BD at the midpoint E of BD. The point His the foot of

the per pendicular from A onto DE, and Hlies in the interior of the segment DE. AH=

40

Su p pose LBCA =

em and CD=

x

90°, CE = 12

em . Find the value of A

c

26

em, EH=

x.

15

em,

28. It is given that

a,

b,

c

and d are four positive prime numbers such

that the product of these four prime numbers is e qual to the sum of

55 consecutive positive integers. Find the smallest possible value of

a+ b+ c +d. ( Remark: distinct. )

The four numbers

a,

b,

c,

d are not necessarily

29. In the diagram below, ABC is a triangle with AB= 39 em , BC = 45 em and CA= 42 em. The tangents at A and B to the circumcircle of

DABC meet at the point P. The point D lies on BC such that PD

is parallel to AC . It is given that the area of DABD is x cm 2. Find the value of x.

30. It is given that

a and

b are positive integers such that

a has

exactly

9 positive divisors and b has exactly 10 positive divisors. If the least

common multi ple

( LCM ) of

a and

b is 4400, find the value of

a

+ b.

31. In the diagram below, ABCD is a quadrilateral such that .!_ABC= 135° and L_BCD= 120°. Moreover, it is given that AB = 2 J3 em,

BC= 4 - 2 -/2 em, CD= 4 -/2 em and AD= of x2- 4x.

c

B

A

x

<-------"'

28

D

em. Find the value

Singapore Mathematical Society

Singapore Mathematical Olympiad (SMO) (Senior Section Solutions)

2011

1. Ans wer: (B)

a2 b2 < Since a, b and c are nonzero real numbers and b2+c 2 c 2 +a 2 that b 2 +c 2 c 2 +a 2 a 2 +b 2 > > c2 a2 b2 Adding 1 throughout, we obtain

c2 , we s ee a 2+b 2

--=--

--=---

Thus

<

1 1 1 > > 2 , which implies that a 2 < b2 < c 2. So we have I a I a b2 c

2

<

l bl

2. Ans wer: (A) Note that (s in()+cos ())2= sin2 ()+cos 2()+2 s in() cos ()= 1+s in 2()= 1+a. Since 0::; ()::;� ' we have sin()+cos ()> 0. So s in() +cos ()= v'f+(i". 3. Ans wer: (D) We have � [(a- b)2 + (b- e)2 + (c- a)2 ] 2 [(-1)2 + (-1)2 +22 ] = 3. ·

�.

4. Ans wer: (C) 1 1 X 2y -

1 2x+y

2x+y =1 2y X y X 1 2+-- -- -= 1 X y 2 1 y X 2 X y 2x+y

::::?-

::::?-

::::?-

Now we have

30

-

<

l ei ·

Moreover, 2x

2(log 3+log 5� +log 7i) log 2 log(9 x 5)+log(49i) log(45 x 27 i) > ---,-log 2 log 2 log(45 x 3) log(128) = 7' > �g 2 �g2

------

SO X >

-

-

3. 5.

8. Ans wer: (B) Note that 74-1 = 2400, s o that 74n-1 is divis ible by 100 for any n E z+. Now, 756 7(756-1-1 +1) 7(756-1-1)+7 7(74n-1)+7 ,

where

56-1 '77+ E !LJ . 4 Since 7(74n-1) is divis ible by 100, its las t two digits are 00. It follows that the last two digits of 756 are 07. =

n

9. Ans wer: (A) logx 2011+logy 2011 logxy 2011

log 2011 log 2011 . log xy + ) ( ) log x logy log 2011 1 1 (- +-) . (log x +logy) log x logy log x logy 2+--+-logy log x 4 (us ing A M 2:: G M), (

>

and the equality is attained when log x = logy, or equivalently, x= y. 10. Ans wer: (C) The roots of the equation x2- (c- 1)x+c2- 7c+ 14 = 0 are a and b, which are real. Thus the dis criminant of the equation is non-negative. In other words , (c-1)2- 4(c2- 7c+ 14)= -3c2+26c- 55= (-3c+11)(c- 5) 2:: 0. 11 < < So we have c - 5. Together with the equalities 3 (a+b?- 2ab (c- 1)2-2(c2- 7c+ 14) -c2+12c- 27= 9 - (c- 6)2,

32

15. Ans wer: 8001 Note that J5n- y'5n- 4 < 0. 01 if and only if 4 > 400. y'5; + v'5n- 4= J5n 5n- y'5n- 4 If n= 8000, then J5n+y'5n- 4= v' 40000+\1'39996 < 400. If n= 8001, then J5n+y'5n 4= v' 40005+v'40001 > 400. So the ans wer is 8001. -

16. Ans wer: 1006 The s eries can be paired as 1 1 1 1 1 1 ) + + ( ) + ( + )+( + 1 11 · · · 11 1 +11- 1+ 111 · 1 + 11-20 1+11-2oo9 1+112oo9 1 +1120 Each pair of terms is of the form 1 1 ----,-+--= 1. 1 1+ a 1+a There are 1006 pairs of s uch terms , and thus the s um of the s eries is 1006. 17. Ans wer: 54 X

7 77r . 4 (-7r ) +cos 4 (-7r ) +s in 4 (8) +cos 4 (87r) 8 8

Sill

sin 4 (i) +cos 4 (i) + s in 4 (i) +cos 4 (i) . 2 (47r) = 3 . 2 (87r) + COS 2 (87r) )2- 4 Sill . 2 (87r) COS 2 (87r) = 2- Sill 2(Sill 2 Thus 36x = 54. 18. Ans wer: 1005 By the laws of s ine and cos ine, we have s in A s inE sinG and c b a Then 1 cos C cote cosAsi B + �os B sinA � sinC cot A+cot B smAsm B s in A s in B cos C sin(A+B) s inC s in A s in B ( . 2 ) cos C Sill 0 (ab jc 2) s in2 C a 2 +b2- 2 ) )( ( 2ab s in2 C a 2 +b 2- c 2 2c 2 2011- 1 = 1005. 2 .

34

·

22. Ans wer: 3 Note that 1 1 -+X y

1 =? xy- 211x- 211y 211

= -

=

0 =? (x- 211)(y- 211)

=

2112.

Since 211 is a prime number, the factors of 2112 are 1, 211, 2112, -1, -211, -2112. Thus the pairs of integers (x, y) s atis fying the las t equation are given by: (x- 211, y- 211)

=

(1, 2112), (211, 211), (2112 , 1), (-1, -2112), (-211, -211), (-2112, -1).

Equivalently, (x, y) are given by (212, 211 +2112), (422, 422), (211+2112 , 212), (210, 211- 2112), (0, 0), (211- 2112 , 210). Note that (0, 0) does not s atis fy the firs t equation. Among the remaining 5 pairs which s atis fy the first equation, three of them satis fy the inequality x � y, and they are given by (x, y) (422, 422), (211+2112 , 212), (210, 211- 2112). =

23. Ans wer: 93 By long divis ion, we have x 4 +ax 2+bx+c

=

(x2+3x- 1) · (x 2- 3x+(a+10))+(b-3a- 33)x+(c+a+10).

Let m1 , m2 be the two roots of the equation x2 + 3x- 1 0. Note that m1 -=1m2 , since the dis criminant of the above quadratic equation is 32-4 · 1 · 1· (-1) 13 -=10. Since m1, m2 als o s atis fy the equation x4 +ax 2+bx+c 0, it follows that m1 and m2 als o satis fy the equation (b- 3a- 33)x+(c +a+ 10) 0. =

=

=

=

Thus we have (b- 3a- 33)m1 +(c +a+ 10)

=

0,

and (b- 3a- 33)m2 + (c +a+10) 0. Since m1 -=1m2 , it follows that b-3a-33 0 and c+a+lO 0. Hence we have b 3a+33 and c -a-10. Thus a+b+4c+100 a+(3a+33)+4(-a-10)+100 93. =

=

=

=

=

=

=

24. Ans wer: 1120 Let m and n be pos itive integers s atis fying the given equation. Then 3(n2- m) 2011n. Since 2011 is a prime, 3 divides n. By letting n 3k, we have (3k)2 m + 2011k. This implies that k divides m. Let m rk. Then 9k2 rk+2011k s o that 9k r+2011. The smalles t pos itive integer r s uch that r+2011 is divis ible by 9 is r 5. Thus k (5+ 2011)/9 224. The corres ponding values of m and n are m 1120 and n 672. =

=

=

=

=

=

=

=

=

36

=

=

� = c2�b2• Since GN is parallel to AD and G is the centroid of the triangle ABC, we have MD/MN= 3. It follows that c+b= aJ3. Thus , AB = aJ3-b = 15-6= 9.

28. Ans wer: 28 The s um of 55 pos itive cons ecutive integers is at leas t (55 x 56)/2 = 1540. Let the middle number of thes e cons ecutive pos itive integers be x. Then the product abed= 55x = 5 11 x. So we have 55x 2': 1540 and thus x 2': 28. The leas t value of a+b +c +d is attained when x= 5(7). Thus the ans wer is 5+11+ 5+ 7 = 28. ·

·

29. Ans wer: 168 Firs t L_BD P= L_BCA= L_BAP s o that P, B , D, A are concyclic. Thus L_ACD = L_ PBA= L_ PDA= L_DAC s o that DA= DC.

By cos ine rule, cos C = 3/5. Thus DC = � AC jcos C = 21 x 5/3 = 35. Hence ED = 10 and BC = 10 + 35= 45. Thus area(L,ABD ) = �� x area(L,ABC). By Heron's formula, area(L,ABC)= 756. Thus area(L, ABD)= �� x 756= 168. 30. Ans wer: 276 Since the number of pos itive divis ors of a is odd, a mus t be a perfect square. As a is a divis or of 4400 = 24 x 52 x 11 and a has exactly 9 pos itive divis ors , we s ee that a= 22 x 52. Now the leas t common multiple of a and b is 4400 implies that b mus t have 24 x 11 as a divis or. Since 24 x 11 has exactly 10 pos itive divis ors , we deduce that b = 24 x 11 = 176. Hence a+b = 276. 31. Ans wer: 20 Firs t we let /!, be the line which extends BC in both directions . Let E be the point on /!, s uch that AE is perpendicular t o f. Similarly, we let F be the point on /!, s uch that DF is perpendicular to f. Then, it is eas y to s ee that BE = AE = v'6, CF = 2J2 and DF = 2v'6. Thus EF = y'6 + 4- 2J2 + 2J2 = 4 +v'6. Now we let G be the point on DF s uch that AG is parallel to £. Then AG = EF = 4 +v'6 and

38

which is a contradiction to the given fact that each element of S is les s than or equal to 2011. Case 2. 1 is in A. We may let a16 = 1. Then a 1, a 2 , As in Cas e 1, we have

· · ·

, a 1 5 are compos ite numbers .

which is a contradiction. Thus we have shown that every 16-element s ubs et A of S s uch that all elements in A are pairwis e relatively prime must contain a prime number. Hence the smallest k is 16. 35. Ans wer: 12 Let the extens ions of A Q and BP meet at the point R. Then LP R Q= LPAB= LQPR s o that QP = QR. Since QA = QP, the point Q is the midpoint of A R. As A R is parallel to LP, the triangles A RB and LP B are similar s o that M is the midpoint of PL. Therefore, N is the centroid of the triangle P LB, and 3 MN= BM.

B

Let LABP= e. Thus tanB= A R/AB= 2A QjAB= 5/6. Then B L= PB cos B= AB cos 2 0. Als o BM/BL = BQ/BA s o that 3MN = EM = �� AB cos 2 0 cos 2B( QM + 3MN). Solving 'for MN, we have MN= 3 �:;_; e = 3x (�f6)2 = 12.

40

Singapore Mathematical Society Singapore Mathematical Olympiad (SMO)

2011

(Senior Section, Round 2 solutions)

1. There is an error in this problem. The triangle is not neces s arily equilateral. In fact

we s hall prove that the altitude at A, the bis ector of LB and the median at C meet at a common point if and only if cos B= � where BC= a, CA = b and A B= c. a

c

Let D,E and F be the points on BC,CA and A B res pectively s uch that AD is the altitude at A , BE is the bis ector of LB and CF is the median at C. Suppos e that AD,BE,CF meet at a common point. The point of concurrence of AD,BE and CF mus t lie ins ide the triangle A BC. Since F is the midpoint of A B, by Ceva's theorem CE : EA = CD : DB. Us ing the angle bis ector theorem, CE : EA =a : c. Thus CD= a2/(a+c) and DB= acj(a+c). Thus cos B= �� = � a



A

B

Convers ely, if cosB= � then LB is acute and BD= c cos B= acj(a+c)

' c

So given a and c, the acute angle B and hence the triangle A BC is determined. If a -=/- c, then the triangle A BC is not equilateral. 2. Yes , in fact, for any k E

N,

there is a s et 8k having k elements s atis fying the given condition. For k = 1, let 81 be any s ingleton s et. For k = 2, let 82 = {2 ,3}. Suppos e that 8k = {a1, ... , ak} s atis fies the given conditions. Let b1 = a1a2 ak bj = b1+aj -1, 2 :::; j :::; k+ 1. · · ·

Let 8k+1 = {b1, b2, ... , bk+1}. Then the fact that 8k+1 s atis fies the required property can be verified by obs erving that lm- nl = (m, n) if and only if (m- n) divides m. 42

Note that a1 a2 ··· an

=

1. It suffices to show that

since it is equivalent to 1 1 1 �n- 1. + +···+ 1+an 1 +a1 1+a2

--

We shall show that ( * ) is true for n 2:: 2. The case n 2 is obvious. We will now prove it by induction. Suppose ( * ) holds for n k. Now assume a1 ··· ak+l 1, ai rel="nofollow"> 0 for all i. To prove the inductive step, it suffices to show that =

=

which can be verified directly. Note: This is an extension of the problem :

44

=

Throughout this paper, let lxJ denote the greatest integer less than or equal to x. For example, l2.1J = 2, l3.9j = 3 ( This notation is used in Questions 7, 9, 19 and 20). 1.

A circular coin A is rolled, without sliding, along the circumference of another stationary circular coin B with radius twice the radius of coin A. Let x be the number of· degrees that the coin A makes around its centre until it first returns to its initial position. Find the value of x.

2. Three towns X,Y and Z lie on a plane with coordinates (0, 0), (200, 0) and (0, 300) respec­ tively. There are 100, 200 and 300 students in towns X,Y and Z respectively. A school is to be built on a grid point (x, y) , where x and y are both integers, such that the overall distance travelled by all the students is minimized. Find the value of x+y. 3. Find the last non-zero digit in 30!.

( For example, 5! = 120; the last non-zero digit is 2.) 4. The diagram below shows �A BC , which is isoceles with A B = AC and LA 20°. The point D lies on AC such that A D = BC. The segment E D is constructed as shown. Determine LA B D in degrees. A =

cos4 a sin4 a cos4 f3 sin 4 f3 . 5. G1ven that �(3 + -.-2- = 1, evaluate --2- + -.-2 - . cos sm f3 cos a sm a 6. The number 25 is expressed as the sum of positive integers XI, x 2, , Xk, where k What is the maximum value of the product of XI, x 2, X3, , and Xk ? ·

·

46

·

·

·

·

::;

25.

18.

A collection of 2011 circles divide the plane into N regions in such a way that any pair of

circles intersects at two points and no point lies on three circles. Find the last four digits of N.

19. If a positive integer N can be expressed as lxJ + l2xJ + l3xJ for some real numbers x, then we say that N is ''visible"; otherwise, we say that N is "invisible". For example, 8 is visible since 8= l1.5J + l2 (1.5 )J + l3 ( 1.5 )J , whereas 10 is invisible. If we arrange all the "invisible" positive integers in increasing order, find the 2011th "invisible" integer. 20. Let A be the sum of all non-negative integers

n

satisfying

Determine A. 21.

A triangle whose angles are A, B, C satisfies the following conditions sin A+ sinE + sinG cos A + cos B + cosC and

12 7'

12 . 25 Given that sin C takes on three possible values 81, 82 and 83 , find the value of 100818283 . sin A sin B sin C =

22. Let x

>

1,

y

>

1 and

z >

1 be positive integers for which the following equation 1! + 2! + 3! + . . . + x!=

y

z

is satisfied. Find the largest possible value of x + y+ z. 23. Let A BC be a non-isosceles acute-angled triangle with circumcentre 0, orthocentre H and LC = 41 o. Suppose the bisector of LA passes through the midpoint M of OH. Find LHAO in degrees. 24. The circle 'Yl centred at 01 intersects the circle 'Y2 centred at 02 at two points P and Q. The tangent to 'Y2 at P intersects 'Yl at the point A and the tangent to 'Yl at P intersects 'Y2at the point B where A and B are distinct from P. Suppose PQ · 0102= P01 P02 and LAP B is acute. Determine the size of LAP B in degrees. ·

25. Determine

n

1 . L (n). -+oo . n lim

i=O

( Note: Here

(�) 't

denotes

n�

.1 't. n

(

" ' 't .

)



fori= 0, 1, 2, 3, · · ·

48

,

n.

)

Thus, 30! 30! 107

1 226 . 3 4 . 5 7 . 74 . 112 . 132 . 17 . 19 . 23 . 29 1 1 2 9 . 3 4 . 7 4 . 112 . 132 . 17 . 19 . 23 . 29 1 6 4 . 25 . 74 . 112 . 132 . 17 . 19 . 23 . 29 6 (2) (1) (1) (9) (7) (9) (3) (9) (mod 10) 2 (-1) (-3) (-1) (3) (-1) (mod 10) 8(mod 10),

showing that the last non-zero digit is 8. D

4. Answer.

10

Solution.

Let E be the point inside f:,A BC such that f:,E BC is equilateral. Connect A and D to E respectively. It is clear that f:,AE B and f:,AEC are congruent, since AE = AE , A B = AC and BE= CE. It implies that L BAE = LCAE= lOa . Since A D = BC = BE , LE BA = L D A B = 20a and A B = BA, we have t:,A BE and f:, BA D are congruent, implying that LA B �= L BAE = lOa .

D

5.

Answer.

.

Solutwn.

1 cos2 a sin2 a cos4 a sin4 a . (3 --:--- . Then 13 and sine= ---Smce �(3 + . 2 (3 = 1, set cose= cos cos sm sm c

---

---

cos (e- a )= cose cos a + sine sin a= cos2 a + sin2 a= 1. 50

Now let x2 +ax+ b= (x- xl) (x- x2), where x1:::; x2. Then the set of integer solutions of x2+ ax+ b< 0 is {k : k is an integer, x1 < k< x2}. By the given condition, {k : k is an integer, x1 < k< x2}= {k : k is an integer, -11< k< 6} = {-10,-9, . .· , 5}. Thus -11 :::; x1 < -10 and 5 < x2 :::; 6. It implies that -6 < x1 + x2 < -4 and -66 :::; X1X2< -50. From x2+ax+b= (x-x1) (x-x2), we have a= - (x1+x2) and b= x1x2. Thus 4< a< 6 and -66:::; b<-50. It follows that 54< a - b< 72. Thus

max:Lia- biJ :::; 71.

It remains to show that it is possible that lla - biJ = 71 for some a and b. Let a= 5 and b= -66. Then x2+ab+b= (x+11) (x-6) and the inequality x2+ab+b< 0 has solutions {x: -11< x< 6}. So the set of integer solutions of x2 + ab + b< 0 is really the set of integers in A n B. Hence 10.

max:Lia- biJ = 71.

Answer. Solution.

D

8 We consider the polynomial P(t)= t3 + bt2 + ct+ d.

Suppose the root of the equation P (t)= 0 are x, y and z. Then

-b= X+y+ Z= 14,

and x3 +y3 + z3 +3d= (x+ y+ z) (x 2 +y2+ z2- xy- xz- yz). Solving for b, c and d, we get b= -14, c= 30 and d= -64. Finally, since t3 - 14t2+30tD 64= 0 implies t= 2 or t= 4 or t 8, we conclude that max{ a, ,B, 'Y}= 8. =

11.

Answer.

38

Solution. Let n be an even positive integer. Then each of the following expresses n as the sum of two odd integers: n= ( n- 15) + 15, ( n - 25) + 25 or ( n - 35) +35. Note that at least one of n - 15, n - 25, n - 35 is divisible by 3, hence n can be expressed as the sum of two composite odd numbers if n > 38. Indeed, it can be verified that 38 cannot be expressed as the sum of two composite odd positive integers. D

12.

Answer.

1936 52

We shall show that I AI bijection from A to B:

=

I BI

(63°)

=

=

34220 by showing that the mapping¢> below is a

First, since { a1, a2, a 3 } E A, we have a1+3 implying that {a1, a2- 2, a 3 - 5} E B.



a2 and a2+4



a 3 , and so a1

<

b3 , we have {b1, b2+2, b3+5}

<

a2-2

<

a 3-5,

It is clear that¢> is injective. It is also surjective, as for any {b1, b2, b3 } A and Hence¢> is a bijection and I AI 16.

Answer.

=

I BI

=

E

B with b1

<

b2

34220.

E

0

32

It is clear that 8(cos 40°) 3 -6 cos 40°+ 1 = 0, since cos 3A = 4 cos3 A-3 cos A . Observe that Solution.

1 3 + 64 sin2 20° 2 2 sin 20° cos 20° 2 6 + 32 (1- cos 40°) 1- cos 40° 1 + cos 40° 8 cos 40°+ 4 + 32 32 cos 400 1- (cos 40°) 2 8 COS 40° + 4- 32 COS 40° + 32 ( COS 40°) 3 + 32 1- (cos 40°) 2 1- 6 cos 40°+8(cos 40°) 3 4x +32 1- (cos 40°)2 32, ----

_

==

where the last step follows from 8(cos 40°) 3 - 6 cos 40° + 1 = 0. 17.

Answer. Solution.

0

6029 Given the original equation f(x2 + x) + 2f(x2-3x + 2) = 9x2- 15x,

we replace x by 1-x and obtain f(x2-3x+ 2) + 2f(x2+ x) = 9 (1-x) 2- 15 (1-x) = 9x2-3x-6. Eliminating f(x2-3x+ 2) from the two equations, we obtain 3f(x2 + x) = 9x2+ 9x- 12, thereby

f (x2+ x) = 3x2 + 3x-4 = 3 (x2 + x)- 4,

hence f (2011) = 3(2011)- 4 = 6029. 0

54

20.

Answer.

95004

Solution. We shall prove that for any positive integer nonnegative integer solutions to L�J= La�lJ, then

a, if f(a) denotes the sum of all

f(a)= 16a(a2-1)(a+ 2).

Thus

!(27)= 95004.

Let n be a solution to L�J = La�1J . Write n= aq + r, where 0::; r
A= LL(qa+r) q=O r=q a-1

a-1a-1

= L(a-q)qa+ LLr = = =

q=O r=q a-1

q=O a-1

a-1

q=O a-1

q=O a-1

r=O q=O a-1

q=O a-1

q=O a-1

r=O a-1

a-1

q=O

q=O

r=O

r=O

r

a2L:q-aL q2+ LLr a2L:q-aL q2+ L:r(r+1) a2L q-aL q2+ Lr2+ Lr

1) (a2+ 1 ) · �a( a-1 ) + (1- a)· �a(2a-1)(a2 6 1 = 6a(a2-1)(a+ 2). =

D

21.

Answer.

48

By using factor formulae and double angle formulae: sin A + sinE + sinG 4cos .:1 cos fl. cos Q 12 2 2 2 - --cosA + cos B + cosC 7' 1+4sin 4 sin � sin � and

A B C 12 . . . . A . B . C smA sm B smC = 8 sm 2 sm 2 sm 2 cos 2 cos 2 cos 2= .

25

Solving these equations, we obtain . A . B . C sm-sm-sm- - 0 .1

2

2

2

B C A cos 2 cos 2 cos 2 Furthermore, sin

C

2 = cos

(

A+ B

2

)

= cos 56

A

0. 6

B

A

B

2 cos 2 - sin 2 sin 2'

p

L

be the midpoint of BC . It is a known fact that AH = 20£. To see this, extend meeting the circumcircle of the triangle A BC at the point N. Then AN BH is a parallelogram. Thus AH = NB = 20£. Therefore in the right-angled triangle OLC, OC = OA = AH = 20£. This implies L.OCL = 30°. Since the triangle A BC is acute, the circumcentre 0 lies inside the triangle. In fact L.A = 60° and L.B = 79°. Then L.OAC = L.OCA = 41°-30° = 11°. Consequently, L.HAO = 2L.OAM = 2 X (30°- 11°) = 38°. D Let

CO

24.

Answer.

30

Let P01 = r1 and P02 = r2. First note that 0102 intersects PQ at the midpoint H (not shown in the figure) of PQ perpendicularly. Next observe that L.APQ = L.PB Q = L.P0201, and L.BPQ = L.PA Q = L.P0102. Therefore L.APB = L.APQ + L.BPQ = L.P0201 + L.P0102.

'/'1

'/'2

H and sin f3 = Let L.P0201 = a and L.P01 02 = f3. Then sin a = PrQ, cos a = 02 r 2 2 2 fr�, cos f3 = 0f1H. Thus sin L.APB = sin ( a + /3) = sin a cos f3+ cos a sin f3 = fr� · 0f1H +

25.

0r2H. E9.. 2 2r1

=

Answer.

2

Solution.

PQ(· 01 H+02H) = PQ0 · 102 2r1r2 = 21. Since L.APB is acute it is equal to 30°. 2r1r2 '

Let

Assume that

n



3. It is clear that

() � ; -1

an=

2+

n

�=1

58

-1 rel="nofollow">

2.

·

D

Singapore Mathematical Society Singapore Mathematical Olympiad (SMO)

2011

(Open Section, Round 2) Saturday, 2 July 2011

0900-1330

1. In the acute-angled non-isosceles triangle ABC, 0 is its circumcentre, His its orthocentre and AB > AC. Let Q be a point on AC such that the extension of HQ meets the extension of BC at the point P. Suppose BD= DP, where Dis the foot of the perpendicular from A onto BC. Prove that LODQ = 90°. 2. If 46 squares are colored red in a 9 x 9 board, show that there is a 2 x 2 block on the board in which at least 3 of the squares are colored red. 3. Let x, y, z > 0 such that

� +� + �

<

x!z·

Show that

2z 2x 2y + + yl + x2 y'l + y2 yl + z2 4.

Find all polynomials P(x) with real coefficients such that P(a)

5.

<

E Z

implies that

a E Z.

Find all pairs of positive integers (m, n) such that m+n-

3mn m+n

60

2011 3

3.

which leads to a contradiction. On the other hand, s uppos e that r1, r3, r5, r7, r9 2:: 6. Then the s um of any 2 cons ecutive ri's is :::; 9. Again we get a contradiction as (r1+r2)+

·

·

·

+(r7+rs)+rg

:::; 4

9+9=45.

x

Let r = 1/x, s= 1/y, t= 1/z. There exis ts a < 1 s uch that r + s + t=a2rst or a(r + s + t)=a3rst. Let a=ar, b=as, c=at. Write a= tan A, b tanB, c = tan C, then A+ B + C 1r. It is clear that 3.

=

=

1 2

- X

1 1 1 + ----= + -;= ::;;;: y1+r2 v1+s2 v1+t2 1 1 1 < + ----= + -;= ::;;;: v1+a2 v1 + b2 v1+c2 = cos A+ cos B+ cos B A+ B + C < 3 cos =�=� x RHS. 2 2 3

LHS=

(

)

2nd soln: Note that

1 1 1 -+ - + x y z

Hence

2x =;;;= y1+x2

<

1 xyz

:::::?

xy+yz+xz

2x = == Jx2+xy+xz+yz

----r=

< -r-::::::==

=

By AM-GM we have

==

<

1.

2x J(x+y)(x + z)

X 2x X -=;= < . + J(x+y)(x+z) - x+y x+z

----r-.;======:=;=

= --

Similarly, z z 2z =;= < -- + --. V( Z+X) (z + y) Z+X Z + y

2y Y + __ Y < __ J(y + z)(y+x) - y + z y + x'

----r-.;======:=;=

=

The des ired inequality then follows by adding up the three inequalities. 4.

Let P(x)=anxn + +a1x+a0. Define Q(x)=P(x + 1) - P(x). Then Q(x) is of degree n- 1. We'll prove by contradiction that IQ(x)l :::; 3 for all x. This will imply that n :::; 1. As s ume that IQ(a)i > 3 for s ome a E R Then IP(a + 1) - P(a)l > 3. Thus there are 3 integers between P(a) and P(a + 1). Hence there exis ts three values ·

·

·

62

Singapore Mathematical Society . Singapore Mathematical Olympiad (SMO)

2010

(Junior Section)

Tuesday, 1 June 20 10

0930-1200

Important:

Answer ALL 35 questions. Enter your answers on the answer sheet provided. For the multiple choice questions, enter your answer on the answer sheet by shading the bubble containing the letter (A, B, C, D or E) corresponding to the correct answer. For the other short questions, write your answer on the answer sheet and shade the appropriate bubbles below your answer. No steps are needed to justify your answers. Each question carries 1 mark. No calculators are allowed.

PLEASE DO NOT TURN OVER UNTIL YOU ARE TOLD TO DO SO.

1

Multiple Choice Questions

1. Among the five real numbers below, which one is the smallest? (A)

v2010;

'JJYJ_J�

(B)

v2009;

(C) 2010;

20!.0MIV\n

(D)

2010 ; 2009

(E)

2009 . 2010

2. Among the five integers below, which one is the largest?

3. Among the four statements on real numbers below, how many of them are correct? "If a
0 then !b < ! a

"If a
"· '

"If a
(A) 0;

4.

(B)

(C) 2;

1;

(D)3;

(E) 4.

What is the largest integer less than or equal to (A) 2009;

(B)

2010;

(C) 2011;

(D) 2012;

.{/(2010)3 +3 x (2010)2+4 x 2010+1? (E) None of the above.

5. The conditions of the road between Town A and Town B can be classified as up slope, horizontal or down slope and total length of each type of road is the same. A cyclist travels from Town A to Town B with uniform speeds 8 km/h, 12 km/h and 24 km/h on the up slope, horizontal and down slope road respectively. What is the average speed of his journey?

12kmfh ----+

Town A

(A) 12 km/h;

(B)� km/h;

Town B

(C)

16 km/h; 2

(D) 17 km/h;

(E) 18 km/h.

6. In the diagram, MBC and f).CDE are equilateral triangles. Given that LEBD LAEB X0, what is the value of x?

=

62° and

=

A

D (A) 100;

(B) 118;

(C) 120;

(D) 122;

(E) 135.

7. A carpenter wishes to cut a wooden 3 x 3 x 3 cube into twenty seven 1 x 1 x 1 cubes. He can do this easily by making 6 cuts through the cube, keeping the pieces together in the cube shape as shown:

. . . . :·

.

.

:

·······.·······.·······

What is the minimum number of cuts needed if he is allowed to rearrange the pieces after each cut? (A) 2;

(B) 3;

(C) 4;

(D) 5;

(E) 6.

8. What is the last digit of 7(77)? (A) 1;

(B) 3;

(C) 5;

(D) 7;

(E) 9.

3

9.

Given that n is an odd integer less than many such integers are there ? (A) 3;

10.

(B)

4;

(C) 5;

(D) 6;

1000 and the product of all its digits is 252.

How

(E) 7.

What is the value of

(A) 451856;

(B)

691962;

(C) 903712;

(D) 1276392;

(E) 1576392.

Short Questions

11.

Let x and y be real numbers satisfying y=

2008x+2009 + 2010x- 2011

2008x+2009 +2010· 2011- 2010x

Find the value of y. For integers ah . . ., an

12.

E {1, 2, 3, . .., 9}, we use the notation a1a •••an to denote the number 2 10n-1a1+10n-2a2+· · · +10an-1+an. For example, when a = 2 and b = 0, ab denotes the number 20. Given that ab = b2 and ache = (ba)2• Find the value of abc.

13.

Given that (m- 2) is a positive integer and it is also a factor of 3m2- 2m+10. Find the sum of all such values of m.

14.

In triangle ABC, AB = 32 em, AC = find the length of AM in em.

36 em and BC = 44 em.

A

B

M

4

c

If M is the midpoint of BC,

15. Evaluate

678+690+702+ 714+...+ 1998+2010 3+9+15+21+... + 327+ 333

16. Esther and Frida are supposed to fill a rectangular array of 16 columns and 10 rows, with the numbers 1 to 160. Esther chose to do it row-wise so that the first row is numbered 1, 2, ..., 16 and the second row is 17, 18, . .., 32 and so on. Frida chose to do it column­ wise, so that her first column has 1, 2, . .., 10, and the second column has 1 1, 12, ..., 20 and so on. Comparing Esther's array with Frida's array, we notice that some numbers occupy the same position. Find the sum of the numbers in these positions.

... .. . ... ... ... ... ... ... 145 146 147 ... 2 18 ... .

1 17

.

3 19

.

... 16 . . . 32 . . . .. . . ... .. . 160 .

.

1 2 .. . .. .

11 12 ... . ..

10

20

.. ... . .. .. . 30 ...

21 22 ... ...

.

... ... .. . ... ...

151 152 .. .. .

.

160

Frida

Esther

17. The sum of two integers A and B is 2010. If the lowest common multiple of A and B is 14807, write down the larger of the two integers A or B.

18. A sequence of polynomials an(x) are defined recursively by a0(x) a1 (x) an(X)

=

=

=

1, x2+x+1, (� + 1)an-I(X)- an_z(X), for all n 2 2.

For example,

az(x) a3(x)

=

=

=

(�+1)(�+x+ 1)- 1 x4+x3 +2x2+x, (� +1)(x4+x3 + 2�+x) (� +x+1) x7 +x6+2x5 +2x4 + x3 +�- 1. =

-

Evaluate a 010(1). 2 19. A triangle ABC is inscribed in a semicircle of radius 5. If AB of s2 where s AC+BC. =

5

=

10, find the maximum value

102009>.

20.

Find the last two digits of 2011<20

21.

Your national football coach brought a squad of 18 players to the 2010 World Cup, consisting of 3 goalkeepers, 5 defenders, 5 midfielders and 5 strikers. Midfielders are versatile enough to play as both defenders and midfielders, while the other players can only play in their designated positions. How many possible teams of 1 goalkeeper, 4 defenders, 4 midfielders and 2 strikers can the coach field?

22. 23.

Given that value of x.

169(157-77x)2 +100(201-100x)2 = 26(77x-157)(1000x-2010),

Evaluate

find the

(20202-20100)(201002-1002)(20002+20100) 20106-106

24.

When 15 is added to a number x, it becomes a square number. When 74 is subtracted from x, the result is again a square number. Find the number x.

25.

Given that x and y are positive integers such that 56::::; x+y::::;

26.

the value of f-x2.

59 and 0.9

<

:: y

<

0.91, find

Let AA' and BB' be two line segments which are perpendicular to A' B'. The lengths of AA', BB' and A' B' are 680, 2000 and 2010 respectively. Find the minimal length of AX+XB where X is a point between A' and B'.

B

I

0 0 0 C'l

0 00 1,0

l

A'

27.

The product 1 x 2 x 3 x M= 1! x 2! x 3! x 4! x 5! ·

·

·

2010

n is denoted by n!. For example 4! = 1 x 2 x 3 x 4 = 24. Let 6! x 7! x 8! X 9!. How many factors of Mare perfect squares?

x

x

1

B'

6

28. Starting from any of the L's, the word LEVEL can be spelled by moving either up, down, left or right to an adjacent letter. If the same letter may be used twice in each spell, how many different ways are there to spell the word LEVEL?

L

L

L

E

L

E

v

E

L

E

L

L

'

L ...___

29. Let ABCD be a rectangle with AB 10. Draw circles C1 and C2 with diameters AB and CD respectively. Let P, Q be the intersection points of C1 and C2. If the circle with diameter PQ is tangent to AB and CD, then what is the area of the shaded region? =

30. Find the least prime factor of

10000 .. ·00 1. ""---v-"

2010-many

7





31. Consider the identity 1 + 2 +· · · + n = n(n+ 1). If we set P1(x) = x(x + 1), then it is the unique polynomial such that for all positive integer n, P1 (n)=1 + 2 +· · · + n. In general, for each positive integer k, there is a unique polynomial P (x) such that k P (n) =1k + 2k + 3 k + ··· + nk k Find the value of P2o10C

-

�)

for each n=1, 2, . . . .

.

32. Given that ABCD is a square. Points E and F lie on the side BC and CD respectively. such that BE=CF = AB. G is the intersection of BF and DE. If



Area of ABGD m ----- = Area of ABCD n is in its lowest term, find the value of m + n.

A

.-------�

B E

F

C

33. It is known that there is only one pair of positive integers a and b such that a :::; b and a2 + b2 + 8ab=2010. Find the value of a + b. 34. The digits of the number 123456789 can be rearranged to form a number that is divisible by 11. For example, 123475869, 459267831 and 987453126. How many such numbers are there? 35. Suppose the three sides of a triangular field are all integers, and its area equals the perimeter (in numbers). What is the largest possible area of the field?

8

Singapore Mathematical Society Singapore Mathematical Olympiad (SMO) 2010 (Junior Section Solutions) 1. Ans: (E) It is the only number less than 1. 2. Ans: (D) Other than (D), all numbers are less than 20102010. Now 37 9 3(3 ), the result follows.

>

2010. Thus 20102010

<

(37l

<

3. Ans: (B) Only the third statement is correct: a < b implies a+ c < b + c. For other statements, counterexamples can be taken as a= -1, b= 1; c= 0 and a= 0, b= -1 respectively.

4. Ans: (C) Since (2010+ 1)3= 20103 + 3 x 20102+ 3 x 2010+ 1. The result follows.

5. Ans: (A)

Let the distance between Town A and Town B be 3s. The total time taken for up slope, horizontal and down slope road are �, { and ;4 respectively. His average speed for the 2 3s . 1 . km/h. Wh 0 e JOUrney IS s s s = 12 8 + 2 + 24 1

6. Ans: (D)Observe that !:l.BCD is congruent to !:l.ACE (using SAS, BC= AC, CD= CE and LACE= 60° - LECB= LBCD). Thus LAEC= LBDC. We get x

o

= = = =

360° - LAEC- LBEC 360° - LBDC- LBEC

LEBD+ LECD 62° + 60°= 122°

Thus x= 122.

9

A

D

7.

Ans: (E)

There is no way to reduce the cuts to fewer than 6: Just consider the middle cube (the one which has no exposed surfaces in the beginning), each of the its sides requires at least one cut.

8. Ans: (B)

The last digit of 7k is 1, 7, 9, 3 respectively for k= 0, 1, 2, 3 (mod Since 77 = (-1? = 3 (m�d

9.

Ans: (C)

252 = 2 x 2 x 3

10.

x

3

x

4), the last digit of 7 <77) is 3.

4).

7. We can have: 667, 497, 479, 947, 749.

Ans: (C) We only need a rough estimate to rule out the wrong answers. -fiT� 3.3 and -Y5� 2.2, so the sum is� 5 . 58 + (1.1)8 � 5. 58 = 30.254 � 304 � 810000. Thus (C). Of course the exact answer can be obtained by calculations, for example,

t (�)< «t")l( �)8-1 t (�)< «t")'(- �)8-1 +

=2

.I (�)( m)i( vs)8-i

zeven

10

1 1. Ans: 20 10. Let

a=

2008x+ 2009 20 10x- 20 11·

Then a � 0 and- a � 0 since they are under the square root. Hence a= 0. Thus y= 20 10.

12. Ans: 369.

It is easy to see that b= 5, ab= 25 or b= 6, ab= 36. Upon checking, b = 6, a= 3 and so 2 (b a?= 63 = 3969. Therefore c = 9. Hence abc= 369.

13. Ans: 5 1. Since

2 3m - 2m+ 10

18 m- 2 m- 2 is an integer. Thus m- 2 is a factor of 18. m- 2= 1, 2, 3, 6, 9 , 18, thus m = 3, 4, 5, 8, 11, 20. The required sum is 5 1. -----

14. Ans: 26. Using the Median Formula,A_MZ

=

= 3m+ 4+

2 2 2 2AB + 2AC - BC ·

4

--

. ThusA M= 26 em.

15. Ans: 16. Note that both numerator and denominator are arithmetic progressions. Removing common factors gives

2X

113+ 115+ 117+ ... + 333 + 335 1+ 3+ 5 + ...+ 109+ 111

2 2 X 112 (335 + 1 13) = 5; (111+ 1)

:::::

16.

16. Ans: 322. The number in the r-th row and c-th column has value 16(r- 1) + c in Esther's array and value 10(c- 1) + r in Frida's array. So we need to solve

16(r- 1) + c= 10(c- 1)+ r



5r= 3c + 2.

There are exactly four solutions

(r, c)= {(1, 1), (4, 6), (7, 11), (10, 16)}.

11

17. Ans: 1139. A direct way to solve the problem is to factor 14807 directly. Alternatively, one may hope for A and B to have common factors to simplify the problem. This is a good strategy because of the following fact: "The greatest common divisor of A and B, equals the greatest common divisor of A + B and lcm(A, B). "

2010 is easily factored as 2 X 3 x 5 x 67. Checking that 67 is also a factor of 14807, we can conclude that 67 is also a factor of A and B. The problem is reduced to finding a and b such that 2010 1 07 =30 and ab= a + b= =221. 67 7 Since 221 can be factored easily, a andb must be 13 and 17. So the answer is 17x67=1139.



18. Ans: 4021. an(1) is the simple recurrence relation fn = 2fn-1 - fn- • fo = 1 and !1 = 3. Using standard 2 technique or simply guess and verify that fn=2n + 1. So a oio(1)=f2010 = 2(2010) + 1. 2 19. Ans: 200. ABC must be a right-angled triangle. Let x = AC and y = BC , by Pythagoras theorem r + r =102•

i =(x + y)2 =r

+ y2 + 2xy=100 + 2 x

area of ABC.

Maximum area occurs when x=y, i.e. LC AB=45°. So x=y= -{50.

20. Ans: 01. Note that 2011 = 11 (mod 100) and 112 = 21,11 3 (mod 100). Since 20102009 is divisible by 10,

20112 0102009

=

1110 x

·

·

·

x 1110

=

=

31 (mod 100) etc. So 1110

1

_

1

(mod 100).

21. Ans: 2250.

(i) x (;) x (�) choices for goalkeepers, strikers and midfielders respectively. The remaining

midfielder and defenders can all play as defenders, hence total number of possibilities are

3

X

lOX 5

12

X

(�)

=2250.

22. Ans: 31.

Let a= 1001x '- 2041 and b= 1000x- 2010.

Then the equation becomes a2+b2= 2ab. Thus (a- b?= 0. The result follows. 23. Ans: 100. Let x= 2010 and y= 10. The numerator becomes [(x+ y)2- xy] (�i ·

-

ii )

·

[(x - y)2+ xy]

= (� + xy + i ) iCx- y)(x+ y) (� - xy + i ) = Y2(� -l)(x3 +l) = y2(x6 -l). ·

·

Hence the answer is 100. 24. Ans: 2010. Let 15+x= m 2 and x-74= n2• We havem 2- n2= 89= 1x89. (m-n)(m+n)= 1x89. Let m-n= 1 and m+n= 89. Solving m = 45 and n= 44. Thus the number x is 452-15= 2010. 25. Ans: 177. From 0.9y < x < 0.91y, we get 0.9y + y < x + y < 0.91y + y. Thus 0.9y + y < 59 and 0.91y + y> 56. It follows that y < 31.05 and y> 29.3. Thus y= 30 or 31. If y= 30, then 27 < x < 27.3, no integer value of x. If y= 31, then 27.9 < x < 28.21, thus x= 28. Thus y2 - x2= (31 + 28)(31- 28)= 177. 26. Ans: 3350. Take the reflection with respect to A' B'. Let A" be the image of A. Then the minimal length is equal to the length of A" B = 20102 + (2000+ 680)2= 3350.

.V

27. Ans: 672. M

= l!x2!x3!x4!x5!x6!x7!x8!x9! = 28 X 37 X 46 X 55 X 64 X 73 X 82 X 9 = 230 X 313 X 55 X 73

A perfect square factor of M must be of the form 22xx32Yx52zx72w, where x, y, z and ware whole numbers such that 2x � 30, 2y � 13, 2z � 5, 2w � 3. Hence, the number of perfect square factors of M is 16x7x3x2= 672. 13

28. Ans: 144. There are total 12 ways starting from any of the L's to reach the middle V. Hence the total number of ways to spell the word LEVEL is 122 = 144. 29. Ans: 25. Let N be the midpoint of CD. Then LPNQ = 90°. So PQ = 5 Vi.

B

A

D

N

c

Then the area of the shaded region is

30. Ans: 11. Clearly 11 is a factor and 2, 3, 5 are not. We only need to rule out 7. 102011+1 = 4 (mod 7) because 103 = -1 (mod 7). 31. Ans: 0. Let k be a positive even numb�r.

14

k Define j(x) = Pk(x) - Pk(x- 1). Then j(n) = n for all integer n � 2. Note thatj is a . polynomial. We must have j(x) = xk. In particular, for integers n � 2, Pk(-n+ 1)- Pk(-n)

=

Pk(O)- Pk(-1)

=

j(-n+ 1)

=

(n- 1)\

Pk(-n+ 2)- Pk(-n+ 1) = j(-n+ 2) = (n- 2i,

Pk(1)- Pk(O)

k O , k j(1) = 1 . j(O)

=

=

k k k Summing these equalities, Pk(l)- Pk(-n) = 1 + O + 1 +

Define g(x) = Pk(-x) + Pk(x - 1). Then g(n) polynomial, g(x) = 0. In particular, Pk(

-�)+ Pk( -�)

32. Ans: 23. Join BD and CG and note that

=

0, i.e., Pk( -�)

��

=

A

=

=

·

·

·

+ (n- 1)k. That is,

0 for all integers n � 2. Since g is a

0.

2.

....-----.

B E

F

Assume the length of AB is 1. Let the area of ABGE and AFGC be x andy respectively.

Then the areas of t1EGC and ADGF are 2x arid 2y. Since the area of ABFC is



· we have

1 1 2 1 . . 3x+ y= 6. Smnlarly, 3y+ 2x=the area of AVEC= 3. Solve x = andy= . Thus 42 21 Area of ABGD Area of ABCD

=1

So m= 9 and n= 14. The result follows.

15

_

1 3( x + y)=l _ 5 42

=

�· 14

33. Ans: 42. Since a � 1,20 10 =a2+b2+8ab � 1+b2+8b. b2+8b-2009 � 0. However b2+8b-2009=0 has an integer solution 41. So a= 1 and b=41. The result follows. 34. Ans: 3 1680. Let X and Y be the sum of the digits at even and odd positions respectively. Note that 1 + 2 + 3 + + 9=45. We have X+ Y=45 and 11 divides IX- Yl. It's easy to see X= 17 and Y = 28; or X = 28 and Y = 17. Hence we split the digits into 2 sets whose sum is 17 and 28 respectively. ·

·

·

There are 9 ways for 4 digits to sum to 17: {9,5,2, 1}, {9,4,3, 1}, {8,6 ,2, 1}, {8,5,3, 1}, {8,4,3,2}, {7,6,3, 1}, {7,5,4, 1}, {7,5,3,2}, {6,5,4,2}. There are 2 ways for 4 digits to sum to 28: {9,8,7,4}, {9,8,6,5}. Thus the total number of ways is 11 x 4! X 5! =3 1680. 35. Ans. 60. Let the three sides of the triangle be a,b,c respectively. Then

where s=



a+ + c

�s(s - a)(s - b)(s - c)=a+ b + c=2s, . Note that s is an integer; otherwise s(s-a)(s-b)(s-a) is a non-integer.

Let x=s - a, y= s - b and z=s - c. Then x,y,z are positive integers satisfying xyz=4(x + y + z). Assume that x � y � z. Then xyz � 12x, i.e. , yz � 12, and thus z � 3. If z = 1, xy = 4(x + y + 1) implies (x - 4)(y - 4) = 20 = 20 (x,y)=(24,5),( 14,6),(9,8).

·

1

= 10 2 = 5 4. So ·

·

If z=2, 2xy=4(x+ y+ 2) implies (x- 2)(y- 2)=8=8· 1=4 2. So (x,y) =( 10 ,3 ),(6,4). ·

If z=3, 3xy=4(x + y + 3) implies (3x - 4)(3y - 4)=52, which has no solution x � y � 3. The area is 60, 42, 36, 30 , 24, respectively; and the largest possible value is 60.

16

Singapore Mathematical Society Singapore Mathematical Olympiad (SMO)

2010 -'

(Junior Section, Round 2) Saturday, 25 June 2010

0930-1230

INSTRUCTIONS TO CONTESTANTS

1.

2. 3.

Answer ALL 5 questions. Show all the steps in your working. Each question carries

10

mark.

4. No calculators are allowed.

ABC D intersect at S and let P be the midpoint of AB. Let M be the intersection of AC and PD and N the intersection of BD and PC. A circle is incribed in the quadrilateral PMSN. Prove that the radius of the circle is MP- MS.

1. Let the diagonals of the square

2. F ind the sum of all the 5-digit integers which are not multiples of 1 1 and whose

digits are 1, 3,

4,

7, 9.

3. Let a1, a2, . . . , an be positive integers, not necessarily distinct but with at least five

si <j s

distinct values. Suppose that for any 1 from

i

and

j such that ai + aj



ak +

a.e.

n,

there exist k, £, both different

What is the smallest possible value of

n? 4. A student divides an integer that

m

by a positive integer m ____: n

=

0

·

167ala2

·

·

·

n,

where

n

s 100, and claims

.

Show the student must be wrong. 5. The numbers

f, �, . .. , 20\ 0

are written on a blackboard.

A student chooses any

two of the numbers, say x, y, erases them and then writes down x

+y+

xy.

He

continues to do this until only one number is left on the blackboard. What is this

number?

17

Singapore Mathematical Society Singapore Mathematical Olympiad (SMO) (Junior Section, Round

2

2010

solutions)

Let 0 be the centre and r the radius of the circle. Let X, Y be its points of contact with the sides PM , MS, respectively. 1.

Since OY j_ MS and LYSO= LASP=45°, SY=YO=r. Also LOPX = LPDA ( since O P II DA) and LOXP= LPAD= 90°. Therefore 60X P c:::: 6 PAD. Hence OX/XP = PA/AD= 1/2. Hence PX = 2r. Therefore PM - MS = 2r + MX MY-r=r. c

D

p

A

B

First note that an integer is divisible by 11 if and only if the alternating sum of the digits is divisible by 11. In our case, these are the integers where 1,4 and 7 are at the odd positions. Let S be the sum of all the 5-digit integers formed by 1, 3, 4, 7, 9 and let T be the sum of those which are multiples of 11. Then 2.

s=4!(1 +

3 + 4 + 7 + 9)(1 + 10 + 100 + 1000 + 10000)

=6399936

T=2!2!(1 + 4 + 7)(1 + 100 + 10000) + 3!(3 + 9)(10 + 1000)=557568. Thus the sum is 6399936-557568=5842368. are the two smallest values. Then a1 = x and let s be the smallest index such that a8 = y. Now there are two other terms whose sum is x + y. Thus we have a2 = x and as+I = y. Since a1 + a2 = 2x, we must have a3 = a4 = x. Similarly, by considering the largest two values w < z, we have an =an-I =an-2 = an-3 = z and another two terms equal tow. Since there is one other value, there are at least 4 + 2 + 4 + 2 + 1=13 terms. The following 13 numbers 3.

a1 :::; a2 :::;

·

·

·

:::; an. Suppose

x

< y

18

satisfy the required property: 1, 1, 1, 1, 2, value of n is 13.

4. We have

m < 0 167 < 0 168 n ·

·

2, 3, 4, 4, 5, 5, 5, 5.

Thus the smallest possible

167n::; 1000m < 168n.

=>

Multiply by 6, we get

1002n::; 6000m < 1008n

6000m- 1000n < 8n::; 800.

=>

But 6000m -lOOOn 2': 2n > 0. Thus 6000m - 1000n 2': 1000 since it is a multiple of

1000. We thus get a contradiction.

We shall prove by induction that if the original numbers are

5.

the last number is

(1 + a1)

·

·

·

n 2': 2, then

(1 + an) - 1.

The assertion is certainly true for n

=

2, the base case. Now suppose it a1, ..., ak+l written on the board.

is true for

k + 1 numbers After one k numbers. Without loss of generality, we can assume that the student erases ak and ak+l and writes bk ak+ak+l +akak+l (1+ak)(1+ak+l)-l. After a further k operations, we are left with the number n

=

k

a1, ..., an,

2': 2.

Consider

operation, we are left with

=

=

This completes the proof of the inductive step. Thus the last number is

( 1 + �) ( 1 + �) ... ( 1 + 20 10 ) - 1 1

19

=

20 10

Singapore Mathematical Society Singapore Mathematical Oly mpiad (SMO) 2010 (Senior Section)

Tuesday, 1 June 2010

0930 - 1200 hrs

Important: Answer ALL 35 questions. Enter your answers on the answer sheet provided. For the multiple choice questions, enter your answer on the answer sheet by shading the bubble containing the letter (A, B, C, D or E) corresponding to the correct answer. For the other short questions, write your answer on the answer sheet and shade the appropriate bubble below your answer. No steps are needed to justify your answers. Each question carries 1 mark. No calculators are allowed.

PLEASE DO NOT TURN OVER UNTIL YOU ARE TOLD TO DO SO

20

Multiple Choice Questions

1.

. F mdthevalue of (A)

(B) (C)

2.

1

(1x2x3)+ (2 x4x6) + (3x6x9)+···+ (335 x670 x1005) (1x3x6)+ (2x 6xl2)+ (3x9x18) +··· + (335 x1005 x20 10)

-

3 2 3 1 -

6

(D)

1 2

(E)

-

4 9

If a, b, c anddare real

numbers suchthat

b+c+d= a+c+d= a +b +d= a +b+c = r, a d b c findthevalue of r. (A)

(B) (C) (D) (E)

3.

If

3 1 -1 3 or 1 3 or-1

1 a a nd tan x+- - =--,where a, b a nd c 2 4 tan x b-;rc are positiv e integers, find thevalue of a+ b +c. O< x<

(A)

(B) (C) . (D) (E)

;r

a nd sin x-cos x=

7r

8

32 34 48 50

21

(A) (B) (C) (D) (E)

5.

104 224 312

336 676

In the figure below, ABC is an isosceles triangle inscribed in a circle with centre 0 and diameter AD, with AB = AC. AD intersects BC atE , and F is the midpoint of OE. Given that BD is parallel to.FC and BC = in em. (A)

(B) (C)

(D) (E)

6.

D A

Find the number of ordered pairs (x, y), where x is an integer and y is a perfect square, such that y = (x-90)2 -4907. (A) (B) (C) (D) (E)

7.

3/5 2 J6 2J3 J7 2./6

2J5 em ,find the length of CD

0

1 2 3

4

LetS= {1, 2, 3, . , 9,10 }. A non-empty subset ofSis considered "Good" if the number of even integers in the subset is more than or equal to the number of odd integers in the same subset. For example, the subsets {4, 8}, {3, 4, 7, 8} and {1, 3, 6, 8,10} are "Good". How many subsets ofSare "Good"? . .

(A) (B) (C) (D) (E)

482 507 575

637 667

22

8.

If the graph of a quadratic function f (x) = ax

2 + bx + c (a i- 0) passes through

two distinct points (r, k) and (s, k), what is j(r + s)? (A)

(B) (C)

(D) (E)

9.

None of the above

Find the number of positive integers k < 100 such that 2(36n ) + k (23n +l) -1 is divisible by 7 for any positive integer n. (A)

10

(B)

12

(C)

13 14 16

(D) (E) 10.

2k c k-c 2k - c

Let

ABCD be

a trapezium with AD parallel to BC and LADC = 90° , as shown in

the figure below. Given that M is the midpoint of AB with CM = BC + CD+ DA

(A)

(B) (C) (D) (E)

=

13

2

em

17 em, fmd the area of the trapezium ABCD in cd.

26 28 30

A

,------,.-,

33 35

B

23

D

c

and

Short Questions

11.

12.

The area of a rectangle remains unchanged when either its length is increased by 6 units and width decreased by 2 units, or its length decreased by 12 units and its width increased by 6 units. If the perimeter of the original rectangle is x units, find the value of x. For r = 1, 2, 3, ..., let

2

1 -+

u,

=1 + 2+3 + ... + r. Find the value of 3 100 + ··· + + . + + + ··· +

(:J (: + J (: : :,) (: : .l

,

13.

If

14.

If

15.

2010!=Mx10k,

,

+

,

,

,

.�.J

.

where Mis an integer not divisible by 10, find the value of k.

1 1 a > b > 1 and -- + -- = -J1229 , find the value of log b 1ogb a a

rl

1 ·

logab b

1

logab a

For any real number X, let X denote the smallest integer that is greater than or

LxJ denote the largest integer that is less than or equal to x (for J1.23l = 2 and LL23J =1). Find the value of

equal to x and example,

I: r 2010 _ 2o10 o

k=ll

x2o1o

l Jl · k

k

.

Fmd the value of + (1_x)2010•

16.

Let f ( x)

17.

If a, b and c are positive real numbers such that

=

x2o1o

ab + a + b = be+b +c ca +c +a = 35, find the value of (a+ 1)(b + 1)(c + 1). =

24

18.

In the figure below, AB and CD are parallel chords of a circle with centre 0 and

radius rem. It is given that AB = 46 em, CD= 18 em and LAOB= 3 x LCOD. Find the value of r. A

B

19.

Find the number of ways that 2010 can be written as a sum of one or more positive integers in non.,.decreasing order such that the difference between the last term and the first term is at most 1.

20.

Find the largest possible value of n such that there exist n consecutive positive integers whose sum is equal to 2010. ·

21.

Determine the number of pairs of positive integers n and m such that 2 1! + 2! + 3! + . + n! = m • .

22.

·

.

The figure below shows a circle with diameter AB. C and D are points on the circle on the same side of AB such that BD bisects LCBA. The chords AC and BD intersect at E. It is given that AE = 169 em and EC = 119 em. If ED = x em, find the value of x.

25

23.

Find the number of ordered pairs (m, n) of positive integers in and n such that m + n = 190 and m and n are relatively prime.

24.

Find the least possible value of j(x) =

25.

Find the number of ways of arranging 13 identical blue balls and 5 identical red balls on a straight line such that between any 2 red balls there is at least 1 blue ball.

26.

LetS= {1, 2, 3, 4, . . . , 100000}. Find the least possible value of k such that any subset A ofSwith IAI = 2010 contains two distinct numbers a and b with Ia- bl::; k.

27.

Find the number of ways of traveling from A to B , as shown in the figure below, if you are only allowed to walk east or north along the grid, and avoiding all the 4 points marked x.

9

1+cos2x over all real numbers for which j(x) is defined.

+

25

1-cos2x

, where x ranges

North

L

East

A

28.

Two circles C1 and C2 of radii 10 em and 8 em respectively are tangent to each other internally at a point A. AD is the diameter of C1 and P and Mare points on C1 and Cz respectively such that PMis tangent to Cz, as shown in the figure below. If PM=

.fiO em and

LPAD

=

xo, find the value of x.

26

29.

30.

31.

Let a, b and cbe integers with a > b > c > 0. If b and c are relatively prime, b + c is a multiple of a, and a+ cis a multiple of b, determine the value of abc. Find the number of subsets {a, b, c} of {1, 2, 3, 4, ..., 20} such that a< b - 1 < c- 3. Let f(n) denote the number ofO's in the decimal representation ofthe positive

integer n. For example-, /(10001 123) = 3 and /(1234567) = 0. Let M = /(1)x2f(I)

+

j(2)x21<2>

+

/(3)x21<3>

+ . . · + f(99999)x21<9999� .

Find the value of M- 100000.

32.

33.

Determine the odd prime number p such that the sum of digits of the number p4 -5 p 2 + 13 is the smallest possible . The figure below shows a trapezium ABCD in which AD II BC and BC = 3AD. F is the midpoint of AB and E lies on BC extended so that BC = 3CE. The line segments EF and CD meet at the point G. It is given that the area of triangle GCE is 15 cm2 and the area of trapezium ABCD is k cm2• Find the value of k.

c

B

34.

Let P (x)= a0 + a1x + a x2 +

E

+

anxn be a polynomial in x where the coefficients a0, a1, a ,•••, an are non-negative integers . If P (1) 25 and P (27) 1771769 , 2 2

· · ·

find the value of ao + 2ar + 3� + ... + (n + 1)an.

27

=

=

35.

Let three circles rl' r2' r3 with centres At, Az, A3 and radii Tt, rz, r3 respectively be mutually tangent to each other externally. Suppose that the tangent to the circumcircle of the triangle A1AzA3 at A3 and the two external common tangents of r1 and r2 meet at a common point P, as shown in the figure below. Given that r1 = 18 em, r2 = 8 e m and r3 = k em," find the value of k.

p

28

Singapore Mathematical Society Singapore Mathematical Olympiad (SMO) 2010 (Senior Section Solutions)

1.

Answer: (A)

(1x2x3) + (2x 4x6) + (3x6x 9) +···+ (335 x 670x 1005) (1x3x 6) + (2x 6x12) + (3x 9x 18) + ···+ (335 x1005 x2010) =

=

2.

(1x2x3)[13 +23 +33 +···+3353] (1x3x6)[13 +23 +33 +···+3353]

�--��--��------�

1x2x3 1 = 1x3x6 3

Answer: (E) From the given equations, we obtain

a + b +c +d = a(r +1), a + b +c +d = b (r +1), a +b +c +d =c(r +1), a +b +c +d =d (r +1). Adding these four equations gives

4(a + b +c +d) = (a + b +c +d)(r +1), that is,

(3-r)(a +b +c +d) =0.

Thus r = 3, or a +b +c +d =0. If a +b +c +d =0, then we see from the original given equations that r = -1 Hence the value of r is either 3 or -1

.

.

3.

Answer: (E)

2

1[ . xcos x= . x-cosx)2 =-, We have (sm which .rmp1"1es that . sm 16

Therefore we obtain

tanx+

1 --

=

tan x

sin x --

cos x

cos x

1

sin x

sin xcos x

+--=

=

32 16-:r2

Hence a+ b+ c = 32+ 16+ 2 = 50 .

4.

Answer: (C) First, we note that 4n3

=

[n(n+ 1)]2 - [n(n- 1)]2 • Thus

29

.

16 -1[2 ----

32

Therefore 143+ 153 + . . . + 243 + 253 142 x152 132 x142 + = 4 4 152 x162 142 x152 + . . .. + 4 4 242 X 252 232 X 242 + 4 4 252 X 262 242 X 2s2 ---

��-

=

4 252 x26 2

4 132 x142

4 = (32 X 13)(18 Thus

5.

= (25 x 13+ 13 x7)(25 x 13-13 x7) 4 X 13) = 9 X 64 X 13 2 •

�143 +153 +163 +...+243 +253

= 3x8x 13 = .312.

Answer: (B) Since the diameter AD perpendicularly bisects the chord BC,

BE = EC =

.JS.

Also, given that BD II FC, we have LDBE = LFCE. Thus MDE is congruent to !1CFE, so DE = FE. As F is the midpoint of OE, we have OF = FE = ED. Let OF = x. Then AE = 5x. Using Intersection Chord Theorem, we have

AExED = BExEC, which leads to 5x2 = 5. Consequently we obtain x = 1. Now CD2 = C£2+ ED2 gives CD =

6.

.J5+1= ../6 .

Answer: (E) Let y = m2 and (x-90)2 = k?-, where m and k are positive integers. Then we obtain k 2 -m2 =4907 =7x701=1x4907, which gives (k -m)(k+m) =7x701 or (k-m)(k+m) =1x4907. It follows that k-m = 7 and k+ m = 701, or k-m = 1 and k+ m = 4907. Solving these two pairs of equations gives (k, m) = (354, 347) and (k, m) = (2454, 2453).

30

Therefore the ordered pairs (x, y) that satisfy the given equation are: (444, 3472 ), (-264, 3472 ), (2544, 24532 ), (-2364, 24532 ). Hence the answer is 4.

7.

Answer: (D) Let the number of even integers in a "Good" subset of S be i, where i = 1, 2, 3, 4, 5, and the number of odd integers in that subset be j, where j 0, 1, 2, ... , i. Then the number of "Good" subsets of S is =

t<(:J�(�) (:}(�) (:) +(�}(�) (:) (�) (�)<(�) (:) G) =

+

+

+

+

+

+. . . +

+. . .

=5(1 + 5) + 10(1 + 5 + 10) + ...+ (1 + 5 + 10 + 10 + 5 + 1) =30 + 160 + 260 + 155 + 32 =637.

8.

Answer: (B) Let g(x) = f (x) -k. Then g(r)

f (r)- k = k-k = 0. Similarly, g(s) = 0. Therefore r ands are roots of the quadratic equation g(x) =ax2 +hx+c-k =0, from which =

b

we deduce that r + s = - -. Hence a

b a

b a

b a

j(r + s ) = j(--) = a (-- )2 +b(--) +c = c.

9.

Answer: (D) We have 2(36n) +k (23n+l) -1= 2(272n) +2k(8n) -1 = 2( -1)2n +2k(ln) -1 (mod 7) = 2k +1 (mod 7). l Thus, for any positive integer n, 2(36n) + k(23n+ ) -1 is divisible by 7 if and only

if 2k +1 = 0 (mod 7 ). As k< 100, it is clear that the congruence holds for k =3, 10, 17, . .., 94. Thus the required number of positive integers k is 14. 10.

Answer: (C)

31

'

i

Extend DA and CM to meet atE as shown in the figure above. Since AM = MB, LAEM = LBCM and LAME = LBMC, we conclude that �M is congruent to tillCM. Therefore AE = BC and CM=EM. Thus CE = 2CM= 13.

Let the area of trapezium ABCD be S cm2 . Then S =_!_ (DE)(DC), and we have

2 (DE + DC)2 =DE2 + DC2 + 2(DE)(DC) =C£_2 + 4S =132 + 4S =169 + 4S. Now DE+DC=DA+AE+DC=DA+BC+DC=17. Hence 172 =169 + 4S, and it follows that S = 30. 11.

Answer: 132 Let the length and width of the original rectangle beL and W respectively. Then

LW= (L + 6)(W -2)

and

LW = (L-12)(W + 6).

Simplifying the above equations, we obtain

L-2W=12

and

3W-L=6 .

Solving the simultaneous equations, we get L=48 and W=18. Hence the perimeter of the rectangle is 132 units.

12.

Answer: As ur

2575

r r + 1) , we have =1 + 2 + 3 + ... + r = ( 2 2 =t �-� =2-__2_=�. =t t_!_ +1 +1 r�l r ( r + 1) r�l r r + 1 r�l ur

(

) i i i:-/ =i:( : J=i:(i +12 )=_!_ ( (n2 1) + n)=�(4 n + 3).

Hence, Sn : =

i i�l i+l

i�l "" __L i�l L...J u, r�l In particular, slOO =2575.

13.

2

n

+

Answer: 501 The number k is the number of the factor 10 that occurs in 2010!. This number is given by the number of pairs of prime factors 2 and 5 in 2010!. Now between 1 and 2010, there are: • • • •

402 integers with 5 as a factor; 80 integers with 25 as a factor; 16 integers with 125 as a factor; 3 integers with 625 as a factor.

32

Therefore the total number of prime factor 5 in 2010! is 402 + 80 + 16 + 3 =501. As there are clearly more than 501 prime factor 2 in 2010! , we obtain k =501.

14.

Answer: 35 First note that since a> b > 1,

1

--

logabb

-

1 logab a

1

1

logabb

logab a

·

>

0. Then

= Iogb ab -loga ab = (Iogb a+ 1) (loga b +1) = logb a-loga b

1

1

loga b

Iogb a

·

=-----

[�-�)' = [�+ : J { : X : J =

-

lo , a

lo . b

lo , a

= ..)1229-4

= .J1225

= 35.

15.

Answer: 1994

l� J

If

= 2010 - 2 10 =0, so Xl= .If k 12010, then X: r O k 2 10 - 2 10 y k ' 2010, then 0 < y:= < 1 so r l =1.

Consider k

=

1, 2, ..., 2010.

� l� J

'

Since the prime factorization of 2010 is 2 x 3 x 5 x 67, we see that 2010 has 16 distinct divisors . Hence

o l2010

Ik=l I 16.

k

l Jl = klL20!0JXl+kl2:20!01 l y

- 2010 k

=number of non-divisor of 2010 among k =2010 -16 =1994.

Answer: 1005 _

Observe that j(x) + j(l -x)-

It follows that

(l- xyolO x2o1o -1 . + x2oJo+ (1-x )2010 (1-x )2010+x20JO _

33

=

17.

1005.

Answer: 216 Adding 1 to both sides of the given equation ab + a + b

=

35, we obtain

(a+ 1)(b+ 1) = 36. Likewise, adding 1 to the other two given equations gives (b+ 1)(c+ 1) = 36 and (c+ 1)(a+ 1) = 36. Now multiplying the three resulting equations above leads to 3 [(a+1)(b+1)(c+1)] 2 = 36 = 66• 3 It follows that (a+1)(b+1)(c+1)=6 =216.

18.

Answer:

27

A

B

Let M and N be the midpoints of AB and CD respectively and let L CON = x. Then LAON=3x and 3 23 AM r s 3x 3sin x � 4sin x = =3_4sin 2 x . = = sm x r sm x 9 CN 23 1 1 1 . Thus sm 2 x = "4 3 g = g , and so sm x= "3 . Hence r=

--

(- )



.

CN =27. sin x

34

19.

Answer: 2010 Consider any integer k where 1 :::; k:::; 2010. By division algorithm, there exists unique pair of integers (q, r) such that 2010 = kq+ r with 0:::; r:::; k- 1. We rewrite this as 2010 = (k- r)q+ r(q+ 1). That is, k- r copies of q and r copies of q+ 1 add up to 2010. Thus there is one desired expression for each value of k, which is clearly unique . Hence there are 2010 such expressions in all .

20.

Answer: 60 Let a be a positive integer such that the sum of n consecutive integers a, a+ 1, ... , a+ (n- 1) is 2010, that is,

a+(a+1)+···+(a+n -1) 2010. =

n(2a+n -1) . =2010, or Th.I S gives

2 n(2a+n -1) =4020=22 x3x 5x67.

(1)

Since n < 2a+ n- 1, we have

n<.J22x3xSx67 =2.JlOOS <2x 32=64.

(2)

Now (1) and (2) imply that n E{1, 2,3,4,S,6,10,12,1S,20,30,60}. Sincenand 2a+ n- 1 have different parities, it follows that nand we have If n

21.

=

4020 n

have differ�nt parities. Consequently,

n E{1,3,4,S,12,1S,20,60}.

60, then 2a+n -1 =

4020 =67, so a 4. Thus the largest possible value of n is 60. 60 =

Answer: 2 First note that if n 2: 4, then

1!+2!+3!+···+n! =1!+2!+3!+4! (modS) =1+2+1+4=3 (modS).

Since the square of any integer is congruent to either 1 or 4 modulo S, it follows that 1 ! + 2! + 3! + + n! i= m2 for any integer m in this case. So we consider n< 4. Now we have · · ·

1!

=

12,

1! + 2!

=

3, 1! + 2! + 3!

=

32•

Hence we conclude that there are two pairs of positive integers (n, m ) , namely, (1, 1) and (3, 3 ), such that 1! + 2! + 3! + + n! = m2• · · ·

35

22.

Answer: 65 EC 119 = = . Thus we can let BC = 119y BA EA 169 and BA = 169y for some real number y. Since LBCA = 90°, we have Since BE bisects LCBA, we have

BC

AB2 = AC2+ B C2

(169yy =(169+119) 2 +(119y) 2 y 2 (169 -119X169+119)= (169+119) 2 y2 =

169+119

169-119 12 y=5

=

144 25

Hence, from triangle BCE, we have BE =

�119 2 + (119y) 2 =119 x 13 . 5

Finally, note that f).A[)E and MCE are similar, so we have AE xCE 169x119 =65 cm. = ED= 119x 153 BE

23.

Answer: 72 First we find the number of ordered pairs (m, n) of positive integers m and n such that m+ n = 190 and m and n are not relatively prime. To this end, write m = ka and n = kb, where k, a and b are positive integers with k > 1. Since m+ n = 190, we see that k is a factor of 190 = 2x5x19 with k t 190. We consider six cases: (i) (ii) (iii) (iv) (v) (vi)

k = 2. Then a+ b = 95, and there are 94 such pairs (a, b) of a and b such that the equation holds. k = 5. Then a + b = 38, and there are 37 such pairs (a, b) of a and b such that the equation holds. k = 19. Then a + b = 10, and there are 9 such pairs (a, b) of a and b such that the equation holds. k = 10. Then a+ b = 19, and there are 18 such pairs (a, b) of a and b such that the equation holds. k = 38. Then a+ b = 5, and there are 4 such pairs (a, b) of a and b such that the equation holds. k = 95. Then a+ b = 2, and there is 1 such pair (a, b) of a and b such that the equation holds.

(

It follows from the above cases that the number of ordered pairs m, n) of positive integers m and n such that m+ n = 190 and m and n are not relatively prime is 94+37+9-18-4-1 = 117.

36

Since the total number of ordered pairs (m, n) such that m + n = 190 is 189, we conclude that the required number of ordered pairs (m, n) where m and n are relatively prime is 189 - 117 = 72.

24.

Answer: 32 For all real values of x for which] (x) is defined, we have

9

j(x) =

l+cos 2x 1-cos 2x

= _!_

2

= 17 + _!_

� t an2

� 17 +

2

=

9

=

� t an2 x+9) + _!_ (25 cot

= 17

+

32.

2

� ( ��

2

X

+ 25 coe

2

X

x+ 25

)

) ))

X

t an2 x 25cot2 x

(AM-GM Inequality)

least possible value of j(x) is 32. fi) = 32. Thus the .

�3

Answer: 2002 First we place the 5 red balls on a straight line and then place 1 blue ball between 2 adjacent red balls. With this arr angement fixed, the condition of the question is satisfied. We are now left with 9 blue balls. We can place the remaining 9 blue balls into the spaces before, after or in between the 5 red balls. The number of ways that this can be done is the answer to the question. Including the two ends, there are 4 + 2 = 6 spaces into which these 9 blue balls can be placed. The number of ways of distributing the 9 blue balls into the 6 spaces is 4

( : J ( � J c :J 9

26.

25

+ 2cos2 x 2sin2 x

_!_(2J9x 25) 2

Note that j(t an-1

25.

25

+

+

-!







2002.

Answer: 49 Consider the following subsets of S: S1

= {1, 2, 3, ..., 49, 50},

{51, 52, 53, ..., 99, 100}, s3 = {1 01, 102, 1 03, ..., 149, 150}, S2 =

Szooo =

{99951, 99952, 99953, ..., 99999, 100000}.

In other words, Si = {50i- 49, 50i- 48, ..., 50i} fori= 1, 2, ..., 2000. Note that Sis partitioned into these subsets St, Sz, S 3 , ... , Szooo.

37

By Pigeonhole Principle, for any subset A of S with where 1:::; i:::; 2000, such that I AnSi � I 2. Let

a,b EAnsi. It

is clear that

IAI = 2010,

there exists i,

Ia- bl:::; 49.

To show that 49 is the least possible value of k, we find a subset A � S with lAI = that Ia- bl2: 49 for any distinct a,b EA. Let

2010 such

A={ 49j+1: j=0,1, 2, ... , 2009}={1,50, 99,1 48, ... , 98442}.

Then A is a subset of S with

27.

Answer:

IAI = 2010

and

Ia- bl2: 49

for any distinct

a,b EA.

11 2

E �--+---+---4----r--� n

Figure 1 A

c

We observe that to avoid the four points marked x, the path must cross either C, D orE as shown in Figure 1 above. Further, the paths that cross C, D orE are exclusive, that is, no path can cross both C and D or D and E, or C and E. There is only 1 way to get from A to C and from A toE. It is easy to see that there are 4 ways to get from A to D. 1

1 1

1

1

c

6

5

..

4 .. 3

2 1

21

• • •

B

15

10 6

Figure 2

3

1

To count the number of ways to get from either C, D orE to B, we note that the number of ways to get to a certain junction is the sum of the numbers of ways to get to the two junctions immediately preceding it from the left or from below (as shown in Figure 2). Therefore there are 21 ways to get from C to B. Similarly, there are 21 ways to get from D to Band 7 ways to get from E to B. Hence the number of ways to get from A to B that • pass through C: number of ways from A to C x number of ways from C to B= 1 X 21 = 21; • pass through D: number of ways from A to Dx number of ways from D to B= 4 X 21 = 84; • pass through E: number of ways from A to E x number of ways from E to B=1x7=7. It follows that the total number of ways from A to B is 21 + 84+7= 1 12. 38

28 .

Answer: 60

Let 0 be the centre of

C2

and let PA intersect

C2 atB .The

homothety centred atA

It mapsB toP. Thus AB = !__ (This 10 AP 10 can also be seen by connecting P to the centre 0' of C1 so that the triangles ABO and APO' are similar.) The power of P with respect to C2 is PM2 = 20. Thus mapping

Cz

to

C1

has similitude ratio

!__

PB·PA = 20,or equivalently (PA - AB)PA =20. Together with AB = !_ ,we AP 10 obtain AB =8 and AP = 10 . Consequently,the triangle ABO is equilateral,and hence LPAD = LBA0 = 60°.

29.

Answer: 6 We shall show that a =3,b =2 and c =1. Note that 2a > b + c.As b + c is a multiple of a ,it follows that a =b + c. Let a + c =kb . Then kb =a + c = b + c + c,so 2c = (k- 1)b . Since c < b,we must have k =2 and therefore b =2c. Since b and c are relatively prime,this implies that c = 1 and b =2 . Thus a =3 . Hence abc = 6.

30 .

Answer: 680 For any 3-element subset {a ,b,c},define a mapping f by f ( {a ,b,c}) = {a ,b - 1,c - 3}. Now observe that {a ,b,c} is a subset of {1, 2, 3, 4, . . . , 20} with a < b - 1< c - 3 if and only if j( {a ,b, c} ) is a 3-element subset of { 1, 2, 3, . . . , 17}. Hence the answer is

C:)

= 680.

39

31.

Answer: 2780 Note that 0 ::;; f(n)::;; 4 for 1 ::::;n::::; 99999. Fork= 0, 1, 2, 3, 4, let ak denote the number of integersn, where 1 ::::;n::::; 99999, such thatj (n) = k. Then 4 4 k k M= kak 2 = kak 2 . k =O k =l.

L:

L:

By considering the number of 2-digit, 3-digit, 4-digit and 5-digit positive integers with exactly one 0 in their decimal representation, we obtain � = 9+ 9 X 9 X 2+ 9 X 9 X 9 X 3+ 9 X 9 X 9 X 9 X 4=28602 .

Similarly, we have

a =9 + 9 X 9 X 2

( �)

+9 X 9 X 9 X

(�)

= 4626,

a3=9+9 2 x4=333, a =9. 4 Hence 3

M =1x28602x2+2x4626x22 +3x333x2 +4x9x24 =102780,

and it follows that M- 100000 = 2780. 32.

Answer: 5 Let n = p4 -5 p 2 +13. When p = 3, we haven= 49 and so the sum of digits is 13. When p= 5, we haven= 513 and the resulting sum of digits is 9. Now let p > 5 be a prime. We have n= p4 -5p 2 +13= ( p-2)( p-1)( p+1)( p+2)+9. Since p- 2, p- 1, p, p + 1 and p + 2 are five consecutive integers, at least one of them is divisible by 5. Since pi= 5, we must have 5 divides one of the numbers p- 2, p- 1, p + 1 and p + 2, so 5 divides the product (p- 2)(p- 1)(p + 1)(p + 2). Observe that at least one of the numbers p + 1 and p + 2 is even. Therefore we see that (p- 2)(p- 1)(p + 1)(p + 2) is divisible by 10. It follows that for any prime p > 5, the number n has at least two digits and the units digit is 9, so the sum of the digits is greater than 9. Consequently the smallest possible sum of digits is 9, and this value is attained when p= 5.

40

33.

Answer: 360 X

ExtendBA andCD to meet at X .LetH be the point onCD such thatFH II Let AD

=

CE

=

a. Then

BC

=

ECG,

3a , andFH

( ) CE

=

1

2

(AD +

BC)

By the similarity of trianglesFHG and we have 2 FH x area of (i) area of FHG = (ii)

Ll CG CE

HG

FH

Ll ECG

=

It follows from (i) and (ii) that the area of triangleFDH

(iii)

XA

XF

AD

XAD LlXAD ( )

60 cm2;

=

3

2

� x 60

2

HG

=

.

90 cm2 .

bey cm2 . By the similarity of triangles XAD and

1

- =- =

FH 2 area of ilXDF

(iv) area of LlXFH

, so that XA =AF and hence =

2x area of AD

FH

=

2

y

=

=

2y cm2;

4y.

It follows from (iii) and (iv) that the area of triangleFDH 2 Since the area of triangleFDH is 90 cm , we get y = 45. Finally, by the similarity of triangles andABC, area of

2a.

2 , so thatHG = 2CG andDH =HC =

-=-=

Now, let area of triangle XFH , we have

=

BC.

LlXBC (BC)2 =

AD

Hence the area of trapeziumABCD

XAD

y =9y . =

8y = 360 cm2•

41

=

4y-2y

=

2y cm2 .

34.

Answer: 75 First we note that every positive integer m can be written uniquely (in base 27) as m=b0 +b1 x27+b2x27 2 +···+b7 x277, where r and b0, bp b2,

• • •

, br (depending on m) are non-negative integers with

bi < 27 for i =

0, 1, 2, ..., r. Since the coefficients of P(x) are non-negative integers and P(1) = 25, we see that ak ::;; 25 < 27 for 0:::; k:::; n. Thus by the above remark, the polynomial P (x) is

uniquely determined by the value P(27) = 1771769. Writing 1771769 in base 27, 3 we obtain 1771769 = 2+11x27+9x27 +3x274• Therefore 3 P (x) =2+11x+9x +3x4• Hence a0 +2a1 +3a +···+(n+1)an =2+2x11+4x9+5x 3=75. 2 35.

Answer: 12

p

N,. Let P be the point of concurrence of the

First we shall show that r3 =

tangent to the circumcircle of the triangle ��As at As and the two external common tangents of f1 and f • Note that the line joining � and � also passes 2 through P. First we have PA; =P� · PA1 , so

( ) PAs

p�

2

=

PA1

PAs =..2. That is, = p� r2 p�

On the other hand, PAs sinLAg� P sin� r1 +r3 = -= =-sin�As P sin� r +r3 p� 2 1+ 3 Thus = ' ' • Solving for r3, we obtain r3 = + 2 ' ,3 r2 =8 , we get r3=12.

--

N,.

3 v;;

42

3. v;;

Substituting r1 =18 ,

',

Singapore Mathematical Society Singapore Mathematical Olympiad (SMO) 2010 (

. Senior Section, Round 2) Saturday, 25 June 2010

0930-1230

INSTRUCTIONS TO CONTESTANTS

1.

2.

3.

Answer ALL 5 questions. Show all the steps in your working. Each question carries

10

mark.

4. No calculators are allowed. 1. In the triangle ABC with AC > AB,

D is the foot of the perpendicular from A onto BC and E is the foot of the perpendicular from D onto AC. Let F be the point on the line DE such that EF DC = BD DE Prove that AF is perpendicular ·

·

.

to BF. 2. The numbers

t, �, . . . , 2 0\ 0

are written on a blackboard.

A student chooses any

two of the numbers, say x, y, erases them and then writes down x

+

y

+

xy. He

continues to do this until only one number is left on the blackboard. What is this number? 3. Given

a1 2': 1 and ak+l ;::: ak + 1 for all k 1, 2, , n, show that af + a� + + a � 2': (a1 + a2 + + an) 2 . ·

·

·

·

·

·

4. An infinite sequence of integers, a0 , = for any n 2': 0, where

an, a1

·

=

an+l an - bn,

·

·

a1, a3 , with a0 > 0, has the property that bn is the number having the same sign as .

.



but having the digits written in the reverse order. For example if =

1089 and

all n;:::l.

a2

=

-8712, etc. Find the smallest value of

a0

5. Let p be a prime number and let

so that

a0

=

1210,

an i= 0 for

a1, a2 , ... , ak be distinct integers chosen from n 1, 2, ..., p 1. For 1 ::; i ::; k, let r } ) denote the remainder of the integer nai upon n division by p, so 0 ::; r} ) < p. Define n S { n : 1 :S n :S p - 1, rin) < < r )} -

·

=

Show that

S has less than k2_f1

elements.

43

·

·

k

.

Singapore Mathematical Society Singapore Mathematical Olympiad (SMO) (Senior Section, Round

2

2010

solutions)

Since we are supposed to prove LAFB=goo, it means that the 4 points A, B, D, F are concyclic. Note that AC > AB implies that LB > LC. If TD is the tangent to the circumcircle w of the triangle ABD with B and T lying opposite sides of the line AD, then LADT = LB > LC = LADE so that w intersects the interior of DE at F. Therefore F can only be in the interior of DE. Now observe that the triangles ADE and DCE are similar so that AD/AE =DC/DE. By the given condition, this can be written as AD/AE = BD/ EF. This means the triangles ABD and AFE are similar. Thus LABD = LAFE. This shows that A, B, D, F are concyclic. Therefore LAFB=LADE=goo. 1.

A

c

B

2.

See Junior Section Question 5.

We will prove it by induction. First, it is clear that suppose it is true for n terms. Then 3.

ar 2:: af since a1 2:: 1.

n+l n n La� 2:: a�+l +La� � a�+l + (Lak) k=l k=l k=l n n+l (Lak) +a�+1-a�+1-2an+1Lak. k=l k=l 2

2

=

44

Next,

a�+l-a;+l-2an+l.L.:�=lak :?: 0. Since ak+l- ak :?: 1, we have a�+l- a� :?: ak+l + ak. Summing up over k 1, ... , n, and using ar - al :?: 0, we have n a�+l- at:?: an+l+ 2 L ak - a1 k=l To complete the induction, we'll now show that

=

4. If

ao has a single digit, then a1 0. Thus a0 has at least 2 digits. If a0 ab 10a+b, then a 1 9(a-b) which is divisible by 9. it follows that all subsequent terms are divisible by 9. Checking all 2-digit multiples of 9 shows that eventually 9 appears ( Note that ab and ba give rise to the same sequence, but with opposite signs ) : 81 63 27 45 9. =

=

=

=

---+

---+

---+

---+

ao abc, then a1 99(a - c). Thus if suffices to investigate 3-digit multiples of 99, 198, ..., 990. Here we find that 99 will eventually appear: 990 891 693 297---+ -495 ---+ 99. If ao =abed, then a1 999(a-d)+90(b-c). If b, care both 0, then a1 and all subsequent terms are multiples of 999. However, if such numbers appear in the sequence, eventually 999 will appear: 9990 8991 6993 2997 -4995 999. For 1010, we get 909 and for 1011 we get -90. For 1012, we get 1012 -1089 -8712 6534 2178 -6534 and the sequence becomes periodic thereafter. Thus the smallest a0 1012. If

=

=

i. e. ,

---+

---+

---+

=

---+

---+

---+

---+

---+

---+

---+

---+

---+

---+

=

5. Let

rbn) 0 and Tk�1 =

S' Note that

=

=

p.

Set

k {n: 1:::; n:::; p - 1, .2.: lr���- r�n)l p}. i=O =

k

L lr���- r�n) I i=O

=

P

iff

Tbn)

:S;

rin)

:S;

·

·

·

:S;

rk�1.

n E S', l r��)1 - r�n) I r���- dn) n(ai+l- ai) (mod p) and p f (ai+l- ai), the numbers r��)1- r�n), 1 :::; n:::; p- 1, are all distinct. Therefore Thus

lSI

Pl8'1

=

=

IS' I ·

Since for

=

lri�i- ri"ll t I.; lri�i- rl"ll t�j t nEB' i=O i=O nEB' i=O j=l I.;

Therefore

lSI



<



k2_J1.

45





(k + l)IS (IS'I + l).

Singapore Mathematical Society

( SMO ) 2010 1)

Singapore Mathematical Olympiad

( Open Section,

Round

0930-1200 hrs

Wednesday, 2 June 2010

Instructions to contestants 1. Answer ALL 25 questions. 2. Write your answers in the answer sheet provided and shade the appropriate bubbles below your answers.

3. No steps are needed to justify your answers. 4. Each question carries 1 mark.

5. No calculators are allowed.

PLEASE DO NOT TURN OVER UNTIL YOU ARE TOLD TO DO SO

46

8n3- 96n2 +360n- 400 . is an integer . Find 1. Let S be the set of all mtegers n such that 2n - 7 the value of lnl.

L

nES

2. Deter mine the largest value of x for which



lx2 - 4x - 396011 lx2 +4x - 3960 11.

3. Given that find the value of L



X=

1 1 1 L1 /3j + L2 /3j + L3 /3j

+ ... + L7999

1 /3j,

J, where LYJ denotes the greatest integer less than or equal to 1 0 (For example, L2.1J = 2, L30J = 30, L- 10.5J = - 1 1.)

4. Determine the smallest positive integer C such that 6n.�

:::;

y.

C for all positive· integers n.

.

5. Let CD be a chord of a circle r1 and AB a diameter of r1 per pendicular to CD at N with AN> NB. A circle r2 centr ed at C with radius CN intersects r1 at points P and Q, and the segments PQ and CD intersect at M . Given that the radii of r1 and r2 ar e 6 1 and 60 r espectively, find the length of AM. 6.

50

Deter mine the minimum value of

L Xk, k=l

positive number s x1, . . . , X5o satisfying

where the summation is done over all possible

50

L __!_ = 1. X k=l

k

4

7. Find the sum of all positive integer s p such that the expression (x- p)(x- 13 ) + can be expressed in the for m ( +q) ( +r ) for distinct integers q and r.

x

x

� - :2 - : 2

8. Let Pk = 1 + , wherek is a positive integer . Find the least positive integer n 3 such that the product P P Pn exceeds 20 10. 3 ·

·

·

9. Let B be a point on the circle centred at 0 with diameter AC and let D and E be the circumcentres of the tr iangles OAB and OBC respectively. Given that sin L.BOC = g and AC = 24, find the area of the tr iangle BDE. 10. Let f be a real-valued function with the rule f(x) = x3 +3x +6x+1 defined for all real value of It is given that a and bare two r eal numbers such that f(a ) = 1 and f(b) = 19. Find the value of ( a +b

x.

1 1. If cot a +cot ,6 +cot'/'

2

4

)2.

=

- 5,4 tan

cot a cot ,6 +cot ,6 cot')' +cot')' cot a

=

=

17 and B 1 -5, find the value of tan(a +,6 +')').

a +tan ,6 + tan')'

7

47

12.

The figure below shows a road map connecting two shopping malls A and B in a certain city. Each side of the smallest square in the figure represents a road of distance 1km. Regions C and D represent two large residential estates in the town. Find the number of shortest routes to travel from A to B along the roads shown in the figure. 8 km

.B

c 10 km

D A

1 3.

Let

a1

=

+ an> 2

14.

1, a2

=

2

and for all

·���� for all

n 2m,

n 2 2, an+l

where

m is

=

n-1 2n an + + 1 an-1· n n 1

It is known that

a positive integer. Find the least value of m.

It is known that

V9 - 8sin 50° =a + b sinc0 for exactly one set of positive integers

(a, b,c

+ ), where 0 < c < 90. Find the value of b c.

15. If a is a real root of the equation x5 -x3 + x -2 the least positive integer not exceeding x.

a

=

0, find the value of

La6 J, where Lx J is

16. If a positive integer cannot be written as the difference of two square numbers, then the integer is called a "cute" integer. For example, 1, 2 and 4 are the first three "cute" integers. Find the 2010th "cute" integer.

( Note: A

the square of a positive integer. As an illustration, 16 are the first four square numbers.) square number is

1, 4,

9 and

17. Let f (x ) be a polynomial in x of degree 5. When f (x ) is divided by x - 1, x- 2, x - 3, x -4 and x2 -x -1, f (x ) leaves a remainder of 3, 1, 7, 36 and x- 1 respectively. Find the square of the remainder when f (x ) is divided by x + 1.

48

18. Determine the number of or dered pairs of positive integer s (a, b) satisfying the equation 100( a +b)= ab- 100.

( Note:

As an illustr ation, ( 1, 2) and ( 2, 1) are considered as two distinct ordered pairs. )

19. Let p= ab +ba If a, b and p are all prime, what is the value of p? . 20. Determine the value of the following expression:

l J l

J l

J l

l

J

J

11 X 3 11 11 X 2 11 X 4 11 X 2009 +. . . + + + + 2010 2010 2010 2010 2010 '

where

LYJ

denotes the gr eatest integer less than or equal to y.

21. Numbers 1, 2, . . . , 2010 are placed on the circumference of a cir cle in some order. The numbers i and j, where i =F j and i,j E { 1, 2, . . . , 2010} form a friendly pair if

( i) i and j are not neighbour s to each other, and ( ii) on one or both of the arcs connecting i and j

along the circle, all numbers in between

them are greater than both i and j.

Determine the minimal number of friendly pairs. 2 2. LetS be the set of all non-zer o r eal-valued functions f defined on the set of all r eal numbers such that f(x2 +yf(z)) = xf ( x ) +zf(y) for all r eal number s x, y and z Find the maximum value of£( 1 2345) , where f E S. . 23. All possible 6-digit number s, in each of which the digits occur in non-increasing order from left to r ight ( e. g. , 966541) , are written as a sequence in increasing order ( the first thr ee 6-digit number s in this sequence are 100000, 110000, 111000 and so on) . If the 2010th number in this sequence is denoted by p, find the value of L J, where LxJ denotes the greatest integer less than or equal to x.

:0

24. Find the number of permutations a1 a2a3a4a5a6 of the six integers from 1 to 6 such that for all i from 1 to 5, ai+l does not exceed ai by 1. 25. Let A=

( ) c�� )) ( ) C )) ( ( ) c )) ( ( ( (����) - G���)) ( ) () ( (;) ) 2010 0

o

2

+

O10 o

2010 1

2

+

010 1

20 10 2

2

2

+. . . + Determine the minimum integer s such that sA �

( Note:

For a given positive integer n,

other values of r, define

n r

=

= 0.

49

40 20 2010

·

n! _ r ).' for r = 0, 1, 2, 3,

' r. n

·

·

·

,

n; and for all

Singapore Mathematical Society Singapore Mathematical Olympiad (SMO) (Open Section, Round

1)

2010

1. Answer: 50 27 8n3- 96n2 + 360n- 400 . 4n2- 34n + 61 + _ . Smce Note that 4n2 - 34n + 61 is 2n _7 2n 7 an integer for all integers n, we must have that 27 divisible by 2n- 7. Hence, 2n - 7 -1,1,-3,3,-9,9,-27,27, so that n 3,4,2,5,-1,8,-10,17. Hence the required sum equals 50. =

=

=

2. Answer: 199 By direct computation,

lx2- 4x- 396011 � lx2 + 4x- 396011 -¢:::=:?- (x2- 4x- 39601)2- (x2 + 4x- 39601)2 � 0 -¢:::=:?- 2(x2- 39601)(-8x) � 0 -¢:::=:?- x(x + 199)(x - 199):::; 0, we conclude that the largest possible value of

x is 199.

3. Answer: 1159 Note that X

-

L11/3j + L21/3j + L31/3j + ... + L79991/3j L Lkl/3j + L Lkl/3j + L Lkl/3j + ... +

L:

L:

1+

2+

L:

3 + ... +

L:

L

Lkl/3j

19

(23- 13) + 2 (33- 23) + 3 (43- 33) + ... + 18 (193- 183) + 19 (203- 193) 19 19(8ooo) - L: k3 k=l 19 20 2 19(8000) -

(

X

:. LlOOJ

)

;

115900 =

1159.

50

4. Answer: 65 6n Define f( n) =I for n=1, 2, 3, ···. It is clear that f( l ) =6, f( 2) =18, f(3) =36, f( 4 ) =54 n.

and f(5) =64.8. For all. n ;:::: 6, f( n) =

6n n!



6 6 6 6 6 � l X 2 X J X Ll X S X 6 =6 X 3 X 2 X 1.8=64.8

5. Answer: 78 Extend DC meeting r2 at H. Note that DN =NC =CH = 60. Since M is of equal power with respect to r1 and r2 Thus MN ·MH = MC ·MD . That is MN(MC+60) = . MC(MN +60) giving MN =MG. Thus MN =30. A

The power of N with respect to r 1 is DN · NC = 602, and is also equal to N A ·N B = NA·(AB -NA) =NA·(1 2 2 -NA). Thus NA·(1 2 2 -NA) =602. Solving this quadratic equation, we get NA = 7 2 or 50. Since NA > NB, we have NA = 7 2. Consequently AM =vfNA2+MN 2=vf7 22+302=78. 6. Answer: 2500 By Cauchy-Schwarz inequality,

and equality holds if and only if Xk =50 fork=1, . . . , 50. Therefore the required value is 2500. 7. Answer: 26 Let (x- p)(x- 13)+4=(x+q)(x+r ) . Substituting x=-q into the above identity yields (-q - p)(-q - 13) =-4, which becomes (q+p)(q+ 13) =-4. Since p and q are integers, we must have the following cases:

( a) ( b)

q+p=-4, q+13=1;

( d)

q+p=- 2, q+13= 2

q+p=4, q+13=-1;

(c) q+p= 2,q+13=- 2;or

51

() q = -14,p= 8 and hence (x- p)(x- 13)+4 = (x - 14)(x- 17); (b ), we obtain q = -12,p = 8 and hence (x- p)(x- 13)+4 = (x- 9)(x- 12); (c ), we obtain q =-15,p= 17 and hence (x- p)(x- 13) +4= (x- 15)2; which

For case a , we obtain -For case

For case is NOT what we want;

()

For case d , we obtain q = - l l,p = 9 and hence (x- p)(x- 13)+4 = (x- 11)2; which is also NOT what we want. Hence the two possible values of pare 8 and 18, the sum of which is 26.

8. Answer: 8038 First, note that

( ) (1+.!.k ) 2= (k - 1)(k+1)2 k3

! ! ]__ ]__ Pk = 1+ k _ k 2 _ k3= 1 - k Therefore,

1· 32 2· 42 3· 52 (n- 1)(n+1)2 (n+1) 2 = P 2P3··· Pn=� ·�·43··· n3 4n (n+1)2

> 2010 is equivalent to n2-8038n+1 > 0, which is equivalent Next, observe that 4n to n(n- 8038) > -1. Since n is a positive integer, the last inequality is satisfied if and only if n 2:: 8038. Consequently, the least n required is 8038.

9. Answer: 45 Let d = AC = 24. First, it is not difficult to see that LDEB =LACE and LEDB = LCAB, so that the triangles DEE and ABC are similar.

Let M and N be the feet of the perpendiculars from B onto DE and AC respectively. As M is the midpoint of OB, we have BM = �· Also BN = BO sinLBOC = � x � = 2r Therefore DE = AC x �lfr = d x �� = 5 . Thus the area of the triangle BDE is ! x BM x DE=! x � x 58d = s;;. Substituting d = 24, the area of the triangle BDE is

l

45.

52

10. Answer : 4

We note that f(x) = (x + 1)3 +3(x+ 1 ) + 10. Let g(y) = y3 + 3y, which is a str ictly increasing odd function. Hence f(a) = 1 implies g(a + 1) =-9 and f(b ) = 19 implies g(b +1) =9. Thus, a+1=-(b + 1), implying a+b =- 2, so that (a+b)2 =4.

11. Answer : 11 Let x=tan a, y =tan/3 and z=tan'Y. Then xy+yz+zx xyz x+y+z x+y+z xyz

-

-

4 5 17

6

-

-

17 5

-� and xy+yz +zx=�· It follows that

From the above three equations, xyz= tan(a +pa +'Y) =

x +y +z- xyz 17/6- (-5/6) = =11. 1- 2/3 1- (xy+yz+zx)

1 2. Answer: 2 20 23

Include the points E, F, G, H, I, J, K, L, M, Nand Pin the diagr am as shown and consider all possible routes: E

8km F

G H

10 km

B

c J

I

L

K

M

D A

For the route A

N

p

---+ E ---+ B,

there is 1 way.

For the route A ---+

F ---+

B, there are

For the route A ---+

G---+

B, there ar e

C2) · (D c ) (�) ° 2

53

·

=80 ways. =1260 ways.

For the route

A� H �I� B,

For the route

A� J �I� B,

For the route

A� J



K � B,

A� L �I� B,

For the route

A� L � K � B, A� L � M



=

=

·

=

=

there are

·

=

·

For the route

A� P � B,

there are

=

=

ways.

ways.

ways.

11 ways.

22023 ways.

Hence, by adding up, there are altogether

4021 an+l - an=

Rearranging the recurrence relation yields

n � 3, we have an- an-1 =

ways.

·

B, there are

there are

ways.

1176 ways.

there are

A� N � B,

for

=

there are

For the route

1 3. Answer:

ways.

there are

For the route

For the route

(�) · (�) 980 (�) · (D · (�) 5880 (�) (D (!) · (�) · (�) 7350 (!) (D (D 4410 (!) (�) 490 (�) · c21) 385 ell)

there are

-ll

n (an- an-1) for n � 2. Thus, n+ .

. n- 2 n- 2 n- 3 (an-1- an-2) = an-2- an-3 ) ( · n n n- 1. n- 2 n- 3 2 1 a1) ··· = n · n -1 ··· 4 · 3( a22 2 2 -n(n- 1 ) n -1 n --

--

--

--

3. Using the method of difference, we obtain an= 3 - � for n � 3. Given that n 2009 , w have an > 2 + 2010 e 2 20091 3- > 2 + 32010 -:;;, 2010' yielding n > 4020. Hence the least value of m is 4021. for

n



14. Answer:

14

We have

8 + 8 sin 10° - 8 sin 10° -.8 sin 50° 9 + 8 sin 10° - 8 (2 sin 30° cos 20°) 9 + 8 sin 10° - 8(1 - 2 sin2 10° )

9 - 8 sin 50°

=

.·. v9 - 8 sin 506 Thus,

a = 1, b=

4 and

c=

16 sin2 106 + 8 sin 10° (1 + 4 sin 10° )2 1 + 4 sin 10°.

10, and hence

b+c

.

54

a

-

=

14.

+1

15. Answer: 3 It can be easily seen that the given equation has exactly one real r oot a, since (1) all polynomial equations of degree 5 has at least one r eal r oot, and (2) the function f(x) = x5- x3 + x- 2 is strictly increasing since f' (x) = 5x4- 3x2 + 1 > 0 for all r eal values of x. 1 It can also be checked that f(�) <0 and f(2) > 0, so that 2
a6 <4 since

a6 <4

-¢::::::} -¢::::::} -¢::::::} -¢::::::}

In addition, we claim that a6

a6

:::=::

:::=::

3

a4- a2 + 2a<4 a5 - a3 + 2a2 <4a 2a2 - 5a + 2 <0 1 2

3 since -¢::::::} -¢::::::} -¢::::::}

a4- a2 + 2a :::=:: 3 a5 - a3 + 2a2 - 3a 2a2 - 4a + 2 :::=:: 0,

:::=:: 0

the last inequality is always tr ue. Hence 3 :::; a6 <4, thereby showing that L a6J

=

3.

16. Answer: 8030

Any odd number greater than 1 can be written as 2k+1, where 2k+1 = (k+1)2-k2 . Hence all odd integers greater than 1 are not "cute" integers. Also, since 4m = (m+1)2-(m-1?, so that all integers of the for m 4m, where m rel="nofollow"> 1, ar e not "cute". We claim that all integer s of the form 4m + 2 ar e "cute". Suppose 4m + 2 ( for m:::=:: 1) is not "cute", then

for some integers positive integers x and y. However, x + y and x - y have the same parity, so that x - y and x + y must all be even since 4m + 2 is even. Hence 4m + 2 is divisible by 4, which is absurd. Thus, 4m + 2 is "cute". The first few "cute" integers are 1, 2, 4, 6, 10 For n > 3, the nth "cute" integer is 4n- 10. Thus, the 2010th "cute" integer is 4(2010)- 10 = 8030. ·

·

· .

17. Answer: 841

We have f(1) = 3, f(2) = 1, f(3) = 7, f( 4 )

= 36

and

f(x) = (x2 - x- 1)g(x) + (x- 1),

where g(x) is a polynomial in x of degree 3. Hence g(1) g(4) = 3. Thus

f(x)

Thus,

=

f(-1)

( X2 _ X _

[

-29,

so

-3, g(2)

=

0, g(3)

=

1, and

1). (-3) . ( x- 2)(x- 3)(x- 4) + (1). (x- 1)(x- 2)(x- 4)

]

(-1)(-2)(-3) (x- 1)(x- 2)(x- 3) · . + (3). . + (X _ 1) (3)(2)(1)

=

=

that its square is 841.

55

(2)(1)(-1)



18. Answer: 18

Solving for b, we get b = 10��t0�00. Since a and b are positive, we must have a > 100. Let a = 100 + m, where m is a positive integer. Thus b = lOO(lOO m)+lOO = 100 + 10!00. Therefore m must be a factor of 10100 = 101 x 2 2 x 5 2• Conversely, each factor r of 10�00) of the equation 100(a + m determines a unique solution (a, b) = (100 + r, 100 + b) = ab -100. There are 18 = (1 + 1)(2 + 1)(2 + 1) factors of 10100. Consequently there are 18 solutions of the given equation. In fact, these 18 solutions can be found to be (a, b) = (101, 10200), (102, 5150), (104, 2625), (105, 2120), (110, 1110), (120, 605), (125, 504), (150, 302), (200, 201), (201, 200), . (302, 150), (504, 125), (605, 120), (1110, 110), (2120, 105), (2625, 104), (5150, 102), (10200, 101).



19. Answer: 17 If both a, b are odd prime numbers, then p is even and p that p is prime. Thus a =2 or b=2.



4, contradicting the condition

Assume that a =2. Then b i= 2; otherwise, p =8 which is not prime. Thus b is an odd prime number. Let b

We shall show that k=1.

=

2k + 1, where k is an integer greater than 1. Thus

(

Suppose that k � 2. If k = 1 mod 3), then b > 3 and b = 2k + 1

=

(

O mod 3),

contradicting the condition that b is prime. If k ¢. 1(mo d 3), then

a contradiction too. Thus

k =1

and

b= 3.

Therefore

20. Answer: 10045 The number of grid points (x, y) inside the rectangle with four corner vertices (0, 0), (11, 0), (0, 2010) and (11, 2010) is (11 -1)

X

(2010 -1) =20090.

There are no grid points on the diagonal between (0, 0) and (11, 2010) excluding these two points, since for any point (x, y) on this segment between (0, 0) and (11, 2010), we have 2010x

y=--·-uBut for an integer x with 1 ::; x ::; 10,

0l�x is not an integer.

2

Hence the number of grid points (x, y) inside the triangle formed by corner points (0, 0), (0, 2010) and (11, 2010) is (11 -1)

X

(2010 -1)/2 56

=

20090/2 =10045.

(11, 2010),

For any grid point (x, y) inside the tr iangle formed by corner points we have

1

1

::::::

y ::::::

2009, 1

:S: X <

11y 2010'

(0, 0), (0, 2010)

and

Thus, for any fixed positive integer y, there ar e the number of grid points satisfying the is condition that :S: x <

.,i-�{0

11 l2 o :o J ' 1�{0 1 (0,2009. 0), (0, 2010) (11, 2010) 11 11 2 11 3 11 4 2009 . 11 l2010J l 2010 J l 2010 J l 2010 J l 2010 J 10045. 21. 2007 3. 3 3. 3. 3, 0. 3 3. as 2 is not an integer when :S: y :S: the tr iangle for med by cor ner points X

+

Thus the number of grid points (x, y) inside and is

X

+

X

+

+. . . +

X

Therefore the answer is

Answer:

Consider n distinct numbers where n 2: of fr iendly pairs is always n- for n 2: also n-

We shall show by induction that the number Hence the minimal number of fr iendly pairs is

If n = then there are no fr iendly pairs as each number is adjacent to the other two. Thus the number of fr iendly pairs is Assume that the number of fr iendly pairs for any arr angement of n distinct numbers on the cir cle is n- for some n 2: Consider n+ distinct number s on the cir cle. Letm be the largest number. Now consider the n numbers on cir cle after deletingm. The two numbers adjacent tom which originally for m a fr iendly pair do not for m a fr iendly pair anymore. Any other fr iendly pair r emains a fr iendly pair when m is deleted. On the other hand, any fr iendly pair aft er m is deleted was originally a fr iendly pair. By induction hypothesis, there are n- fr iendly pairs after m is deleted. Therefore, there are (n+ fr iendly pairs originally.

22.

Answer:

3

1) - 3

12345

1

We are given the equation f(x2+yf(z))

0 (1), 0 (1),

=

xf(x) +zf(y).

(1) 0. (2)

Substituting x = y = into we get zf(O) = f(O) for all real number z. Hence f(O) = Substituting y = into we get Similarly, substituting x = Substituting y =

1 (3) into

0 (1) in

xf(x) we get f(yf(z))

(3)

zf(1)

(4)

we get f(f(z))

for all r eal z. Now, using

zf(y).

(2) (4), and

=

we obtain

57

(5)

Substituting y =z =x into (3) also yields f(xf(x)) = xf(x).

(6)

Comparing (5) and (6), it follows that x2f(1) = xf(x), so that if x is non-zero, then f(x) =ex for some constant c. Since f(O) = 0, we also have f(x) =ex for all real values of x. Substituting this into (1), we have that c(x2 + cyz) = cx2 + cyz. This implies that c2 = c, forcing c = 0 or c = 1. Since f is non-zero, we must have c = 1, so that f(x) = x for all real values of x. Hence f(12345) =12345. 23. Answer: 86422 The number of ways of choosing r objects from n different types of objects of which + repetition is allowed is (n � 1 . In particular, if we write r-digit numbers using n digits allowing for repetitions with the condition that the digits appear in a non-increasing + order, there are (n �- 1 ways of doing so. Grouping the given numbers into different categories, we have the following tabulation. In order to track the enumeration of these elements, the cumulative sum is also computed:

)

)

+ n r (n �

Numbers Beginning with

Digits used other than the fixed part

1

1,0

2

5

2

2,1,0

3

5

3

3,2,1,0

4

5

4

4,3,2,1,0

5

5

5

5,4,3,2,1,0

6

5

6

6,5,4,3,2,1,0

7

5

7

7,6,5,4,3,2,1,0

8

5

From 800000 to

5,4,3,2,1,0

6

5

3,2,1,0

4

4

855555 From 860000 to 863333

-

1

)

�� =6 � �) =21 )

l!

0

I

5

27 83

=126

209

=252

11 =462 5 1 =792 o

6

=56

: �) 1 ;) c) (:) l

Cumulative

= 252

=35

461 923 1715 1967

2002

The next 6-digit numbers are: 864000,864100,864110,864111,864200,864210,864211,864220. Hence, the 2010th 6-digit number is 864220. Therefore, x =864220 so that l J =86422.



24. Answer: 309 LetS be the set of permutations of the six integers from 1 to 6. Then lSI=6! =720. Define P( i) to be the subset of S such that the digit i + 1 follows immediately i, i =1,2,3,4,5. 58

Then =

� IP(i)l (�)5! _I; 1P(i1)nP(i2)1 2

21 <22

(� )4!

=

. �. 1P(i1) nP(i2) nP(i3)1 (� )3! . _I; 1P(i1)nP(i2)nP(i3)nP(i4)1 (!) 2! . IP(1)nP(2)nP(3)nP(4)nP(5)1 (�) 1!.

21 <22<23

=

=

21 <22<23<24

=

By the Principle of Inclusion and Exclusion, the r equired number is

25. Answer: 2011 Note that A =

(COo10) - (2��0))2+ ((2010)- (2010))2+ ((2010) - C010)) 2 2 0 1 1 +...+ (G���) G���) ) 2 � f ((2010k ) _ (���))2 . k=O =

_

Obser ve that

is the coefficient of

x2010 in the expansion of the following expression:

We also have

2011 ((2010) - ( 2010 )) k 2011 (2010) k - 2011 ( 2010 ) k L L k L k k 1 k 1 k=O k=O k=O (x+1)2010 _ x(x+1)2010 (1 _ x)(x+1)2010 2010 ((2010)- ( 2010 )) k 2010 (2010)xk-! 2009 (2010 )xk+1 + L1 k k 1 x k=O L k k 1 L1 + k=k=(x+1)2010 _-(1 x+1)2010 (1 _ 1/x)(x+1)2010. 59 X =

=

and

X

=

=

=

X

X

=

X

The coefficient of x2010 in the expansion of the following expression: (1 - x)(x + 1)2010 (1 - 1/x)(x + 1)2010 = (2 - 1/x - x)(x + 1)4020 is 2 Hence

( ) ( ) ( ) ( ) ( ) ( ) ( ) ( ) 4020 2010

- 4020 2011

A=

- 4020 2009 4020 2010

Consider the inequality:

sA 2:

=

_

2

4020 2009 ·

4020 2010 '

s(1 - 2010/2011) s

2:

Hence the answer is 2011.

60

4020 - 4020 2 2010 2009 .

2011.

2:

1,

Singapore Mathematical Society Singapore Mathematical Olympiad (SMO)

2010

(Open Section, Round 2) Saturday, 26 June 2010

0900-1330

INSTRUCTIONS TO CONTESTANTS

1.

2. 3.

4.

Answer ALL 5 questions. Show all the steps in your working. Each question carries

No

10

mark.

calculators are allowed.

1.

Let CD be a chord of a circle r1 and AB a diameter of r1 perpendicular to CD at N with AN> NB. A circle r2 centred at C with radius CN intersects r1 at points P and Q. The line PQ intersects CD at M and AC at K; and the extension of NK meets r2 at L. Prove that PQ is perpendicular to AL.

2.

Let an, bn, n = 1, 2, . . . be two sequences of integers defined by a1 for n;::: 1,

an+1 bn+1

=

=

=

1, b1

=

0

and

7 an + 12bn + 6 4an + 7bn + 3.

Prove that a� is the difference of two consecutive cubes. 3.

4.

5.

Suppose that a1, ..., a15 are prime numbers forming an arithmetic progression with common differenced> 0. If a1> 15, prove thatd> 30, 000. Let n be a positive integer. Find the smallest positive integer k with the property that for any colouring of the squares of a 2n x k chessboard with n colours, there are 2 columns and 2 rows such that the 4 squares in their intersections have the same colour. Let p be a prime number and let x, y, z be positive integers so that 0 < x < y < z < p. Suppose that x3, y3 and z3 have the same remainder when divided by p, 2 show that x + y2 + z2 is divisible by x + y + z.

61

Singapore Mathematical Society Singapore Mathematical Olympiad (SMO) 2010 (Open Section, Round

2

solutions)

Extend DC meeting r2 at H. Let the radius of r2 be r. Note that DN = NC = CH =r. Since M is of equal power with respect to r1 and r2. Thus MN MH = MC ·MD. That is MN(MC + r) =MC(MN + r) giving MN=MG. Thus M is the midpoint of NC. 1.

·

A

As K lies on the radical axis of r1 and r2, the points C, N, A, L are concyclic. Thus LALC=LANG=goo so that AL is tangent to r2. It.follows that AC is perpendicular to NL at K, and hence MN=MC=MK. Now let PQ intersect AL at T. We have LTAK = LKNM = LNKM = LLKT and similarly LTLK = LAKT. Consequently, 2LKTL = 2(LTAK + LAKT) LTAK + LAKT + LLKT + LTLK= 180°, which means LKTL=goo. First we shall prove that a� is the difference of two consecutive cubes. To do so, we shall prove by induction that a� = (bn + 1)3- b�. When n = 1, this is true. Suppose for n 2: 1, this is true. We have 2.

(bn+l + 1)3- b�+l =3b;+l + 3bn+l + 1 =3(4an + 7bn + 3)2 + 3(4an + 7bn + 3) + 1 =48a; + 147b; + 168anbn + 84an + 147bn + 37 =(7an + 12bn + 6)2 + (3b; + 3bn + 1)- a; a;+1 + (bn + 1)3- b�- a�=a;+l =

where the last equality is by induction hypothesis.

62

Lemma: Suppose p is prime and a1, ... , ap are primes forming an A. P. with common difference d. If a1 > p, we claim that p I d.

3.

Proof Since p is prime and every

ai is a prime > p, p does not divide ai for any

the pigeonhole principle, there exist 1 � i < j � p so that ai aj aj - ai = (j -i)d, and p does not divide j - i. So p must divide d.

( mod p) .

i. By Now

Apply the Lemma to the sequences at, ... , ak fork= 2,3, 5, 7,11 and 13. Then all such k's are factors of d. So d > 2 3 5 7 11 13 > 30, 000. ·

4.

·

·

·

·

The answer is 2n2 -n + 1.

Consider an n-colouring of the 2n x k chessboard. A vertical-pair is a pair of squares in the same column that are coloured the same. In every column there are at least n vertical-pairs. Let P be the total number of vertical-pairs and Pi be the number of vertical-pairs with colour i. Then P = P1+ + Pn � nk. Thus there is colour i with Pi � k. There are = 2n2 -n pairs of rows. Thus if k � 2n2 -n + 1, there is a pair of rows that contains two vertical-pairs with colour i.

e;)

·

·

·

Next for k = 2n2 - n, exhibit an n-colouring where no such sets of 4 squares exists. Note that it suffices to find such ann-colouring for the 2n x (2n-1) board. We can then rotate the colours to obtain n of these boards which can then be put together to obtain the requiring n-colouring of the 2n x (2n2 -n) board. For each i = 1,2,...,2n- 1, let Ai = {(i,2n-1+i),(i+1,2n-2+i),...,(n-1+i,n+i)}, where 2n+k, k > 0, is taken to bek. Note that the pairs in each Ai give a partition of {1,2,...,2n}. Moreover, each pair of elements appears in exactly one Ai. Now colour the squares of column i using n colours so that the two squares in each pair of Ai receive the same colour and the colours the 2n pairs are mutually distinct. This gives ann-colouring of the 2n x 2n -1 board with the required property and we are done.

5.

It is given that

p I x3-y3= (x -y)(x2 + xy + y2).

Moreover,-p < x-y < Oand sop f x-y. Thusp I x2+xy+y2. Similarly,p I y2+yz+z2 and p I x2 + xz + z2. Hence

p I x2 + xy -yz -z2 = (x -z)(x + y + z). Since p f x -z,p I x + y + z. Note that 0 < x + y + z < 3p. So x + y + z = p or 2p. We will show that p I x2 + y2 + z2. Assuming this for the moment, the proof is complete if x+y+z = p. If x+y+z = 2p, then x+y+z is even and hence x2+y2+z2 is even. So both 2 and p divide x2 + y2 + z2. Moreover, p > 2 and so 2 and p are relatively prime. Thus 2p divides x2 + y2 + z2 and the proof is also complete in this case.

63

Fg{^Q„€‰Zž?QcZvQgUQrž F€UgZ’ž Qmwf`zy{d‚N`~gds`~ma`q‚Prszm`c‚ QNP‚ F<
-( ʥŭʥ

@ Žʥ ʥȩʥ -ʥ



#"$&!&)

,371:BMMo '/o 0534#.,3 ,41:9.51:,3713:.,:4":,371:3"4:/1.6#: .1:4":*5)4&):".#:0534#.,3 : ,41:9.51:,3713:.,:4":,371:3"4:9:3"#,!:4": 5):.,4#,#,!:4":)441: : Fo Go =ʥ.1::.23/.,#,!:4.:4":.114:,371 .1: 4": .4"1: 3".14: 0534#.-3 : 71#4: 9.51: ,371: #,: 4": ,371: 3"4: ,: 3": 4": //1./1#4:5): ).7:9.51:,371 : .:34/3:1:,:4.:'534$ :9.51:,3713: ":0534#.,: 11#3: o *1( .:)5)4.13:1:)).7:

  )))) ) )) ) ))))

ʥ

'&#)")'%&"!%)

;

iʥ4dʥ  ʥ4_ʥʥ ))ʥ)Bʥ*ʥ )ʥ#bs ] ʥ ʥ„ʥ]ʥ ʥ$B ʥ  ʥ  ʥ*ʥ ]ʥ*]#ʥA) ʥ]ʥʥ*ʥ)Bʥ*ʥ )ʥ "ʥbs )ʥ])ʥ$B ʥ] ʥ ʥ ʥ*ʥ*]ʥ14;  ʥ1 ";

%aʥ %Fʥ %Yʥ %jʥ ;

ʥ ʥ "ʥ +ʥ *ʥ*ʥ]ʥ *&ʥ

Uʥ]ʥ )  ʥB*ʥ]ʥ )ʥ*ʥ<  ʥt =ʥ)ʥʥ  ʥ]ʥ )ʥ*ʥ<  ʥ t94ʥ)ʥ+‡ʥ“)&ʥ] ʥ©4t =; ʥ ß7ʥɐ ʥ]ʥ  ʥ*ʥ]ʥ]  ʥ)* 94=#ʥ 1;

ʥ ";

%aʥ %Fʥ %Yʥ %jʥ &o

Êʥ

Êʥ

£Êʥ

"Êʥ *ʥ*ʥ]ʥ *&ʥ

iʥkžʥ ʥ ʥŀʥA) ʥ]ʥ ^)ʥ& Bʥ*ʥkž]ʥ] ʥ]ʥ**) )< B);ʥ]*B Dʥ

˜™š k&ž T&Œ



%aʥ ʥ %Fʥ žž •ž %Yʥ %ȡʥ Ǒʥ

ʥ

*o

@ŭʥʥʥ ʥ ʥ ʥ   ʥʥŰʥ$&ʥʥ ƪʥ 3ʥ  ʥ4ʥʥ ʥ®ʥUʥʥ„ʥVXoʥ ʥʥʥʂ ʥ2ʥ֜ ʥʥʥʥW go

T&Œ

%aʥ %Fʥ %Yʥ %jʥ .o

ʥ  ʥ ʥ "ʥ +ʥ

‡Œ :;; s!ʥ 2 ‡Œ oŒ ‡Œ ‡Œ ‡Œ “Ä&ʥʥ œʥʥʥ< Źʥ ʥ ʥ ʥ!ʥ ʥ ʥeo ʥʥʥʥ Ä&ʥœʥ Äʥ

 ! ֜ ʥʥ ^ʥ& ʥʥ: HO

T&Œ

%aʥ %Fʥ %Yʥ %jʥ 0o

( ʥ P-ʥ 

(£ʥ mʥ ʥ &ʥ

@ʥ<ʥ/6; Ãʥ/6;*6;:5;  ʥ/8; ; -­ʥA ʥ/98 T&Œ

%aʥ %Fʥ %Yʥ %jʥ

 ʥ  - £ʥ -ʥ £ -ʥ ££-ʥ



3o

Fʥʥʥ ʥŻʥ ʥ   ʥʥ ʥ&Žʥ ʥ ʥʥ´ðʥ$ ʥʥ  ʥ ʥʥʥ^ʥʥ‘ʥA ʥʥ$ ʥʥʥ$ ʥ ʥʥ & ʥŽʥʥ‘ʥ

T&Œ

)'ž To^o ž Š‘

%aʥ ž U Fʥ  #To %Yʥ  #Uo %jʥ  #T 7o

“&ʥ ʥbo  6ʥ ʥ ʥʥ Ãʥʥųųɭʥ< Dʥ

žbeož6 ʥ ;)žž+

bo

ž  ʥ b!o ž'‚  n‘

e ʥʥ& ʥʥ G‚bo6ž  ʥ ž#G‚ ‚ T&Œ

%aʥ Fʥ %Yʥ %jʥ



#Gž

)ž"ž )žGž ʥ ž ʥHž

“&ʥ ʥ6ʥ



T&Œ

%aʥ Fʥ %Yʥ %jʥ

b #!( #božʥbož#´ð ʥʥʥ& ʥ 6‘

   

P-ʥ

#

;

@ʥʥʥ$LLgʥL ʥLʥ0; h+Œ 3; :  LÃL  ,ʥ  : ,ʥ; J Lʥʥ LLʥ ʥ/;;hŒ;3; ;:Lʥ T&Œ

# #

Yʥ

#



# # 



"$&)'%&"!%) ;

@ʥ ʥÝʥ ʥʥiFYʥgLLʥʥ ‘ʥŝʥLʥ ʥį ʥLįʥ   ʥ ʥ LʥʥʥLʥ ʥƛ ʥcƚʥL ʥ ʥ@ʥ Lʥʥʥʥʥʥ Lʥʥʥ   ʥ ʥLʥ#0 7ʥLʥ ʥʥʥL ʥʥ ʥLʥ "ʥ/  @gLLʥʥ ʥ  ʥŽʥʅʥʥʥɠʥ L  2ʥʥʥ ʥ  gʥʥ ʥ L  ʥɗʥʥ ʥ„‘ʥUʥʥ Lʥʥʥ  ʥʥʥ   ʥ  ʥʥ ʥʥʥL ʥ ʥLʥ!; / ʥɏ ʥʥg ʥʥ!

;

@ʥ L  ʥʥʥ ʥ$ ʥ ʥ$ʥʥLʥ!;  ʥLʥ9|ʥ  ʥ ʥ  ʥ  ʥ$ʥ ɳʥLʥ9ʥ  ʥLʥ ZŒ  ʥg $‘ʥ @ʥg $$ ʥ Lʥ9ʥLʥ "ʥʥʥ$ ʥ  ʥʥʥ ‘ʥUʥʥ LʥʥLʥ!;  LʥLʥʥ2ʥ´ð ʥʥg ʥʥ ʥ$ ʥʥLʥ4ʥʥʥʃ ʥLʥ+: ,: +,:

!;

9ʥ 1;

£ʥ

%o

*o

.o

-ʥġ ʥ ʥ Ùʥ ʥʥʥ*$ʥʥġʥ*ʥ ʥ<*ʥA ʥ ʥʥʥ*ʥ ʥ$ʥ< ʥ*ʥ  ʥ*ʥ$ʥʥ ʥ  ʥʥ  ʥ ʥ šʥʥ*ʥ ;ʥ*ʥ  ʥʥ*;ʥ  ʥ"ʥʥʥ ʥ*ʥʥ ʥʥ ʥʥ ñ ʥ*ʥ*ʥʥʥ*;ʥ *ʥʥ ñ ʥ*ʥ*ʥ*;ʥʥ#y [o A ʥʥ & ʥ*ʥ[



!#$; ʥ ʥ  ʥ ʥʥ,#!$; -ʕ®ʥAo < ʥʥ* ʥ*ʥʄʥ  ʥ!#;  ʥ#$; ʥ*#ʥšʥ  ʥ*ʥʥ< ʥ!#%'; ʥŋʥ>ʥ  ʥʥ  ʥ*ʥ ʥ< ʥ#$)+;ʥ "ʥ>#ʥA ʥʥɁ ʥ*ʥ ʥ%#+;ʥ>ʥ

+;

:

';

0o

1;

!;

ʥ

ʥ 'Œ

ʥ rʥ šʥVu *ʥ ʥY  Y ʥ'ŒʥY ʥ 'Œ 'Œ 'Œ ʥY ʥ ʥY ʥY # 888Œ ʥY ʥY ʥ ʥY ʥY "ʥRV 5ʥ ʥʥ*ʥǝʥA ʥʥ& ʥ*ʥj'Œ5­ʥ

j

4o

7o

8o

“&ʥ ʥg'Œ

ʥ; hŒ'ŒhŒ ( ʥ  ʥg1 hŒ,ʥ ʥ 2ʥ´‹ ʥʥ& ʥ*ʥgh0 g'Œh8

Uʥ p !ʥF‚'„Œ'Œ6ʥ3ž ʥ  ʥ lŽž* Žž, !ʥ;'ƫʥ֜ ʥʥ& ʥ* !ʥ,ʥŽ' ž Œ -r €ʥÚʥ  ʥÈʥ$ʥ*ʥ*&ʥ$ʥʥA*ʥ*ʥɌɟ^ ʥʥ, ʥʥ ž,:( 2ʥ5ʥ( -2ʥŋʥ( 52ʥ'5ʥ( Ĥʥ $ʥ ʑ2ʥ Ú2ʥÈ2ʥ ÈĤ2ʥʥ*ʥ  ;ʥʥ ʥ* #ʥA ʥʥ& ʥ*ʥ5®ʥ



o

Ac ʥ ʥ ʥ ʥ  ʥ $ ʥ ʥ$&ʥ c…ʥ ơ!2ʥ6ʥ  ʥ  ɓʥ ʥ < ʥ !ʥ$ž'Œ 6ʥž'Œ -ʥP -!

o



-6ʥP -ʥ0ʥ#

A ʥʥʥ$ ʥʥ

ʥ

ʥ 'Œ ʥ

ʥ

ʥ

ʥ

ʥ

ʥ ʥ,ʥ ʥ'Œ "ʥ,ʥ 'ʥ'Œ +ʥ,ʥ -ʥ

o

@ʥ   ʥBʥʥ ʥ Bʥ 3Òđ2ʥʥʥ  ʥʥ ¤=2ʥ3¤TX2ʥ=TÀđ  ʥX ¦§ʥ ʥ ʥ_2ʥʥ_2ʥ+ ʥ_ʥ  ʥ ʥ_ʥ$&B;­ʥA ʥʥ  ʥ §Àś _ʞʥ 3ʥ

ʥ =ʥ

¦ʥ

đʥ

%o

*o

śʥ

Òʥ

j& B ʥ ''ʥ( ʥżʥ ,ʥ ''ʥ,ʥ ʥżʥ ʥ A ʥʥʥʥʥʥʥʥžʥ ʥ 2ʥ2ʥ###ʥ2ʥ -Ĥʥʥʥʥʥ ʥ ʥ

ʥ 'ʥ

.o

“S&ʥ ʥ ʥ ʥ.ɵʥSʥ ʥSʥ…ž3žʥJʥ ʥ###ʥʥ5# #ʥ Aœ ʥʥ& BGʥ 5ʥʆʥ ʥ.ŷʥ,ʥ._ʥ,ʥ. ʼnʥ,ʥ.~ʥ,ʥ

.ʥ zƽdʥ0ʥ"P 'Œ.z ă_ʥ



!!!O

1o

ìʥʥ S  ʥq 3ʥSʥ ʥS BʥSʥȮ q3ʥ3ž?ßʥ  ʥq3ʥ Jʥï#ʥvʥo Šʥ ʥ ʥ $Sʥʥ 3ʥGʥ ʥ " vʥ vŠʥ3žJJʥŠ3#ʥìʥʥ&ğS BʥSʥʥvŠʥ3žʥï × ʥʥ  ʥʥʥS BʥqvŠʥ„ʥ_#ʥ 0O



ʥ



EEEEEEEEEEEEEEEFGGOCCCCODO

Šʥ



qʥ 3o

ļ

 ʥ 



Jʥïʥ

iʥ[dʥ!_ʥ!\ʥ[~ʥ ʥʥGʥʥʥʥ
7o

@ʥS ʥq 3ʥq ¤ʥ  ʥq3¤ʥʥ ʥ  ʥq 3¤ʥ ʥS B ʥS Bʥ S##ʥ©¾q3ʥ3ž©¾q¤ʥ3ž©3q¤ʥ3ž?ß#ʥ“S&ʥ ʥt ʥ3ž'ʥq3ʥ3ž ʥ  ʥq¤ "ʥ´ð ʥ ʥ& BGʥʥ H ʥʥėĘq 3_ʥH ʥʥėĘq Î _ʥ, H ʥʥėĘq3ďĀ_ʥH ʥʥ™3Î _#ʥ 0O

8o

5ʥĂJ AS ʥ ʥ B ʥ$SS&ʥSʥ5ʥ ʥSʥ ?5ʥ ,ʥJJʥ  S#ʥ +ʥ

QU

ʥļŻʥ GSBʥ

%o

A ʥ8ʥ& BG8ʥoʥ8ʥ BB8ʥ$&8ʥ88ʥOʥGʥ ʥ8ʥ8
!

,ʥ Oʥʥ ʥx , JssOʥ-ʥ ʥ3ž

 ʥB;ʥ88ʥBG#ʥ %o

% o

%%o

&+o

%.o

Uʥ ʥ B8ʥ!#$; 8ʥB8ʥoʥ8ʥ BG 8ʥ!%;  ʥ#(; 8ʥʥ  ʥJ ʥ8$8&8B;#ʥ A ʥ8ʥB 8ʥ$B8ʥ88ʥ& BG8ʥoʥ8ʥB8ʥoʥ8ʥ ʥ Bġ 8ʥ$) ; TŒ oGʥ ʥG8ʥʥoʥÝʥ ʥ$ ʥoʥ8$8 8 ʥ ʥoʥ8^ $B8ʥ JJ2ʥ " "ʥ  ʥ''#ʥA ʥ8ʥ BʥG8ʥoʥGʥ$B8ʥG8ʥ ʥ 8ʥ &B8ʥ;ʥ'ʥʥJJʥGʥʥ#ʥ

Oʥ  ʥ5ʥ 8ʥʥ$&8ʥc88ʥ ɍʥJʥċČčʥOʥċČčʥ5ʥċČčʥ#ʥA ʥ8ʥG8ʥoʥ $ ʥoʥO2ʥ5ʥGʥ ʥ8ʥ$ GʥO5ʥʥ &B8ʥ;ʥ#ʥ ĕʥ8ʥ ʥ ʥJ2ʥ 2ʥʥ  ʥ2ʥ× ʥ8ʥG8ʥoʥJ ʥ8


Fg{^Q„€‰Zž@QcZvQgUQrž F€UgZ’ž Rmwf`zy{d‚O`~hdt`~ma`r‚Prtzm`c‚ ROP‚ F<
HōʥF|ʥ Hʥʥ•ʥʥ   ʥBʥʥ ʥ"ʥBʥ ʥ ʥ ʥʥʥ$  ʥ1

FG‘ V‘ g‘ ‘ q‘ r‘ s‘ u‘

BC‘

(

)*+ + HIJ‘ DE‘



‘ ‘ ‘ }‘ ~‘ t‘ v‘

w‘ x‘ y‘ z‘ ‘ ‘ U‘ V‘

v‘ t‘ t‘ }‘ ‘ ‘ ‘ ‘ ‘

KLMNOP‘ =>?@ABO "O

o

HȊʥHʥ Ş ʥɡǡ 3ʥ-ßʥ BÙʥ ʥʥ$ʥ ; ʥ&B $ʥʥ %$ 1;

1;

;

ʥ

ʥ"ɶɅ( "Êʥ0ʥ Ê# ŏ#ʥV  ʥ  0ʥ8‚





%o

÷µÛ¸ʥ$Œ _ʥ

žžʥ™ ž 0ʥʥʥ ʥ !( '(!ʥ 0ʥ ʥʥ ʥ "# ( !(#Ø  Zʥ ^)ʥ& ʥ*ʥyʥ1ʥ ʥʥ ʥ ʥ 0 •ʥ *o

÷ܸʥB%Œ iʥvʥʥʥ) $*)ʥ*ʥĖʥ  ʥŠʥʥʥ$*)ʥÜʥʥ)ʥVYoʥʥ) ʥ )#ʥ fo

@ʥ;ʥ)) ʥſ ʥ0ʥJ ʔ –ʥv3ʥ0ʥJ ʥZĖ0ʥ"o0ʥ ʥ _ ( J _ʥ0ʥ  .o

÷µÛuʥ#Œ Jʥ Jʥ 61ʥss!ʥ ƾ! – ! 0 ʥJ( 6ʥJ h  * 6ʥJĎʥJ –(s h6 hs# V)ʥ* !ʥ  6ʥ ʥ)  6ʥʥJʥ)ʥ ʥ *ʥ*ʥJ ʥ)ʥ6ʥ0ʥ(- ʥ(+2ʥ(" ʥ(#ʥ @ʥ ^)ʥ& ʥ* 6ʥ)ʥ(ʥ

0o

÷ܸʥ ; ; ʥ J iʥhyŒ0ʥ]y0 ]y2?Œ 0ʥ5 _ʥ M‚hyŒ 1ʥ M‚5 _ʥ1ʥJʥ JJʥ0ʥ - # ;

ySŒ

ySuŒ

ēʥʥ*ʥ   ʥ H‚hyŒ 1ʥ/;8; /8; –ʥ/;8;1ʥ - ʥʥ/8;0ʥ-Ł ySŒ

J ʥ

3o

Huʥ

FS ʥʥ<SB ʁ BʥS Bʥ ʥ;ʥñSʥʥʥʥʥ ñ ʥ#ʥ ìʥʥ ;ʥʥʥ ʥʥ< ʥ$ ʥSʥ&ʥ;ʥʥ$ ʥʥʥ  Bʥ& ʥ;ʥʥʥʥ ; > “(…ž ʥ ɬĨ# ”ʥ &ʥ!ž…>ʥy Ĩʥ1ʥ !ž

7o

Huʥjʥ K>ʥ,ʥǤʥ1ʥ"ʥ#Kʥ,ʥ6>ʥ0ʥ ʥ,ʥ"ʥ Vʥ Kʥ,ʥžʥKʥ,ʥ6>ƺ "ģʥ,ʥ 6ʥ0ʥ  ʥ,ʥĢ > > :; K , 6 , K , 6 , 1

, "ĢʥéK , 6 ,  1  , Ģɋ

8o

HDʥjʥ 6ʥ1ʥK> P ">ʥK> P >ʥ1ʥK~ P — K>ʥ,ʥ ǿ"ʥ1ʥƞʌ> ( "ƣ> ¬ -­ VʥĝęSGʥ& BGʥʥ6ʥ1P -#ʥ

o

HuʥƢHʥ  g,Œ h+Œ 3; x1ʥ 77'7 7ʥ 77+7 7ʥ 77-7 +7ʥ 77 7 7ʥ 777 7ʥ 77"7 #ʥ

o

Huʥ'ʥ

"Kʥ ʥ V   ʥ

” ʥ

ʥ'ʥ ʥ H ʥʥV   ʥ >ʥ  "Kʥ>ʥ,ʥ-Kʥ>ʥ1ʥ ʥ>ʥȚʥ'Kʥ1ʥ {ʥ .#ʥH ʥ ”S ¡  "K-K¡ ʥ{
ʥ


 o

M…Dʥʥ'ʥ ʥ iʥʥ  ʥƒʥʥÇ ʥ  ʥʥ ʥʥvʥ  ʥĔʥÇg;#ʥ ʥ v1 ʥĔ#ʥV i1 ʥvȀ'ʥĔʥ1 "ʥ –ʥi 5ʥ1ʥ'# ”ʥ gʥ "ʥ ,ʥ Ŀʥ NJNj ǭʥ Ŀ DZʥ

%o

M…Dʥʥ ʥ ȵʥʥƒʥ ʥÇʥʥ Ȇʥ,ʥ ʥ1ʥ #ʥ

*o

MDʥ  ʥ Aʥʥ  ʥʥ"ʥʥʥ"Ħʥ ;#ʥm^2ʥʥ ʥ'ʥÇ ʥʥʥ"ʥ ʥʥʥʥʥ;#ʥZʥk 3 žȂFĈʥy Ħʥ1ʥ  #

.o

MDʥ"ʥ ”ʥ ʥȝ3 4ʥ-ßʥ ɣʥ ʥʥÇʥ3ʥʥʥȜ34Įʥ¦#ʥ "(+ʥ1 ” ¦ʥ Xʥ

³ʥ

0o

MDʥ ȁʥ

ʥ

1;

;



ʥ

ʥ

ʥ Ą 1ʥ mʥ ʥ 5ʥy 5ʥ,ʥ ʥ ʥ 5ʥw 5ʥʥ  5ʥ,ʥ ʥy 5ʥ ʥ ʥ #ʥ y 5ʥ,ʥ ʥ

ʥ

ʥ

ʥ # Zʥ ʥY  Y ʥ,ʥʥY ʥ , ,ʥ ,ʥ ,ʥ

 Y ǬʥY ʥ ʥY ʥY " ʥY ʥ ʥY ʥY "ʥ

ʥ



ʥ ²ʥǰʥ ʥY ʥ( ʥY "ʥ 1ʥ "ʥ#

;

;

;



;

3o

M¸ʥ ʥ

ʥ( ʥ¹º ( ʥ

iʥKʥ0ʥ.ʥʥ 7ʥ6ʥ0ʥbʥā ʥKʥ( 6ʥ 7ʥʥKʥ( ʥʥĄX ʥ;6ʥʥ ʥʥƻ6ʥ »ʥKʥʥX Wʥ6ʥĄ6ʥ

; :;Wʥʥ¹º»ʥʥ0ʥ

¹º»ʥK(6ʥ ( 7o

123

.b (.ʥbʥ0ʥ #ʥ

Muʥ  ʥ V$$ʥ6ʥno #ʥAʥ¨ʥ6ʥ* 6ʥʥK ʥ ; 'ʥʥ gʥKʥ0ʥ'ʥ  ʥ•ʥ G‚K¨ʥKʥʥ6ʥ0ʥ ʥ

ʥ  g 6 0Pʥ ; hʥ]o "7Œ Vʍhʥ] V$$ʥKʥaoʥAʥ G‚K¨ʥKʥʥ6ʥ0ʥ ʥʥ gʥ6ʥ0ʥĎʥʥ !Œ7Œ V !Ďʥ#

ʥ —ʥ Ńʥ Zʥʥʥʥʥ< Źʥʥ ( 6ʥʥK ʥ ;'ʥ ʥ ʥ Kʥʥ6ʥ0ʥ ʥ¹º»ʥKʥ0ʥI‚ 6ʥW Ă7

8o

Huʥ'ʥ V)ʥÚʥʥÈʥ0ʥ"5( '7 ÚʥʥÈʥ0ʥ, -ʥ)ʥ)ʥ ʥV Ú 0ʥ 7ʥÈʥ0ʥʥ  ʥ,:Wʥ'®ʥ

o

Huʥʥ"ʥ mʥ ʥ Kʥ–žʥ6ʥIžʥ -ʥ( -KʥP -6ʥ¡ 0ʥIžʥ– -ʥ % -ʥ 7ʥʥK6ʥ0ʥ -®ʥ ȉʥK7 6ʥWʥ 7ʥ -7ʥ'7ʥ +'7ʥ— 7ʥ—-7ʥ—-7ʥ— 7ʥ +'7ʥ'7ʥ -7ʥ ®ʥ

o

Muʥ +"ʥ

ʥ

ʥ

ʥ

ʥ

ʥ

ʥ

ʥ iʥÒʥ0ʥ ʥ ʥ —ʥʥ ʥʥ "ʥʥ 'ʥʥ +ʥʥ -ʥ#

ʥhʥL ʥ‚hʥ +'Ń 'ʥ hʥÒʥhʥ 'ʥ 789 +"ʥ<‚ F Žʥ -ʥ ʥ

o

HDʥ "ʥ iÙʥ ʥ ƒʥƒʥʥ  ʥƒʥ ʥ 3Òȧ7ʥʥ gʥ 3Ñʥ0ʥ 3=ʥʥ=¦Ȩʥ ¹ºƠ» ¾3ďĀʥ34¥Ŗʥʥ¥X §Śʥʥ§ÀÑʥ0  3ďĀʥ 4=ʥ=¥Àǚħʥ¥X §ŚʥʥX¦ŗʥ X ¦ŗʥ 456 34¥Ŗʥʥ§ÀÑʥ0ʥ 4=ʥ =¥řǛħʥ ¹º»ʥʥʥ§ÀÑʥ0ʥ ʥʥ—+ ʥʥ ʥ 789 §řÑʥWʥ "®ʥ

—ʥ

%o

ÓÔµʉDʥ'ʥ i Â1ʥ ''ʥ¬   ʥî1ʥ ''ʥ  ›ž  1 ʥ Âʥî‡ʥ >ô ʥ;-ʥ \ʥ1 ÂʥȺ\ʥ1ʥÂ\ʥʥȹ\ʥʥȷîÂʥʥȻ V)ʥÂ\ʥʥî\ʥ1ʥ Ėʥ1ʥ \ʥ1ʥ ʥʥ ' ʥèʥ \ ( ' ( ʥ;ʥè  ( ' >ʥʥ' ʥʥ ʥ1ʥʥ 1ʥ'# :;

*o

ÓÔܸʥ ʥ Áʥ) ʥʥʥʥ)uʥ  ʥ ʥ ---ʥÁʥʥʥʥʥ )gʥ)ʥʥ.ʥʥ:ʥ`1ʥ

ʥ dd \ d d\ Lo )ʥ ã ă F\Ŧdʥ1ʥ F>ʥ1ʥ'+ʥmʥ ʥ

ʥ ʥʥʥ$)ʥ ʥ

ʥʥ   Vʥʥʥʥ)ʥ)ʥ'+ʥ( ʥ( "ʥ1ʥ"-ʥ ʥ  ʥ ---ʥÁʥʥʥʥʥɫ )gʥ)ʥʥ.ʥhŒʥa1ʥ ‡ʥ Uʥ)ʥdǧã\ădF\Ŧdʥ1ʥd>F>ʥ1ʥ""ʥmʥ ʥ ʥ ʥʥʥ$)ʥ ʥ ʥVʥ ʥʥ)ʥ)ʥ""( ʥ1ʥ"ʥ ʥ  ʥŠŒ -ʥÁʥʥ ʥŽʥ -ʥ )´‹ʥʥ$$Žʥ NO

NO

ŏ‡ʥ@ʥ ʥʥʥ)ʥ )Ã)ʀʥ$$Žʥ)ʥ"-ʥʥ"ʥʥ ʥ1ʥ ʥ

.o

ÓÔµÛDʥʥʥ ĭ .z1ʥ ʥʥ >ʥʥ \ʥ‡‡‡ʥʥ zʥ1ʥ zãȤ ĸ  i !ʥ1ʥ ʥµÛʥ gʥ.ŷʥʥ.dʥ.ʥ >ʥʥʥ ʥ  . 1  H .d1  ʥʥ51ʥ ʥ z "#"#ʥ 5ʥ "#ʥ ʥ"ʥ( ʥ ʥ ʥ ʥ ʥPdŒ zãʥdʥ ʥ#"# ʥ ;O

0o

ÓÔµʊDʥ "ʥ

ʥôʥʥnʥw ʥbh; "ʥ> mʥ ʥ  ʥʥſ tvŠʥ1ʥ2‚ 

3o

ÓÔÜDʥ'ʥ Aʥ)gʥ< )ʥ!dʥʥ!>ʥʥ[\ʥʥ[~ʥ1ʥʥ1ʥ!dʥ!>ʥʋ\ʥʥ!~ʥʥ[\ʥ[~ʥ!ɞʥʥ!>ʥ1ʥP-ʥ  !dʥ!>[\[~ʥ1ʥ( -‡ʥmµÛʥ!dʥ!>ʥ1ʥ-ʥèʥ[\[~ʥ1ʥP ʥVʥ!dʥʥ!>ʥ1ʥ ʥ!\ʥ[ʥ ~ʥ1ʥ( ‡ʥ Zʥyʥ² [øʥ[>ʥ[ʥ øʥ[\ʥ[øʥ[~ʥ[ʥ >ʥ[\ʥ[ʥ >ʥ[~ʥ[ʥ \ʥ[~ʥ1ʥ-ʥʥ ʥY ʥ( ʥ(  ʥ1ʥ'#ʥ



7o

Muʥ  ʥ m*ʥ ʥMŒ *«t 9>pʥMŒ ʥ*ʥ«t 4>ʥMŒ ʥ*ʥ«t9ë >ʥM ʥ*™9ë >ʝ V*ʥMŒ ʥ*ʥ«t 9>ʥ, MŒ ʥ*ʥ«t Î >ʥM ʥ*ʥ«t9ë >ʥMŒ ʥ*ʥ™9Î>ʥ ʥw žMŒ ʥ*ʥ«t ë >ʥM ʥ*ʥ«t94>ʥ, MŒ ʥ*ʥ™9Î>ģ Wʥ ʥY žʥ ć ʥY 'ʥY "eʥ ć ʥY 'ʥY >ʥʥ ć ʥY ʥY "Øģʥ  # n‘



8o

M…Dʥ

ʥ Ǚʥ -l  N N ʥWʥ-ʥʥ  ʥ #ʥÐʥ 5P ʥ )ʥ ʥ*ƹź*ʥ )Nʥ• )*7 F*ɝ ʥ lʥ ʥ l ʥ -lʥʥ

ʥ ʥlʥ ʥ)ʥ N*ʥ ʥ*ź*ʥ )Nʥ• )*ʥèʥi ʥ$*))&ʥ)ʥlʥWʥ

# :O

%o

:O

MDʥ-ʥ K>ʥʥ Oʥ,ʥʥKʥ,ʥ NssO ʥ-ʥ ʥK Kʥ POʥ,ʥʥio OP ØP - #ʥ@) ;)N ʥ)ʥ*N)*ʥ)ʥ  ʥ*N;ʥ)ʥO( Ø( -ʥ)ʥ ʥ$ʥ< 7ʥ ;ʥl >ʥ ZʥO( ØPl >ʥWʥ -ʥWʥ'>ʥw  ʥéʥ n‘



¨ʥO (ňʥ,ʥlʥpʥ -ʥ  ʥ¨ʥO (ň(lʥWʥ 7ʥ*ʥ ò O( ò51 +'  ʥò OP òPlp '7 *ʥ ¨ʥOʥP ʥ¨ʥʥ,p : ʥ-ʥɀ ʥ¨ʥOʥ( ʥ) lʥWʥ # V*N&c2ʥlʥWʥʥjo 7ʥʥjo '7ʥʥioʥé@ʥ NNʥ$*))&ʥlʥ)ʥ-#ʥ

%o

Muʥʥ MŒ ʥ*ʥʥ) Nʥ) w =ʥw 94 Wʥw 4ʥw 9³ W w 9ʥw 4X V)ʥ =pʥʥ  ʥ9³ Wʥ 7ʥ94 Dʥ 4ʥ ʥDʥ ǜʥi 4ʥ K7ʥʥ94ʥ Kʥ n‘





ĕ)ʥ@) NʥÐ< N);7ʥ 9ʥhʥ 4ʥʥ94ʥ  ʥ94ʥhʥ 9ʥʥ 4ʥșʥ Kʥhʥ 9ʥhʥK#ʥ A*ʥ4Xʥpʥ

ʥhʥ4Xʥhʥ"ʥé@ʥĜ ʥ)ʥ& Nʥ*ʥ4Xʥ)ʥ#



;

MDʥ-'ʥ ”ʥNʥʥ ʥ ))ʥ ))ʥʥ ʥ  ʥ9ʥ)ʥ ʥȐ ȋȌʥ-ʥ¼½ʥ9¼½ʥ-ʥ  ʥ ʥ? 9ʥ F ʥ ʥDʥ 9 9ʥpʥ  ʥw 9ʥ@ʥ ʥ-ʥw -ʥpʥ+ ʥ$*))N))ʥ Z*&ʥ 9ʥʥ*ʥʥ ʥN)$Nʥ*ʥ'ʥ ʥ$*))N))ʥ^N )ʥ'ʥ  ʥ''ʥ~ @ʥ ʥ+ ¡ ʥ1ʥ"-ʥ F ʥ uʥ 99ʥ1ʥ

ʥw  ʥss ʥ, 9‡ʥ@ʥ ʥ

ʥ$*))N))ʥ ʥ ʥ *ʥ*ʥ'ʥ  79ʥpʥ  ʥ ʥ ʥ+ʥ"ʥʥ" ʥ"-ʥ'ʥ+ʥ-ʥ F ʥDʥ 99 ʥpʥ

ʥw - ʥʥ sʥ 9ʥ9ʥʥʥʥ*ʥ'ʥ@ʥ ʥ 'ʥ$*))N))ʥ Ȉ­ʥ@* Nʥʥ$*)Nʥpʥ"-ʥʥ

ʥ,ʥ 'ʥpʥ-'#ʥ

;

MDʥ"ʥ ÐlʥpʥʥOʥpʥ ʥ ʥʥʥʥ ʥɲʥʥ ʥ ʥ$ )ʥ Ð Oʥ1ʥʥlʥpʥʥʥʥʥʥ*ʥʥ ʥ+ʥ$ )ʥ ÐlʥOʥ? ʥʥ &ʥ ʥ Dʥ F ʥ DʥOʥ²ʥ.ʥlʥ1ʥN Nbʥ.ʥ>

ʥ  ʥbʥ= ʥʥ ʥȑʥ.¼½ʥN Nb¼½ʥʥ V*ʥ)ʥbʥ² ʥ.1ʥ ʥ ʥʥ  ʥ)ʥbʥpʥ ʥ.Ʃ1ʥ ʥ ʥʥ##ʥʥ'ʥ@ʥ ʥ ʥ$ ) F ʥ ŌʥOʥ²ʥN N.ʥlʥ1ʥbʥ.ʥ> ʥ  ʥbʥ<

ʥʥ ʥȒʥ

.¼½ʥb¼½ʥʥ V*ʥ)ʥ.ʥ² ʥbʥ1ʥʥʥ"ʥ'ʥ+ʥ-ʥ ʥ ʥ ʥ  ʥ)ʥ.1ʥ ʥbʥ;+ʥ-ʥ ʥ ʥ ʥ @ʥ ʥ-ʥʥp ʥ ʥ ʥ$ cŒʥ Zʥʥ &ʥ N*ʥ ʥʥ+ʥʥ ʥʥ ʥ1ʥ"ʥ$ c‡ʥ

;

MDʥ ʥ ”ʥ NNʥ ʥ<ʥ ʥ )Ãʥʥ* ))*ʥ ʥá** ùʥ<ʥ iʥ zʥ *ʥʥʥ*ʥá** Ɖʥ<ʥ ʥ ʥ)ʥ)ʥʥ*ʥʥ 9zʥ *ʥʥʥ*á** ùʥ<ʥ ʥ ʥ)ɪʥ)ʥ ʥ*ʥʥ 4zʥ *ʥʥʥ*ʥá** ùʥ<ʥ ʥ ʥ)ʥ ʥ ”ʥ &ʥ  ʥ zĴʥ1ʥ9zʥ  ʥ ʥ<ʥ)ʥ zĴʥ  ʥʥ*& ʥ*ʥ ʥ<ʥ )ʥ9zʥ;ʥ N)ʥ)ʥN ʥ ))ʥ



% ʥ

#6;‘  !6;ʥ $6;  ʥ ʥ<ʥc…ʥ!6;  ʥʥ& ʥʥ ʥ <ʥʥ#6;‘  ;ʥ ʥ ʥ ʥ%ʥʥ ʥʥ ʥʥʥ ʥʥ%ʥʥ ʥʥ ʥ  ʥʥ ʥ  ʥ ʥ<ʥʥ$6; ʥʥ& ʥʥ ʥ<ʥ #6; ;ʥ ʥ ʥ ʥʥ ʥʥʥʥ

%ʥ

$7;‘ 1ʥ#6;  ʥ ʥ<ʥʥ$6;‘   ʥʥ& ʥʥ ʥ<ʥ ʥ#6;;ʥ ʥʥ ʥ ʥʥ ʥ

Zʥʥ ʥʥ ʥ#6; 1ʥ#  ʥ5ʥmo ʥ FÙʥ ʥ#; 0ʥ ʥ  ʥ3>ʥ0ʥʥʥ#6; 1ʥ ʥY ʥ6;  ʥ#6;1ʥʥY ʥ6 } kʥ!4;ʥ#4;ʥ14; 0ʥ#4;ʥ#4;1ʥ ʥY ʥY Ĉʥʥ ʥY Ǽʥ²ʥ # ‘

‰‘ 9O



%.o

HDʥ%'??'ʥ F ;ʥOʥ  ʥ,: ʥʥ ʥʈ#ʥ m^ʥʥ ʥʥ$ɒʥ ʥʥÙɨʥO5ʥ1ʥ ʥw Ĉʥw ʥw 'ʥw

>ʥw ?ʥw ' mʥʥʥ ʥ ʥʥO5ʥʥʥʎʥ ;ʥ ʥ S‚Oʥ  ʥ S‚5ʥaʥʥe ʥʥO5ʥʥ ʥ– i ʥ ʥʥOʥʥʥ%ʥʥ  ʥ ʥ ʥʥ5ʥʥ ʥ%ʥʥ"ʥʥ+ʥ HʥǸ T‚O5ʥʥ?ʥ & ʥ ʥ ʥʥʥOʥ  ʥr ŝʥʥʥ  ʥ? U‚Oʥ–ʥ? U‚5 k ;ʥ

ʥE‚Oʥ–ʥ

ʥ&‚, kʥ,:0ʥ ?+yʥ@ʥʥ ʥ ɽʥʥyʥʥʥʥ?#ʥ kʥʥ ʥ ʥʥ'ʥ ?ʥ'ʥ ʥ ʥʥ ʥ ʥʥyʥʥʥ?ʥ ”ʥ &ʥ;ʥʥʥ Dʥy 0 'ʥ ʥw 'ʥʥʥw 'ʥw ?ʥʥʥw ?ʥw '#ʥ Ş ʥ ʥʥՋʥ ʥʥ5ʥʥʥʥȍȎȏʥ ?+yʥhʥ"ʥ–ʥyʥ1ʥ'ʥw 'ʥ Zʥ5ʥ1ʥ ?+y1ʥ + ʥ  Oʥ0ʥ + ʥ–ʥOʥʥ5ʥ1ʥ'??'#ʥ



Cey\O‚~‡Xž =O‹`XtO‹eSOpž C~SeX‹ž Efz]PƒˆYž>PŒaYuPŒfTPqžAq‘uƒfPVž E>A ž E;;Q % !"' $! ' !% ' '

 Eq“ @:=? “ J:6“"J=“I\}“ %1&1_ X\“}b\“m\qa}b|“t]“c}|“|c[\|“ =?“?: “:= “ y\|u\Y‹ }c„\mˆ“ Oyt„\“}bW} 1 % 6“ &&1' 1

 Tb\“ |\}“ t]“ "[cac}“ cq}\a\y|“ Wy\“ [c„c[\[“ cq}t“ }…t“ |\}|/“ }b\“ |\}“ Qo Ytq|c|}cqa“ Wmm cq}\a\y|“\WYb“t]“…bcYb“YWq“X\“y\uy\|\q}\[“W|“}b\“uyt[‚Y}“t]“}…t“ [cac}“cq}\a\y| Wq[“ }b\“ |\}“ Ro …bcYb“ Ytq}Wcq|“ }b\“ t}b\y“ cq}\a\y|“ VbcYb“ t]“ }b\“ |\}|“ Qo Wq[“ R Ytq}Wcq|“ oty\“\m\o\q}|8  Q‚uut|\“ %%1 1%1 c|“W“ "+[cac}“ cq}\a\y“|‚Yb“ }bW}“]ty“\WYb“ #6“ “" “““"( }b\“ "[cac}“ cq}\a\y“ %(%()1 Ytq}Wcq|“ $o [c|}cqY}“ uyco\“ ]WY}ty|“ Bcq[“ %++,01 Mt}\/ -./1   “[\qt}\|“ Wq“cq}\a\y“ …bt|\“ [cac}|“Wy\“ -1.1/1  “ )

I\}“R“X\“}b\“|\}“t]“cq}\a\y|“}bW}“YWq“X\“…yc}}\q“cq“}b\“]tyo“&p“1$\o …b\y\“p“Wq[ q Wy\“qtqq\aW}c„\“cq}\a\z|“Bty“\‡Woun\“ $o &“ &$“Wy\“Wmm“cqR“ Bcq[“}b\“|‚o“t]“Wmm ut|c}c„\“cq}\a\y|“qt}“c{’“R

-

I\}“ %1&1 X\“ ut|c}c„\“y\Wm“ q‚oX\y|“ |W}f|`cqa“ %1&16“ “ Qbt…“ }bW}“ c]“ -1-1

11- Wy\“ut|c}c„\“y\Wm“ q‚oX\y|“|‚Yb“ }bW}“ --1

1-16“ “ }b\q %-11&%-1&0001%-1 &1# “



Dey\O‚~‡Xž =O‹`XtO‹eSOpž D~SeX‹ž Efz]PƒˆYž>PŒaYuPŒfTPqžAq‘uƒfPVž E>A ž E;;Q % !"' $! ' !% ' ' #!%$! #'

K\}“ BGx X\“ }b\“ Wqam\“ Xc|\Y}tx“ t]“ MBx Tb\q“ 3BCDx89:;x 3GBDx Ub‚|“ BCx"xGBx@ BD"GDx @x CD\BDx I\}“ HGx7“lx Wq[“ GDx7“m x Ub\q“ W"lx @x V\mx @x T\Vx Tb‚| WVx@ TlxV+x@xTmx Tb‚|“V+x xWVx@ Tl Tx mxWq[“ b\qY\“ VVx W @ xT+x



N



 V\“|tm„\“}b\“a\q\xWm“YW|\“t]“"r[cac}“cq}\a\x|“ …b\x\“r“#“ "“ Tb\x\“Wx\“ %#+i  %#+i' "r[cac}“cq}\a\x|“ Ub\x\“Wx\“ %#ix ki'x r[cac}“cq}\a\x|“ >tq|c[\x“ Wmm“}b\“uxt[‚Y}|“t] uWcx|“t]“r[cac}“cq}\a\x|“ Ub\“}t}Wm“q‚oX\x“ So t]“|‚Yb“ uxt[‚Y}|“|W}c|_\| _

 ki(x x A kix 



%#+i  %#+i'  %#+i(  %#+i+ kix x ki'x "“ "“ Ub\|\“uxt[‚Y}|“cqYm‚[\“Wmm“}b\“q‚oX\x|“cq“ Nx Ub‚|“ KNLx A LOx  T…t[cac}“ q‚oX\x|“ …bcYb“Ytq}Wcq“ }bx\\“ [c|}cqY}“ uxco\“ ]WY}tx|“ Wx\/ $“@x "Œ$& %"“@x "Œ$Œ) '“7%Ž$Œ& ''“7"“ $Œ “)“ @x "Œ&) )*“@x "$ $ *%7“%Œ$Œ) Bzto“b\z\“ …\“YtqYm‚[\“ }bW}“ %(16“'“ ]tzg6“ 11   ““"(“Wq[“ %1 c|“\c}b\z“'“tz“  1

)#x

E]“ -1 c|“ }b\“ |oWmm\|}“ cq}\a\z“ cq“R“ |‚Yb“ }bW}“ -1  # ot[“$ “ }b\q“ -11$j“ IoR“ Wq[ - $ j“ 11 “R“ ]tz“Wmm“ j“ JKŒ  1 V\“bW„\“$“ c|“}b\“|oWmm\|}“o‚m}cum\“t]“$“ }bW}“c|“cq“R2“  c|“|oWmm\|}“q‚oX\z“cqR“}bW}“c|“ J 1 ot[“$ “ Wq[“ 1 c|“}b\“|oWmm\|}“q‚oX\z“cqR“}bW}“ c|“ 1 ot[“$ “ Tbƒ|“ }b\“ut|c}c„\“q‚oX\z|“ qt}“ cq“R“ Wz\“ 1%“   ““+(“ Wq[“ 11   ““%(“ Tb\cz“|‚o“c|“ $$+(“11 '"“1%( “ 1 1  1 1 )

1O

-

Tb\“m\]€“bWq[“|c[\“c|   % -$1

1$11% &-$$$11$$$$11‡Œ‡Œ ‡Œ 1$$$$   1% & -$$11$$$11‡Œ‡Œ‡Œ 1$$$     1% & --11$$11‡Œ ˆŒˆŒ 1$$11%& -11-11‡Œ ‡Œ‡Œ 1-11&  1 1   1 1 1 1 # % 1% &11% & 1% & 1%& 1& !% 1& i‘ 

Tb\“mW|}“c|“}z‚\“|cqY\“Xˆ“ 9L CL“ cq\w‚Wmc}‰“  1 $$$$11$$$$11‡Œ‡Œ ˆŒ 1$$$$1 JKŒ $$$$$  6“  $$$11$$$11‡Œ‡Œ‡Œ 1$$$1 # $* $$$-*"1 #  $$11$$11‡Œ‡Œ‡Œ 1$$1 JKŒ $$$$$ *" 1  $11$11‡Œ ‡Œ‡Œ 1$1 JKŒ -*$$ $$  1 /O

/O

/O

Mt}\“ Btz“W“uztt]“t]“}b\“a\q\zWm“YW|\“ |\\“Q\qctz“P% “

1

Bdx[N}†Wž
cƒn€lf†*ŒDŒ^ƒxnŒD::PŒ



:PM:/ >D::Œr€Œ

#"$&!&)

"'+&- - LNΠ%)'(#"'

"(&-,#)&-"'+&'-#"-(-"'+&-'(-$&#*#&- (- !) ($ - #- %)'(#"'- "(&- ,#)&- "'+&- #"- (- "'+&- '(- ,'"- (- ) -#"(""-(- ((&-  - V(ΠY)Π[Π#&- - #&&'$#""-(# (-#&&(-"'+&#&- (- #(&- '#&(- %)'(#"'- +&(- ,#)&- "'+&- #"- (- "'+&- '(- "'-(-$$&#$&(-) -  #+-,#)&-"'+&#- '($'-&-"-(#-)'(-,#)&- "'+&' - %)'(#"- &&'- =Π!&

#- ) (#&'- &- #+-

  )))) ) )) UΠ )))

ʥ

_‚ts{tmŒXqzsjmŒa‚m~szw~Œ



k$$ʥ ʥp ʥ ʥ$ ʥ  ʥ ʥ  ʥ9ʥ ʥʥ$ʥʥʥ$ ʥp Uʥʥ  ʥʥ ʥ  ʥ9ʥʥʥnʥʥ ;ʥʥ ʥʥʥʥ$ ʥ ʥ ʥʥ  ʥʥ ʥʥ  ʥ ʥʥ'ʥïʥ  ʥʥ  ʥÝʥ  ʥʥ  ʥ9ʥʥ "ʥnʥ$&;țʥ %Hʥ ʥ %aʥ ʥ %Fʥ ʥ %Yʥ ʥ jʥ Ue;ʥ ;ʥ



iʥ6ʥ=‚% '(!% ?P !% ?ʥʥ!% 'ʥ!ʥ ʥʥ! ʥ ʥ ʥʥA ʥ  ʥ$ʥ& ʥʥ6ʥ %Hʥ Ƭ ?"ʥ %aʥ P ?ʥ %Fʥ P ?ʥ %Yʥ P ?ʥ jʥ P ? ʥ

ʥ

UʥÝʥ ʥʥ]  ʥR ʥ  ;ʥʥ•ʥʥ& ʥ%ʥ 7ʥՋ ʥʥ $ ;ʥ ʥʥ< ʥ!/ʥ( JK‚! ʥ ʥRʥ>‚ʥ ʥ ʥ %Hʥ ʥ +ʥ %aʥ ʥ

"ʥ %Fʥ ʥ

"ʥ %Yʥ ʥ ʥ jʥ ʥ ʥ



Uʥ!ʥ  6ʥ ʥ ʥʥʥʥ n!mžʥ!ʥʥ 6ʥ ʥ>‚ ʥ  ʥ 4J 6ʥʥ! 'ʥ Ջ ʥʥ & ʥ !ʥʥ6ʥ %Hʥ (ʥ %aʥ P ʥ %Fʥ ʥ %Yʥ ʥ jʥ ʥ .O



#

Uʥ ʥ) ʥ 342ʥ)ʥ 0 ʤʥ  ʥʥ3ʥ1ʥ 3‚ A) ʥʥg ʥƒʥʥ4# ʥ

ʥ "ʥ

"ʥ %Hʥ ąą "ʥ "ʥ aʥ " "ʥ %Fʥ " "ʥ " %Yʥ "ʥ "ʥ " # ą (Ƽ "ʥ "ʥ

"

@ʥ  ʥƒʥ ʥ) ʥ 34ʥSʥʥɸɦ#ʥíSʥ=2ʥ³ʥ  ʥX ʥʥS ʥ 32ʥ34ʥ  4 ʥÇ)g;2ʥ ʥʥ)ʥʥ×ʥ#ʥUƒʥ =1ʥʥn2ʥ=31ʥʥn2ʥ  ʥ  ʥƒʥó) ʥ 3¥ʥ)ʥ< ʥʥʥ  ʥƒʥ< )  ʥ=3¥X2ʥ× ʥʥ ũ ʥƒ ) ʥ Ƞ4ʥ)ʥ>#ʥ %Hʥ

ʥ aʥ ʥ %Fʥ ʥ %Yʥ ʥ

ʥ \Œ

=ʥ '#ʥ

A) ʥʥg ʥƒʥ %Hʥ ƭ ʥ Eʥ aʥ ʥ( ʥ ʥ E

ʥ( ʥ WŒ ʥ E %Yʥ ( ʥ Eʥ

ʥ( ʥ # ʥ E

ʥ ʥ ##ʥʗ ,ʥ ,ʥ ,ʥ

E EEʥ EEEʥ E E Eʥ

!

"

#

$

%

!

&

%



+

@ʥ ʥʥ&$ʥ ʥ ʥʥ+ʥA ʥʥʥʥ ;ʥʥʥ   ʥ ʥʥ  ʥʥ  ʥʥʥ ʥʥ$ʥʥʥ&$ʥ  ʥ ʥ&$ʥ ʥ^ ;ʥʥʥ  ʥʥʥʥʥʥʥ &$ʥ cʥʥ ʥʥʥʥ ʥʥʥʥʥʥʥ &$ʥ ʥʥʥğ Hʥ ʥ %aʥ Ƕʥ %Fʥ  %Yʥ 

jʥ "

?

Ycʥʥʥʥ   ʥó ʥ%ʥ ʥ ʥ ʥʥ ʥ?ʒ ʥʥ ʥ ʥ%ʥ ʥ ʥ$&ʥʥ  ʥʥ ʥ ʥʥŒʥ ʥʥ ʥ ʥ Hʥ ʥ %aʥ ʥ %Fʥ ʥ %Yʥ "ʥ jʥ 'ʥ



i 94=ʥʥ ʥ<   ʥc ʥʥ ʥcʥʥ  ʥ 4ʥ  ʥ T  ʥʥʥ$$  ʥ•ʥ=ʥʥ 9ʥ ʥʥʥʥeʥʥUʥ = 0 =4ʥ  ʥʥ  ʥʥ< Ɇ  ʥ 94=ʥʥ ʥ>ʥe ʥʥʥʥ=Tʥʥn ŕʥ

Hʥ %aʥ %Fʥ %Yʥ Œ

ǎʥ Ēʥ Ǐʥ ǐʥ "ʥ



bqz|Œa‚m~szw~Œ

#ʥ A ʥʥʥ $&ʥ &ʥʥ  +\ʥijʥʥw +ʥw ?ʥ,ʥ >

ʥ k$$ʥ ʥ.ʥRʥ  ʥ` ʥ ʥʥ ʥ ʥ #ʥA ʥʥ& ʥʥ

ʥ ʥ ʥ

ʥ 

ʥ,ʥ

ʥʥw}‚

ʥʥɃ}ŧ

;7;

;8;

ʥ A ʥʥ  ʥʥ  Eʥw ʥʥ Eʥy ʥʥEʥw ʥʥ‡ ‡ ‡ Œʥ +"Eʥw +"ʥʥ &  ʥ;ʥ ?ʥ °ʥ °

#ʥ A ʥʥ& ʥʥ   , ǕȪE  ( ɯǞE ʚ

ʥ €ʥ.:ʥ ʥʥ

5ŒAœ ʥ ʥ& ʥɊʥ.ʥ A‘

ʥ



ā.(ʥ  [W‘

"ʥ UʥʥՋʥʥ 34ʥʥ ʥ ʥ  ʥ=ʥʥ ʥ$ʥʥ ʥ34ʥí Tʥʥʥ  ʥ 3ʥʥ ʥ=Tʥʥʥ ʥʥʥ.&;  ʥ$ʥXʥʥʥ ʥ 4ʥ ʥ ʥ=Xʥʥʥ ʥɾɮʥʥȯ =4 #ʥA„ ʥʥ& ʥʥ Tʥ#ʥ3=ʥʥ4Xʥ # T3ʥ =4ʥ X ʥ ʥ

'#

A ʥʥ& ʥ %CǹƓʥ( %Cúʥ(ǩ%CƔʥ( %CCúʥ( %C úʥ %C ĸ  ʥ 

+

O

A ʥʥʥʥCʥʥÞ.ʥRŸʥʥ žʥ ʥ ʥʥ6 ʥʥ??ʥ Ÿʥʥ  .Rʥ. , Rʥ ʥʥ ʥ$ʥ ' "ʥ

?#

€ʥ!ʥʥ ʥ ʥʥʥ ʥ!/ʥ( !ʥ‚   ʥAĚ ʥʥ& ʥʥ! Ëʥ‚ -O

X

#ʥ ĐʥʥŪ‹ʥʥ 34ʥʥ ʥ ʥ 3ʥ0ʥ ʥnʥ  ʥ34ʥ ʥn#ʥíʥ=ʥ

 ʥTʥʥʥ ʥ 4ʥ  ʥ$ʥXʥʥ ʥ34ʥʥ ʥTȢʥʥ$  ʥʥ 3ʥ  ʥ=Xʥ ʥ$  ʥʥT3#ʥ“&ʥ ʥ3Tʥʥ ʥ ʥʥʥȰ34ʥ  ʥ ʥ Ȟ0ʥ ʥnʥÖ ʥʥʥʥ4=ʥʥnʥ Eo n‘

ʥ €ş 0ʥž ʥ ʥʥ‘ʥ7ʥ"7ʥ"ŸʥYɈʥʥʥʥ  ʥ$ʥ%!ʥ6ʥʥ ʥ ʥ! 6ʥPşʥ!ʥ’ʥʥ  6’ʥ ʥ ‘

“&ʥ ʥ_un‚>‚ _u‚ ʥv‚?‚ ʥ ʥʥ###ʥĵʥ  ʥDx@‚_o‚@‚ ʥ Ū‹ ʥʥ& 

ʥv_uX‚_u‚

ʥ



Đʥʥeʥʥ 34ʥʥ ʥó ʥ 3ʥ0ʥʥnʥ34ʥ?‚ ʥnʥ  4 0ʥ ʥnʥí vʥ  ʥŠʥʥʥ  3ʥ  4 $&;ʥʥ ʥ

 ʥʥ™vŠʥ>‚]‚ “&ʥ ʥʥ ʥ$ʥʥʥvŠʥʥyʥnʥe ʥ

 ʥʥ™34ʥ ʥ & ʥʥy­ʥ ʥ





1;





Uʥ!ʥ6ʥ  ʥʥ ʥ ʥʥʥ ʥ!ʥ,ʥ6ʥ,ʥʥ ?‚?ʥ  ʥco ,ʥ6ʥʥ!ʥ?‚ ʥe ʥ ʥ$ʥ& ʥʥ #ʥ



A ʥʥʥsP ʥ ;ʥ<ʥ ʥ;ʥ^ʥsûʥ  ʥ^ʥ ʥûʥʥ  ʥʥsûʥ ʥʥAʥ^ $Ĝʥ

    ʥʥʥ ʥ<ʥ

 

  ʥ  ʥ

 

 ʥ ʥ

"ʥ Uʥ

 †ʥ

(ʥʥ ţʥʥ ţʥʥ†ʥ

#O

e !

'

Ac ʥʥʥʥ$&ʥʥ!ʥʥ!ʥ ?ʥʥ   hʥ"ʥ ʠ

+#

€ʥ5ʥʥʥ$&ʥʥʥ ʥƕ

ʥ

ʥ

ʥ ʥ  { {ʖʥ N Nǟ  Ǡʥ ȱ  łȬʥ A ʥʥ& ʥ 5#ʥ

?

UʥʥՋʥʥ 34=ʥʥ ʥ ʥTʥʥʥ $ʥʥ =ʥ  ʥXʥʥʥ  $ʥʥ4³ʥUʥʥ  ʥʥ ʥ3=Xʥɚʥ ʥ_ʥe ʥʥ  ʥʥ  34=ʥʥ_ž



Uʥ ʥʥʥʥ"Ʊ ʥ$&ʥ¸ʥʥʥ+ ++

 ++ʥ&;ʥ ʥʥʥʥ $$ ʥ ʥ ʥʥA ʥʥʥʥ " ʥ$&ʥ

 

€ʥ!ʥ  6ʥʥ$&ʥʥʥ ʥ '!ʥ,ʥ6Œʥ?ʥA ʥʥ  $ʥ& ʥʥc

M

/(



ʥ

: ¢ ʥ ?

 

Yʥʥ×ʥʥv - ʥʥ^$ ʥʥ% ʥ‚v* ‚v ‚v-

#

Aʥn ʥ ʥʥʥʥʥ]u‚I n  ‚ ʥ  ʥʥbu‚ ʥʥ ɿʥĝ &ʥʥ.fʥ  ʥ]o A ʥʥ ^ʥ& ʥʥbu‚ ʥr  ʥ&ʥ  $&ʥʥ



Za@‘



Ǘʥ ĕʥʥ ʥ ʥ ʥʥʥʥ"ʥ'ʥ+ʥʥ ʥʥ+Eʥ0ʥ ʥ+ ʥʥʥ ʥʥʥ ʥ ʥ ʥ #ʥAʥ ʥ?o[o?o ʥʥ]e ʥʥyʥ ʥʥʥʥ ʥ   ʥʥ ʥ Dʥ

"'+ "+' '"+ʥʥ +'" ȕʥ  ʥʥ ] "'+ʥ ]  "+'ʥ ʥ ]#" I+'" ʥA„ ʥ] - ] +| -O



-O

-

*;

Zʥ :`6 iʥvʥ ʥ$&ʥʥ  ʥʥ ]I :¯Qʥv5 ž  ʥbʥI ¯Qʥ ʥʥ ʥʥʥ ʥʥ< ʥʥ` A„ ʥʥB ʥ$ʥ& ʥ  ./(R / žʥ



Qmwf`zy{d‚N`~gds`~ma`q‚Qyamd~‚ bswpe{z}mŒ_eqmvesjetŒ`t…v{sekŒ b_`Œ 8o bmwsz}ŒbmjszwŒbzt‚szwŒ

#

HDʥF Uʥʥ$ ʥp   ʥ ʥʥʥ ʥ'ʥnʥ ʥ ʥ ʥ  ʥ ʥʥʥ  "ʥnʥ ʥ ʥ3

UʥZoʥ ʥʥʥʥ$ ʥp   ʥʥ ɺ ʥʥZo  ʥ ʥʥ'ʥnʥ  ʥʥ  ʥʥZo  ʥ3ʥʥ "ʥn2ʥʥ Zo ʥʥ  ʥʥʥʥ F ;2ʥʥ ʥʥʥʥʥ$ ʥ ʥ ʥ  ʥʥʥ2ʥ ʥʥ ʥʥeʥ &#ʥ 

MDʥH ”ʥ & 6: Æ ' /ʥ( !/  ?/ʥ( !/ʥ :ʥ! ± ā ' /ʥ‚ ?/ !/ʥ ' /ʥ{ ?/ :ʥ! ±ʥ( "!/ʥ‚ / :ʥ!ƅʥ 4/:€‚‚4/4‚ 4/9‚ Zʥʥ ʥ$ʥ& ʥ6ʥʥ / ( /ʥ:ʥʥ( "+ʥ:ʥ( ?"ʥ 

#ʥ

HDʥHʥ @ʥ ɛ ʥʥʥ< ʥʥg0 R#ʥ @ʥʥ< ʥɕ ʥ ʥʥʥ  ʥ;ʥʥgŒGŒRʥ @ʥ  ʥ$ ʥʥʥeʥʥʥʥ ʥ ʥʥ $ʥʥ  ʥg+Œ Rʥʥ ʥʥhʥ\ Rʥ@ ʥ

 ʥg‹ŒR#ʥHʥʥ  ʥʥʥ  ʥ$ ʥʥ

#

.;

ʥʥ ʥʥ< ʥ$ ;ʥʥ\‚ + ʥ

ʥ O

!#

ʥ !#

,

MDʥ%Y Uʥ!ʥ?oʥʥ›!›ʥ0ʥP!ʥ  ʥʥ ʥ•ʥ›!›ʥʥ!ʥʥ6ʥ ʥ ʥ6ʥ:ʥ‚ @ʥ6ʥ ʥ  ʥ $&ʥʥŴÏ(6ʥ !ʥ0ʥ'ʥ&ʥ!ʥWʥ'ʥʥʥ ʥó ʥʥ!Œʥʥ @ʥʥʥ &ʥvM ʥF<;ʥ›!›ʥʥ!ʥʥ6ʥWʥ ʥ&ʥʥ<ȿʥ !ʥʥ6ʥ0ʥ ʥ Ɯ ʥ Uʥ6ʥGŒʥʥŴÏʥP 6ʥ,ʥ!ʥ0ʥ'ʥ&ʥ!ʥ 2 o kʥ!ʥWʥ'ʥʥ›!›ʥʥ!ʥ,ʥ6ʥWʥ ʥʥ ʥ6ʥ:ʥ( ʥʥ ʥŽžGŒʥZʥʥʥ &ʥ6ʥ’ʥʥ  ʥʥ ʥ •ʥʥ< ʥ›ȸÏʥP 6ʥ,ʥ!ʥ0ʥ'ʥ ʥ !( 6 0ʥ'ʥ % ʥ k&ʥ< ʥ% ʥ  ʥ% ʥ&ʥ!ʥ:ʥ  ʥ6ʥ:( , L@ʥ!ʥ,ʥ6ʥ0ʥ ʥ ʥ n‘





M…Ŏʥ%F

ʥ ʥ Ɨʥ ʥ ʥ 9ʥŔʥƳ ʥ Ɩʥ ʥ ʥ ʥŔʥ¢ʥ å$ʥåʥʥ ʥʥ ,L ʥʥ9ʥ ʥ å$ʥåʥʥ ʥÄŵʥ ʥ ,Lʥ(¢ʥ ʥ ȶÄɤɥʥ :ʥƲ ʥ ʥ ʥ



ʥ ʥʥ’ʥ9ʥ’ʥ +†ʥ­ʥ Uʥʥ :(ǒʥʥ%  ʥ 3:ʥc… 3ʥʥ  3ʥ:ʥj.‚ ^‚ ʥhʥ ʥ  ʥ  ʥ ęʥʥʥ$ʥʥʥ’ ʥʥ3’ ʥ ʥ +öʥʥ ʥɴ ʥ@ʥʥʥ &ʥ ʥ : ǓʥF<;ʥʥ4ʥ0ʥ +†ʥĂ% ʥ9ʥʥ & ʥ ĭʥ ʥ4ʥ:ʥ(%ʥ ʥʥ9ʥ:ʥ(ʥ ʥʥ9ʥijʥʥ ʥʥ3ʥ (¢,L ʥ¢ʥ ʥ: "ʥ ʥ ʥ ʥ ʥ "ʥ m‘ 

3O

-

"ʥ

K

L



MDʥjʥ k…ʥ  ʥʥ ʥ 9Tʥʥ< ʥʥ

 ʥʥ<   ʥ=9TXʥʥʥ ʥ

 ʥʥšT ʥ0ʥ  ʥʥšTXʥ@ʥ $ʥ ʥ=Tʥʥ$  ʥʥ X#ʥ 4Tʥ Ĺ: =ʥ ʥ ʥ @ƴ: ¢ʥkÄŵ 43ʥ 3ʥ +ʥ

Ũ ūʥŜTêʥ êTʥ ʥ ʥ

Ũ ūʥŜ9êʥ ê9ʥ

 ʥʥ™T4ʥ0ʥi‚w , ʥ_ʛʥ +ʥ -O

 ʥ

1;

=



'

HŎʥ%F Aʥũʥ ʥ%5¡ Eʥ%5ʥ( Eʥ5E˜ʥ%5( Eȼ ʥ%5ʥP ʥ5ʥ %5ʥ( Ⱦʥ˜ʥ5 /%5( E# @ʥʥ&ʥʥ ʥʥʥ  # 5( ʥ # ʥ # 5ʥ : : *ž5ʥ Œ5ʥP üʥ#ʥ *ž5ʥ5ʥ iŒ ( %ʥ5ʥ( üʥ iŒ ( üʥ ʥ *ž5ʥ5ʥ ʥ   ʥ ƶĺ ʥ : * " #: 5Eʥ *ž %5ʥ( Eʥ 5Eʥ

ʥ ʥ  :  ŸʥĺP kkl ǘ ʥʥʥʥ$Ÿʥɩ2ʥʥʥ ʥʥ : Eʥ Eʥ ˆc\‘

ˆd]‘

ˆe^‘

+

M

4M

ˆfŽ‘



ˆf\‘





HDʥYʥ @ɖʥ  :ʥ'ʥ  ;ʥ $Ěʥʥ  ʥ ʥğʥ  ʥ





ʥʥʥ&$ʥkʥ ʥʥ ʥʥʥʥ‡ ‡ ‡ Œʥ+ʥ "2ʥʥ ʥ+ʥ ʥʥʥ ʥʥʥʥʥʥ&$ʥ ʥʥ ʥʥʥ< ʥʥ +ʥ ʥʥ ʥ< ʥʥʥʥʥʥʥʥʥ&$ʥ ʥ ʥ2ʥ ;2ʥ%+ʥ'ʥ ʥ ʥƝ+ʥ"2ʥ2ʥ ʥ%+ʥ2ʥ2ʥ ʥ%+ʥ2ʥ2ʥ ʥ%'ʥ"2ʥ2ʥ  %'ʥ"2ʥ2ʥ ʥ%'ʥ2ʥ2ʥ ʥ  ʥ"2ʥ2ʥ2ʥ#ʥZʥĚʥʥ ʥʥ<Ű ʥ   ;  '(+ʥ:ʥ # ʥ +O



?

HDʥ €ʥʥ ʥʥʥ   ʥ ʥʥ!Ŷ2ʥŽ|ž!Ŷ2ʥʥʥ 

ʥʥ!ɰʥ@ʥʥ &ʥ!ʥŽʥ žʥ!ʥ +ʥ  ʥʥ’ʥ!ʥ0Ž./ž!ʥ’ʥ?ʥAʥʥ < ʥ!ʥ0žŽž0ž!2ʥʥ ʥ!ˆ‰ʥ!ʥŽʥ žʥ!ʥ0ž'!ʥ  ʥʥʥʥ• ʥeʥ< ʥ ʥ +ńŅʥ!ńŅʥ"ʥkʥ! ʥ ʥʥ  ʥ!ʥ’ʥ?ʥʥ   'ʥ  ʥ!ʥ "ʥ 'ʥ +ʥ ?#ʥZʥʥ ʥʥ   ʥ ʥʥ ʥ  $&;ʥ% "Žʥ'"ĥ2ʥ'+† ʥ% Šʥ' ʓʥ+Ğʥ% +Žʥ"+ĥ2ʥ+ĥʥ  ʥ % ?†ʥ"ʥ†ʥ+Šʥ n‘

+O

;

 ʥ



MDʥa

ŕʥ

€ʥÉʥʥʥ ʥʥʥʥʥʥ k = W =4  ʥ©¾=4 : ? ýʥ ©¾4=: ý#ʥ@ʥ©¾9=: Ƙ#ʥHʥ ©=T9ʥ:ʥ?ýʥʥ$ʥ ʥ=T1ʥ9T#ʥ€ʥ!ʥW =T W 9ʥ T#ʥkʥ94 ȖʥT=ʥ  ʥ

 ʥʥ<   ʥ 94=ʥB‚  ʥʥȲT=ʥʥ  ʥʥȳ9=ʥʥ  ʥʥȴ4= ʥ &ʥ (‚ /ʥ (‚ { 94 { !: (‚Ļ% 994!ʥ ( ‚ Ļ{ % 9(!! , (‚¢!ʥ¢ 1ʥ ʥ ʥ ʥ ʥ ēʥʥʥ  ʥ ʥt = ʥʥ$ɷ  ʥʥ 4ʥ  ʥ

 ʥ <    94=ʥ ?‚  ƙʥ™94ʥ,  ʥʥ™4= ʥ &ʥ ʥ:ʥ[‚ 9ʥ 94ʥZ ʥ ‚ 4ʥ{ʥt = :ʥ ʥ[‚{ʥ 9ʥ 94ʥƨʥÉ/ % ʥ ʥ ʥ ʥ mʥ< ʥ% ʥ  ʥ 9 /ʥ,ʥ94 /ʥ:ʥ 4 /ʥ:ʥÉ/ʥ$;ʥ ʥ % 994 /ʥçÉ/ʥ, { 9 {ȟ4: É/ʥ% ( É/: ?" #ʥ @ʥ 9ʥʥ94ʥB‚ Ē #ʥZʥ•ʥ< ʥ (‚2ʥʥ ʥ!ʥA‚ Ē # JO

((

IO

JO

MDʥ ? ʥ €ʥ_‚?‚ +ʥ@ʥ % + ʼnʥʥ 6‚ w +ʥw ?ʥ‚(kB‚ _5‚‚6_ _‚,ʥ( ‚,ʥ(k

A _5 6_0  6_ (k‚ ? _  ‚  ĉʥ?‚ ? ĉʥ?‚l0‚{ʥ ĉʜ Zʥʥʥʥ$&ʥ &ʥʥ (1‚ʥ %"ʥ,ʥ( ‚?‚? ʥ

66‚

%(

MDʥ *

%L

%L

%LLwǯ‚ 7 ; %LL

%Lʥ %L

%L %L

%L

 ;

ʥ‚}ʥ ʥª /.ʥ}ʥ ʥ}w ‚ʥ /ªw}ʥ‚ʥ  ‚  ʥ ªʥ ʥ ./Rʥʥ :L wʏʥR Rʥ %L ʥ %L ʥ %L w}‚ʥþ.Rªÿʥ Ʉ}ʥ }wʥ : ʥw‚ʥ:)L;Lʥw‚ʥR /ªʥʥw‚ʥª /ʥ: :ʥǪw‚ʥ:;<+L:ʥ*L

%*

MDʥ (##0 ē&ʥ ʥ@L @L @L @LL% %L @LL% @L@ %L %LL(L (LL*L *LL L(0.L (0.L (L%LL* (LL, L!*Lʥʥʥ(0/ (0. Œ‘





Œ‘







JJJO



+O

(0/ %L kʥ(##1L (0/L / L(0/ %L %L (##0L ʥ(##1LUʥʥ ʥʥ   ʥʥ(##0L *O



%,

‘





MDʥ €ʥX H (-LʥGE+Lʥ(-L%#87&+KL@ ! °ʥ:ʥ(DL%L#LL(-L G*L (-)L %##-L9&+L(DLʥG>+LL(DL G>+L ʥ&ʥ!+L3-#L%-H Lʥ!(! /ʥʥ !ʥʥ%#L L #L@ʥ< ʥ ʥ; ʥ ʥʥ!ʥ -L -O

*O

%-

MDʥ %#(, :4L%Lʥ$%‚ &ʥ(:L %/ʥ:ʥ(##1 Lɘʥ$e ʥʥ :/ :4L-#( L

(L :+L -#*: DAAIBL:ʥ::/ʥ : -#(:)L : DAAIBL :ʥ:: /ʥ :-#(:)L : -#((FL #Lʥ# L ʥ('$L:ʥ%#(,L



H

*,L

"#

HDʥ

kʥ=³ʥʥʥ ʥʥʥPHo ʥ &ʥ Tʥ:ʥCoʥk ;7ʥ ³3ʥ 3=ʥ Do Co Tʥ‡ 3 3=ʥ‡ $); =Xʥʥʥ ʥʥʥ.$; : ĹʥZʥ :ʥ  4Xʥ =4ʥ T3ʥ =4ʥ X ʥ

'

HDʥ + Aʥʥ ʥʥ ʥ3ʥ:ʥƆ7ʥ ʥ ʥ:  %ʥ ,ʥ3:ʥ  ʥ ,ʥ  3ʥ 7ʥ  ʥ

(    3ʥ

ʥ¬  ʥ ( ʥ  ʥ3ʥ:ʥ ʥ ʥ ʥ3ʥ#ʥF<;7ʥ  ( 3   3ʥ:ʥ    3ʥ: # % (%3( : ʥ (     3ʥ    3ʥ Zʥ % ƇʥƷ % †ʥ( % †ʥ( Ĝ% †ʥƸ % Ğʥ( % †ʥ( ʥ :ʥ% ƈʥ¡ % öʥ(% †ʥ( % Ğʥ( % öʥ( % öʥ( ʥ : +ʥ

+

HDʥ " mʥ ʥ.Rʥ, . ,ʥRʥW %.  ʥ %Rʥ,ʥ P ʥ@ʥ.Rʥ. , ʥRʥʥ ʥ$ʥ 'ʥ

 ʥ;ʥʥ%.ʥ %Rʥ( ‚ ʥ @o ʥ% ʥ'# € 0ʥÞ 7ʥ 7ʥ7ʥʥ2ʥ--2ʥ Ÿ7ʥ  ʥ ů ʥ:ʥJHLJo D ʥHL@oÅʥ% ʥ'õʥÅ1ʥ7ʥ 7 7ʥ###ʥ7ʥ"#ʥLoʥ ;ʥʥ&Ãʥ ʥʥ ;ʥ HLJo ɜ ʥ  ʥeo Jo ǫ7ʥʥˆ‰Åʥ ʥ0j./ž"7 co@o ʥ% ʥ'ʥʥ  ʥ;ʥʥÅʥ:ʥjžKo ʐ 7ʥ"õ7ʥʥ#: Wʥ ʥ  ʥjžWʥ7ʥʥ#: Wʥ6‚  ʥjž0ʥ @ʥʥ ʥŬʥ ʥ F ʥ#.ʥ ʥ7ʥRʥ ʥ ʥJo ůʥʥÅʥP J% L"õʥʥ @ʥ.7ʥRʥJo ŮʥʥÅʥJo JA LõʥHʥŲ ɱʥ¿ʥ:ʥŲ ɻʥ¿ʥI  7ʥʥʥʥʥʥÞ.7ʥRŸ ʥ :ʥ +

&%1M

F ʥ Dʥ. , ʥ ʥ  ʥRʥ,ʥ ʥ ʥ  ʥʥ /ʥ  ʥ Ëʥ $&;7ʥʥʥʥ  ʥʥ @ʥ.ʥ  ʥRʥ ʥ  ʥʥ ¯ʥ  ʥ °ʥ$&;7ʥʥʥʥʥ ʥʥ kʥ¿ ¯æʥ:ʥ ʥ  ʥ¿ °æʥ  7ʥʥʥʥʥʥÞ.7ʥRŸʥʥ ʥ ʥ1ʥ ʥ F ʥDʥ. ʥ ʥ  ʥRʥ,ʥ ʥ ʥ  ʥʥ °ʥ  ʥ Ǻʥ$&;7ʥʥʥʥʥ  ʥʥ @ʥ.ʥ  ʥR ʥ  ʥʥ /ʥ  ʥ Ëʥ $&;7ʥʥʥʥʥ ʥ mʥ ʥ¿ /æʥ:ʥ ʥ  ʥ¿ Ë æ  ʥ@ʥʥʥʥʥʥÞ.7ʥRŸʥʥ ʥ

ʥ1ʥ #ʥ Zʥʥ ʥʥ + ʥ,ʥ ʥ ʥ " ʥ Œ‘

-O

Œ‘

-O

Œ‘

*O

ʥ

4E' _Ÿ©¶ƒ§FÏ=EB6B xŸ€ƒÏ¸5Ï4?¸Ï Ï4WÏ Ï2Ï¸Ï ÏY‚WÏ4?'Ïz‹ƒ§ƒ†£§ƒ ¸>Ï Ï X¸ ƨÏ=¸5Ï Ï A Ï 4Ï5Ï CA Ï(Ï 4Ï !;=ϸ ÈÏ Wϸ ÇÏ W4?>Ï=¹4?5Ï 6Y=EB6B'Ï 62' _Ÿ©¶ƒ§FÏ6= W

Eo

xŸ€ƒÏ ~ŸÏ ©ƒ€«©Ï ¶ƒÏ¶ƒÏ ‹~´ƒÏ ZlƒŸ€ƒÏ Ï4ÏFÏ='Ïi±§¬‹ƒ§›£§ƒ4ÏHÏÏ©Ÿ€ƒ © ƒƒÏ « ‹~«Ï ©ƒ€«©Ï =' Ï Ï oƒ«Ï Zϸ½Ï~ŸÏ z‹ƒŸÏ¶ƒÏA‹'Ï~´ƒÏ ¸Ï ¼Ï[«Ï4;'?Ï ƒ¦±~«£Ÿ©Ï ƒ˜©Ï¸Ï B'¼?'ÏÏ~Ÿ¼ZÏ m«Ï†£˜˜£¶©Ï ‹~«Ï ~ŸÏ==¼Ï¸ÏZÏ?¼'6='ÏÏx£˜´ŸŠÏ«‹ƒÏ 64' b©¶ƒ§H ÏCE==2Ï z‹ƒÏŸ±›ƒ§Ï £†Ï©±€‹Ï£§ƒ§ƒÏ«§¥˜ƒ©Ï¸ϼÏ ¶«‹Ï¸ ¼Ï©Ï ÉW62C2Ï z‹ƒÏŸ±›ƒ§Ï£†Ï©±€‹Ï£?§ƒ§ƒÏ«§¥˜ƒ©Ï¸ϼÏ ¶«‹Ï¸Ï ¼Ï©Ï 6Ï ÏÊ ÏWÏCB;A2' lƒŸ€ƒÏ«‹ƒÏ~Ÿ©¶ƒ§Ï©Ï62C2Ï ÏCB;A2Ï CE==2)Ï #'; ');o! #; !';

-!#$; %);o'#; %'; ;

!'; ;'$;;!#; ;#$; %); -')$ ;

%'; ;%$;

;

$%;

zŒ

zŒ

Y

,O

;AÏ

,O



66* _Ÿ©¶ƒ§FÏ4EE24 4Ï WÏ 'Ïz‹ƒ§ƒ†£§ƒ i•©«ÏŸ£«ƒÏ«‹~« 

M 4Ï 4Ï W;  WÏ4EEÏ6Ï622ÏW4EE22' 9ž 3M 4Ï 4 4EE22ÏWÏ4EE24' ,,

,,

mF [m[m

[m[m 

Y

mG

6;' _Ÿ©¶ƒ§FÏAÏ ` ?5 Ï425Ï4;5ÏWÏ 44 WÏ {ƒÏ‹~´ƒÏ€£©Ï` WÏ`c5Ï6 ``Ïce5ce5Ï `eÏ 6?42Ï 6?Ï oƒ«Ï` vÏ ¸Ï„›Ï~ŸÏ`ÏwÏ ¼Ï„›'ÏxŸ€ƒÏ~§ƒ~Ï£†ÏpvwÏWÏ"6ς¸¼Ï©ŸÏ`Ï~ŸÏ~§ƒ~Ï£† pceÏWÏ!6ς`Ï c`Ï ÏeÏ©ŸÏ`ÏY#Ï6ςÏ?42ÏÏ©ŸÏ`Ï‚¶ƒÏ£«~ŸÏ?2ÏYÏ#=Ï‚«‹~«Ï©ϸ¼ÏXÏ6?Ï ' 6Ï lƒŸ€ƒÏ vw5ÏWϸ5Ï Ï¼5 6¸¼€£©`ÏY¸Ï5Ï Ï 6? A6?Ï Ï44ÏNOPÏ6ϸ1 6?Ï Ï44ÏWÏ6?Ï Ï44ÏX;ÏA' 7Ï  Wϸ Ï=¸Ï =¸Ï5Ï 5Ï d£Ÿ©ƒ¦±ƒŸ«˜½ÏvwÏ:A϶«‹Ï«‹ƒÏƒ¦±~˜Ž«½Ï~««~ŸƒÏ¶‹ƒŸÏ¸ÏXϼÏXÏ *O

+O

co



MO

Ï ?Ï 6=' _Ÿ©¶ƒ§I xŸ€ƒ¸Ï ϼÏ$MEÏ¾Ï 6=¾¸Ï ώ ž 6=ϾEÏ¾Ï ¾5ÏE¾ 6='Ïr£¶ÏYŸ£«ƒÏ «‹~«Ï¸Ï~Ÿ¼Ï~§ƒÏ§££«©Ï£†Ï«‹ƒÏ¦±~§~«€Ïƒ¦±~«£ŸÏ­5Ï Ï¾E­Ï Ͼ5E¾Ï Ï6=Ï 2+ _©Ï¸Ï~Ÿ¼Ï~§ƒÏ§ƒ~˜϶ƒÏ‹~´ƒÏ¾ÏE5Ï=¾5ÏE¾ Ï6=NOPÏ2϶‹€‹Ï©›¥˜‡ƒÏ«£Ï ¾5Ï ¼ 6 Ï ¾ Ï ?, Ï  A¾Ï Ï ? QR Ï 2 'Ïx£˜´ŸŠÏ « ‹ƒÏ  Ÿƒ¦±~˜«½Ï ½ ƒ˜©Ï4Ï S Ï T ¾ ST Ï ?' Ï {‹ƒŸÏ ¸ Ï lƒŸ€ƒÏ«‹ƒÏ˜~§Šƒ©«Ï¥£©©˜ƒÏ´~˜±ƒÏ£†Ï¾Ï©Ï?-Ï co

*O

*O

+O

+O

;BÏ

+O

*O

6@' i§©®Ï _Ÿ©¶…§H¥±®ÏÏ;®‹…Ï©ºÏ4Ï @BÏ ©ŸÏ£Ÿ…Ï©…¦±…Ÿ€…'Ïz‹…ŸÏ®‹…§…Ï~§…Ï!MŠ~¥©Ï…†£§…Ï®‹…ψ©®Ï4Ï …®¶……ŸÏ ® ¶£Ï ~ –~€…Ÿ®Ï 4  Ï ©~ŸÏ ~ ‰…§Ï ® ‹…Ï ˜ ~©®Ï 4 ' Ï i £§Ï … ~€‹Ï © ±€‹Ï Š ~¥ Ï ¶…Ï € ~ŸÏ ¥ ±®Ï ~ Ï ©ŸŠ˜…Ï2Ï£§Ï …£±˜…Ï t©Ï® ‹~®Ït©©Ï22'Ï®‹…ŸÏ Ï ®‹…§…Ï~§…Ï…º~€®˜½ÏA 6Ï©”Š˜…Ït©϶‹…§…Ï m†Ï®‹…§…Ï ~ §…Ï º~€®˜½Ï Ï  £±˜…Ï 2Ï4Ï6Ï !8 !M ;'Ïz‹…§…†£§…Ï ®‹…ÏŸ±›…§Ï£†Ï©±€‹Ï”~§½Ï ©…¦±…Ÿ€…©Ï ¶®‹Ï…º~€®˜½ÏÏ£±˜…Ï <Ï   Ï !8 !M  Ï t©Ï©Ï .Ï AÏ6¿Ï.Ï2M'Ïl…Ÿ€…Ï®‹…Ï~Ÿ©¶…§©Ï©Ï8ž 'Ï AÏ6¿Ï2 ;@B'Ï 6A'Ï _Ÿ©¶…§JÏE@Ï €£©˜ttu €£©422ÁÏ €£©5Ï@2©Ÿ5Ï@2ÁÏ 4=©Ÿ6@3π£©6@Áπ£©@2ÁÏ 46©Ÿ@2Áπ£©@|Ï €£©@2ÁÏ©Ÿ@2Á5Ï €£©@2ÁÏ Ï©Ÿ@2ÃÏ 4Ï Ï®~Ÿ@2ÁÏ €£©@2ÁÏ  ©Ÿ@2ÁÏ 4®~Ÿ@2ÁÏ 0ʥ ®~Ÿ=@ÁÏ Ï®~Ÿ@2ÁÏ ®~ŸE}'Ï 4®~Ÿ=@ÁÏ®~Ÿ@2ÁÏ l…Ÿ€…»Ï E@'Ï 6B' _Ÿ©¶…§HÏ66;Ï ˜£Š‘ MKMA Ϙ£Š ËÏ0M /Ï » ˜£Š>M; 1M UÏAÏ Ï˜£Š<ÏEϘ£Š<Ï» » ˜£ŠÏ<ÏEÏ0M ˜£ŠÏ»5Ï <Ï ˜£ŠÏ;Ï <Ï UA ˜£Š<E˜£Š<»'Ï ˜£Š<Ï»Ï ˜£Š<ÏEÏ 4ÏUÏD²϶‹€‹Ï©Ï…¦±´~˜…Ÿ®Ï®£ o…®Ï²Ï ˜£Š<Ï»0Ïz‹…ŸÏ®‹…ύŸ…¦±~˜®½Ï…€£›…©Ï6²Ï ²6Ï M ²5ÏD²Ï Ï4@Ï 2Ï'Ï £Ï´œŠÏ 'υ¦±~Ïn®½ÏŠ’´…©Ï²ÏVÏ6Ï£§Ï;ÏVϲÏVÏ@Ï‹Ï . Ï . Ï ® Ï…Ï œ Ï ®Ï~®Ï n © ‹Ï ² 6Ï ˜£Š<Ï»ÏUÏ6Ï£§Ï;ÏUϘ£Š<Ï»ÏUÏ@'Ïm®Ï†£˜˜£¶©Ï®‹~®Ï2ÏVÏ»ÏVÏEÏ£§Ï6BÏVÏ»ÏVÏ6=;'Ïl…Ÿ€…Ï®‹…§…Ï ~§…Ï66;Ï©±€‹ÏŸ®…Š…§©'Ï #:

+O

O

f

cET 

'O

+O

C

B

+O

: :

:

;DÏ

(O

6C' aŸ©¶ƒ§F Ï/2L ®‹~® i§©®Ï£©ƒ§µƒÏ %L

W

z‹ƒ§ƒ†£§ƒÏ

(2 L

C‚

H( H L %L %L %L  -‚ 5

M

2

aŸ©¶ƒ§KÏ2. O O O O O O O O O O O O O O O O

oƒ®ÏyσϮ‹ƒÏ~§ƒ~Ï£†Ï§ƒ€®~ŸŠ˜ƒÏ`%Lcfg'Ïz‹ƒŸÏ¶ƒÏ‹~´ƒÏ%L %L %L ~§ƒ~Ï£†ÏšfgjÏWÏ!(L¹Ï~§ƒ~Ï£†Ï™fghÏWÏ!(L¹!yÏ WÏ"y'Ï ʥ +ʥ %L %L %L %L Ï sƒº®¶ƒ‹~´ƒÏ~§ƒ~£†Ïqfj\!¹~§ƒ~£†Ïqfh]¹!yWy' (L (L (L ʥ s£¶Ï%(L[~§ƒ~Ï£†ÏqgjÏ^~§ƒ~Ï£†ÏqfgÏ  ~§ƒ~Ï£†Ïqfj~§ƒ~Ï£†ÏšfgjÏ WÏ}‚ [Ï}‚ ʥ +ʥ +ʥ (L V}‚W}‚ lƒŸ€ƒÏ®‹ƒÏ~§ƒ~Ï£† §ƒ€®~ŸŠ˜ƒ`cfgÏ[Ï2.L€›8ÅÏ *2L

;2, _¡©¶ƒ§F Ï# kŽ§©«Ï ¡ £«ƒÏ « ‹~«ÏŽ † Ï ƒ´ƒ§½Ï  ŽŠŽ«ÏŽ¡Ï « ‹ƒÏ A #ŽŠŽ«Ï ¡ ±›ƒ§Ï ~ ¥¥ƒ~§©Ï ~ «Ï˜ƒ~©«Ï ° Ž€ƒÏ « ‹ƒ¡Ï « ‹ƒ§ƒ €~¢¡£«Ï ƒÏ†£±§Ï Ž©«Ž¡€«Ï ŽŠŽ«©Ï Ž¡Ï«·£Ï ¯‹ƒÏ¡Ž©«Ž¡€«Ï ±›ƒ§'ŽŠŽ«©Ï ÏĐʥ£«‹ƒ§Ï ¶£§©Ï «‹ƒÏ¡±›ƒ§Ï €~¡Ï£¡˜½Ï ƒÏ†£§›ƒÏ  ½Ï±©Ž¡ŠÏ £ ¡ƒÏ ŽŠŽ«Ï £ §Ï « Œ§ƒƒÏ  Ž©«”€«Ï ŽŠŽ«©Ï §ƒ©¥ƒ€«Ž´ƒ˜½, Ï z‹ƒ§ƒ†£§ƒÏ ¶ ƒÏ € £¡©Žƒ§Ï « ‹§ƒƒÏ € ~©ƒ©, Ï d~©ƒÏ«‹ƒÏ #z‹ƒÏA$ŽŠŽ«Ï¡±›ƒ§ÏŽ©Ï†£§›ƒÏ½Ï£¡˜½Ï£¡ƒÏŽŠŽ«'Ï z‹ƒ¡Ï ¡ ±›ƒ§Ï Ä £ †Ï©±€‹Ï A $ŽŠŽ«Ï ¡ ±›ƒ§©ÏŽ©Ï€˜ƒ~§˜½Ï E ' Ï d~©ƒÏ6«‹ƒÏ FÏz‹ƒÏ A$ŽŠŽ«Ï ¡±›ƒ§Ï Ž©Ï†£§›ƒÏ ½Ï«¶£ÏŽ©«¡€«ÏŽŠŽ«©'ŠŽ´ƒ¡Ï Ï ŽŠŽ«©ÏÏ~¡Ï— kŽ§©«Ï ¡ ±›ƒ§Ï £ †Ï©±€‹Ï A $ŽŠŽ«Ï ¡ ±›ƒ§©Ï†£§›ƒÏ½Ï«¶£Ï ¶‹ƒ§ƒÏ#ŐőʥÏV—STÏEώ©Ï Ì M L ͪ¤Ï rƒº«Ï « ‹ƒÏ ¡ ±›ƒ§Ï £ †Ï©±€‹Ï A $ŽŠŽ«Ï ¡ ±›ƒ§©Ï † £§žƒÏ  ½Ï 2 Ï~¡Ï ~ Ï Š Ž´ƒ¡Ï  ŽŠŽ«Ï ¶ ‹ƒ§ƒÏ # ST ÏEώ©Ï =>o

z‹ƒ§ƒ†£§ƒÏ«‹ƒÏ«£«~˜Ï¡±›ƒ§Ï£†Ï©±€‹ÏA$ŽŠŽ«Ï¡±›ƒ§©Ï†£§›ƒÏ½Ï«¶£ÏŽ©«Ž¡€«ÏŽŠŽ«©Ï ι?2 E¹Ï6?ÏGM926?' d~©ƒÏ«‹ƒÏ¡±›ƒ§Ï Dʥz‹ƒÏA$ŽŠŽ«Ï¡±›ƒ§ÏŽ©Ï†£§›ƒÏ½Ï«‹§ƒƒÏŽ©«“¡€«ÏŽŠŽ«©1Ï k§©«Ï £ †Ï©±€‹Ï A %ŽŠŽ«Ï ¡ ±›ƒ§©Ï†£§›ƒ‚Ï  ½Ï«Œ§ƒƒÏ Š Ž´ƒ¡Ï  ŽŠŽ«©Ï  —Ï~¡Ï k ¶‹ƒ§ƒÏ ŐőʥÏV—ÏVÏk0žEώ©Ï QU

G ),H. "<$ŽŠŽ«Ï M ¡±›ƒ§©Ï†£§ƒÏ½Ï2Ï~¡Ï«·£ÏŠŽ´ƒ¡ÏŽŠŽ«©ÏÏ~¡ rƒº«Ï « ‹ƒÏ ¡ ³ƒ§Ï £ †Ï©±€‹Ï A —϶‹ƒ§ƒÏ STϐÏV—STÏEώ©Ï A¤ ' G (+H. z‹ƒ§ƒ†£§ƒÏ«‹ƒÏ«£«~˜Ï¡±›ƒ§Ï£†Ï©±€‹ÏA&ŽŠŽ«Ï¡±›ƒ§©Ï†£§›ƒÏ½Ï«Œ§ƒƒÏŽ©«Ž¡€«ÏŽŠŽ«©Ï #

QU

#; ¹ÏE2Ï,ʥÏÎÀÏA2ÏGMEB62 lƒ¡€ƒÏ«‹ƒÏ~¡©¶ƒ§ÏŽ©ÏEÏ,ʥ626?ÏÏEB62Ï  # +O

 #

 #

Muʥ  ŀʥ ʥ6ˆ‰ʥ-2ʥʥ &ʥ6ʥàʥ-( '!ʥ#ʥLoʥ  Vœʥ '!ʥ ʥ /ʥ / / àʥ-!( '! 'ʥ!(!/:ʥ '  ¬ !(  {ʥ ʥ ʥ ʥ ʥ ʥ

    9

'O

;

@2ʥʥ !(

)0; ʥ2ʥʥ!ʥ ʥʥ!ˆ‰ʥ 2ʥ lo

    9M

 /ʥ. ,‚  / h  #ʥ 'ʥ +‚ àǵʥ U !ʥ0ʥ "2ʥʥ6àʥ-( ' "ʥà 'ʥ@ 6œʥ 2ʥ  œ  ʥ V ;2ʥʥ!ʥ0ʥ '2ʥʥ6ˆ‰ʥ 2ʥ  ʥˆ‰ʥ #ʥ U !ʥ0ʥ +2ʥ 6ˆ‰ʥ 2ʥ  ˆ‰ʥ #ʥ U !ʥ0ʥ -2ʥ 6ˆ‰ʥ 2ʥ  ˆ‰ʥ +#ʥ Uʥ2ʥʥ ^ʥ& ʥʥʥʥ 2ʥʥʥ   ʥ ʥ!ʥ0ʥ +ʥ  6ʥ0ʥ

#ʥ  #

HDʥ "  mʥ  Æ!Ɗʥ!Ƌʥ!ƌ ƀʥ:ʥ

F%IAƁÆ!ƍʥ!ĩƎ#

ʥ ʥ ʥʥ#:M ʥʥ#:@ 2ʥʥɔʥeʥʥ!/Ȅʥ„ʥʥ^$ ʥʥ · ! ȓʥ ʥʥ!/ʥʥ!±Ţʥʥ#ʥmʥ ʥ ʥʥ#:ʥ&2ʥʥʥeʥʥ!/ ȅʥʥ ^$ ʥʥ!ǷȔʥ ʥʥ!/ʥʥ! ±ʥŢʥʥ ʥǥ#ʥ@ʥʥ;ʥ ʥʥ ɇʥʥ Ɏ‹ʥʥ!Ƃʥ„ʥʥ^$ ʥʥ ʥ!Əʥʥ!ĩĪʥ ɉʥ#:0ʥ#ʥ Lo

&MID -M

Áʥ%:0ʥ2ʥʥ &ʥ ƃÆʥ!īʥ!Ĭ Īʥ:

%IB

%"CƄÆ!īʥ!ĬƐ

" /ʥ ±ʥ ǔ 0ʥ ʥ7M [ʥ #ʥ [ʥ[ʥ

"  0ʥ 5!/ ɹɼ 4 !/ ə ƥ!/ ȥ ʥ  Y‘

{E

o

ZE

o

 ʥ

Lo

'*MEM˜ʥ ʥZ`ʥʥ Iʥʥ:#@M ʥ:ʥ"Cʥ #

?

 ʥ` Iʥ ʥʥ`e`ʥʥ!±ʥʥʥ^$ ʥʥ r /űʥ ʥʥ!/űʥ dDS ho

M…Iȇʥ  ʥ AIʥ `ʥ5 ʥCʥʥ#rʥʥ`ʥxfʥ˜ʥ` .f.fƦȦʥʥ &ʥxfʥÏʥ.fʥ  ʥxfʥÏʥ.fIJěʥ šGʥxfʥŘʥ.fƧě(.fƑʥ ʥʥxfʥŘʥ5ʥ ʥ/ʥ ʥ(5 /ʥ ʥ`ʥ&ʥxfʥǢC5ʥ ʥ  ʥ# Z`ʥxfʥņC5Ǯʥʥ (5C5ʥ ʥ  ʥʥ cʥxfʥņC¬5#ʥLo ʥ  xfʥǨCCĶ5ʥʥC5ʥ,ʥ ʥ ʥʥxfʥǣ ʥF
+O

X

*O

r

M…IŌʥ 

AcIʥʥ Ic./QQÌʥ  ʥ./QQĊʘʥVG$$ʥ] ʥ./QQÌʥçʥ!¯!/! °!±! ·!Ŋ!ǽ!ÌʥI ]ʥ!Ůƒʥ Iʥ c…`ʥ ʥʥž ʥCʥʥʥʥ"ʥ'ʥ+Ÿʥrʥ iʥ Íʥž.ɢōʥkž:ʥ Cʥr#rʥǦCŸ V`ʥ'Eʥ ʥM C+ʥʥ  G`ʥ ʥ!¯ʥ˜ ʥ ʥ ʥIʥ IʥIʥ ʥC+ʥ GIʥʥ ʥG`ʥ ʥ]ʥeIʥ ʥʥ rʥ HʥCʥ "Eʥ@ C+ʥ@ ʥ "Eʥʥ &ʥ!/ʥçʥʥ ʥIʥ Iʥʥ ʥC+ʥGIʥ ʥG`ʥ ʥʥeIʥ ʥʥ ʥ  ʥʥ` ʥ ʥʥCʥIʥ ʥGʥIʥ IʥIʥ  ʥC+ʥGIʥcʥ ʥG`ʥ ʥʥ´‹Iʥ ʥʥ ʥ  ʥʥ` ʥ ʥʥCʥʥIʥ r V I;ʥc`ʥCʥ "Eʥ,ʥʥ Eʥ@ C+ʥ@ Cʥ "Eʥʥʥ Eʥʥʥ ʥʥI ʥ!Ǵʥʥ'rʥa;ʥI$ ʥʥ IGʥ  ʥGʥʥ
'"ââââʥʥ ʥʥeIʥ $$ Iʥ ʥʥeɑʥ ʥʥ./QQ·ʥʥʥGIʥ I II  ʥ„ʥ`I c…ʥI IʥF
Œ‘

Œ‘

Œ‘





+O



‹‘









Œ‘

Œ‘



Cʥ













ʥ #

HDʥ ʥ Áʥ 5::ĆQʥx šʥĆQʥ ʥ: ¡t Vʥ.ʥ::ĆQʥx6 žʥ & W % |ʥ 5: 0 . ,ʥdo ʥʥ ˆ‰ʥdo ; #ʥ Vĝę Ž2ʥʥbʥÍʥ:ž ¬ 5:5žʥ & ʥ( 5:0ʥbʥ,ʥ6o ʥʥ ʥ;
ʥ

Dey\O‚~‡Xž =O‹`XtO‹eSOpž D~SeX‹ž Efz]PƒˆYž>PŒbYuPŒfTPqžAq‘uƒfPVž E>A ž E;;Q  !"' $! ' !% ' '   '

$%"&' ' % ' ' ' ' '  !# # ,o    !# # # # # " # ! # #  # # o 

 #  # # !

 I\}“ Nx Wq[“ Ox X\“uscq}|“sq“ |c[\|“ BCx Wq[“ BEx s]“}xcWqak\“ BCEx x\|u\Y}c„\kˆ“ F]

HNx EOx

?x % NBx sOBx x |bs…“ }bW}“ NxOx uW||\|“}bxs‚ab“}b\“Y\q}xsc[“s]“ BCEx  Bcq[“Wkk“uWcx|“s]“us|c}c„\“cq}\a\x|“r “g }bW}“|W}c|`“ }b\“\w‚W~csq“ ,xt)fx x%xh‘ r!  I“ \}“ Bx X\“ Wq“ r\k\o\q}“ |‚X|\}“ s]“ n%x)x   xx)##5px …c}b“ }b\“ uxsu\x}ˆ“ }bW}“ }b\“[c]‹ ]\x\qY\“ X\}…\\q“ Wqˆ“ }…s“ q‚oX\x|“ cq“ Bx c|“ qs}“ W“ uxco\“ q‚oX\x“ Bcq[“ }b\“ kWxa\|} us||cXk\“ „Wk‚\“ s]“ r“ Bcq[“ W“ |\}“ …c}b“ }bc|“ q‚oX\x“ s]“ \k\o\q}|“ Ms}\,“ %x c|“ qs}“ W uxco\ q‚oX\x ‚ )

I\}“ TxUx# L #x |‚Yb“ }bW}“ Tx xUx x # x\Wk“q‚oX\x|“|‚Yb“ }bW}“ l&l*x xl0

/O

/O

%x Qbs…“ }bW}“ c]“c]“ l&xl* x   xxl0x Wx\“ us|c}c„\ %x }b\q

TlXx xUl&x WxTlw xUl*x WuutxTlq xUl0x Wx JKΠ%x

-

Eq“ Wq“WxYb\xˆ“ Ysou\}c}csq “ }b\x\“Wx\“ ,#x Ysq}\|}Wq}|“ Tb\“}Wxa\}“c|“[c„c[\[“ cq“ }…s Šsq\|“ 9“ bc}“ W}“ Šsq\“ %x c|“ W…Wx[\[“ %#x uscq}|“ …bck\“ W“ bc}“ W}“ Šsq\“ )x c|“ W…Wx[\[“ 1 uscq}|“ Ms“uscq}“c|“W…Wx[\[“ ]sx“ W“oc||“ AWYb“Ysq}\|}Wq}“ |bss}|“ %2x Wxxs…|“ 9}“ }b\ \q[“s]“}b\“ Ysou\}c}csq“ |}W}c|}cY|“ |bs…“ }bW}“ osx\“bWq“ 1#x s]“}b\“Wxxs…|“bc}“Šsq\ ) Tb\“q‚oX\x“s]“Wxxs…|“}bW}“bc}“Šsq\“ %x Wq[“oc||“}b\“}Wxa\}“Wx\“\w‚Wk“ Oxs„\“}bW} }b\x\“Wx\“}…s“Ysq}\|}Wq}|“…c}b“}b\“|Wo\“|Ysx\“

((o

Cey\O‚~‡Xž =O‹`XtO‹eSOpž C~SeX‹ž Efz]PƒˆYž>PŒaYuPŒfTPqžAq‘uƒfPVž E>A ž E;;Q  !"'$! ' !% ' ' #!%$! #'

 I\}“ Gx X\“}b\“od[usdq}“s]“ BDx QdqY\“ Mž A %xOx kd\|“dq“ }b\“|\ao\q}“ DGx I\}“ Ix X\ }b\“dq}\x|\Y}dsq“ s]“ CGx Wq[“ NxOx ;ˆ“ L\q\lW‚|“ Tb\sx\o“Wuuld\[“ }s“}b\“kdq\“ NxOx Wq[ }xdWqak\“BCxGx

GIxx CNxx BOx @x% ICx NB OGx

Tb‚|“



  

CIx CNxx BOx DO x BO %x IG NBx OG OBx OG OxB DOx )DG DOx DOx OGx OGx )OGx ?x ?x)r OGx Tb\x\]sx\ “ Ix d|“}b\“Y\q}xsd[“s]“BCDx 1O

N

O

 V\“bW„\“ g*

2O

^

%x os[“ ,x Tb‚|“ gx?x,]x x%x sx“ ,]x x)x ]sx“|so\“qsqq\aW}d„\“dq}\a\x

]x d “ gx?x,]x x%x 9]€\x“|doukd`dqa “ …\“bW„\“)fx?x,] *x x)]x?x],]x x)x Tb‚|“]x Wq[ ,]x x)x Wx\“Xs}b“us…\x|“s]“ )x F}“ d|“Yk\Wx“}bW}“ ]x?x )x d|“ W“ |sk‚}dsq“ Wq[“ ]x @x %x d|“ qs}“ F] ]x?x)Qx …b\x\“v345“ )x }b\q“ ,]x x)x?x ),x‡Œ )Q`x x%x d|“qs}“W“us…\x“s]“ )x W|“ ,xrx)Q ax x %x d| s[[“ V\“bW„\“sq\“ |sk‚}dsq-“ q h‘ ( “g ?x % dd “ gx?x ,]x x)6x 9aWdq“ …\“bW„\“ )f ?x,]*x x/]x x%x?x,]x x%]x x%x Wq[“Xs}b“ ]x x% Wq[“,]x x%x o‚|}“X\“us…\x|“s]“)x ;s}b“]x?x  “ %x Wx\“|sk‚}dsq|“ Vb\q“ ]x?x  “ g @x  “ …bdYb d|“ qs}“ W[od||dXk\“ Bsx“ ]x L %x …\“ bW„\“ ,]x x%x @x )]x x]x x%x L )]x x)x Wq[“ }b\x\]sx\ %“]x x%x L ,]x x %x L )x]x x%x D\qY\“ d]“ ]x x%x ?x )Qx ]sx“ |so\“ us|d}d„\“ dq}\a\x“v “ }b\q %&“

)R *x L ,^x x%x L )R &x Wq[“ …\“ZsqZk‚[\“}bW}“ ,^x x1 ZWqqs}“X\“W“us…\x“s]“)x Tb‚|“}b\x\ c|“sq\“ |sk‚}csq“ cq“ }bc|“ZW|\.“ r p “ 1 ,x /O

\} :“X\“Wq r\k\o\q}“|‚X|\}“s]“o %x)x   xx)$$5px …c}b“}b\“uxsu\x}ˆ“}bW}“}b\“[c^\x\qZ\ X\}…\\q“Wqˆ“ }…s“q‚oX\x|“cq“:“ c|“qs}“W“uxeo\“ q‚oX\x“ Bcq[“}b\“kWxa\|}“us||cXk\“„Wk‚\“ s]“r“ Bcq[“W“|\}“ …c}b“}bc|“q‚oX\x“s]“\k\o\q}|“ Ns}\,“ %x c|“qs}“W“uxco\“q‚oX\x x  H]“ r“ Io : “ }b\q“ r“ x # |‚: “  )x,x1x(“ 9osqa“ r“ x%xr“ x/xr“ x2x W}“ os|}“ sq\ ZWq“ X\“ cq“ :“ Tb‚|“ Wosqa“ Wqˆ“ 4x Zsq|\Z‚}c„\“ cq}\a\x| “ W}“ os|}“ )x ZWq“ X\“ cq“ S“ D\qZ\ G : G 01“ ) b)$$5"4cx i‘ 1$/x Q‚Zb“W“|\}“c|“ o/^x x% 7 x^x i‘ $x%x! ! ! xx1$)p x /O

)

V\“ac„\“W“uxss]“s]“}b\“a\q\xWk“ ZW|\“ >sq|c[\x“ }b\“\†uWq|csq“s]

TlXx xUl&x xWTlwx xUl*x W r r r x Tl< xUlhx Wx Y

Tb\“}\xo“cq“ T exW _x …b\x\“# xh“ x^x

$O

r“c|

Y

Tb\x\“Wx\“Wk}sa\}b\x“ ,ž h[ x }\xo|“cq“}b\“‘ ‚ooW}csq“ V\“Zbss|\“ # ]WZ}sx|“]xso“…bcZb …\“ }Wi\“ Tl= x Bxso“ }b\“ x\oWcqcqa“ r“  # ]WZ}sx| “ …\“ Zbss|\“ h“ }s“ }Wi\“ }b\“ }\xo|“ Ul-x <ˆ“ |ˆoo\}xˆ “ }b\“ q‚oX\x“ s]“ }\xo|“ Zsq}Wcqcqa“ l>x c|“ W“ Zsq|}Wq} “ W|“ c|“ }b\“ q‚oX\x“ s] }\xo|“Zsq}Wcqcqa“ }b\“}\xo“SY vx Tb‚| “ …b\q“}b\“}\xo|“cq“ }b\“|‚ooW}csq“ Wx\“o‚k}cukc\[ }sa\}b\x “ …\“a\}“ l&l*x!   S x h Rx 1 ]sx“ |so\“  # Bsx“s‚x“ u‚xus|\ “ c}“c|“qs}“q\Z\||Wxˆ“ }s“ Y Y Zsou‚}\“  # Hq“ ]WZ}“ # i‘ )  ,‰1 x h[ x x JAZ xw—h ž h‘ ,ž F[ x <ˆ“ }b\“ 9LCL “ cq\w‚Wkc}ˆ “ …\“bW„\“ $O

D\qZ\“

Tb\“ q‚oX\x“ s]“ Wxxs…|“ }bW}“}bW}“ bc}“Šsq\“ %x c|“ A ,$x %2"/x %)$x H]“Zsq}\|}Wq}“  bc}|“Šsq\“ %xTYx }co\| “ Šsq\“)xUYx }co\|“Wq[“oc||“}b\“}Wxa\}“WYx }co\| “ }b\q“}b\“}s}Wk“|Zsx\“c| %$TY 1UYx 1TY 1TY UY  x 1TY 1%2 WYx 4$ 1TY  WY rx Q‚uus|\“}b\“|Zsx\|“Wx\“Wkk [c|}cqZ} “ }b\q“ }b\“ ,$x q‚oX\x|“TY  WYx o‚|}“Wkk“X\“[c|}cqZ}“ <ˆ“ }b\“uca\sqbsk\“uxcqZcuk\ bWk]“ s]“ }b\|\“ ,$x q‚oX\x|“ Wx\“ \c}b\x“ us|c}c„\“ sx“ q\aW}c„\“ V\“ Zsq|c[\x“ }b\“ us|c}c„\ ZW|\“ Vc}bs‚}“ ks||“ s]“ a\q\xWkc}ˆ “ k\}“ TYx WYx L $x ]sx“ # %x ! xx%1x Tb\q“ TYx WYx JKŒ   Tb\x\]sx\“ TYx # # D\qZ\“ T&x xrxrxrx xT&0x HIŒ %)$x <‚}“T&x xrxrxrx xT.jx A %)$x Wq[“ …\“bW„\“W Zsq}xW[cZ}csq“ Tb\“ q\aW}c„\“ ZW|\“c|“|cockWx“ -

LO

/O

/O

$O

/O

/2x

/O

                   

>CBHCLB@PR'R;NHCR #5R

5' #R

:HLMJNAMEIHLR MIRAIHMCLM@HMLR 

"'-&22 ,R %+'*#"'2



&*2/#+&2 "'-&'2 "2*2 "'-&2 '*2$&#,2"2'2*2$$&#$&*2+ '2  #/#+&2"'-&'

%

#2 '*$'2&2 "2 *#2+'*2/#+&2"'-&'2

*

2%+'*#"2&&'2 R !&2

,

#2 + *#&'2&2  #-2



?A ?A ?A ?A +'A ';34'55,20A A5,0A  %25A  %25A  %25A %!0A#'A 5,/3.,)'&A!5A25,0A  2 :+'4'A 2 !1&A $!4'A325,7,9'A,07'*'45A:,7+AA  2  A ,0&A7+'A9!.8'A2(A A2 



'7 A AAA6A!0&AA#'A 5,;A32,075A20A!A%,4%.'A,0A7+,5A24&'4A58%+A7+!7A77"RA 7"7&R

7&7+RA 7+7/R !0&A7/72R 727R :+'4'A 77"R &'027'5A7+'A !4%A.'0*7+A 2(A7+'A !4%A 77"R '7%A 7A ,5 !.52A -02:0A 7+!7A <77&7/RA  @A ,0&A7+'A5,>'A 2(A =7+727"R ,0A&'*4''5

$

 AA  A #'A  A&,57,0%7A 32,075A 21A 7+'A 5'*/'07A 89R 2(A!A74,!0*.'A 689R :+'4'A 68RA 69 A A 9!.8!7'A 7+'A 58/A R  R  

)

'7'4/,0'A 7+'A .!4*'57A 9!.8'A 2(A ?R (24A:+,%+A

'7A

 -

'7A (



 

 

R R O FFOR2  2!O 21O .1



O

R



 AO  A

2 (2  2 2 .2   . 2 2  0 2 2) #'A !A32.=02/,!.A,0A OR:+'4'A  2 12  2  2  2 !4'A

%2057!075A!0&A"6A A

+'0A &,9,&'&A#=AO A O A O A O A !0&A O A

(<A.'!9'5A !A 4'/!,0&'4A 2(A A  A A  A !0&A A 4'53'%7,9'.=A ,0&A7+'A 9!.8'A2(A( A

0 R

,0&A7+'A 9!.8'A 2(A

KDG4QR KDGRQR

RKDGR4QR

(3R



'7P7NE@G7]P>7]GQE17N]J8] 5@=@P]KJO@P@R7]@GP7=7NO] OQ4>]P>/P]/;7N]57B7P@G=] /GV]JG7]5@=@P]P>7 N7E/@G@G=]]5@=@P]GQE17N]@O] 6@R@O@1B7]1V]

 $

*P] @O] =@R7G] P>/P]

     S>7N7] 0] /G6] $ /N7] N7/B] GQE17NO ] )@G5] P>7] E/T@EQE] KJOO@1B7



R/BQ7]J8]0 3



+7P] $%&] 17] /] PN@/G=B7] S@P>] O@57O] $%]



]%&]



] /G5] $&



] $] QG@MQ7] 4@N4B7] 4/G] 17] 5N/SG]

PJQ4>@G=] P>7] O@57] $&] /G5] P>7] B@G7O] %$] KNJ5Q475] /G6] %&] KNJ5Q475] +7P] '] 17] P>7] 47GPN7] J8] P>@O] 4@N4B7] )@G5] P>7] R/BQ7] J8] %' ]

 



#$





*8] #

$

+7P-#] Z]]] 

]][] '7P7NE@G7] P>7] GQE27N] J8] R74PJNO] U]W] 

P>7]R/BQ7]J8]

 S@P>] U]W]  -]

OQ4>]P>/P U]"] /G5W"]  "] 



+7P] 8H] 17] P>7] GQE17N] J8] ,O] @G] P>7] 574@E/C] N7KN7O7GP/P@JG] J8] P>7] KJO@P@R7] @GP7=7N] H] )JN 7T/EKB7]8]



]/G5]8]

 



 )@G5]P>7]R/BQ7]J8

8]  8]  8]    8]



OQ4>] P>/P] U] A] R/BQ7O] J8] !

$



*P] @O] =@R7G] P>/P] $ @O] /] KJO@P@R7] @GP7=7N] GJP] 7T4775@G=]  ].>7N7] /N7] GJ] G/PQN/B]GQE17NO] U] /G5W

+7P]-]



] )@G6] P>7] 6@887N7G47] 17PS77G] P>7] E/T@EQE] /G5] E@G@EQE] KJOO@1B7

Z]]]]$ ]][] *G]7/4>] J8]P>7] 8JCBJS@G=] OQ1O7PO] J8]-] Z[] Z]][] Z]]]]][] Z]] ]]]]]][]

P>7] OQE] J8] /BB] P>7] 7C7E7GPO] @O] /] EQBP@KB7] J8]  ] )@G5] P>7] PJP/B] GQE17N] J8] GJG7EKPV] OQ1O7PO] $ J8]-] OQ4?]P?/P]P>7] OQE] J8] /BB] 7B7E7GPO] @G] $ @O]/] EQBP@KB7] J8] ]

 $

$]
 $  O/P@O:7O] P?7]N7D/P@JG] 8U8X]



8UW]  ]  8U]  W 8U]  U]S>7N7]

U]W] )@G5]P>7] R/BQ7] J8 8]





+7P] Z0I[] 17] /] O7MQ7G47] J8] LJO@P@R7] @GP7=7NO] OQ4>]P?/P]0]

$    '7P7NEF7] P?7] R/BQ7] J8]  





] 0]



]/G5]8JN] H]  ]



 



'7P7NE@G7] P>7]GQE17N]J8]S/VO] J8]P@B@G=]/] T]N74P/G=B7] 1V]P@B7O] J8] O@Y7]]T



)@G5] P>7] GQE17N] J8]  5@=@P] KJO@P@R7] @GP7=7NO] OQ4>] P>/P] P>7] 6@=@PO] 8NJE] B78P] PJ] N@=>P] /N7] GJG\ @G4N7/O@G=] (T/EKB7O] J8]  5@=@P] GJG@G4N7/O@G=] GQE17NO] /N7] ] /G6] !] $G 7T/ELB7]J8]/] GQE17N]P>/P]@O] $ GJG@G4N7/O@G=] @O]]

$



*&

!4 *4  &(*4 $&4 "- '4 ()4 * !4 

, #4 *4 #&



4

$#(*04!*&(4 !4 $, (4 ' +'4 , "4 ,  ,

*&

!4 *4  (*4 #"), ",4 #,)-4* *4 )*+''',  

 

 



 1&4  !4

,

 

 

#&4 !34$#(*04 & 4 ". &(4         ,

 ,

*#4, !/(04 *44*4 $&# *34* *4 *4  (*4 *&4 #4 *4 !/ &(4 &4 #!(/*04 !4*40 /4 #4    #*4  !#*(4 *4& *(*4!*&4 ((4* !4#&4%/ 4*#4  24 !/

&(4 &4 & !#34 (*4&#

0 / *4



!, 



*&



, ", * !4

,

,

!4 *4  &(*4$&



4*4!*&(4

,

4 *#&4#4 *4 (/

4



  ! ",

 ,

 

*4 &$, ", 4 4(%/!4#4 & 4 !/

&(4(/4* *4

$, #&4 0&34$#(*04!*&(4  *&







, ,





,  $",   $, , ,

!4 *4 0 /4 #4  4 $ , $(%,

,



, !



&4

 

$  " " ""  "  "  " " $  " $  " "  "   "  "  "  " " "  "  "  " !" $$ $ "    " "

!" " " "  "  " "  "  "  " "  "

 "  " " " "

 " "  "

"  "$  "  " "  " "   "

" "  "  " 

" "  "

 " 

 " "  " " " " "  "  "  " "

  



$





$

$ 

 



$



$



$

$

$



$ $



$



$

$



$

$



$



$

                  

       



!% '  #

 # #  # #  # #  # #

  ! !  !  !  !# 

 # #

# !# # !# # !#  !#  #  !#  !#  !#  #  !  !#  #  !  !#  # '

 !  !  !  !# #

# 







  !# !%' ""'# # ' 



 #

 #

!% ' #  !"' ! $' ""'   '      '   '







 " "#



"# '   '

"     '

"# !#"' '   '





" #





  

"   &' #!'

3

38<*7@ 

43897:(9@&@(07(1*@  (*397*)@&9@)3 <09.@7&)0:8@)*3(3)+3 08@,.) !3 <.0(.@08@&184@*6:&1@94@ *,/,.+3

".:8@



!3

".*@54<*7@4+@,.3 <09/@7*85*(9@94@ 

3

,.) *,.3 2,.+(3 !

3



 ".*7*+47*@9.*

8:2@*6:&18@$3 03 @(3  3

38<*7@  !03(*@ =@ @=  @=@ =@@ @ =@ @@= @ 9.*@-0;*3@*6:[email protected])8@0+@&3)@431>@0+ 

!03(*@  



 @ 0+@&3)@431>@0+@'3 ".*@&'4;*@03*6:&109>@7*):(*8@94 = = = =@!  %&3

!03(*@ ?=@= = @= =@@!  %&3 



 !3%@$

<*@(43(1:)*@9.&9@9.*@1&7-*89@;&1:*@4,@=08@ @

3

38<*7 @  #*@.&;* 



1 -1  1 "1 #3

 

3 3 3 3 3

1 1 1 "1 #3

3 1 1  1 "1 #3 3 3 3 3 1 #3

3 1 1 -1    3 3

3 3 3 1 1 -1  1"3 3

 

 

3

  3      

  !4@9.&9@+@ @ 

3

3

 $

'.4- 6 $





 

  

 

/"'6

    



 

  /"'6

 



    

      

    

 

 



  



  









 $



'.4- 6  06$



  

$#$ 6 .1 66'1&-6 56$0#'66 '6 6 4606    $#$ '6   $#$

-.)0#3$56 (0 6(6 0 &6-6 #3#.#%656 $ '60 #-6 #-'6 $  6 #.6 %.(6 #3#.#$656$ 0 -(-6 6 6#.6#3#.#%656 $ 560 6.#&#$-6 -1&'06  6    $

 ! "$ !$-6 #3#.#%656  $ '6(0 -64(-6 %%6 0 6 ##0.6(6$-6('-1'06&(1$(6  $ 

$('0#'.6##0.60 06-6-0-60 '6 $ 0 '6('6'6.10-06$ -(&6 6 ##060(6 066'46 '1&-6$  '6$.0#..60 6 -,1#-&'0.6#'60 6,1.0#('6#6'6('%56#6$ (.6 #'6$$6 0 6 ##0.6-6 ('-2'06&(1$(6 $ #06 -&#'.60(6 ('.#-6 '1&-.6(60 6(-&6 ********6 4 -6+6



*******6

$ $ $ $ $ $ $ 56$0#'66##06 #'60 #.6'1&-6 46 060 6 '1&-6 

*6 4 # 6#.6 #3#.#%6 56 $ #6'6('$56#+6 #.60 6 ##06$ (-6  $ (43-6

0 6 -.0604(6 ##0.6(6$&1.06 6 $.#'6 0 6'1&-6$!.6 $ ##0.6'60 06 '56'1&-6 46 06 56 $0#'66##06 #'6$ .6 $##0. 6 '6 0 6 (0 -6 '6 0 6-&#'#'6 $##0.6'66 $ $ (#.6(6.1 6'1&-.

#')''0%56$ (-6  $ ('.,1'0$56 0 -6 -6 

$



'.3-7 7

  

    

    

       



'.3-7

070#7 $-&73$0#7'0-7 * '7-$1.7  0*1#7 0#7 0("'07 %$'.7 **+-*17'7  +-*17 07 0#7 +*$'0.7 *

*(7 !*-.+0$2&67 #(7 *  * * * '7 0-$'"%.  *(7 !*-7*'"-1'07 '7 #'7 "**"** "* 7#2 */7



*



 

*

 

 

7  '7#'7 / )



*





* * 

*7 !(7  37 #27

* ** 3#-7 * '*0.7 0#7 -7 * 70-$'"%7 * 07 '7

  



&(* $( %(* )) $) %) &* 3#-7  *  * 





*%2$("7 37 "07   #* *(.$-$'"7 0-$'"%7 

* #27 







*37 #27  *** #1.7 37

7

'.3-7  7

$'57







$.77 -**07 * 70#7,10$*'7 ' 40

 







3#$#7 $.7 (*07 +*..$%7 #1.7



 



37 #27











7 .7 0#0

7 #1.

*

*-7







 7









7



49>(8H  H !+(8(H #8(H;>6H %#9(9H;6H %649,&(8H #9(H H 6:  DAH BFH#4&H#9(HH 6:  EH BF H 68H#9(H H



;+(8(H #8(H : H   H



>#@9H#4&H)68H#9(HH ;+(8(H#8(H



>#@9 H (4%(H ;6;#0H 4<3$(8H6)H>#@9

49>(8H  H (;:H D H   H  H HH FH #4&H:-H H D5H 

68H H .H  ,)H5H 

!: )5HH.FH)68.GH H !+<9H

!': #4&H5H +#9H(?#%;1@H/H&,*,;9H,4H;+(H&(%,3#1H8(78(9(4;#;,64H ;+(4H (?#%;0@H

>(H +#=(H/.H&,*,;9H #8(H 464C(86 H !+<9H 



"(/ )(0:

    )(:

!+(4H ,;H,9H %0(#8H;+#;H

 

49>(8H  6;(H ;+#;H   H  :HH #4&H : )68H #00H 4#;<8#0H 4<3$(89H

# :





!+<9H

 :



 H <7769(H /HH  H 36&H H6;(H ;+#;H

6:*:    36&  36&H $<;H 

(4%(H ;+(8(H #8(H 46H 4#;<8#0H 4<3$(89H 6: #4&H  9<%+H ;+#;H  !+(8()68(H 3#?D/F  3,4D/FH

 



#: H  H36'HH

H



 *:



 36& H

H ,)H /H,9H #H 3<0;,70(H 6)H H

 H

49>(8H   H (;H 



96!:

"H .H36&HFH )68.H   H H 6;(H ;+#;H !4+:



H !,+:H H#4&H ! :H H (;H  $(



;+(H 9(;H 6)H#00H 9<$9(;9H: 6)H !:9<%+H;+#;H 7&$:8:,9H #H3<0;,70(H 6)H H 6;(H;+#;H)68H#4@H:  !:

:

 8:   83" 3": 15%



( :  

!+<9H :   ,)H#4'H 640@H ,)H 2H 2:!-.: :2! :36'H H !+<9H ,;H,9H %0(#8H;+#;H









(#!'** F1,?,

               *

 

*

*

*

*

*

*

*



*

*

",;*,L   L  L   L #3;*,LF,LF(;AL<;9IL;<;,:=AIL@B)@,A@L F,L1(D,L



;@F,?L  !3D,;L-G-L  -GJLLLL-GL L J -GLLGL@


-GJLLLL-GL L J -LLJ

#B)A?(*A3;0 LF,L1(D,L -GLLGL



-KLLJL -
*<;@A(;AL 9,AL3AL),L4L #<L-GL  GLL4 #B)@A3ABA,L)(*5LAB(A3<;LF,L1(D, GLL4JLL4L  HL LLL4LL&GLLJLL4' GLL4LLG GJLL7LL8LL  6GLLJ GLLJL

GJLLLL4LLGLLJLL4 G 4LLG





4  L

4 4LLL  GLLJ 4 4 4LLLLGLLJL L -






;@F,?L 

"& $"& 

"& * "& $    "&)*





  "& $ 

?<:L1,?,L F,L@,,LA2(A

"&*

3@L(;L(?3A1:,A3*L@,>B,;*,LF3A1L.?@ALA,?:L

"&*

L(;+L*<::<;L+3--,?,;*,L L $1C@L

"& %* 

  L

L (;+LA1(A

"&* "*



*  *



 * (;+L



 *  *

"

"* "*  "  

"*

 L 



*



 * %,LA1,?,-


&HQW9P#]  +9S];]49] S>9] HUF49P] M:] W2ZQ]M:] SAEAH=] 2] ] Y] I] P95S2H=E9 ] &EQM] E9S] 49] S>9] HUF49P] M:] W2ZQ] M:] SAEAH=] 2] ] Y] I] P95S2H=E9] WAS>] S>9] SMN] MP] 4MSSMF] SWM] QOU2P9Q] AH] S>9] E2QS] 5MEUG] FAQQAH=] 2H7] E9S]  49] S>9] HUF49P] M:] W2ZQ] M:] SAEAH=] 2] ] Y] I] P95S2H=E9] WAS>] S>9] SMN]2H7] 4MSSMF] QOU2P9Q] AH] S>9] E2QS] 5MEUFL] FAQQAH=]

-9S] UN] 2] QZQS9F] M:] P95UPP9H59] P9E2SAMHQ] AHVMEVAH=] ;] ]  2H7]  4Z]

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

         

   /AS>] AHASA2E] 5MH7ASAMHQ<]



      ] 2H7]    ]

-MEVAH=;]P95UPQAV9EZ] W9] M4S2AH]

 ]



&HQW9P#] " '25>]  7A=AS] 4AH2P[] Q9OU9H59] 5MHS2AHAH=] 9Y25SEZ] HAH9] ,Q] 2H7] Q9V9H] ]Q] 52H] 49] F2S5>97] UHAOU9EZ] SM] QU5>] 2] ] 7A=AS] AHS9=9P] MP] ] 2Q] :MEEMWQ#] '25>] ] WAEE] 49] P9NE2597] 4Z] 2] 7A=AS] :PMF] ] SM]"] AH]S>AQ]W2Z#] S>9] HUF49P]M:] ,Q] SM] S>9] PA=>S]M:]2] N2PSA5UE2P] ] AH7A52S9Q] S>9]V2EU9] M:] S>9] 8B=AS] (MP] 9Y2FNE9] ] .>UQ] P9OUAP97] HUF49P%]







!!] 2H7] ]



"""" ]



&HQW9P$] ]

     

AQ] 2] NPAF9] S>9H]  ] -UNNMQ9] I]          

  ] W>9P9]925>] AQ] 2] 5MFNMQAS9] S>9H]   -AH59]





:25SMP] MH] T?9] PB=>T] AQ] =P92S9P] S>2H]  5MHSP27A5SAH=] S>9]:25S] S>2S]3K] ] AQ] 2] NPAF9 ] .>9P9:MP9] I]  ] FUQS] 49]2] NPBF9] .>9] E2P=9QS] NPAF9] I] QU5>] S>2S] ]  ]AQ]] )MW9V9P]  ]%]]]  ] Y] !"] X?C6@] 

JMS] 2] NPAF9 ] -BH59]  ]

OU9QSBMH] BR] 





] AQ] 2] NPAF9] HUF49P] S>9]2HQW9P] SM] S>AQ]



$)-( 0 ,

!!

, &

,

 !



!



,

%*0 **0 &,, %,

,

! 

! !

!! !



,

*, 0 .*0 )*0 

 !  !

,

  

,

 ! 

!!

/0 *0)#0 ()%$$0 &, !,

, 0 !#0 **0 &,! , +&&%)0**0 &, !  , *$0-0 #+)*0 ,0

, ,&,, &, ! ,, , , , -0)0 )+(0 $0 %+(0 "#0 **0 &,! , )0*(+0 ) 0 )!!0 $.*0 )%-0**0 #,)0 *0!)*0 &%))!0 %+$0%(0 & , %$)(0 *0 $+#()0 +( , ' , !

!

-(0 " , ,  ,  ,, , , (*0 %#&+** $0/ !")0 $,

! 

 !

, ! , $0 ', )0 %)$0  ! !  !



*%0 0(*((!/0 "%)0 *%0  , *)0.&())%$0 *)0 (*((!/0 !%)0*%0 ,



$)-( 0 , 0 $+#(0 %0 )!*%$)0 )+0 **0 $%0 *-%0 %0 *0 $+#()0 (0 %$)+*,0 , 0 $+#(0 %0 $(/0 )'+$)0 %$*$$0 , !,)0 $0 , )0



 

0 $+#(0 %0 )!*%$)0 )+0 **0 .*!/0 *-%0 %0 *0 $+#()0 (0 %$)+*,0 % $(/0 )'+$)0 %$*$$0 0!! , , !,) $0 ,)0



 



0 $+#(0

0 $+#(0 %0 )!*%$)0 -*0 .*"/0 *-%0 )*)0 %0 *-%0 %$)+*,0 $+#()0 +*0 $%0 *(0 

0 $+#(0 %0 $(/0 )'+$)0 %$*$$0 , ,)0 , ,)0 $0 ,

$+#()0 (0 %$)+*,0 

)0

 

0 $+#(0 %0 )"*%$)0 -*0 .*"/0 *(0 )*)0 %0 *-%0 %$)+*,0 $+#()0 +*0 $%0 *(0 $+#()0 (0 %$)+*,0



0 $+#(0 %0 $(/0 )'+$)0 %$*$$0 , ,)0 $0 ,)0 ,

 

,





#%

       







 









 

   





   







     

       





'

  

              













#%



 

%  %% %







 % "%%

 '

   ' 

     '

     %

'

 !'

   '    '



   

       



      $% #"'



  %% %  ' ' %  % % ! % #% '

  % %&'  ' #'$' % %  % %

  '

'

(

$% ( % % #%  %  $% #%

% !

 

#(



"( #  %(( (  % % #%  %   (    ( " ( 

   

$% $%  %    %  !% "%  % % ! #( #'$(

(





"(

 %(( ( ( " ( 



 ( ((

(( (

$%  (

% "% %  % "% "%

  ( ( (  ( (  (  (   & ( ( ( ( ( 

%   (

% "% % "%

( %  (   ( ( (  (( (  ((  ( ( ( 

   (( ((  ( ( 

%   (

% % "% %  %%%"%     ((   (

% %( (%  (

 (



(

 "(  $ $ (  $&(

 !#( % % ( '! (  (  #  # !# #  !"#  !"#  #

  #

 # #   #

 "&

&  !   "& & &  & & #!& "  & ! & &  

 & ! & & "  #(  ( #( #( ( #(  ( !( ( ( $!(  % #!( ( ( (

( !( ( !!(

%%% ( % !#%'( !( ##( (

( ( 

##( #( #( !( ( &( ##(  ( (  ( (    ( $( !( ( $( &( !( $( &( #( ( ( #!( #! !( %!( %( ##( #( ##( !!(  





(#!( #'

'( ( $!(  (  ( !#%( #( ( % (

%



 %(( '(

% % %   % % % %( ##( % !( ( #( ( ( % $&  ( % !( ( ( ( '( ( %  (  % %



 ( #( !#( !##( % !$( ##

%

 !%    % %& 

% % ( ( !#%( ( $!( ! % %!% !$( ## 





!%  $%  !%%    % 





 ( ( #!( ! %"% ( #% &#( % & !% & "% & #% "$( ##

!"%





 ( # % !#%







 ( " % "#%



 ( !%



 ( %

 ' ! " '   "$'   ' # # '% ' '     

      





'#9 %&4/#9 1'-619 '*$9 -$9 3'#9 /'-+ 419 6'(!'9 (19 ,9 (1-1!#*#19 3/(,&*#9 6'#/#9 9 9

/#9 .-(,319 -$9 3,&#,!79 -$9 3'#9 !(/!*#9 3-9 3'#9 1("#19 9 9 ,"9  9 /#1.#!3(5#*79 -3#9 3'39  9 #9

 9# 9

 9# 9

,9 ./3(!4*/9 #    # ! # 

'419  9 !##  # 9 #,!#9 3'#9 3/(,&*#19 99 ,"9 99 /#9 1(+(*/9 39 $-**-619 3'39 9 99 #  9 

(+(*0*79 -,9 3'#9 *-6#/9 '*$9 -$9 3'#9 /'-+ 419  9



9 9 '#,9   9  # 9 

,"9 '#,!#9 3'#9 3/(,&*#19 9 ,"9 9 /#9 1(+(*/9 '419  9#  9 (,!#9 9 )19 ./**#*9 3-9 9 (39 $-**-619 3'399 (19 ./**#*9 3-9 9



#39 ###   # #**9 1'-69 3'39 3'#/#9 /#9 (,%,(3#*79 +,79 # 14!'9 3'39 # !-,1(131

-$9 -,*79 3'#9 "(&(39 # (#9  # ###  ## #

')19 7)#*"29  #



#

+-"9  # '419 ,79 .-1(3(5#9 (,3#&#/9 -$9 3'#9 $-/+9

+-"9 # (19 (,9 3'#9   ),!#9  #

 #  #  "   #  #



 # ,"9  #

+-"9 # $-/9 ,79  '419 3'#/#9 /#9 (,%,(3#*79 +,79 .*),"/-+(!9 ,4+8

#029



#

6'#/#

+-"9 # 6#9 '5#

#9 '5# #  ####  # #  ### #

) ! ) %) !)

=F "F =F $F -> #F ==F F"->F*F$=F "F %9 # ==F "F ->="=->#F*F$=%9 #F =F F"=F $F -> #F="=->#F+F$=F F"%9 #F F$=%9 # ) ! ) ") %) ! )

$1= ->F4 ==F "F F

1=F+F"BF $F BF F F=7F +F$:F F"F

%)



 

>F >F $1=F*F0=6B 6BF+F 0= "6B 6BF

)





%) ) ! !)

   =F ! ) "

==F F"F $1=F " ) ->F "F ) $F  )  ) ->F ) @55F



) ) !) ) )

1=F  ) @55F

=F

) !)  ) )

@55F ") ) @55F

   =   )  ) =$BF   ) =F "

> %F 1=F 0F=6BF 6BDFBF  ) =F 



" )

->F

) $

   =

&F  )  ) =F F"!$BF   )   " F #

> # %F 1=+F 0F 8,#F " )



->F 

"F BF F 4?F <@5 = ;F

) $

 !) !) ! !

CAF 2E F&2#2'"F*2/ F2E "'&2#2'F" +F2F# F 2F F'F"FF2F#F 2F 'F%F F 2F# F F2'F "F F&CF#F2'F2F#F 2F 'F "F *FC#F F2' "3C#C'%F FC#F "F C'F   !)  ) # ) !) () ') %) !)

C#F CF 'F+F"F

C#F FC'F+F"F ) C#F+F 2'F+F"F "

) %) # !)  $)

) !') )  !$) !) %) "& ) !) %) ! !) !) %)  )  )

(&F

$ $  $ $ $  $  $ $  $ %*.



*..* . $ 

%*..%*.  %.



$ #$ $ $ *.+.,. $ $ $   $  $





"$ $

.

. 

*.+.



*.,.



+.,.



 .

$

. . *. .+.

.,. 

 $ $ $  $  $ !$ $ "$ $ $

*+..*,..+, . .   $ *. +. , .  $ *.+.  $ ,. $  !$ $ !$  $

*+ *, +,!. ()$*+, . $ !$  $ *+..*,..+,.   $  $

 $ *.



&*+,. $ $ $&.  .

  $ #$ *+,. !$

. . . .  " &#'. ,. +. *. *+,.

.+. .,. $ !$  $ 

 $ $  $ *.



. $ $ #$

. . .   *. +. 



-



$ $ $ $ $  $  $

. .

. .  +. ,.

. . . +.

+.  $ !$  $ $ #$  $ $ $ ,. $ .  $ .  $ $ . *. .+. .,. $ $ $ *.+.,. . .  $ .  .  $ .  $ . $ $   #$ $ $ #$ $ $ *.+.,. .  .

! $ $  $







.

         Yxƒrc‰†Œm›Tc’um€c’xgc}›V}•€‰xch› YTV›5));›  +%!&(-*!&%-

–«œªš˜¯¹ *ǎ˜¯¹

; Ô ǎŸª¹

$'&(*%*-

'A=J5021M )=AMG42M;I:G5>:2M/4=5/2M@I2DG5=>A=>A5-G2M.I..:2M.2:=KML=IAM-DM-A2M<22020MG=M8IDG7ML=IAM-
  - -- -  -  - -  -  - ---



+"*!'"-  &!- +)*!&%)>¹

UǎQ7ǎêǎ ǎǎ  ǎ %ūǎ %ŪǎQ ǎ×ǎQ ǎ8} 'ǎ 3ǎ +ǎ .1ǎ 9ǎ



x8HQǎ 'ǎ 3ǎ +ǎ .1ǎ 9ǎ



M

–

=





ǎƖQ ǎǎH ǎǎ® •.š

&;ǎ &#ǎ &;ǎ  ;ǎ &ǎǎǎ ƉQǎǎǎHǎ

 ǎ #ǎ *;ǎ ǎ ǎ

>ǎ8Q ǎQǎ*ǎQ11u¹ M;ǎQ21u¹ 8ǎ 8H8  ǎ7ǎ#"ǎ8 'ǎ 3ǎ +ǎ .1ǎ 9ǎ



ǎMLM

>ǎQǎǎ8QǎQǎǎQ ǎǎQ 8Qǎ8ǎ ǎ8ǎ .'¡ǎ .3ǎ +ǎ .1ǎ 9ǎ



%ǎ ;ǎ ǎ ǎ ǎ

ǎ "ǎ ǎ #ǎ ǎ

ƃğQǎǎ±ǎǎ Qǎ 8QǎQ ǎğǎ%ǎ Qǎ 8Qǎ8Qǎ8ǎ Sǎ UǎSǎ±ǎ

8Qǎǎǎ8ǎ SǎǎQ ŁǎQǎ™ǎǎǎ8ǎ  QBǎiǎ 8ǎǎ8 87ǎǎǎ ǎH ǎǎǎǎ 8QǎSQǎǎ8ǎ*ǎ Q}ǎ # %ǎ  3ǎ ǎ ǎ +ǎ %ǎ ǎ .1ǎ & %ǎ ǎ 9ǎ ǎ 'ǎ











rǎ  eǎ ǎ /¹ ǎ rǎ eeǎ ùǎ $ $$  ǎ$ ¬rǎ 

ee¬ǎ eǎĐǎJǎej ǎ eeƼǎ¬ǎ$ j}ǎ

‰¹

   $



   $

$

'ǎ 3ǎ +ǎ 1ǎ 9ǎ p¹

>rǎ ǎeǎrǎeǎǎrǎ›aǎ!ǎ+2 4› e 'ǎ 3ǎ +ǎ 1ǎ 9ǎ



#ǎ ¥ǎ aǎ a"ǎ a#ǎ

ĐǎJǎeǎ eǎRǎIǎÏǎǎrǎǎrǎ<eǎRIÏǎ\š a¥}ǎ 'ǎ 3ǎ +ǎ 1ǎ 9ǎ

>0¹

ǎ "ǎ %ǎ #ǎ *ǎ

[ǎRǎ Iǎ ǎÏǎjǎŗ ǎj-ǎ FǎRǎ!ǎIǎ\š ǎ I!ǎÏǎ\š ¥ùǎ[ǎ hǎ\š Rǎ!ǎ˜-š irǎeǎrǎǎǎrǎsǎ ǎrǎeǎ  ǎǎh}ǎ 'ǎ 3ǎ +ǎ 1ǎ 9ǎ



#ǎ *ǎ ¥ǎ > ǎ

ǎ #ǎ > aǎ %ǎ

>rǎ ǎǎ ǎ >4((:›  'ǎ 3ǎ +ǎ 1ǎ 9ǎ

 ǎ a ǎ " ǎ # ǎ ¥ ǎ



 &(*- +)*!&%)>>¹

G  ǎǎ  ǎǎ āŭǎ!ǎRǎ!ǎǎ ǎ  ǎjJǎǎ!ǎ B

>O¹

G  ǎǎs yǎ Tyǎǎ ǎ& ""ǎ !ǎ *ǎ ǎ

>Z¹

G ǎ  Tǎ  Tǎ ǎ ǎ ǎ @ǎ ǎ  ǎ ǎ ǎ Tǎ  Tǎ  ǎ



G  ǎǎ ǎǎ ǎTǎ Tǎ >a¹

#ǎÍ ǎǎ   ǎĕǎŀǎǎTƠ ǎ  ǎ ǎ#ǎT S ǎ

 G  ǎǎ ǎ  jTǎ yjǎ ǎ y ǎ ǎ  ǎT S ǎ  ǎ ǎ 

fǎ

>f¹

'ǎǎ ǎǎ yŞǎǎS ǎ ǎ ǎǎJǎǎǎ   ǎƵyǎ ǎǎ “š ǎ j ǎ ǎ ǎ ǎcǎǎS ǎǎǎǎ  ǎƛ¯ǎÆƟ ǎ ǎyjǎǎ jTǎT nǎ  ǎ™ǎǎ  y ǎ fǎ

>l¹

x ǎǎ à!ǎĦǎ^š *ǎ ǎàĦǎ\š *]ǎ„ ǎǎ Tyǎ I›àC ƋǎI›

>o¹

9 Tyǎ ǎ Tǎ¡ ǎ(ǎ*

>s¹

ǎ!ǎ*ǎ ǎ(ǎ*ǎ02

G  ǎǎyǎǎTTǎǎ  ǎ ǎTǎǎ;;;ǎǎǎ  jTǎjJǎ % M

>x¹

'ǎ ǎǎǎƦ ǎǎ   ǎǎǎ$  ǎǎ$ ǎǎ ǎ ǎ ĭǎǎ -ǎ Fǎ$   ǎǎíJǎ"ǎ ǎ ǎ íǎƲǎ ǎǎ $   ǎ %Òǎ  ǎǎǎJǎ"ǎ ǎǎǎG ǎǎǎ ǎ.ǎ

ǎǎǎǎǎJǎ ǎ ǎǎǎǎƧ -ǎ

P0¹

'èǎ  Jǎ S ǎ„ ǎnËǎǎǎ ǎ  %ǎ #ǎ ##ǎ #ǎ * ǎ ǎ Eǎ  ǎ   ǎ ǎ ǎ  ǎ  ǎ ǎ ǎ Çd ǎ ǎ ǎ  S ǎ ǎ  ǎ ǎ Bǎ §ǎǎ ǎ ǎǎǎ  S ǎ ǎ#ǎ  ǎE]ǎ Çd ǎ ǎ ǎǎ ǎ nǎĞǎnǎ

P>¹

Fǎǎ<š GH›• —¹H› .M I› \š ǎǎǎ  ǎ ǎǎ ǎ —¹ "#  G ǎǎ  ǎǎ .

PP¹

G ǎǎ  ǎǎǎǎ=ǎǎǎ ǎǎǎ ǎ<ǎ  ǎǎ -ǎ Rǎ \š ǎ š =Iǎ =Rǎ \š ǎ š I-ǎ

PZ¹

>ǎǎǎǎ—?¹—Q¹ —\¹,,,¹¹ —uu¹  ǎ ǎǎǎǎǎ< ǎǎ&ǎ & -ǎx ǎǎ—@Q¹!ǎ—|›š**+š š —uuQ¹ ^š ǎ„ ǎǎ  ǎǎ—@c¹(ǎ—Qc¹(ǎ-Bǎ(ǎ—u –› 8š



<@š

DGš



Pa¹

G ǎǎ  ǎǎ



A

Pf¹







”š





GtIš



  Iš Jš



”š





JKš









<22D<22Gš ”š

1 1 J L



8š <22Gš







<22I <22Jš

!

<22Iš

8š " <22Jš <22Kš

'ǎǎǎ ǎǎǎǎ² ǎǎǎǎǎ"EEǎ%€ ǎ>ǎ Jǎǎ €š

ǎǎ   ǎJǎ*ǎǎ ǎǎŔ =ǎ ǎǎ ǎǎG ǎǎ  ǎǎ €š $= Pl¹

>ǎ ǎǎǎ ǎ  $ ǎǎ  ǎ ǎ /¹  ǎ ǎǎYǎǎ   $ ^š ǃǎ'ǎ  ǎǎ ǎƶYǎǎǎǎ ǎĞǎǎǎ /¹ $  ǎ$ G ǎǎ  ǎǎˆ,š $

 ǎ



$



Oo¹

Os¹

Ox¹

>ǎ  ǎ ǎ ǎ ǎ ǎ  ǎ  ǎ ǎ ǎ    ǎǎRǎ ǎ ǎǎ -ǎG ǎǎ  ǎǎRǎ [ǎ wš  ' ǎǎǎǎR@ & "Rǎ?ǎ˜ǎ^š ǎǎ˜ǎǎ ǎǎ§ǎwš ǎ ǎǎR@ǎ?ǎ"R& ˜ǎ\š ǎ„ ǎǎ  ǎ b'› [ǎ€ š=ǎǎǎ ǎǎ ǎYš €š Ććǎ=-ǎ1Çd

§ǎhǎ \šâǎǎ?ǎâǎǎ?â"ǎǎ ǎ ?ǎBBǎ?ǎŠǎǎÇd ǎǎ  ǎǎ ahǎ Z0¹

Z>¹

ZO¹

ZZ¹

[ǎ2ǎ  ǎ6ǎǎǎǎǎR@ǎ?ǎRǎ?ǎ ǎ ǎ ǎ˜ǎ ǎáǎǎǎǎǎ R@ & Rǎ? ǎ ǎ\š fǎ G ǎǎ  ǎǎ2 ?ǎ˜6ǎ?ǎ˜2C á6C áĺÕ @L

"ǎ  Sǎ ǎ"ǎǎ ǎ ǎǎ ǎ ǎǎ ǎǎǎǎ Gƞ ǎǎ ǎǎJǎǎ ǎǎ ǎǎ ǎǎ ǎǎǎǎ  ǎĝƳǎ¯ǎǎ ǎǎ   ǎ  Sǎ x ǎǎ=ǎǎǎŕ ǎǎã î ǎǎ *RIǎ ǎRǎ¯ĝƿǎ ǎ Jǎ ǎ ǎ ǎǎ ǎ ǎ ǎ ǎ Eǎ ˆǎ Jǎ  ǎ ǎ=ǎ ǎ ǎ ǎǎ

  ǎJǎ}ǎ ‰¹ ƽ ǎ$$ \š 6K& ǎYǎ$ \š 2ÿǎYǎ ǎ $ ^š 2ǎYǎǎ2ǎ¯ ǎ6



 ǎǎǎǎƆG ǎǎ  ǎǎ 2& 6ǎ Za¹

=ǎ@ǎC Eǎ ˆǎJǎ ǎǎ=ǎǎ ǎĆćǎ=ǎŰűǎ ü ǎǎǎ ǎ =ǎ@ǎŖ ǎ *  ¯ǎǎǎ ǎ}ǎ

Zf¹

[ǎ=ǎǎǎ ǎǎ ǎǎ= @ǎ?ǎ E=ǎ?ǎ"ǎǎǎ ǎ<ÕǎG ǎ  ǎǎ=ǎ



Yxƒrc‰†Œm›Tc’um€c’xgc}›Y†gxm’•› Vdj`Ylkm^{QYta^iYtdZYg{SgyildY\{VQS { O00s¹ Ovjdkn{V^Ztdkj{Vkgvtdkjq {

/

4

8gnsOl-… 9

4>?{ \š >2. {/>-<{ \š 4<{ “š <{ “š >B{ IgN…4--C8{/ǎ4D{ “š 4>/8{ Ch…pVO…ilhNqMp…MhgpIWgn RIMphln… %…IgN…'+ …sVWMV…ilhNqMO…/-08

8gnsOl-…T 4u…šw… š u  E{ \š - { 4u…(ǎw…/ǎ-{ … u…/E{ ǎ hl…E hl…&ǎIgNw…\š4u…\š Ŏ#ǎhl…#ǎ/0w…

u  E{ ^š -{ Ch…sO…VIrO…u…\š 

-

8gnsOl-…1 EVO…SZlnp…WgpOUOl…Wn…4//{ IgN…pVO…_Inp…Wn…4--/ { Ch…4//{š< //-{/ǎ4--/{ STš <M/ǎ/C-

"

8gnsOl-…' *ǎ5..B{ \š C{  /{5..B{ \š #"4þǎ š /{ Rhl… nhbO… WgpOUOln… l0{ CWbW_Il_y … sO… VIrO E5..B{[š C{š/ 5..B{/ǎ#"4@ǎš/{ Rhl…nhbO…WgpQUOln…]! …=OgMO…pVO…lObIWgNOl…Wn…4

>

8gnsOl-…1

ǎ 4{ 4{ ǎ ǎ >{ "ǎ › "ǎ —›  “  “  \ 

#

8gnsOl-…' @p…Wn…M_OIl…pVIp…ƒ/ǎ/{ Ch…$š$ \š//{ UVš$\š4 {=OgMO…$š$ MIg…LO…ǎšC{ hl…"ǎš*ǎhl >{š#ǎhl…k¹š>{ hl…*ǎš"ǎhl…C{(ǎ

*

8gnsOl-…3 {4{št{5{ \š "ǎ?ǎ4u{šǎ\š *ǎš4u{ IgN…{4{ "2 *ǎ 4t{Ch…{4{šu{ 5 ?ǎ4{ t5{/ǎ/ " *¹ CWgMO…-{ Yš 4{ tL{/ {-{ Yš 4{ u5{Yš /{ /0pVO…_OInp…WgpOUOl pVIp…Wn…UlOIpOl…pVIg…›4{š…"…Wn…& ?L

C!

8gnsOl-…T ©¹ ©¹ ©¹ ©¹ +¹ u…?ǎw…š˜š \š E{š 3—› Chu4…/{š 3›w…/ǎ E 3›IgN…˜š/ǎ/{š 3—› CcMO…u …w …™š z{¹ - { sO VIrO…4{ h ¹ ǎ¹/C{

E

8gnsOl-…1ǎ 4--C{/ǎ48{ “š 4>/{;hgnWNOl… Fu › I \š 4U1{“š 4>/{oY{ `w…F \š 4U6{“š 4>/{o6{IgN… I›˜š ſ/ǎ4U9{ “š 4>/{o:v{EVOgjm…šV šV \š ǎIgN…¦§¹š¦Q¹š¦ \š/ {nh…pVO…gqbLOl…hR…>=D6G6J2M WgpOUOl 5{ 8{ 8{ nh_qpWhgn…Wn…?T5{ rš 8T5{ \š ;-{EhUOpVOl…sWpV…pVO…"ǎihnnWL_O…NWnplWLqpYhgn…hR…pVO…nWUgnn¹

.?]ǎ?]ǎ?ǎ]ǎ .?]ǎ ]ǎ  ǎ

/-"



 .&]ǎ?] &]ǎ.&]ǎ&] ?]

ǎ<ǎǎ ;-{ “š ={vǎ/4-{ 

Zgǎ 3ǎ ˆǎ ǎ Jǎ ǎ „ ǎ J7''H{ . ǎ /--%{ ėǎ ǎ ǎ ǎ J7{ /. ǎ /--{ vǎ C/ <{ ?{ J . ǎ/--{vǎ@/ {J . ǎ/--{/ǎ=/ {J H{. ǎ/--{ [š 4/{  ǎJ '{. ǎ/--{/ǎ-/% ˆ ǎJ7''H{ JH{. ǎ/--{/ǎ4/% 

//%

Zgǎ/ >ǎš ǎǎ.ǎ /{ 7''H{?4ǎ --C{ = {?4ǎ --C{ ^š/%

/4%

'gǎ ;= Gǎ'ē|xē\ǎ2 ?ǎ A44 qš

Ĵǎ  / ?ǎ,2

?6-ǎˆ ǎ r/==

2 4 A44 /=={vǎ;=#{

/;%

Z\ǎ4C [ǎ ǎǎ ǎ  ǎ [š 6]ǎ ǎ /ǎ6ǎ Fǎ6@ǎvǎC{ PQRš 6ǎvǎ4%{ ˆ  vǎ/=e{/ǎ4C%

/=%

Z\ǎ4= [ǎ oǎvǎ ² ǎǎ S ǎšš€ǎ  ǎOǎvǎ ² ǎǎģšS ǎ

€ >ǎǎ ǎǎǎ ǎǎǎ S ǎǎǎ

 

ǎǎoǎ|š O]ǎ ǎ@- ;@{/ǎ4=%

/>%

Z\ǎ ;4 >ǎSǎ ǎ ǎǎbns{<ǎǎǎ  ǎǎ4-{  ǎ.ǎ ǎ=  ǎǎǎ XǎǎǎSƻǎǎƗǎǎǎ<]ǎ ǎ™ǎ4   ǎ ǎǎ ǎǎǎJǎǎǎ@={&)*4{[š ;4{ ǎ<

/@

Zgǎ ; Xǎǎvš  ǎ-›ǎǎǎǎǎ<ǎr 7{ /Ar{?ǎA-{/ǎ-%{ Fģ šǎǎ  r{/ǎA{ ǎ/{-%{ˆ ǎ I›vš -›I›vǎ;%{

/A%

Z\ǎ4--C [ǎ r{vǎ ]ǎ  ǎ ǎNJ  J  ǎ XǎR@ǎ/ǎ 4--C{ ?ǎ4--Ar{ MNOš  r{  4--C r{ ?ǎ/{ [š -#{ ˆ ǎ ǎ  J  ǎǎr{/ǎ4--C%{

/C%

'\ǎ ; ;/@>{ >ǎ< ǎǎǎ/>{?ǎ;-{?ǎ=>{?ǎ%%%{ {JJ({/ǎ/> 44//{/ǎ;;/@>${

/ E%{

'\ǎ4/@-{ ˆ¹  ǎǎǎǎǎ ]ǎ ǎǎǎ ǎǎǎǎ oǎ ǎǎ   ǎǎ=#{ėǎǎǎǎǎ;{ g q¹

W… $ǎRlhb…nVhi…$ IdN… ǎRlhb…nVhi…Oąǎ #$$ǎ ‘š %Òǎ/ǎ $0 WW… "ǎRlhb…nVhi…$ IdN…ǎRlhb…nVhi…Oąǎ $$ǎ(ǎ #$$ǎ ‘š %Òǎ/ǎ #$0 WWW… ǎRlhb…nVhi…$ IdN…ǎRlhb…nVhi…O…ǎ #$$ǎ(ǎ#$$ǎ/ǎ$$0 pVO…_hsOnp…ilWMO…Wn… #$ $

8dnsOl-… E @R sO…pI^O…bhNq_h…%0ǎpVO…nOrOd…WdpOUOln…%0ǎ ##0ǎ#0ǎ* 0ǎ0ǎty¹ UWrO…0ǎz{ 0ǎ0ǎ 0 0ǎ …=OdMO…pVO…lObIWdWdU…pVlOO…WdpOUOln…MId…hd_x…LO…##0ǎ* ǎIdN…-

 

8fnsOl-…$$ EVO…OkqIpWhd…Wn…OkqWrI_Odp…ph…Wǎ& 2ǎ\Ž/ǎ.M°¹ $$ ;InO… lǎ @R….M Yš $$0ǎ pVOd…Wǎǎ& 2ǎ\Ž 5ǎ.MC $$ǎ VIn…dh…lOI_…lhhp…nWdMO….MC $$ǎ Yš $0 IdN…Wǎǎ& 2ǎWǎ5ǎ.M(ǎ$$ǎVIn…Ip…bhnp…ǎlOI_…lhhpn ;InO…lǎ @R….M eš $$0ǎ pVOd…LhpV… æC 2®ǎ5ǎ. $$ǎ IdN… æC 2®ǎ5ǎ.M(ǎ$$ǎVIn… lOI_…lhhpn … sVWMV…UWrOn…"ǎ 05DH5
;InO…lǎ @R….M5ǎ$$0ǎ pVOd… \Ž’ 2ǎ\Ž 5ǎ. $$ǎ/ǎ$ǎ VIn…hd_x… ǎ lOI_…lhhp…ǎ520ǎ IdN ǎ ǎ.M(ǎ$$ǎ/ǎ"$ #ǎVIn…ǎlOI_…lhhpo…ǎ/ 2DŽǎ"$ #E & 2ǎW/ =OdMO….M/ǎ$$ 

8dnsOl-…Cǎ 8NNWdU…pVO…ǎ OkqIpWhdn…IdN…nWbi_WTWdU…UWrOn…=ǎ(ǎC LM5ǎǎ?OdMO…WR…=ǎ/ǎC0 sO…UOp…$ǎ/ǎǎ sVWMV…Wn…WbihnnWL_O



8dnsOl-…$$ AOp…Pǎ hR pVOb…LO…CǎIdN…=ǎhR pVOb…LO…C ǎEVOd…Pǎ(ǎ=ǎ/ǎǎIdN… &É Pǎ( CƩÉ = /ǎ$ǎ Ch_rWdU …Pǎ/ǎ"0ǎ=ǎ/ǎ#"ǎ =OdMO…CĂ Pǎ(ǎ& Ă =ǎ/ǎ$$-

"

8dnsOl-… $$"

‘ǎ ‘ǎ C ‘ǎ ‘ǎ IdN… pVO… Ìǎ Ìǎ(ǎ:¹ ÌǎķÌǎ(ǎ‘ĸ Ch… pVO… dqbOlIphl…5ǎ J¹ Y¹ M d¹ J--h¹ J--n¹ ‘ǎ J--q¹ N…OdhbcIphl…/ǎY¹ M d¹ h¹ s…V…V… ljš U[GOn… 5ǎ $$" J--n¹ J--q IL





%







/ / 0 L





0L

0L





  



8dnsOl-…# EVOlO…IlO…* ǎbq_pWi_On…hR…*0ǎ"%ǎbq_pWi_On…hR… ǎIdN…#ǎbq_pWi_On…hR…**ǎ pVIp…IlO…_Onn pVId…%$$Bǎ Ch…pVOlO…IlO…* ǎ(ǎ"%C #ǎ/ǎ $ǎ dqbLOln…Wd…pVO…nOp…sVWMV…IlO…bq_pWi_On…hR $ǎ ǎ *ǎhl… ǎ=OdMO…ilh…L…L_… I…px…/ǎ•¹/ %$ -



'…ǎC + ǎ ǎ ǎ$  ǎ ǎ $ ǎ <  -ǎ F   7ǎ $  <  -ǎU ǎ$$ 5ǎ$O{$ Oǎ aš $ ǎǎ ǎǎǎ  ǎǎ ǎǎ ǎ/¹$  ǎOǎ+  7ǎ‡šO{$ 5ǎC{Y-

4@$

$

Ǝ ǎ

4A$

'Ùǎ=4 ǎ iǎ  [ǎǎU+qǎǎǎ¹/C{ ǎ4ǎ ǎ  î 5ǎ42ǎ/C{5ǎ46ǎǎ .2ǎ6 5/{ [+Ýǎǎǎ¹/C{O{lcf!{ qǎ  ǎ ǎ lcf  4ǎ5ǎ34-!{ +  7ǎ ǎcf /{fO{/{ ǎ .426 ð 4ǎlf{O{l{ aš  . $ǎ3C{5ǎ4ǎ `š lO{@${U ǎf{5ǎ;{ 

5"B

4C

'…ǎ-{

d{ ǎǎǎǎǎ< ǎ@ & "ǎ {g{O{-{ ǎd 5 =d{ {g{O{-{  ǎd{ ǎǎǎ ǎ<  @ǎ {"& g{5ǎ-{ ǎd 5{ {=   d g{5ǎ-${iǎ ǎ4g{5ǎ-{ bš dk{5ǎg R-$

4 E${

'gǎ4--A{

8 / 8 8 3 8

j m{n{5ǎ m{

;-#

š

š

š )))š š

8 / 85ǎm{ /(



j 4{n{ j{ ;{n{ ,{ ={n{ {$ $${ j{ n{n



4--A{ h·ǎ { ={4--C{ ${$${ ,{ 4--C{4--C{P{ ,4{4--C{ ,{  ;{4--C{ ,

n{

 4{

?L

'ůǎ ;4-@= Xǎǎcf{5ǎgh{53ǎ {c {f{5ǎ4---{  ǎg{ {h{\š 4--C${ Fǎǎ 

c {gf{ g{ c hf i›5ǎXcf{ c {fg{ g{ 5\Xcf c {fh{ {h5\ Q{ 3{ 4---g{ g{ 73{ 4{ ---h{ {h5{ O{ Cg{ {=--Ci{ 5ǎ ;4-@=#{ :.¹

 -

'…ǎ*# ǎ

Å ǎǎ•   ǎǎc  ǎ ǎ9s  ǎǎ 7ǎ\š #{ǎ "{ǎ E{ǎ & "{"{ǎ! "{ǎ! {ǎ& {ǎ5 %C  & #!#!ǎ  & #/ *# 

'…ǎ ǎ/ǎǎ š Bǎ Fǎǎǎ ǎǎ!ǎ*ǎ!ǎǎ!ǎǎ!ǎǎ!ǎIǎ†ǎ Eǎ!ǎǎ!I ǎ ǎ 4ý-ǎ ǎ

ǎ*ǎ!ǎǎ!I ǎ & ǎ!ǎ ! ǎ ǎǎbšI& & ǎ\š ǎ4@ǎ_š& »¼½ǎ 4@»¼½ǎǎ†ǎ4@ǎuǎǎF Iǎ/ǎǎ!ǎ ǎbš ǎ!ǎǎ/ǎ4Ö-ǎßǎ4Öǎ/ǎ0ǎǎ/ǎǎ I5ǎ-ǎßǎ4ýǎ/ǎ 0ǎǎ\š %ǎ

®¹Áǎ#ǎßǎ4Öǎ\š 0ǎǎ/ǎǎ IÁǎE@L



'…ǎ Å ǎ>  ǎc< 7ǎ2Vǎ!ǎ2ǎ“ǎ6V& ǎ†2!ǎ ǎ& 62ǎ!ǎ ǎ! 6“ǎǎF  ǎ2! ǎ!ǎ6“ ǎ ǎǎ¾¿À2 !ǎ ǎ& 6ǎ“ǎǎ¾¿À2 C 6ǎ·¹ -ǎÅ ǎ>  ǎc< 7ǎ ǎ2ǎ!ǎ6V& ǎ“ǎ2Vǎ¾¿À 2C ǎ& 62 & ǎ! 6‚ǎǎF  ǎ2 & ǎ!ǎ6ǎ“ǎǎ¾¿À2 & ǎ& 6ǎ‚ǎ ǎ†2 & 6 »¼½ǎǎU ǎ2ǎµ 6ǎuǎ

"

Ź…ǎ"#  ǎ.=@ǎ& E0ǎ=@ǎ& *ǎ \š ǎ †ǎ ǎ.=@ǎ& E0ǎ ǎ\š -ǎ U ǎ=@ǎ  Eǎ ǎǎǎ

ìǎ & ǎ ǎ†ǎ=ǎ ǎ ·ǎ F  ǎ ǎ "#$M <M»¼½ǎ 0ǎ ǎǎ ǎ !ǎ ǎuǎ"#ǎ      ǎ ǎ=

%-

'…ǎ = @ǎ!ǎ E=ǎ!ǎ"ǎ\š .=ǎ!ǎ=ǎ!ǎ #-ǎcǎ=ǎ!ǎǎ \š 4ǎǎǎ ǎ40ǎǎ=ǎ!ǎ #ǎ\š ǎ ǎ #E44ǎ!ǎ ǎ ǎǎǎ<·ǎFǎ ǎǎ 4ǎ!ǎ ǎU ǎ.=ǎ!ǎ =ǎ!ǎ #ĹIJ/ ǎ ǎ=ǎ!ǎǎ†ǎ ǎ.=ǎ!ǎ0ǎ=ǎ!ǎ #ǎ/ǎ ǎ†7ǎ ǎǎǎ ǎ<Bǎ [ǎ=ǎ!ǎǎ/ǎP@ǎF  ǎ=ǎ!ǎ #ǎ ·¹ Pǎ!ǎ @ǎ\šP @ǎ!P ! ǎ ǎ\š =ǎ!ǎǎ! P!ǎ 0ǎǎ  PŴǎ#ǎ+ S ǎPǎ \š 0ǎ0ǎ"0ǎ%ǎ ǎ#0ǎ=ǎ!ǎ #ǎ\š P @ǎ!ǎ ǎ\š *0ǎ0ǎE0ǎǎ ǎ"E    7ǎU ǎPǎ5ǎ#ǎ ǎ=ǎ\š ǎ

:;¹

Xw‚qaˆ…‹l› Sa‘tla‘wfa{› X…fwl‘”›

$&%'2(#(!2!*#&2 2 6&&G %( "( %( ( #%'( ( %(  (

  (

!#%"!( "( #!"#!( 6(š "' ) 

) Fš &"$") ;&š

') ) %) "%") ) (& ) ' )

?'š ) &"%) 

") 61š  )

) ) &% ")  ) ')

6+Ž

9k‚ (,.0‚ A,0.‚ Nu‚ƒŽ 1‚ ]t‚ wWS‚ Tnnw‚ nT‚ wWS‚ Ofw]wzRS‚Trnj‚0‚ wn‚ ,.‚ OkR‚ 4‚]t‚ wWS‚ pn]kw‚ nk‚ wWS‚ t]RS‚ .0‚ tzQW‚ wWOw‚ 04‚

L(

.1#'‚ Crn{S‚ wWOw‚ ,1 ‚04‚LŽ ,4‚

=Sw‚ nŽoŽqŽrŽ PS‚ pnt]w]{S‚ rSOf‚ kzjPSrt‚ tzQW‚ wWOw‚ qrŽLŽ6+Ž Crn{S‚ wWOw‚ wWSrS‚ ]t‚ Ok‚ ]kwSVSr‚  tzQW‚ wWOw‚ noIJŽ :IJŽ n



>L

ŽqoŽ r&



9k‚ wWS‚ qzORr]fOwSrOf‚ DEFG ‚ , ‚ .‚ 0‚ OkR‚1 OrS‚ wWS‚ j^Rpn]kwt‚ nT‚ wWS‚ t]RSt‚ DE ‚ EF ‚ FG OkR‚ GD‚ rStpSQw]{Sf‚ OkR‚ BC¹ ]t‚ wWS‚ j]Rpn]kw‚ nT‚ 01‚ HzppntS‚ [Ž ]t‚ wWS‚ pn]kw‚ nk‚ wWS‚ f]kS ,hN&‚ tzQW‚ wWOw‚ [0‚LŽ .0‚ Crn{S‚ wWOw‚ fQ[}7ŽL u€



H]~‚ R]tw]kQw‚ pnt]w]{S‚ ]kwSVSrt‚ nŽoŽqŽrCŽ} š _{ OrS‚ V]{Sk‚ ;OQd‚ OkR‚ ;]ff‚ QOfQzfOwSR‚ wWS‚ tzjt‚ nT‚ SOQW‚ pO]r‚ nT‚ wWStS‚ kzjPSrt‚ ;OQd‚ QfO]jt‚ wWOw‚ WS‚ WOt‚ %$‚ pr]jS‚ kzjPSrt‚ |W]fS‚ ;]ff‚ QfO]jt‚ wWOw‚ tWS‚ WOt‚ !M pr]jS‚ kzjPSrt‚ OjnkV‚ wWS‚ tzjt‚ MWn‚ WOt‚ wWS‚ QnrrSQw‚ QfO]j+‚



2SwSrj]kS‚ Off‚ pr]jSt‚ …š tzQW‚ wWOw

mkŽ @Ž /Ž…



]t‚ O‚ pSrTSQw‚ tqzOrS ‚ ]S ‚ wWS‚ tqzOrS‚ nT‚ Ok‚ ]kwSVSr‚

6J¹

Xw‚qaˆ…‹l› Sa‘tla‘wfa{› X…fwl‘”›

$&%'2)#)!2!*#&2 2 6&&G %( "( %( ( %"!(



?Su‚ PTLŽ †Ž WTLŽ„+Ž IWSk‚ SXŽ

„4<Ž OkR‚ „Ž †Ž Ž PQŽKp+Ž -‚ t]j]fOr‚ ur]OkVfSt‚ |S‚ Oftn‚ WO{S‚ SPŽLŽoŽLŽ "{ >Su‚ ZŽ PS‚ uWS‚ on]ku‚ nk‚ PYŽ tn‚ uWOu‚ YZŽ SY+Ž 7rnj‚ PSY
=L

>L

|S‚ WO{S

PY:ŽLŽSP:Ž ŽSY:Ž

¡'›' ¹

PZŽ „Ž 4<:ŽLŽp†Ž „Ž :Ž4@ŽLŽ„Ž ‡Ž †Ž „Ž :Ž4@+Ž

IWSrSTnrS‚

PZPZŽ Ž„ŽLŽ††Ž¹ „/ 7rnj‚ WSrS‚ ]u‚ Tnffn|t‚ uWOu‚ PZŽLŽ†Ž OkR‚ |S‚ OrS‚ RnkS‚

L) MS‚ kSSR‚ un‚ orn{S‚ uWOu‚ Z› 9 Ž |}¹ uW]t‚ ]t‚ Sqz]{OfSku‚ un

poŽ r1Ž

-‚ tqzOr]kV‚ PnuW‚ t]RSt‚ OkR‚ t]jof]T‚

[AB› os Žnr4<+Ž H]kQS‚ os Žnr4<Ž U¹ š›LŽ Z› uWS‚ ornnT

]t‚ QnjofSuS



7]rtu‚ nPtSr{S‚ uWOu‚ PRSTŽ ]t‚ O‚ oOrOffSfnVrOj‚ P‚ KOr]Vknkt‚ uWSnrSj‚ ?Su‚ uWS‚ S~uSkt]nkt nT‚ PgŽ OkR‚ RSŽ jSSu‚ Ou‚ h1Ž H]kQS‚ PTŽ ]t‚ oOrOffSf‚ un‚

ShŽ 2PTŽLŽ eghS$Ž H]kQS ePgTŽLŽ ehgSŽ OkR‚ 8TŽLŽ ^vSŽ gPTŽ OkR‚ ~whSŽ OrS‚ QnkVrzSku ‚ tn‚ uWOu ShŽLŽ TPŽLŽ SRŽLŽ [S0Ž IWzt‚ [Ž f]St‚ ¸¹ uWS‚ Q]rQfS‚ QSlurSR‚ Ou‚ SŽ |]uW‚ R]OjSuSr‚ Qh& 8SkQS‚ eQ[ŽgŽMŽH5Œ &

_$¹ HzoontS‚ xŽnT‚uWS‚ j¹ kzjPSrt‚ OrS‚ S{Sk‚ H]kQS‚ uWS‚ tzj‚ nT‚u|n‚ S{Sk‚ nr‚ u|n‚ nRR‚ kzjPSrt‚ OrS‚

<Ž uWS‚ nkf‚ S{Sk‚ or]jS ‚ |S‚ tSS‚ uWOu‚ uWS‚ kzjPSr‚ nT‚ or]jSt‚ OjnkV‚ uXS‚ tzjt‚ ]t‚ Ou‚ jntu‚ xD x+Ž -‚ QWSQd]kV‚ Tnr‚ xŽLŽ 5Ž6Ž   ‚ŽDŽ |S‚ tSS‚ uWOu‚ uWS‚ jO~]jzj‚ {OfzS‚ nT‚ xD xŽ ]t‚ !M OuuO]kSR‚ |WSk‚ xŽLŽ X& IWzt‚ ;OQdt‚ Okt|Sr‚ ]t‚ RSU_k]uSf‚ |rnkV‚ <]fft‚ Okt|Sr‚ ]t‚ QnrrSQu‚ PSQOztS‚ !M or]jSt‚ QOk‚ PS nPuO]kSR‚ Trnj‚ uWS‚ Tnffn|]kV‚ j¹ kzjPSrt)‚ E¹ ^¹ r ¹ M M ! S{Sk‚ OkR‚ uWS‚ tzj‚ nT‚ u|n‚ R]tu]kQu‚ ont]u]{S‚ ont]u]{S‚ ]kuSVSrt‚ ]t‚ dš

M



=Su‚ BjŽ ¹ >

HL

…?Ž

9L

†:$Ž IYSk‚

IWzt‚

†Ž 9…:Ž

;L

BFŽ †Ž'9…:Ž Zš ¨‰Ž

5fZjZkOuZkV‚ †Ž }S‚ VSu‚ >…:Ž Zš

BGB‰"ˆ 6%Ž

}WSrS‚ -¹ ¶¹ Šš Wš Žš OkR‚ Šš'Žš

:L

…Ž

9T‚ Šš dš -¹ uWSk‚ BŽ \Ž >…:,Ž IWzt‚ …Ž Zš

BŽ OkR‚ uWS …Ž OkR‚ }S‚ WO{S‚ BjŽ >…:Ž Ž6$ MS‚ tWOff‚ orn{S‚ P‚ ZkRzQuZnk‚ uWOu‚ ByŽ dš >x:Ž¹ 6Ž Tnr‚ S{Sr‚ ZkuSVSr‚ xŽ‚ 9$Ž IWS‚ ZkSqzOfZu QSruOZkf‚ WnfRt‚ Tnr‚ xŽ 9+Ž Hn‚ }S‚ OttzjS‚ uWOu‚ Zu‚ WnfRt‚ Tnr‚ tnjS‚ xŽ‚ 9%Ž BnuS‚ uWOu‚

VZ{Sk‚ S~orSttZnk‚ Zt‚ ZkRSSR‚ O‚ tqzOrS ‚ 9T‚ Šš Zš - ¹ uWSk‚ š

;L

;L

;L

>xŽ¹ 6OŽ'6Ž >x:Ž¹ 6

:L

>x:Ž Ž6Ž lxŽ >Ž Wš 6Ž Ž6Ž'6Ž Wš B ¹ ' >x:Ž¹ 6Ž >x:Ž'6Ž >x:Ž¹ 6Ž

IWSrSTnrS‚

¨¢8¹

;L

B .Ž ¨¢¹ dš B>x:Ž¹ 6Ždš >xŽ'6:Ž¹ 6/

M

Tnre‚ 9-Ž

Wvp`‡„Šk›R`sk~`ve`z› W„evk“› !%'&(- * $*!#-#,$'!- - 4$$E{ %!&(-*!&%-

Wu]p[Xx{5C{ PXx{5%%F{

%I;% .5%%{bp{

$'&(*%*-

$*.)0 0 :>{ (,*+&$* $+)0 /&,)0 $*.)*0&$0+0 $*.)0 *+0')&-0 &)0 +0 #,"+'"0 &0 (,*+&$*0 $+)0 /&,)0 $*.)0 &$0 +0 $*.)0 *+0 /0 *$0+0 ,"0 &$+$$0+0 "++)0  0 0 L{ N{ &)0 0 &))*'&$$0 +& +0 &))+0$*.)0 &)0 +0 &+)0 *&)+0 (,*+&$*0 .)+0 /&,)0 $*.)0 &$0 +0 $*.)0 *+0 $0 *0 +0'')&')+0 ,"0 "&.0 /&,)0 $*.)0 &0*+'*0)0 $0+&0 ,*+0/&,)0 %*.)* 0 0(,*+&$0 ))*0 -{ #)! &0","+&)*0 )0""&. 0

  - --- - - - -  ----

<e¹

Qwgtdlg^{MakdZ^{Uw^qtdkjq{



ĵǎ ǎ(ǎǎ(ǎDǎ(ǎ*ǎ(ǎ ǎ(ǎ;;ǎ 7¹ Gš ǎǎǎ ǎǎǎ ǎ ǎ ( M(ǎ"ǎ!# ǎ ǎ £Bǎ! $$ǎ ǎ ǎ! "ǎ .'ǎ ";ǎ ";ǎ 3¡ǎ D$ǎ D$ǎ .+ǎ D ǎ ;ǎ Ó1ǎ ;;ǎ $$ǎ 9ǎ $ ǎ



Fǎǎǎ  ǎ LM ǎ ǎŽǎǎ Ƙǎ ǎǎ  ǎ

  lǎ C LMAǎ"C_ŵǎ "CǎLM’ǎ!ǎ M _ Ŷǎ ǎeš ®¹ǎ iǎǎǎ  ǎ   _} .'ǎ _ǎeš& ǎ _ǎ‚ ǎ ǎ 3¡ ǎ .+ǎ _ǎ‚& ǎ Ó1ǎ _ǎeš ǎ 9ǎ _ǎ ǎǎ7ǎ ǎǎ



cǎ ™ › 3¡ǎ .+ǎ

ǎ& ¦¦ 0ǎ Ɖ    Ê.  ǎ 2?@›Ž.Ċċ ǎ Dǎ ĈĉǎŽ.Ĉĉõ "ǎ ƀ¦Ê.¦ǎ ( ˜› "ǎ

Ó1ǎ

ŏĊċÊ .¦ ǎ

9ǎ

ǎ M› õČÊ.ČŐLjǎ

M

M

M

M

"Bǎ

cǎ _  ǎd›ǎǎ ǎ ǎǎǎǎǎǎǎ< āǎ  ?2 ?ǎd›\š &›„ ǎǎ  ǎǎ _›d› .'ǎ 6› .3ǎ ñǎ .+ǎ & .1ǎ ǎ .9ǎ 6›

Dǎ

'ǎǎ ǎ6&› ǎǎǎ ǎ 0ǎǎ››6&›>ǎ ǎǎ  7

ǎǎǎBǎq ǎǎ 7ǎǎǎǎǎǎǎǎǎ

  ǎ7ǎ6› ǎ .'ǎ ǎ ǎ .3ǎ 6 *ǎ .+ǎ ;ǎ ;ǎ .1ǎ ;ǎ ǎ .9ǎ *ǎ

#-ǎ

'3+1ǎǎǎ<ǎǎ'3ǎ]š 2]ǎ ǎ'9qxǎǎǎ  ǎ ǎƹǎ9ǎ ǎǎ  3+ǎ ǎ1ǎ ǎǎ ǎqxBǎ cǎ'9ǎ\š d›ǎǎǎ ǎǎ ǎ9q} d› 9 .'ǎ 3ǎ +ǎ _› 6_0 .3ǎ d› qǎ 2ǎ _1 7 d› .+ǎ 6d› _1 ! .1ǎ L d› 0› _ .9ǎ d› xǎ

M

*ǎǎ

ǎ %~ǎ 7¹ ~ǎ *ǎ~ǎ ǎ ~ǎ ďœǎ

ǎǎǎ Hǎǎǎǎ ljœǎ ő!œǎ Œ ó £ ( œǎ œ!œǎ ǎ ǎ ǎ ǎ .'ǎ ǎ ǎ 3ǎ aǎ *ǎ .+ǎ "ǎ .1ǎ aǎ %ǎ .9ǎ aǎ



.L



.L



.L





'ǎ 8 ǎ ǎ 8ǎǎYǎ8ǎ8   ǎ 8 ǎǎ  ǎZ3+ǎǎ©ǎ'+3ǎ ǎ 8ǎ ǎ cǎ'3ǎ^š ;ǎYǎ ǎǎǎǎǎ8 ǎ'3+ǎ8ǎ#ǎ @ǎ`‰ ǎ H ǎ3 .'ǎ aBaǎ 3ǎ a# .+ǎ  .1ǎ " .9ǎ ŝǎ

;ǎ

Uǎ7ǎ  Hǎ8ǎ=ǎǎǎ ǎǎ *=ǎ!ǎ ǎ ǎǎ ǎ<ǎ ǎ =ǎ(ǎ ǎbǎa$$}ǎ .'ǎ #ǎ 3ǎ ;ǎ .+ǎ aǎ .1ǎ %ǎ .9ǎ ǎ

$

ď  ǎǎ8 ǎH ǎǎ.—!ǎ.

(858 48#8ǎ  ǎ ǎHǎ ǎ- 8H š

ƫǎǎ .' 3ǎ #ǎ .+ǎ ;ǎ aǎ .1ǎ %ǎ .9ǎ

 M

—¹

.M

Vakmt{Uv^qtdkjq{

 -

q  ǎǎ ǎ ǎƒš  ǎǎ ƒ{O›& ǎ$ǎeš ǎ x Hǎǎǎ Iǎǎ  Hǎ ǎǎ ǎǎǎ?Ɠǎ Éǎ \š %$$ǎ 

×ǎ?ǎ%›

­¹ ^š %$$0ǎ`d ǎǎs ǎH ǎ 



q ǎǎ ǎ  Hǎ ǎLǎ ǎǎťǎ eš ƒ=š ǎH7ǎ ǎƒš  ²ǎLǎLǎ(ǎ ‡ ¹ Lǎ?ǎK ¹ Lǎ(ǎ ]¹ Lǎ?ǎ"ǎ€ǎ

"

>ǎ ǎǎǎ ǎǎǎ<   ǎǎ$$#ǎYǎ$$*ǎYǎ$$ǎYǎ

ǎY-ǎ cǎǎ ǎǎ ǎ`d ǎǎ ǎ  ǎH ǎǎ

%

q  ǎǎǎǎ Hǎ ǎǎǎ ƙǎƺǎ < 7 ǎ? ǎƾǎ

#

>ǎ ǎǎ   ǎǎǎ   Hǎ ǎǎǎ#| ǎ  ǎ>

7  ǎ ǎǎǎ>ǎ  ǎ ǎ`d ǎǎǎ„ǎǎ ǎǎ èSǎ[ǎ_ǎ  ǎ 7ǎǎǎ  ǎǎ ǎ ǎǎ èSǎ ǎǎ 7  ǎ ǎǎ™ǎ `dǎǎ ǎ ǎ<ǎ ǎǎ 7ǎǎǎ  ǎǎ ǎ ǎǎ Sǎ ǎ

7  ǎ ǎǎǎǎǎ`dǎƚǎ q  ǎǎ ǎ ǎLǎ ǎ ƒƒ_ LŸǎ Bǎ ǎ

B

q  ǎǎǎǎ7ǎǎ%ǎǎǎǎ ǎǎǎ  ǎ ǎ ǎ# ǎǎ ǎǎǎ ǎ ǎ ǎ ǎ ǎǎǎ:ǎ fǎ>ǎǎǎ  B

E

q ǎǎ  ǎǎC þǎ ?ǎ.C ɐǎ ǎ H  ǎ7ǎ@C ǎ(ǎ

$

Fǎǎ'3+ǎ ǎǎ  ǎ ǎ1 ǎǎ ǎǎ ǎÄǎ ǎ'1Áǎ31ǎÁ +1÷ cǎ Œ¹'+3ǎ5ǎijǎ„ ǎǎH ǎǎB



:w¹

2! 38

ƅǎ

cǎ0ǎIǎ) ǎĩǎ):ǎ,  Hǎ :,ǎ, ǎ)ǎ*ǎ!ǎ Iǎ(ǎEǎ^š ¤#¤0ǎ„ ǎǎH) ǎ  $( I( ĩǎ

øǎ

G  ǎǎH) ǎǎ

¤øǎ

ˆ¹ ǎ`d:ǎ 0ǎ'19ǎ ,ǎ)ǎ: ) ǎ ǎ ©ǎ'91ǎ\š$t0ǎ) ǎ3ǎ) ǎ+ǎ):ǎ ,

) "$Ñǎ)#$tǎ )$Ñǎ ) "$Ţǎ(ǎ)#$Ñǎ(ǎ)$Ñǎ

ǎ, ǎƒ¹ , ǎ)ǎ3+9ǎ ,ǎ)ǎƴ ):) ǎ: ) ǎcǎ'3ǎ ^š "ǎYN0ǎ+1ǎ\š #ǎYN ) ǎ3+ǎ^šǎYN0ǎ`d ǎǎH) ǎǎǎ 9ǎ





™¹

"ǎ

F,ǎ)ǎǎ) Iǎ):ǎ,  Hǎ :,ǎ, ǎ)ǎǎ!ǎIǎ^š $$ǎ) ǎIǎ),ǎǎ N)s NNǎH) ǎG  ǎǎH) ǎǎCIǎ

%ǎ

cǎ , oǎ & !›0ǎ`d ǎǎH) ǎǎ#ǎ,›ǎ(… o!ǎ#ǎ ,ǎ oǎǎ ¤ǎ

#ǎ

¨ǎLǎǎǎ,  Hǎ :ǎ:ǎ ǎǎ,Nǎǎ ,ǎǎ,N) ,ǎ) :,ǎ ,ǎ"ǎ) ǎ ǎ,Nǎǎ ,ǎǎ ):,ǎ) :,ǎ ,ǎ$"BǎG  ǎǎH) ǎǎ¹

*ǎ

cǎhǎ\š 2



8L



ǎ& ġĽƪǎ P› 0ǎ`d ǎǎH) ǎǎ ǎƒƒƒh 4! & 4  ǎ Q›

M.¹



'ǎ) :ǎ:ǎ ǎ:ǎ,  Hǎ :,ǎǎǎ ): ăǎ D0ǎ;D0ǎL0 ) ǎ),S ǎ:ǎ M),,ǎǎ NǎǎM),ǎ NNǎNM Mǎǎǎ:ǎN:, ĕǎ,Í ǎN ,:) ǎ Dǎ),ǎ *Dǎ) ǎ N ǎǎM),ǎ NNǎNM Mǎ *D0ǎ;Dǎ) ǎLǎ ,) ǎ>ǎ),:ǎǎ)  ǎ),ǎǎ,)Nǎ),ǎǎ ::  ),:ǎG  ǎǎM),ǎ,, MǎH)MǎǎŽ

;

>ǎ`‰Í:ǎMǎ,,ǎ)ǎ: )Mǎ'3+ǎ:ǎ'3ǎ^š '+ǎ1ǎ) ǎ9ǎ):ǎ ,ǎ , ,ǎÄǎ) ǎ'+0ǎ:,  HMJ0ǎ, ǎ)ǎÄǎ^š "13ǎ) ǎ'+ǎ\š "'9-ǎcǎǎ):)ǎ ǎ<) : M):)Mǎ3+91ǎ ,ǎDǎ N@ǎ) ǎǎ):)ǎǎǎ: )Mǎ‚…†¹ ,ǎRǎ N@0ǎ`‰ ǎRf 'ǎ

$

>ǎ`‰:ǎMǎ,,ǎǎ : M,ǎ ǎ :,ǎ'ǎ) ǎ3ǎ) ǎ)ǎM ǎ[ǎ ǎ ,ǎ) )ǎǎǎ : M,ǎ)ǎ«ǎ) ǎ—ǎF,ǎ)ǎ«—ǎ^š "$ǎYN0ǎ'3ǎ^š " ǎYNǎ) ǎ ):)ǎǎǎ<) : M):)Mǎė«ǎ ,ǎ$$ǎ N@ǎcǎ—¹ ) ǎ6ǎ ǎǎ):),ǎ 

: M,ǎ ǎ :ǎ'ǎ) ǎ :ǎ3ǎ:,  HMJ0ǎ`‰ ǎǎH)Mǎ ›› —¹

 

8 8 8

G  ǎǎN)s NNǎH)Mǎǎǎ,n Rǎ?ǎ ?%ǎ ǎ,  Rǎ?ǎ :)MǎN:,ǎ I5¹

:ǎRǎ:),ǎH:ǎ)MM

£

G  ǎǎǎǎ |  ǎ  ǎ ǎ ǎǎǎ  ǎÈǎ ™ǎǎ  ǎ|  ›ǎGǎs ǎ "%#*EEE0ǎ%%%%%%%%%%%0ǎ"%%%*E



G  ǎǎ ǎ  ǎ ǎƒš  ǎǎ B

"f

¨h uǎ²ǎ 0ǎ0ǎ0ǎÕf ǎ0ǎ €ǎǎǎǎǎ ǎƱ  ǎ ǎǎ ǎǎǎF ǎLǎ ǎǎ ǎ  ǎ ǎ ǎǎs  7ǎ ǎǎǎh ǎ ǎǎLǎ ǎǎ 7ǎǎǎÈǎhǎǎǎǎ ǎǎLǎ

  ǎ -ǎG ǎǎǎǎǎ  ǎǎŽ

%

[ǎ xoš x= š xB š  ǎ ǎǎ< ǎǎ ǎ  ǎ ǎǎǎ   7  ǎǎ*%0ǎǎ x9š bǎx=š bǎxBš b dž´LJ >ǎƉǎƉ ǎǎǎǎ< ǎl x:š 0ǎx=š 0ǎ xBš "0ǎ xEš *0ǎxHš BǎG  ǎǎ  ǎǎ x>33L/š -

7L



7L



7L

II¹

ƒš

ň ǎ?ǎ ?ǎ ǎ ǎǎ 

Yxƒrc‰†Œm›Tc’um€c’xgc}›Y†gxm’•› Vdj`Ylkm^{QYta^iYtdZYg{SgyildY\{VQS { O00s¹ V^jdkm{V^Ztdkj{Vkgvtdkjq {



Šgǎ+ F ǎǎǎǎǎ ǎǎǎ ǎ  ǎǎǎǎ ǎ$ǎ$%ǎ?*ǎ ǎ$ $;;ǎ ǎ %ǎ Aǎ Aǎ ǎ   ǎ$"ǎ$#ǎ$ǎ$$$ ǎ ǎ % ǎ GL GL GL

-

ŠŮǎ1 F ǎǎ  ǎ<ǎC Iǎ^š "ǎ&_ǎ ǎ"& Iǎ/ǎǎ$_ǎǎ  5C %_ I/ǎ C *_£ǎF    Iǎô %_Âǎ C *_ǎ_Âǎ -



Šgǎ3 Vǎ AǎŒǎ >ǎ Vǎ& ŒVǎ Aǎ µ V ǎ ǎ  ǎ [ǎKǎ$ ũ& Œǎ& Vǎ(ǎŒVǎ& ǎ -ǎGǎ ǎǎǎǎ  

Œǎ& Vǎ& "ŒVǎC ǎ'ǎ  ǎ ǎ Œǎµ¹ /› ìǎ "ǎ X ǎŽC  AûÛ û  A  Š ǎŽ‚ ǎ ǎŬ& ƒ  ǎ  Vǎ‚ǎ CV  ǎ ǎ ǎǎ  ǎ   %ǎ & ǎ‚ǎ‚ ǎ ǎǎ>ǎŽǎ ǎ ǎǎH¹  ǎ ǎ ǎ ǎµ¹ Žǎǎǎµ¹   "ǎ

>ǎKǎ$

"

Vǎ‚ǎ ǎ ǎ

ŠgǎŠ Û & ‹ ǎF  ǎ X ǎ      <ǎVǎ  "  $6ǎ   ǎǎ ǎ*&" ‹ǎ$& ‹ǎ2?ǎ6ǎAǎǎƍ:ǎǎ ǎǎ ǎ *ǎ$2ǎ?6& "ǎ$2‹ǎ   ǎ> ǎ  ǎǎ*ǎ(ǎ2ǎïǎ6ǎ/ǎǎ  ǎ"ǎ?2 5 ǎ ǎ ǎ ǎ2 5¹ ǎ  ǎ6ǎ^š ǎªǎ2(ǎ6ǎ/C ǎ  

8L

%-

Šgǎ3 [ǎD… Ðǎ ǎ"ǎ*ǎ   ǎǎǎ€  ǎ D!… Ðǎǎ%ǎǎ- -  ǎǎ;ǎ€  ǎ D%…^š Ðǎǎ#ǎ;ǎ   ǎǎǎ€ǎX ǎ ǎǎǎǎǎǎ ǎ ǎXǎǎ7ǎǎ   ǎǎ2  ǎ6 ǎ Ðǎ ǎǎǎ  öǎǎǎ;Ŀǎǎ€  ǎãǎǎ $ 6ǎãǎ   ǎ7ǎǎ ǎ ǎ 7ǎ  ǎ ǎ Ɲǎ   ǎ Ą  ǎ ǎǎǎ2ǎ ǎ6ǎ ǎǎ“4¹  ǎD!…   7č  ǎ ǎǎ2ǎ ǎ6ǎ ǎǎ”V#¹ $ $ #$  "%ǎ$ ǎ >ǎǎ<ƥ ǎ 7ǎ/ǎ Kǎ # $ $ Aǎ Aǎ   %;ǎ ǎ ?L

@L

IW¹

#f

źlǎ9 Xǎ ǎǎ”31ǎuǎǎǎÞ91ǎ / ǎǎ”9Gfǎ ªǎ @ǎ xš /ǎǎǎ<ǎ'Ż+1 u  ǎ   Þ31ǎ /ǎǎ ǎǎƇ9Gǎ uǎǎǎ Tǎ'9Gxfǎ



3ǎ š

š š š š š š š š

BL CL BL

CL BL CL









BL š š DL š BL š CL L š š L 2L š L CL  L L L š BL š CL š BL  CL š



0{



U ǎ9GǎAò-ǎ 6ǎ xǎ *

'lǎ3 ~ǎ %~ǎ ' ' *ǎmš

' mš –­Ģçǎ'ʼn( ơçǎ Ŋ( Ƣƣǎ'ŋ( –­Ģçǎ ó ǎ ǎ ǎ ǎ  Aǎ nǎ' ~ǎ( ›ǎ' ~  nǎ ~ǎ( ǎ ~ ǎ įǎ ǎ ǎ +L

 - …



+L

8 fǎ ǎ –­Ĥǎ 8›(ǎĎĔ–ǎ …8› mš

6L



6L









-

mš ǎ–­Ĥǎ0{9 ĎĔ–ǎ0{8› ,L



8

'lǎ+

Xǎǎǎǎ”3+ǎ/ǎ'3ǎ(ǎ@ǎªǎǎ ǎ#ǎ/ǎ;ǎïǎ@ǎ ÿǎ(ǎE¢#ǎ   ǎFM ›ǎǎ< ǎ ǎǎ/ǎǎǎǎ/ǎ3G¹ ǎǎ ǎ ǎ f 

Ef

'lǎ9 G ǎǎǎ=ǎ(ǎ ǎbǎǎ T ǎǎ=ǎbǎ##Ef h0{ 3

F  ǎ*=ǎ(ǎ ǎ ǎǎ ǎ<ǎǎTǎ*=ǎ(ǎ ǎ/ǎh1!{ ªǎ=ǎAǎ fǎªǎ*ǎ ǎǎ ǎǎ h 0{ 3

6L

¹  ǎ ǎ *ǎ  ~š  ~š(ǎ  ǎF  ǎ*ǎ ǎǎ ǎ

M`¹

ǎ ǎǎ*ǎǎǎ ǎǎP¢ ǎǎPǎ!ǎ čǎǎǎPǎ/ǎ*4¢ ǎǎ*4ǎ!ǎ ǎǎ ǎǎ4-ǎ PKǎ& ǎ "Eƕǎ& "4 ǎ §ǎPǎ5ǎ*4ô ǎ ǎ ' Aǎ Aǎ *4ǎKǎ&4 ǎ'ǎ ' bǎ##Eǎ¹ ­Ɛ *ǎ *ǎ *j  4ǎYš ##Eǎ  ǎ ǎǎ4/ǎ ǎǎ- - - ǎǎE—› Ķ ǎ ǎ İǎPǎ5ǎ*ǎ4ǎ!ǎ ǎǎǎǎǎ ƒšAǎPKǎ¢ ǎA "E4Kǎ! "4ǎ *4Kǎ!ǎ4ǎbǎ##Eǎǎǎǎ F¹¹ǎǎ7ǎ¹ǎ *ǎ *ǎ  45ǎ ǎǎǎ - - - ǎǎE-ǎ U ǎǎǎ ǎ ǎ ǎǎ





-ǎ

'lǎ+ǎ ǎ " 2!6ǎ ! 2ǎ 6ǎ

 8 

6ǎ "2ǎ 2ǎ 6ǎ



8



8L

6Kǎ! "2Kǎ 6Kǎ! "2Kǎ -ǎ>ǎǎ ¨ǎPǎ 26ǎ 26ǎ 

 ǎ "2Kǎ&P62!ǎ6 Kǎ  - ǎ>ǎ ǎǎǎ7ǎ ǎ  ǎǎPǎ ǎ6ǎ 8L

PK6KǎÔ #6KǎÃǎ - ǎ>ǎǎ ǎPKǎ& #Ãǎ  ǎǎP Ã ǎ" - ǎ

(8+8!ǎ8 %ǎ!Pǎ ǎÃǎEǎU ǎǎ›ǎ  ǎǎE-ǎ

>ǎ2ǎ!6ǎ ǎ





'\ǎ  F ǎǎ< 7ǎ  "›& ǎǎ' ǎǎǎ !  ǎ AU Û U!  ' ıǎ ǎ Ô  U 'ǎ ' ǎǎǎ ǎ ǎbǎ#›bǎ ǎǎ ǎ ' T¹  -

-

'Ąǎ % ǎ IKǎAǎ!ǎIĘ!ǎI Kǎ& Ĩę ºǎ!ǎIºǎ Úǎ!ǎIKǎ¢ Ĩ!ǎ %Ę%&%ęǎAǎ%ǎ 8L



'lǎ % Åǎǎ ǎ  åǎǎǎLǎ/ǎ%-

"

'lǎ # 37ǎ ǎ<T7ǎǎbǎ#ǎ!ǎ*ǎ!ǎǎ/ǎ# -ǎF ǎ ǎǎǎ  ǎǎǎ ǎ ǎ  ǎǎ ǎ#-

%-

'\ǎ % iǎ 

ǎ ǎ ǎ ǎ!ǎ ǎ & bǎ&  ǎ ǎ

 Yš



Ne¹

)@{ 



8{

){

){

ºǎǎ

8{

X  X

8{

 ºǎǎXš8{ )@{ B{ 89,+ X q X  š *  Xš

'ǎ){Xš89,*A{Xš  ǎǎǎǎ ǎ  ǎǎǎ ǎ < 7ǎǎǎ/ǎ 8{    ǎǎ#ǎU ǎǎǎ%ǎǎBǎ #

'\ǎ E _ǎ/ǎ•ƯĜǎǎ åǎWǎ 7  ǎǎ /ǎ"ú #ǎ

ǎ5ǎ•ưĜǎǎ åǎWǎ 7  ǎǎǎ ǎ5ǎ ú "B ƒƒ_ǎ  > ŲųǎB ǎ ǎ ǎǎ ǎLǎ/ǎEBǎ Eǎ ǎ 5L

)@!

Z\ǎ # iǎ  ŘĀǎ ǎtǎ!ǎ Śǎ ǎ"tǎ! ǎ‹¹ ǎt Aǎ ş ǎ³ǎ´ ǎ"tǎ´ Ǎǎǂ

 0 8 ŤŨ 8 & *  tǎ!0  "tǎ  ³ 8 řĀ & * ǎ³ǎ!'  ǎ#tǎ! ǎt8

A ŜƂŧ  tǎō   tǎ!  "³ 8L

5L

A ǎ‹¹ \]›AC Aǎ&# ǎ  ƄǎK ǎ Kǎ U ǎǎǎ ǎ ǎ&#KǎAǎ#ǎ B

'\ǎ 8{# >ǎ7ǎǎ ǎǎ   ǎǎǎǎBǎXǎǎǎǎ7ǎ   ǎǎǎ ǎ ǎ ǎ#& ) {/ǎ ǎXǎǎ ǎǎǎǎ

 ǎ ǎǎ  ǎǎ8{ ǎǎǎǎ  ǎ ǎ>ǎ ǎǎ7ǎǎ  ǎǎǎ  ǎ ǎ ǎ8{ ǎ5ǎ# 

E

'\ǎ ) 37ǎǎ  ǎ  ǎǎ  ǎ)+##{!ǎ& 30##{ AǎKǎ& ǎ!ǎ ǎļě!ǎ6  ǎǎ ǎǎ< ǎ ǎ2(ǎ6ǎ ǎǎ  ǎ ǎ ǎ2ǎ ǎ6-ǎ XǎǎKǎµ ǎ(ǎǎAǎǎ& &ǎ>ǎ ǎ)+##{!ǎ& 32##{ Aǎǎ& &ǎ! ě!ǎ6

Ih¹

•ž8ǎǎ\š ǎǎj8ǎ  ð V¸¸ǎA2(ǎ6ǎ ǎ2(ǎ6ǎ\š •ž8ǎǎ\š ǎǎj8ǎ ǎ&  ţ¸¸ǎAǎ2ǎ(ǎ6ǎ ǎ2ǎ(ǎ6ǎ\š · F 8ǎǎǎ<ž8,ǎ8 ,ǎ2ǎ\š ǎ ǎ6ǎ\š -ǎ 

',\ǎ ; hš

-

G8,ǎǎǎ,8 ǎī1'+ǎ ǎī13+ǎǎ8,,  ,ǎ8 ,ǎǎ ǎ m1'+ǎ m1+'ǎ ǎm13+ǎ/ǎm1+3-ǎXǎj7ǎ ,8 8ǎǎ,žǎǎ ,ǎ8ǎ ë+   ǎ m1'+ǎ(ǎm13+ǎ(ǎm1+'ǎ(ǎm1+3ǎ/ Īǎ m1+'ǎ(ǎm1+3ǎ\š Ī-ǎ >ǎ©'+3ǎ\š ©1+'ǎ(ǎ©1+3ǎ\š ;ǁǎ  

',Ùǎ  F8 ǎǎzǎ ǎǎǎ,88 ǎǎ  *ǎ(ǎzǎ(ǎǎbǎ*ǎ(ǎzǎ(ǎ;ǎbǎ;ǎ(ǎzǎ(ǎ êǎ >ǎ8,ǎ *ǎ(ǎzǎ(ǎǎbǎ#ǎbǎ;ǎšzǎ( — %š >ǎ8ǎ ,ǎ ( z( b ǎ -"ǎ  ǎ(z(Âǎ -%-ǎ F8 ǎǎzǎ ǎǎǎ8,ǎǎj8ǎǎ(ǎzǎ(ǎǎ\š -ǎU ǎ Ɗǎ(ǎzǎ(ǎǎ\š -

-

Z,Ùǎ iǎ,ǎǎǎ 7ǎ8ǎë+ǎ8,ǎ ž|  ǎƸ pǎ´ǎĚ Oǎ Üǎ ǎ ǎp (ǎ Oǎ(ǎÜǎ iǎ ǎ ĚƮǎp(ǎ Oǎ(ǎÜǎ Aǎp(ǎ Oǎ(ǎ ³& p(ǎOǎ Úǎp(ǎ OC pǎ( Oǎ ǎp(ǎ Oǎ Úǎpǎ( ǎOǎ C p  Oǎ ǎ Aǎp(ǎ O & ñ ǎp´ǎ Oǎ GL

-



Im¹

8 8

8 ,8 8

1pId5pId9… pIg5pId9 pId5… pId9… …pId…5…€pId…9 pId5pId9… pId5pId9… 2pId…5…pIg9……pId*z5…9… 2pId…6…pId9……pId:… nh…pVO…lOnq_p…Rh__hsn… 5L

#

7ensOl,…* 







™¹

HO…VIrO…

m'w9ǎ[š *{){…\š |… m9+1ǎ IdN… m'9wǎ *}.mw'9ǎ…~… m+19 EVOlORhlO…Ē'w9ǎWn…nWbW_Il…ph…Ē9ż1ǎIdN…Wp…Rh__hsn…pVIp…'wg w9ǎ\š 9+g+1=OdMO…w+@ǎ[š.w9.9+ǎ\š.'w.+1ǎ\š )&…Mb! …nh…w+ǎ[š*…Pb @L

@L

&

@L

7ensOl-… ' CWdMO…ǎ…w… * …DžIǎVIn…p‚O…bItWbqb…rI_qO…WR…IdN…hd_x…WR *…1…&…BhpO…pVIp…v…VIn…pVO…bItWbqb…rI_qO…WR…IdN…hd_x…WR…v ǎ1… …w…1… … NhOn…EVqn…Cw…\š &…'…\š'… @L

'

7fnsOl.…& FnWdU…pVO…WNOdpWpx…J$…K… $…2J …K J  JK……K …sWpV…J1…n\ …5…IdN K…2…Mhn… …5 ……sO…VIrO nXf…(…5…Mhn(…53…nWd …5…Mhn …5nXd …5…Mhn …5 #nXf …5Mhn …5… 3……#…nWd… …5M… hn… …5…2…ƒnXd… …5 &… 2… Mhn …5…1…  2…

   8 /,8



>OdMO…) nXf…(…5…) Mhn(…5… &… @L

Iq¹

#

*-

Z\ǎ % Xǎǎ ǎ ǎ   7ǎǎ ǎ ǎǎLǎǎ ǎ ǎǎǎǎ±ǎ  L  ǎǎŽ¹ +< 7ǎL  ǎŇ ǎǎǎ ǎ ǎǎŽ¹ >ǎ ǎ Lǎ Aǎ"-ǎ F ǎǎ< ǎ ǎLǎ % ÷ǎ  ǎǎ< ǎ Lǎ ǎ

 - --"- $%%--

@L

Z\ǎ  Ė ’ǎ

EL









ǎ ǎ ǎ C A  ǎ ǎ JK› U ǎ Ėǎ5ǎ ǎ C









Zgǎ *% iǎ Sǎǎǎ ǎ  ǀ ǎǎ %ǎ;%ǎ ǎ *%ǎiǎ  %ǎ[š @ǎ %Øǎ ;%ǎ @ǎ %@ǎ Ɓǎ *%ǎ5ǎǎ %@ǎ F  ǎ [+Ý@ǎ %Øǎ@ǎ %@ǎ ǎR{5ǎ[+Ýǎ %@ǎ ǎ@ǎ %@ǎ ǎR{ ǎǎǎǎ ǎ  ǎ  ǎǎLǎ ǎ%Øǎ ǎ *% 









@L









;

Zlǎ 









@L





Ɣǎ

+ ǎ”1‡ǎ ǎĠ+1‡ǎF  ǎ+‡ǎ Z‡ǎǎǎǎ ǎǎĠ+1‡ǎ[š ǎÎǎǎǎÞ1‡ǎ ǎ @ F   7ǎǎ ǎ ǎ ǎǎƈ+1ǎ5ǎ& Îǎǎǎ”+1-ǎ ǎ Xǎǎǎ”+1ǎ5 ǎǎäZ1‡ǎĻǎǎä+1‡ǎ5ǎ"ǎ @ǎ "ǎ $ ǎǎä3+1ǎ[š Yǎ ǎ "ǎ >ǎǎ ǎǎ< ǎǎ A% ǎ ǎ ǎǎ ǎ ǎ @L

@L





@L

Iv¹

B

Z\ǎ #

[ǎǎ ǎǎǎ  ǎ ǎ ǎ'ǎ ǎ3ǎǎǎYǎ IǎYǎ   7ǎ  ǎ ǎ+ǎǎǎ ǎǎǎ nǎǎ3—ǎ ǎǎ'+ǎ ǎ  ǎǎ«—ǎ >ǎ'+ǎ\š "ǎYǎ ǎ3+ǎ\š  ǎYBǎF  ǎë+ǎ ǎǎ |  ǎ  ǎ  ǎ Ił Vǎ!ǎ" Vǎ " Vǎ 7ǎ•7ǎħǎǎ IC5;-ǎ "ǎ ǎ!I ǎ  ǎ ǎ ǎ Xǎ  ǎǎê ǎ'3—«ǎiǎ ǎ ǎ ǎ Ƒ!ǎIǎ\š %Bǎ F ǎǎǎ  ǎ< ƌǎǎ ǎǎ\š ǎ I/ǎ BǎU ǎ

-



nj #B  f

'\ǎ *

8 8 8 -ǎn8!ǎ ǎ 8!ǎ%ǎn8!ǎ ǎ !ǎ 8 -ǎn8!ǎ ǎ 8!ǎ%ǎƜË8!ǎ ǎ 8 ǎ !ǎ%ǎ ǎ!ǎ )nǎ - ǎŦœǎöǎ 8!=› 8 %,› 8!< 8B ü Vǎ % ǎ U ǎǎs ǎ  ǎ ǎ  !ǎ + -C & *B   ǎn ! ǎ ǎ !%ǎ ǎn ¬!ǎ ¹

‚

‚

B

Z\ǎ *%% iǎǎǎ ǎ ;Ņ  ǎ Jǎ< ǎ   ǎs  7ǎ ǎ ƒŸǎ

  ǎ  ǎŸǎ ǎǎ  ǎ <ƨ7ǎǎ ǎǎ |  ǎ  ǎ ǎ ǎ  nǎ7\ǎ9 ǎ  Ÿǎ ǎǎ   ǎ7ǎǎ  ǎÈǎ ǎǎ;ǎ ǎǎ ǎǎǎǎ Ÿǎǎǎǎ ™ǎǎǎ  ħǎ  Ÿǎ  ǎǎ  ǎǎ

 BǎGǎs ǎ    ǎ ǎ  ǎǎ*;;;;;;;ǎ

W-¹

'

ǎ ǎ

!-

ǎ  ǎǎ ' Š¹  ǎǎǎ< 

Aǎ^DDB



ZËgǎ D ' ' š ' ?ǎ' ǎǎ  ǎǎǎ G ǎǎǎǎ ǎ  ' '  ǎ ' š ' ǎ  -ǎF  ǎ ' ǎǎ  ǎ ǎ ǎ  ǎ š ' ǎ Bǎ¨ǎ ' Aǎ4ǎ  ǎ š '  ǎ  C o› F< ǎǎ ǎǎǎ< ǎǎ ǎ  ' 5ǎ4@ǎ ǎ ' š' ^š ž F² > ǎ   ĥĥ5ǎŽ.j› C Žy Doy ›   š Xǎǎƒš ǎǎǎ ǎ  ǎ ǎ ǎ š ' ǎ '  ' '  ǎǎǎ ǎǎ  ǎiǎÍ ǎ   ǎ ž¹ 4ǎAǎ ' š ž  4ǎAǎ '   ǎ ǎ  ǎǎ ¹  ǎ4ǎ ǎ| ǎǎ< ǎ. ǎǎ>ǎ  ǎ ž¹ 4ǎAǎ  ǎ ǎ ǎ ǎǎ< ǎǎ ' ǎ ž¹š4ǎ C›  ' U ǎ ž¹ ’ǎD F¹ 0{ ' Aǎk  ?ǎ ' AǎD B  ǎ4ǎuǎkEǎ ǎ ' ’ǎ4 š

k

Zgǎ # iǎ`dǎ`d ǎǎ ǎǎ   ǎƤǎǎhǎǎǎ ǎǎŽ +  Jǎ' ǎǎ ǎǎŽ¹ Xǎǎ ǎǎ ǎ4ǎ ǎǎǎ ǎǎLǎǎ4ǎ  ǎǎ ǎǎL  ǎ>ǎǎǎ¹4ǎ¹'   ǎ4ǎ ǎ hǎ4ǎǎǎ  ǎǎLǎǎ ǎǎ ǎǎǎ ǎǎǎǎhǎǎ

  ǎǎ   ǎ4ǎ ǎ4ǎǎǎǎ ǎǎŽ¹ U ǎǎǎ 

' ǎ ǎǎŽ¹ >ǎ ǎ ľǎǎ ǎ5ǎǎ kǎ k  ^ǎ Dǎ^š ǎ Dǎ ' ^š ǎ '' 5ǎkǎ Dǎǎ ǎ ǎǎŽ¹ +< Jǎ  ǎǎ±ǎǎǎh ǎǎǎ ǎǎLǎǎ   ǎǎ   ǎǎ' ǎ ǎ' ǎ  ǎLǎǎ ǎ>ǎǎ   ǎǎ #ǎ ǎ ^ǎǎǎ Jǎ±ǎ    ǎǎhǎǎǎǎ ǎǎŽ¹ U  Lǎ5ǎ×ǎ @ǎ D ^ ' ǎ ' uǎ#k#k ǎǎǎǎ  ǎǎLǎ ^š ǎ #ǎš ǎ(ǎkǎ^š #B 



?L























D

ZËgǎ ^#k ¨ǎ5ǎ"' ǎ¤ǎśǎǎ^kǎ^D€ǎǎǎǎǎ ǎÈǎ' ǎ^D-ǎiǎ`dǎ`d ǎ ǎǎ ǎ ǎǎǎ   Jǎ ǎǎ^Dǎ5ǎǎ D@ǎ [ o5ǎ" ' lǎ ǎǎ ǎǎ #'  ǎOǎ^š "'  ' ' ǎăǎDǎ ǎǎ ǎǎ # ' >ǎ oéǎ Oǎ ǎǎǎ ǎ ǎǎǎǎǎ   Jǎ ǎǎ^DBǎXǎǎ®oǎ W5ǎDǎWOžǎ uǎ Dǎ

 š O 5ǎ Wǎ " '  ' ' ǎ l ǎ Dǎ ǎǎ ǎǎ $' Wǎ 5ǎ D Bǎ3Jǎǎ   ǎǎ   ņs   oǎ ®ǎ æ đ W|WoǎƭOWǎ^š DBǎ>ǎǎƬǎǎ ǎ ǎǎ WǎoǎéOWuǎWođǎš O O®ǎ5ǎ^DC Dǎ kǎ>    Jǎ ǎǎ^Dǎ ǎWƏǎ ǎ CWǎoéǎ ' ' ^.›Aǎ' Akǎ ' ’^ǎ' ' ' '   ' A^k Bǎ XǎJǎ  ǎ  ǎJǎ| ǎ ǎ ǎǎ ǎ < Jǎ ǎ ǎǎ^D4ǎš °ǎǎǎ ǎ4ǎ ǎ°ǎǎ4ǎT¹'  ǎ' €¹ °ǎ¹ ^kBǎXǎ  ^D4ǎš °ǎ^Dǎ5ǎ ǎ ǎ ǎ Jǎ ǎ °ǎ^Dǎ^š ' >ǎǎ< ǎ"!#!&' 

?L

W6¹

¬ǎǎǎ›ǎƷǎǎ $\š *%4ǎ(ǎ$ ǎ4ǎS¹ ǎ ǎ ǎ%&M5%&M"ǎ§  ǎ4ǎ  ǎ7ǎ4ǎ ǎ 4ǎC›

7L

&8

 ǎ5M ǎǎ ǎǎ ' ǎ    ǎ7ǎ"Bǎ>žǎǎ

"8 %ǎ ǎ ^š

5M \š BǎU¬

  $\š *%ǎ % (ǎ!$ *%ǎ(ǎ "ǎ

@L

WI¹

*#"ǎ

' ^š

Xw‚qaˆ…‹l› Sa‘tla‘wfa{› X…fwl‘”›

$&%'2(#(!2!*#& 2 2 6&&G   ( "( &( ( "& '( ( &(  (

  (

!" &"!( "( "!""!( 5š

2 #'!)  ) Fš &#$#)

;&š ') ) $) #$#) ) (&!) '! ?'š ) &#$) !!#) 50š !

) ) &$!#) !) ')

%

WWǎ QS%Ž HzppntS‚ _Ž OkR‚ aŽ OrS‚ rStpSQuZ{Sf‚ pnZkut

@Su‚ PQSTŽ PS‚ O‚ urOpS€Zzj‚ |ZuW‚ PTŽ

nk‚ uWS‚ tZRSt‚ PQŽOkR‚ STŽtzQW‚ uWOu‚ bQPaŽZš cST_/Ž Crn{S‚ uWOu‚ cQaPŽZš cS_T&

D

2SuSrjZkS‚ Off‚ prZjSt‚ ¥¹ tzQW‚ uWOu

Zt‚ O‚ pSrTSQu‚ tqzOrS‚ Z!S‚ uWS‚ tqzOrS‚ nT‚ Ok‚ ZkuSVSr‚



6ZkR‚ Off‚ TzkQuZnkt‚ tŽ  £¹  £¹ tn‚ uWOu‚ Z‚

t9ŠŽ

;L

tŠŽ Ž‹t‹ ŠŽ'tŠ ‹tŽŠ ‹Ž Tnr‚ Off‚ Š Ž‹ŽUz Ž OkR

ZZ‚ tŠŽ=Ž 5Ž Tnr‚ Off‚ ŠŽV{|Ž

_% IWSrS‚ OrS‚ %%‚ QnjjZuuSSt‚ Zk‚ O‚ QfzP!‚ 5OQW‚ QnjjZuuSS‚ WOt‚

M jSjPSrt‚ OkR‚

S{Sr‚ u|n

QnjjZuuSSt‚ WO{S‚ O‚ jSjPSr‚ Zk‚ Qnjjnk‚ HWn|‚ uWOu‚ uWSrS‚ Zt‚ O‚ jSjPSr‚ |Wn‚ PSfnkVt‚ un‚ _ QnjjZuuSSt!



=Su‚ u š oŽ{š R¹ 5+Ž Crn{S‚ uWOu

WW¹

Xw‚qaˆ…‹l› Sa‘tla‘wfa{› X…fwl‘”›

$&%'2(#(!2!*#&2 2 6&&G ( $( %( ( %$!(

PQa`Ž `t‚ QQf`Q‚ IWzt‚ cPTaŽ cP`aŽ Zš 6E5Œ &Ž IWSrSTnrS‚ cQ`aŽ ŽbQSaŽ Zš 6E5Œ bP`ŽaŽ Ž6E5Œ bPTaŽ' 6E5ŒŽ tn‚ wWOw QSa`Ž `t‚ Oftn‚ QQf`Q"‚ 8SkQS‚ cPQaŽ Zš bTS` OkR‚ cQaPŽ Zš 6E5ŒŽ cPQaŽ bQPaŽ ' 6E5ŒŽ cTS` cST`Ž' dS`T.

%‚ 9wt‚ QfSOr‚ wWOw‚

D!

=Sw‚ =T{ > ŽŽ

FL

„?Ž Zš †:&Ž IWSk

IWzt‚ †

9„:ŽZš =D{

9Ž „:Ž

†š

=r{

4L

|WSrS‚ 5Ž ~¹ ‰š Wš Œš OkR‚ ‰š ŽŒš ' „

5Ž wWSk‚ ={Wǎ>„:‚ IWzt‚ „Ž *‚ ={ OkR‚ wWS VZ{Sk‚ S~prStt`nk‚ `t‚ `kRSSR‚ O‚ tqzOrS‚ :T‚ ‹ Zš 5Ž wWSk‚ Œš Zš „Ž OkR‚ |S‚ WO{S‚ =T{ Zš >„:Ž Ž6$ MS‚ tWOff‚ prn{S‚ P‚ `kRzQw`nk‚ wWOw‚ =e{ dš >x:Ž Ž6Ž Tnr‚ S{Sr‚ `kwSVSr‚ xŽ‚ 9+Ž IWS‚ `kSqzOf`w QSrwO`kf‚ WnfRt‚ Tnre*‚ 9'Ž Hn‚ |S‚ OttzjS‚ wWOw‚ `w‚ WnfRt‚ Tnr‚ tnjS‚ xŽ ´¹ 9&Ž BnwS‚ wWOw 3f`j`kOw`kV‚ †š |S‚ VSw‚ >„:Ž Zš =D=so

6&Ž

:T‚ ‰š dš

>Ž >xŽ 6Ž :Ž  6Ž >x:Ž  6Ž ExŽ ' Wš 4š # Ž6Ž 6Ž Ž Wš ={ Ž Ž >x:Ž 6Ž Ž >x:Ž 6Ž Ž >x:Ž 6Ž Ž Ž6Ž IWzt‚

Tnr‚ xŽ‚ 9%

=e f{ Zš ={=e{ dš A>x:Ž 6Ž Ždš >xŽ 6Ž :Ž 6Ž )

 tŽ at‚ S`wWSr‚ QnktwOkwf‚ 5Ž nr‚ QnktwOkwf‚ 639( /fSOrf‚ S`wWSr‚ nT‚ wWS‚ QnktwOkw‚ TzkQw`nmt‚ OPn{S‚ tOw`tU`St‚ wWS‚ rSqz`rSjSkwt"‚ /nk{SrtSf tzppntS‚ wWS‚ V`{Sk‚ QnkR`w`nkt‚ WnfR‚ HSwx`kV‚ ŠŽ ' ‹Ž |S‚ WO{S‚

t9ŠŽ /OtS‚ %‚

>L

t;ŠtiŽ'titŽ 9Š)Ž

tiŽ Zš 5*

IWSk‚ t9ŠŽ Zš 5Ž Tnr‚ Off‚ ŠŽ kš ¤¹ OkR‚ WSkQS‚ tŽ `t‚ QnktwOkwf‚ 5+

/OtS‚ D"¹

tiŽ' {š K{ 5&

IWSk‚

tŽ!9ŠŽ '

6 {š t9ŠŽ zš

ŠŽ Zš 5Ž |S‚ WO{S‚ `k‚ pOrw`QzfOr‚ {š tŽŠŽZš tŠŽ Tnr‚ Off‚ ŠŽ kš ’¹ IWSrSTnrS

HSww`kV‚

Tnr‚ Off‚ ŠŽ kš ’¹





6Ž

{$š

8SkQS‚ {š Zš

649%Ž

9w‚ Tnffn|t‚ wWOw

t9ŠŽZš tŠŽ Ž‹]Š ‹Ž t Ž Š ‹tŠŽ Ž‹Ž*‚ 9tŠŽ'‹tŠ HSww`kV‚ ŠŽ Zš 5Ž |S‚ WO{S‚ 649Ž ' tiŽ Zš 9t‹t

‹Ž kš ¤¹ t`kQS‚ t‹Ž ³¹ 5&

Y`¹

‹Ž*‚ ;t‹:&Ž

IWzt‚ t‹Ž

‹% >L

649Ž Tnr‚ Oif

_$

6nrj‚ Ok‚ ZkQZRSkQS‚ jOurZ~‚

PŽ |WSrS‚ uWS‚ rn|t‚ OrS‚

ZkRS~SR‚ P‚ uWS‚ QnjjZuuSSt‚ OkR‚ uWS

Qnfyjkt‚ OrS‚ ZkRS~SR‚ P‚ uWS‚ jSjPSrt‚ OkR‚ |WSrS‚ uWS‚ [ b‚ Skur ‚ !$' Zš ' ZT‚ jSjPSr‚ c‚ Zt Zk‚ QnjjZuuSS‚ [‚ IWSk‚ uWSrS‚ OrS‚ UZ{S‚ %t‚ Zk‚ SOQW‚ rn|‚ OkR‚ Tnr‚ SOQW‚ [ ‚c ‚ uWSrS‚ S~Ztut‚ xŽ tyQW uWOu‚ !%' Zš $%'

3L

' IWyt‚ uWSrS‚ OrS‚ UZTu‚ UZ{S‚ %t‚ Zk‚ P+Ž LS‚ kSSR‚ un‚ orn{S‚ uWOu‚ uWSrS‚ Zt ' {{=

Qnfyjk‚ |ZuW‚ Tnyr‚ %t‚ LZuWnyu‚ fntt‚ nT‚ VSkSrOfZu‚ OttyjS‚ uWOu‚ "' Zš ' Tnr‚  ¹

9L

/nktZRSr‚ uWS‚ tyPjOurZ~‚ .‚ TnrjSR‚ P‚ uWS‚ UZstu‚ UZ{S‚ Qnfyjkt‚ OkR‚ uWS‚ fOtu‚ uSk‚ rn|t‚ 3OQW‚ rn|‚ nT‚ .‚ WOt‚ Ou‚ fSOtu‚ nkS‚ %‚ 8SkQS‚ .‚ WOt‚ uSk‚ %t‚ 9T‚ uWSrS‚ Zt‚ O‚ Qnfyjk‚ |ZuW‚ uWrSS‚ gt‚ uWSk‚ PŽ WOt‚ O‚ Qnfyjk‚ |ZuW‚ Tnyr‚ %t‚ OkR‚ |S‚ OrS‚ RnkS‚ 9T‚ lnu‚ uWSk‚ S{Sr‚ Qnfyjk‚ WOt‚ u|n‚ %t‚ OkR‚ uWyt‚ SOQW‚ nT‚ uWS‚ QnrrStonkRZkV‚ Qnfyjkt‚ Zk‚ PŽ WOt‚ uWrSS‚ %t‚ -‚ QnktZRSr‚ Off‚ uWS‚ nuWSr‚ rn|t‚ |S‚ tSS‚ uWOu‚ ZT‚ kn‚ Qnfyjk‚ WOt‚ Tnyr‚ %t ‚ uWSk‚ S{Sr‚ Qnfyjk‚ jytu‚ WO{S‚ S~OQuf vWrSS‚ %t‚ -yu‚ uWZt‚ Zt‚ ZjonttZPfS‚ Ot‚ uWSrS‚ OrS‚ UZTu‚ UZ{S‚ %t‚ Zk‚ PŽ Pyu‚ 



n›==

6Zrtu ‚ Tnr‚ Ok‚ rSOf‚ kyjPSr‚ &' |S‚ WO{S

|ZuW‚ SqyOfZu‚ ZT‚ OkR‚ nkf‚ ZT‚ &' Zš ' 9k‚ oOruZQyfOr‚ Tnr‚ knkkSVOuZ{S‚ &'   ' '&'   cš ''&' '&' 

|ZuW‚ SqyOfZu‚ ZT‚ OkR‚ nkf‚ ZT‚ &' Zš ' JWyt

''''' '''' cš ' '''#''' 'c{'''#   Zš ''' ' '''' '' ''  '' # R¹ '''''# =Ž ' ' 3qyOf\u‚ WnfRt‚ ZT‚ OkR‚ nkf‚ ZT‚ 

 ' ''' Zš '

Wd¹

 +   + +

    

966
 $ !$ "$   #$ $ #$  $

  $

 !"! $ !$ ! !! $ (!… .#…

#+". .

;TÂ !(#'#.

"'. ,(". $+"#. . '. $+". #'. "*. . # . '.

" " '. (

$. +. ,(".

$+"%. 0$…

. #' #. ". . '.(#',. ,(". &+"#.



 . !-)#'.  ""#. (…  ".

T&

.  ( '"$. ". +.

5%

v¤Ø¤ÔŹȤé Ø®¤é ÈÛŜ¤Ôé ͧé دԤ¤¤¿¤Å¤ÈØé ×ۜפØ×é ͧé Ø®¤éפØé "ıˆı ˆı#ıTTwı#ı -ı §ÍÔé ß®¹Ÿ® Ø®¤é×ÛÅéͧéØ®¤éخԤ¤é¤¿¤Å¤ÈØ×é¹×é ›éÅۿرϿ¤éͧéB*

=- ’¯¤Ô¤é ›Ô¤é 5* ×ØÛ£¤ÈØ×é ؛½¹È¬é ϛÔØé ±Èé›éśخ¤Å›Ø±Ÿ×éŸÍÅϤرعÍÈ*é k§Ø¤Ôéد¤éŸÍÅϤرعÍÈ Ø®¤ãé £±×ŸÍÞ¤Ôé د›Øé ¤›Ÿ®é ͧé Ø®¤Åé ×ÍÄÞ¤×é ¤á›ŸØ¿ãé ı ÏÔ͜¿¤Å×é ›È£é ›Èãé ı ͧé د¤Åé ×Í¿Þ¤é ›Ø ¿¤›×Øé 4 ŸÍÅÅÍÈéÏÔ͜¿¤Å$陮›Øé¹×éØ®¤éŹȱÅÛÅéÈÛŜ¤Ôéͧé×ØÛ£¤ÈØ×é߯Íé×Í¿Þ¤é›éŸÍÅÅÍÈ ÏÔ͜¿¤Åé ߯¹Ÿ®é¹×é×Í¿Þ¤£éœãé ÅÍ×Øé×ØÛ£¤ÈØ×j B1 xޛ¿Û›Ø¤éØ®¤é ×ÛÅ

0 ()-"+ 



g5b…

ß®¤Ô¤é (0?BAa £¤ÈÍؤ×éØ®¤é¬Ô¤›Ø¤×Øé¹Èؤ¬¤Ô鿤××éØ®›ÈéÍÔé¤Òéۛé‚éØÍé? E- v¤Ø¤ÔŹȤéØ®¤éÈÛŜ¤ÔéͧéÏÍ׹رޤé¹Èؤ¬¤Ô飱ޱ×ÍÔ×éͧéSSNFTTTTé Ø®›Øé›Ô¤éÈÍØéØ®¤é£¹Þ±×ÍÔ×éͧ SSNGUVVP* I$ …¤ØéÑ e #a œ¤é ›éÏÍ¿ãÈÍŹ›¿é߹خéÔ¤›¿éŸÍ¤§ª¹Ÿ¹¤ÈØ×é×۟®éØ®›Øé§ÍÔé ›¿¿éÔ¤›¿é?

ıı8!B 4… 8? Ëı  8?Bı ›È£éÑ "a 4… Né4ı

+

yı v¤Ø¤ÔŹȤéØ®¤é ޛ¿Û¤é ͧé8B .

K, ‚Èéد¤é ØÔº›È¬¿¤é 6) rel="nofollow">ı )ı 8x 6)Dı Đ)ı ( ½ı ›È£é ı ¹×éØ®¤é §ÍÍÚé ͧé Ø®¤é ›¿Ø¹ØÛ£¤é§ÔÍÅé ) ÍÈØÍé nq*é k韹ԟ¿¤é ߹خ飹›Å¤Ø¤Ôé Ăı ¹ÈؤÔפŸØ×éØ®¤éפ¬Å¤ÈØé )ı›Øé wı y¹È£é  ªı )w +

L( ‚Èé Ø®¤é ØÔ¹›È¬¿¤é 6)Dı 6ı 8x Né6)ı :ı ›È£é )ı 8x ı …¤Øé ı œ¤é Ø®¤é ÏͱÈØé ÍÈé 6)ı ×۟® Ø®›Øé ²6ı đ)yı y±È£é Ąå. +

+

O* ‚ÈéØ®¤éØÔ±›È¬¿¤é 6)Dı Ø®¤éœ±×¤ŸØÍÔ×éͧé Ēı ›È£é ² ı Ť¤Øé›ØéØ®¤é±ÈŸ¤ÈØÔ¤é*>ı Ø®¤é¤áؤÈ×¹ÍÈéͧ *ıŤ¤Ø×éØ®¤éŸ±ÔŸÛş±ÔŸ¿¤éͧéØÔ±›È¬¿¤é 6)ı›Øé áı …¤Øé Zı œ¤éØ®¤é§ÍÍØéͧéØ®¤éϤÔϤȣ¹ŸÛ¿›Ô §ÔÍÅé ı ÍÈØÍé >ı ›È£é 8ı›éÏ͹ÈØéÍÈéØ®¤é¤áؤÈ×¹ÍÈéͧé ı×۟®éØ®›Øé*ı ı 8âı v¤Ø¤ÔŹȤ Ø®¤éޛ¿Û¤éͧé 68ı … *ı  Zı … *ı —. +

NUÂ

R* ‚Èé›é ŸÍÈÞ¤áé Òۛ¡Ô°¿›Ø¤Õ›¿é 6) Dı ³6)ı ē) Dı ³6)ı Ĕ) >ı د¤é ¤áؤÈ×°ÍÈ× Í¥é  ı ›È¡é 6)ı Ť¤Øé ›Øé >ı ›È¡é د¤é ¤áؤÈ×°ÍÈ×é Í¥é 6ı ›È¡é )ı Ť¤Øé ›Øé n$ v¤Ø¤ÔŰȤ د¤éޛ¿Û¤éÍ¥ 6s ı T 6)s) +

+

54, yÍÔé ›Èãé ÏÍ×9Ø:Þ¤é ±Èؤ¬¤Õé Cı ¿¤Øé Àz¯Â œ¤é د¤é פØé Í¥é °Èؤ¬¤Õ×é ¥ÕÍÅé ı ØÍé Cı ±*¤* é {¯Â 8x ¼#ı5ı ‡ı sı5ıı…¤ı ŒÍßé›××ÛŤéد›Øé+0 B  ı v¤Ø¤ÔűȤéد¤éśá°ÅÛÅéޛÀÛ¤éÍ¥é ı×۟¯éد›Ø د¤é¥ÍÀ¿Í߰Ȭé°È¤ÒۛÀ°Øã -B ax Fı ¹ı óôı  ı )+ (+

R UB=… RU…

¯Í¿¡×é¥ÍÕé ¤›Ÿ¯é ı Á {¯Â ߱دé ĉĊı B  hı 55) ~Íßé śÈãé ¥ÍÛÕ#¡°¬°Øé ÈÛŜ¤Ô×é ¬Õ¤›Ø¤Ôé د›Èé H444é Ÿ›Èé œ¤é¥ÍÕŤ¡é ¥ÔÍÅé د¤é ¡°¬°Ø×é 4é 5é <

Bé Eé Hé K é Lé O é Ré°¥éÍÈ¿ãéد¤é¡°¬°ØéEéśã霤éդϤ›Ø¤¡j 5<$ “¯Ô¤¤é¬°ÔÀ×éké oé›È¡é té ›È¡éÈ°È¤é œÍã×é›Ô¤é ØÍ霤鿰Ȥ¡éÛÏé°Èé›éÔÍß,é …¤Øé ı œ¤éد¤éÈÛŜ¤Ô Í¥éߛã×éد°×韛È霤é¡ÍȤ鰥é oéÇÛ×Øé¿°¤éœ¤Øߤ¤Èéké›È¡é té ›È¡éké oéÅÛ×Ø霤éפϛ՛ؤ¡éœã ¤á›ŸØ¿ãé Eé œÍã×,é v¤Ø¤ÔŰȤé Ģã:Âċ^ 5B$ v¤Ø¤ÕŰȤé د¤é ÈÛŜ¤Õé Í¥é D#¤¿¤Å¤ÈØé ×ۜפØ×é ¼Rı 5ıÕı…ı Í¥é "‡ı5ı ˆıD  é sı5ı …ı ×۟¯é د›Ø ı  ı ıııŽÂ °×é¡°Þ°×±œ¿¤éœãé . )+ )+

5E* y°È¡é¯Íßé śÈãé دԤ¤é¡°¬°ØéÈÛǜ¤Õ×é ¿ã°È¬é œ¤Øߤ¤Èé 544é›È¡éRRRé °ÈŸ¿Û×°Þ¤ é ¯›Þ¤éØßÍé ›È¡ ÍÈÀãé ØßÍéŸÍÈפŸÛذޤ顰¬°Ø×é°¡¤ÈØ°Ÿ›¿* 5H, y°È¡éد¤éśá°ÅÛÅéțØÛ՛¿éÈÛŜ¤Õé߯±Ÿ¯é›Ô¤é¡°Þ°×±œ¿¤éœãéB4é›È¡é¯›Þ¤é¤á›ŸØ¿ãé B4é¡°¥¨¤Ô¤ÈØ ÏÍ×°Ø°Þ¤é ¡±Þ±×ÍÕ×* 5K, v¤Ø¤ÕŰȤéد¤éÈÛŜ¤ÕéÍ¥é×é›ØéÚ¯¤é¤È¡éÍ¥éد¤éޛ¿Û¤éÍ¥éد¤éÏÔ۟͡Øéı… ‹ı… ı… D遅

)+ )+ )+

…

‹ 

5L) …¤Øé Q^… œ¤éد¤éŸÍ¤¦ª°Ÿ°¤ÈØéÍ¥é Xkƒ °Èéد¤é¤áϛÈ×°ÍÈéÍ¥é a ı ı ıA__Cı ߯¤Ô¤é ı `a "®¯ı  yı y°È¡ د¤éÈÛŜ¤ÕéÍ¥é°Èؤ¬¤Õ×é n2x ®¯ı 'x cdé SSé×۟¯éد›Øé ĪĮı 3… ī˜ģį 5O$ …¤Øé cı œ¤éد¤éŸÍ¤¦ª°Ÿ°¤ÈØéÍ¥é O cı °Èé د¤é ¤áϛÈ×°ÍÈéÍ¥ EaıııEaı—ı0ıEaıŠı!ııawoxııNıısı$EaııNN)ƒ )+

)+

v¤Ø¤ÔŰȤéد¤éޛ¿Û¤é Í¥é ĤN ·!ğh 5R, …¤Øé Öı ›ı‡ı5ı,ı œ¤é¦°Þ¤éÈÛŜ¤Ô×éכذץã°È¬éد¤é ¥Í¿¿Íß°È¬é ŸÍÈ¡°Ø±ÍÈ×Z

ı  ı ııı ı ı,ı 8x Cı ›È¡  ı ı ,ıı,ı,ı ;ı ;,ı ,ıı,ı 8x ‹ ßı y°È¡éد¤éޛ¿Û¤éÍ¥é !ı ı ñèııòéı … ­Œıı, ­Œ <4, …¤Øé A5ı™ı ( ) )x œ¤é›éפÒۤȟ¤éÍ¥é՛ذÍț¿éÈÛŜ¤Õ×é×۟¯éد›Øé A

v¤Ø¤ÔŰȤé ı … __ì9

NWÂ

+

ı ›È¡é¥ÍÔé ı B ı

=5, y°È¡éد¤éÈÛŜ¤Õéͧ餱¬¯Ø¡°¬°Øé°Èؤ¬¤Õ×éŸÍÅÏհװȬéد¤é¤°¬¯Ø顱¬°Ø×é§ÕÍÅé5éØÍéOé×۟¯éد›Ø  XııŠ !a ¡Í¤×éÈÍØé°ÅŤ¡°›Ø¤¿ãé §Í¿ÀÍßé ›Â §ÍÕ雿¿é Xı د›ØéÕÛÈ×é§ÕÍÅé5éØÍéL$ ==* …¤Øé nHtı 8x ħııAOııO ı  !àäݬ§ı ıN O Nı ߯¤Õ¤é ĨRıA5ı Rı!ı ›È¡é Nı ›Õ¤é ŸÍÈ×؛ÈØ× ß°Ø¯é Nı a %.ı š¯¤Èé ¡°Þ±¡¤¡é œãé ıv M%% Dı ı M%%?Dı ıv M%%/Cı ıv M%% ı ›È¡é ı M%%@D Õ¤×ϤŸØ°Þ¤¿ãé nHtı ¿¤›Þ¤×é›éդś±È¡¤Õéͧé M?Ñı /  D  / ı ›È¡é M?ı {±È¡éد¤éޛ¿Û¤éͧé ÁHM%%th =B$ y±È¡éد¤éÈÛŜ¤Õéͧé54¿¤ØؤÕéϤÕÅÛ؛ذÍÈ×éŸÍÅÏձױȬéEé  ×Cı Béž×é Bé  ×é×۟¯éد›ØéÈÍéØßÍ ›¡»›Ÿ¤ÈØ鿤ØؤÕ×é›Õ¤é±¡¤ÈØ°Ÿ›¿$ 



<E* …¤Øé nHtı 8x O !ıı ııŠDı ߯¤Õ¤é ı °×é›éÕ¤›¿éÈÛŜ¤Õ$é }±Þ¤Èéد›Øé د¤é °ÈÞ¤Õפé§ÛȟرÍÈé ͧ n ¤â°×Ø×é›È¡é°×鬰ޤÈéœã YaAı D a

 ıı

Dx

+

*

/ ž2éı;x

A¥!ı ı

Dx

* *

0 ıv

/ ı$;x

A¥!ı

*

߯¤Õ¤é Iı žé›È¡é ı ›Õ¤éÏÍױذޤéŸÍÈ×؛ÈØ×é «²È¡éد¤éޛ¿Û¤éͧé ıoB ıoBBı =I$ o¤Øߤ¤Èé5é ›È¡éO444é °ÈŸ¿Û×°Þ¤é «²È¡éد¤éÈÛŜ¤Õé Í§é °Èؤ¬¤Õ×é ߯±Ÿ¯é›Õ¤é¡±Þ°×°œ¿¤éœãé Ȥ°Ø¯¤Õ 5CéÈÍÕé =5éœÛØ顱ްװœ¿¤éœãé ¤°Ø¯¤ÕéEé ÍÕé K$

@Mé

 +   + +

    

966
$ !$ "$ $ "! $

5$ kÈ×ߤÕ`é M?ID7, yÍÕ·hé 7ƒ8ƒ=ƒ ¿¤Øé 3:Hhé "Oı 4H 8ƒ0 Oı0 8=7ƒ ›È¡é Oı 7x ›Â eq? ƒ …¤ı ’¯¤Èé 43:;Hhé D7&ƒ ‚§ "šıž N + °×é ›é B¤¿¤Å¤ÈØé ×ۜפØé ͧé د¤é ¬°Þ¤Èé פØé د¤Èé Bé ¡°Þ°¡¤×é ıı žéı Nx °§é ›È¡é ÍÈ¿ãé ±§ ¤â›ŸØ¿ãé ÍȤé ͧéد¤é§ÍÀ¿Í߰ȬéŸÍÈ¡°Ø°ÍÈ×é¯ÍÀ¡×[é  ° + ›À¿é ›ıžé Nx ›Õ¤é °Èé 3 @H ÍÕé °Èé 3!H ÍÕé °Èé \ rel="nofollow">  °° + ÍȤé ͧé د¤é Ïıžé Nx °×é °Èé 3 H ›ÈÍد¤Õé °Èé l7é ›È¡é د¤é د°Õ¡é ÍÈ¤é °Èé \>,ƒ ’¯¤é ÈÛŜ¤Õé ͧ B¤¿¤Å¤ÈØé×ۜפØ×éͧé 3:H °×é G H yÍÕ餛Ÿ¯éŸ¯Í°Ÿ¤éͧé ı °Èé 3 H ž°Èé 39ƒ ›È¡é Nx °Èé 3 #H ß¤é¬¤Ø ›é B¤¿¤Å¤ÈØé ×ۜפØé ×۟¯é د›Øé Bé ¡°Þ±¡¤×é ıı žéı N&x ’¯Û×é د¤é ØÍ؛¿é ÈÛŜ¤Õé ͧé B ¤¿¤Å¤ÈØ hé M?ID7.ƒ ×ۜפØ×é "šıžé N + ×۟¯éد›Øé ?ƒ ¡°Þ¸¡¤×é   ıž é ı Nx °×é¤Òۛ¿éØÍé ? () ƒ ıD7@ƒ =, kÈ×ߤÕYé F+ š°Ø¯ÍÛØé¿Í××éͧ鬤Ȥ՛¿°Øãé ߤéśãé›××ÛŤéد›Øé¤Þ¤ÕãéÏÕ͜¿¤Åé°×é×ÍÀÞ¤¡éœãé ×ÍŤé×ØÛ¡¤ÈØ( …¤Øé 3H œ¤éÍȤéͧéد¤é×ØÛ¡¤ÈØ×$é oãé›××ÛÅÏØ°ÍÈ餛Ÿ¯éͧéد¤é Mƒ Íد¤Õé×ØÛ¡¤ÈØ×é×Í¿Þ¤×é›Ø鿤›×Ø ÍȤéŸÍÅÅÍÈéÏÕ͜¿¤Åé›×é3H oãéد¤éÏ°¬¤ÍȯͿ¤éÏհȟ°Ï¿¤é د¤Õ¤é›Õ¤é›Ø鿤›×ØéBé×ØÛ¡¤ÈØ×éß¯Í ×Í¿Þ¤é ›é ŸÍÅÅÍÈéÏÕ͜¿¤Åé ߯°Ÿ¯é °×é ›¿×Íé ×Í¿Þ¤¡é œãé 3H ’¯¤Õ¤§ÍÕ¤é د¤é űȰÅÛÅé ÈÛŜ¤Õé ͧ ×ØÛ¡¤ÈØ×é ߯Íé×Í¿Þ¤é›éŸÍÅÅÍÈéÏÕ͜¿¤Åé ߯±Ÿ¯é°×é×Í¿Þ¤¡éœãé ÅÍ×Øé ×ØÛ¡¤ÈØ×é °×é›Ø鿤›×Øé D-ƒ ‚§ د¤éÅ°È°ÅÛÅé ÈÛŜ¤Õé±×é ¤â›ŸØ¿ãé Dƒ د¤Èé ¤›Ÿ¯éÏÕ͜À¤Åé±×é ×Í¿Þ¤¡é œãé ¤â›ŸØÀãé Dƒ ×ØÛ¡¤ÈØ×$é ‚§ د¤Õ¤é›Õ¤é bx ÏÕ͜À¤Å×é°Èé د¤éŸÍÅϤذذÍÈé د¤Èé Dv ?7+ oÛØéد°×é °×é›éŸÍÈØ՛¡°ŸØ°ÍÈéװȟ¤ Dƒ ¡Í¤×éÈÍØé¡°Þ°¡¤é ?7-ƒ ¤ÈŸ¤éد¤éűȰÅÛÅéÈÛŜ¤Õéͧé×ØÛ¡¤ÈØ×é߯Íé×Í¿Þ¤é›éÏÕ͜¿¤Åé߯°Ÿ¯ °×é ×Í¿Þ¤¡éœãé ÅÍ×Øé ×ØÛ¡¤ÈØ×é °×é ›Øé ¿¤›×Øé F+ƒ š¤éׯ›À¿é ׯÍßé د›Øé Fƒ °×é°È駛ŸØéد¤éÅ°È°ÅÛÅé°È د¤é§Í¿ÀÍß°È¬é ¤â›ÅÏ¿¤é ߯¤Õ¤é ߤé ßհؤé 8=? ƒ ØÍ髛Èéد›Øé›é ×ØÛ¡¤ÈØé ×Í¿Þ¤×é ÏÕ͜¿¤Å×é 8ƒ= ›È¡é ?S 8=TA ƒ 8?D ƒ 8DF ƒ 8FI ƒ 8I= ƒ =BF ƒ =DF ƒ =DI ƒ ?DI ƒ ?FI ƒ +

B* kÈ×ߤÕaé IIIIIŠÍؤéد›Øé PQUK87$Aahé 7ƒ ›È¡é PQVK8S$Bahé #0 °§é =kƒ  + Éé 3… =k9 &ƒ ‘°ÈŸ¤é I=?Jƒ hé =9>ƒı=8D8 ߤ鯛ޤé  ÃU­ É Aahé 8=ƒ §ÍÕé =9>ƒ0 + 0 I=?Jƒ ›È¡éد¤Õ¤é›Õ¤é =8D=ƒ ×۟¯é É×$é ’¯Û× > 7ƒı8=>ƒ# = ƒı==@# => ƒı   ıı88=9>$ =99 ƒı 8= =8D= ƒ 88 =9># =ƒı =>ƒ ı  g ıı=99 ƒ ı8= =8D= ƒ 88 =9># ==9 9 # 8 ƒı8= =8D= ƒ hé IIIII/ƒ =7 =99ƒ ı8= =8D= ƒı=ƒ

C, kÈ×ߤÕ[é MMMMM. ŠÍؤé د›Øé MMKƒhé =ƒ … DMMƒ ›È¡é DMMƒ °×é ›é ÏհŤ$é kÈãé ¡°Þ°×ÍÕé ͧé MMKENNNNƒ ¯›×é د¤é §ÍÕÅ ı =bDMMdƒ ߯¤Õ¤é ı ›È¡é žé›Õ¤é ÏÍ×±Ø°Þ¤é ±Èؤ¬¤Õ×é œ¤Øߤ¤Èé 7ƒ ›È¡é DMMMM&ƒ ’¯°×é ¡°Þ±×ÍÕé  ¡Í¤×éÈÍØé ¡°Þ°¡¤é MMKENNNLƒ ÍÈ¿ãé °Èé ØßÍé Ÿ›×¤×é ߯°Ÿ¯é›Õ¤é ¤°Ø¯¤Õé Ixhé DMMMMƒ ÍÕé žé hé DMMMM&ƒ ‚È د¤é «²Õ×Øé Ÿ›×¤é ı Ÿ›Èé œ¤é =ENNNN=ENNNNDMM=ENNNNDMM>ƒ0 ( (ƒ =ENNNNDMMENNNN1ƒ ‚Èé د¤é פŸÍÈ¡ Ÿ›×¤é ıŸ›È霤éDMMENNNOƒDMMENNNx=ƒDMMENNPN=>ƒ) /)ƒ ƒDMMENNNN=ENNNN-ƒ ‚È餛Ÿ¯éŸ›×¤é د¤Õ¤é›Õ¤ I4444é ÏÍ××°œ¿¤é ޛ¿Û¤×é œÛØé د¤é ÈÛŜ¤Õé =ENNNNDMMENNNNƒ ±×é ŸÍÛÈؤ¡é Øß°Ÿ¤*é ’¯Û×é د¤é ØÍ؛¿ ÈÛŜ¤ÕéͧéÕ¤ÒÛ°Õ¤¡é¡°Þ°×ÍÕ×é °×é =ƒ … F7777# 8 MMMMM&ƒ +

+

@Qé

I$ kÈ×ߤÔYé ;33N$ …¤Øé 4ı ı 'ıı ı ‘Ûœ×Ø°ØÛذȬé د°×é ±ÈØÍé د¤é ¬°Þ¤Èé ¥ÛȟرÍț¿é ¤ÒۛرÍÈé ß¤é ¬¤Ø 'Y'ıYı ı'ı+'ı $ ¥ÍÔ雿¿éÔ¤›¿é.ı ‘±ÈŸ¤é'ı±×é›éÏÍÀãÈÍÅ°›¿é ߤéÅÛ×Ø鯛ޤ 'ı+ 'Yı 5x Iı ߯¤Ô¤é ı°×é›éÔ¤›¿éŸÍÈ×؛ÈØ$é •¯¤éÏÍÀãÈÍű›¿é 8ı 7x 'ıY'+ Ÿ›ÈØé ؛½¤é د¤é כŤé ޛ¿Û¤é ›Øé ›Èé °È«²È°Ø¤é ÈÛŜ¤Ôé Í¥é ¡±×ذȟØé ÏÍ°ÈØ×é ›×é °Øé °×é ¤Þ¤ÈØۛÀ¿ã ÅÍÈÍØÍÈ°Ÿé ÛÈ¿¤××é°Øé °×é›é ŸÍÈ×؛ÈØéÏÍ¿ãÈÍÅ°›À$a Š¤âØé ¿¤Øé 'ı 6x =ıı ı ‘Ûœ×Ø°ØÛرȬ د±×é±ÈØÍé '+ '+ ıı Iı ߤ鬤Øé =ıı=+ ı ×Íéد›Øé =ı 4x Iı ߯¤Ô¤é °×é›éÔ¤›¿ ŸÍÈ×؛ÈØ$é •¯¤Ô¤¥ÍÔ¤é 4ı ı  ı ı ı  ı ‚Øé Ÿ›Èé œ¤é ¤›×±Àãé Þ¤Ô±«±¤¡é د›Øé ›Èãé ÏÍÀãÈÍÅ°›¿ 4ı}ı ıı ııı כذ׫±¤×éد¤é¬°Þ¤Èé¥ÛȟرÍÈé¤ÒۛذÍÈ$é ˜×°È¬é4Bıı Né›È¡é 4ıı A; ߤ鬤Øé 4ı~ı ı oB ıN$é •¯Û×éÐ D3a ı ;33N$ K$ kÈ×ߤÔZé D$ ‘°ÈŸ¤é  ı °×é›é¡°›Å¤Ø¤Ôé ߤ鯛ޤé č ı ı S3èé ×Íéد›Øé ı ±×éϛԛ¿À¤¿éØÍé 



k×é Ď ı±×éױŰÀ›ÔéØÍé ď Iı ߤ鯛ޤé  ı 6x  ı×Íéد›Øé ı   ı …  £ ‘±Å±¿›Ô¿ãé  ı ı ı ‚…

ı ’¯Û×é ı  ă ı  ı ı D* L$ kÈ×ߤÔbé 8AJ$ ‚È鬤Ȥԛ¿é ߤ鿤Øé  ı 6x 5ıı i ž›È¡é ı ı …¤Øé  ı œ¤éد¤éœ±×¤ŸØÍÔé Í¥é $ı ˜×°È¬ Ø®¤é›È¬À¤"œ±×¤ŸØÍÔéد¤ÍÔ¤Åé

ı  ı· ı •¯Û×é

ıı  ı›È¡é ı    oãé‘ؤߛÔØ×éد¤ÍÔ¤Å

 ı ı ı 

ı  

6x

 ııı

߯°Ÿ®é›¥Ø¤Ôé×ÍÀޱȬé¥ÍÔé  ı ¬°Þ¤×



 ı  ;ı Y



O)Â

ı

 ı ı ı #ı  ıÌı 

‘Ûœ×Ø°ØÛذˬéØ®¤éޛÀÛ¤×é괎   ıV ıı {#ıNx ı Kƒ ߤ鬤Øé  ı{ı •¯Û×é W ı °×é°×Íן¤¿¤× ›Ë¡éØ®¤éœ°×¤ŸØÍÕé ı Í¥é $ ı °×éϤÕϤˡ±ŸÛ¿›ÕéØÍé ı ›Ë¡éı °×éد¤éű¡ÏÍ°ËØéÍ¥é ı k×

 }ı >ı ߤ鮛ޤé ıı .ı —װˬéãØ®›¬Í՛×éØ®¤ÍÕ¤Åé ߤ鬤Øé ı ı Ø  ı ı {ı +'+ ı ı  /?ı •®¤Õ¤¥ÍÕ¤ é ?ı ı /ı 

O$ kË×ߤÕYé  …¤Ø×éÏÕÍÞ¤éØ®›Øé W Z8ı °×é ױŰ¿›Õé ØÍé W*ı 8

y°Õ×Øé $* ı ı $ ı ı $±Ā ı k×é $* ı ı $* >ı ß¤é ¯›Þ¤é $* ı  $* ı }ı ı 8.ı •®°×髛Ë×é W* 8ı $

ı  $* ı ×Íé Ø®›Øé $ *āı ı $ *>ı Ø®Û×é *ı   °×é ›é Õ°¬®Ø›Ë¬¿¤¡é ØÕ°›Ë¬¿¤é ß°Ø®é $ġı 8ıı ( ½.ı k×é $8ı °×é ›é ŸÍÅÅÍËé ›Ë¬¿¤ é ߤéØ®Û×鯛ޤ W Z8ı °×é ×°Å°À›Õé ØÍé W* 8^ı ~¤ËŸ¤ é 8 Zı ı *8* ı ı * * £ı tÍËפÒÛ¤ËØ¿ã

 8ı L… *  Zı L… * ı ıh R* kË×ߤÕZé ^ <…



‘±ËŸ¤é $ı °×é ›Ëé ¤àؤհÍÕé ›Ë¬¿¤é ¥ÍÕé Ø®¤é ØÕ°›Ë¬¿¤é  >ı ß¤é ®›Þ¤é $ı ı $ ı $ œı ¯¤ËŸ¤é $ı ı $ ı  $ .ı ‚Øé ¥Í¿¿Íß×é د›Øé $ ı ı $ >ı ×Í Ø®¤é ØÕ°›Ë¬À¤×é  ı ›Ë¡é ±ı ›Õ¤é װű¿›Õ-é yÍÕé Ø®°× é ߤé ͜؛°Ëé ı ı ı oÛØé $ ı ı $>ı ×Íé ß¤é ¬¤Øé ı ı  .ı —ױˬé Ø®¤×¤é ¤ÒۛÀ°Ø±¤×é ß¤é ¬¤Ø  ı  ı ııEı )+

)+

54$ kË×ߤÕ[é (( y°Õ×Øé›××ÛŤéØ®›Øé +0 ¿Â  ı tÍË×°¡¤ÕéØ®¤é¥Í¿¿Í߱ˬéפØéï

  "ı ı[ı«ı $ [ı ı #ı5ı5ı

O3Â

'+ '+ '+

5ı(-Eı

ŠÍÙ¤éÙ®›Ùé&Fı ı%Òı ı #$  $ ›Ë¡ -B

`x

F <ı kıSı

RUB>…

€¤ËŸ¤éÙ®¤é›Ë×ߤÔé±×éËÍÙéÀ›Ô¬¤ÔéÙ®›ËéRR%é ŠÍßé ŸÍË×±¡¤ÔéÙ®¤éŸ›×¤éÙ®›Ùé +0Žı ((ı yÍÔé \ıııı ı

$+ %+ %+

Kı(ı

ĩ’ ı"H\ Qııbîı÷øı bõöı -Sı ŠÍÙ¤éÙ®›Ùé

ŠÍßé›××ÜŤéÙ®›Ùéı ±×é›Ëãé×ܜפÙéÍ¥é $ ß±Ù®é&Fı GH %gı oã鏱¬¤ÍˮͿ¤éÔ±ËŸ±ÏÀ¤é&3m’0ı GH ı ¥ÍÔé×ÍŤé \iı ı Žı \Žı (ı ‡¤Ùé ;¦Kı;7ı A… 3ım’gı –®Ü× -B

`x

& < ùúı0;¦ı ;70ıûüı HH\ QııQ HH\ QııQı kı%Sı

RUB>… R6U…

‰ kË×ߤÔ\é


€…

Nf

‚¥é ?ı ›ÏϤ›Ô×é ¤á›ŸÙÀãé Ù®Ô¤¤éٱŤ×é Ù®¤ËéÙ®¤éËÜŜ¤ÔéÍ¥éߛã×éÙÍ韮ÍÍפé Kı;Kıı ±×é S ~¤ËŸ¤éÙ®¤é›Ë×ߤÔé±×

/ı K… Hı(ı K… Né K… @ı



NéıQı kı /Þ



5<* kË×ߤÔ\é %? …¤Ùé jı œ¤éÙ®¤éפÙé Í¥é ›Ôԛˬ¤Å¤ËÙ×é Í¥é Ù®¤×¤é ¬±ÔÀ×é›Ë¡é œÍã×é ÜË¡¤ÔéÙ®¤é ŸÍË¡±Ù±ÍËé Ù®›Ùé Ôı  ÅÜ×Ùé œ¤é פϛԛ٤¡é œãé ¤á›ŸÙÀãé Cé œÍã×$é yÍÔÅé ›é œÀ͟¾é mı ß±Ù®é ıı ›Ùé Ù®¤é ÙßÍé ¤Ë¡×é ›Ë¡ ¤á›ŸÙ¿ãé ?ı œÍã×霤Ùߤ¤ËéÙ®¤Å/é –®¤éËÜŜ¤ÔéÍ¥éߛã×éÙÍé¥ÍÔÅé×ܟ®é›éœÀ͟¾é mı ±×

ı J… –®¤Ìé ߤ鮛ޤé

Fj&ıkıı J…

*



?]S

*



]ı K… @]

›×é ß¤é Ÿ›Ëé ŸÍË×±¡¤Ôé ×ܟ®é ›é œÀ͟¾é mı ›×é ÍË¤é ±Ù¤Åé ›Ë¡é Ù®¤Ô¤é ›Ô¤é ×Ù±¿Àé /ı œÍã×é ›Ë¡é ÍË¤é ¬±ÔÀ ±.¤%é Êı ŠÍÙ¤é Ù®›Ùé ±Ëé›Ëãé›Ôԛˬ¤Å¤ËÙé ±Ëé jCıı ±×éÍÜÙ×±¡¤é Ù®¤éœÀ͟¾é œ¤Ùߤ¤Ëé ı ›Ë¡é Eı ‚Ëé¤á›ŸÙ¿ã ®›¿¥éÍ¥éÙ®¤é›Ôԛˬ¤Å¤ËÙ×éÍ¥é jı ı ±×霤Ùߤ¤Ëéı ›Ë¡é gı €¤ËŸ¤ +0 kı&jĈıı

*

€¤ËŸ¤éÙ®¤é›Ë×ߤÔé±×é &+ €… ?]ıı %?.

O:Â

€…

]ı €… @]S

5B$ kË×ߤÔ]é ( ¢ yÍÔé [ı ı ııCı À¤Ø ŠÍؤéØ®›Øé

* "*  * * !* * "* * *"* * !*"* ı Iı

ı"bıiıı¿ıbıÀı  ıb 9 [ı ı‡Â $ -ıÝ ı@ı ı@ı ›Ë¡é

‚Ø韛Ë霤é×®ÍßËéØ®›Øé¥ÍÔé ı ı#ılı"ıı$ Dé

$+ %+ &+

ı -Iı ı  ı ıııı ±×顱ޱױœÀ¤éœãé $ ±¥©

ı ¥ÍÔ雿Àé [ı ııýı ÍÔé

±a<"ı ıı-ı

±±a<"ı ıı-ı3ı ±±±a<"ı ıı-ı

$ ¥ÍÔé ×ÍŤé [iııÀı[ı ¿ıı ÍÔ

ıı ›Ë¡é º"ı ıı-ı $ ›Ë¡é

º"ı ıı-ı

ı ¥ÍÔé ›¿Àé \ıııı ÍÔé

ı D%é

±Þa<"ı ıı-ı

’®Û×éØ®¤éËÛŜ¤ÔéÍ¥éD¤À¤Å¤ËØé×ۜפØ×é "ı ıı-ıÍ¥é "ıı$ #ı ııı ±×顱ޱױœ¿¤éœãé $ ±×é

#+ #+ #+

ı -ı×۟®éØ®›Øéı  ı ı

& *&* '"* ( * '"*(* &"*  *  * )" *    * ı







$

ıı



@7ıı







ı ( Tı

5E, kË×ߤÔYé   ’®¤Ô¤é›Ô¤éØßÍéÏÍ××±œ¿¤é¥ÍÔśØ×é¥ÍÔéخԤ¤é¡±¬±ØéËÛŜ¤Ô×éØÍ鮛ޤéØßÍé›Ë¡éÍË¿ãéØßÍéŸÍËפŸç Ûرޤ顱¬±Ø×鱡¤ËرŸ›ÀY ±a ı ß®¤Ô¤é ı > ı ›Ë¡é ı > Iı ÍÔ ±±a  ı ß®¤Ô¤é ı > ı ›Ë¡é ı > . ’®Û×éØ®¤éËÛŜ¤ÔéÍ¥é×۟®é±Ëؤ¬¤Ô×é±×

(ı I… (ıı(ı J… (ıı ¡ 5H$ kË×ߤÔ^é / . ‡¤Øé ı œ¤é›é ˛ØÛԛÀé ËÛŜ¤Ô顱ޱױœ¿¤éœãé B4$é ’®Û×é ı Ÿ›Ë霤é¤àÏÔ¤×פ¡é›×

ß®¤Ô¤é p… >?é Πͥé ı ±×

*

EF $ ›Ë¡é 7ıþÇıVı ›Ô¤éÏÍױرޤé±Ëؤ¬¤Ô×$é ’®¤éËÛŜ¤ÔéÍ¥éÏÍױرޤ顱ޱ×ÍÔ×

 %**  %** 

ı7ıııı!ııııVııı ‘±ËŸ¤é ıı =ƒ … $ … /Iı ß¤é ®›Þ¤é p… ı ı ±¥

7ıı!ııVıı

ı ıT

ııı

€¤ËŸ¤é Ĝı ›Ë¡é ı ®›×é ¤à›ŸØ¿ãé A3é ÏÍ×±Ø±Þ¤é ¡±Þ±×ÍÔ×é ±¥©é ı ı    "$ ›Ë¡é 7ıı!ı Vıı ıı Cı ß®¤Ô¤é 7ı!#ıVı ›Ô¤é›¿ÀéÏÍױرޤ.é yÛÔØ®¤Ô é 7ıı!ııVıııı ß®¤Ô¤é 7# !Vı ›Ô¤é›¿Àé ÏÍױرޤ é ±¥©é "7!V-ıı"- ’®¤éśà±ÅÛÅéޛ¿Û¤éÍ¥é×۟®é Ix ±×é   ıı … 7ı … /Nı ı/ 

OMÂ

5K, kË×ߤÕYé F44& …¤Øé m_x œ¤é د¤é śà°ÅÛÅé °Ëؤ¬¤Ôé ×۟¯é د›Øé =uƒ °×é›é ¥›ŸØÍÕé Í¥é 8ƒ … =ƒ … ?ƒ … Dƒ … … =44Kƒ ›Ë¡é vƒ د¤éśà°ÅÛÅé°Ëؤ¬¤Õé×۟¯éد›Øé Fwƒ °×é›é¥›ŸØÍÕéÍ¥é 8ƒ … =ƒ … ?ƒ … Dƒ … … =44K&ƒ '+ '+ '+

!+ !+ !+

•¯¤Ëéد¤éËÛŜ¤ÔéÍ¥é×é›Øéد¤é¤Ë¡éÍ¥é 8ƒ … =ƒ … ?ƒ … Dƒ … ‚Øé±×é͜ޱÍÛ×éد›Øé qı A v&

"+ "+ "+

… =44Kƒ

±×é¤Òۛ¿éØÍé +B

`x

"q5ıv&ƒ

£Â

Žœ×¤ÕÞ¤é د›Øé =44Kƒ 3… FG'ƒ yÍÕé 8ƒ c b Dƒ د¤é ËÛŜ¤Õé ¡¤ËÍؤ¡é œãé ai*ƒ Í¥é °Ëؤ¬¤Õ×é ±Ë 5ı=44Kƒ ¡¤ËÍؤ¡éœãé 3H ¡°Þ°×±œ¿¤éœãé Fiƒ ±× "#ı=ƒ?ƒD !+ !+ !+

aiƒ 8x p=44K3Fiƒ]ƒ (

‘Íé a9ƒ 8x D48ƒ a 8x K4ƒ a >ƒ @ œãé FEYƒ

+

8Iƒ a

Eƒ 8x ?&ƒ •¯Û×é د¤Ô¤é›Õ¤é¤à›ŸØÀãé ?ƒ ËÛŜ¤Ô×é°Ëé 3H ¡°Þ±×±œ¿¤

¤à›ŸØ¿ãé 8I" ?ƒ 8x 8?ƒ ËÛŜ¤Õ×é°Ëé 3H ¡¶Þ°×°œ¿¤éœãé F@ƒ œÛØéËÍØéœãé FEZƒ ¤à›ŸØ¿ãé K4ƒ# 8Iƒ 8x IDƒ ËÛŜ¤Õ×é°Ëé 3H ¡°Þ±×±œÀ¤éœãé F>ƒ œÛØéËÍØéœãé F@Zƒ ¤à›ŸØ¿ãé D48ƒ# K4ƒ

+

?=8ƒ ËÛŜ¤Õ×é°Ëé m ¡°Þ°×°œ¿¤éœãé Fƒ œÛØéËÍØéœãé F>+ƒ

~¤ËŸ¤é د¤éśà±ÅÛÅé°Ëؤ¬¤Õé vƒ ×۟¯éد›Øé Fwƒ °×é›é¥›ŸØÍÔéÍ¥é 8ƒ … =ƒ … ?ƒ … Dƒ … v

8x

Dƒ M… ?ƒ?ƒ ƒ M… 8?ƒ=ƒ ƒ L… IDƒ8ƒ ƒ M… ?=8ƒ 8x F44)ƒ

5L, kË×ߤÔYé KL* …¤Øé ajƒ œ¤éد¤é ŸÍ¤¥ª±Ÿ°¤ËØéÍ¥é ~iƒ °Ëéد¤é¤àϛË×°ÍËéÍ¥é 8ƒ=ƒ ~ :55&ƒ •¯¤Ë

 

844 aiƒ 8x =ƒi £Â #

•¯Û×é @oa… 8x

@o…

‘Ϳޱˬéد¤é±Ë¤Òۛ¿±Øãé

   



846ƒ Axƒ 8ƒ

*

844 H…

=844# y ƒ :… 4 y8ƒ ¬°Þ¤×éد¤é×Í¿ÛØ°ÍËé yƒ 3… 8MM3?&ƒ ƒØé°ÅÏ¿°¤×éد›Ø

~¤ËŸ¤éد¤é ›Ë×ߤÔé°×é KL$é 5O* kË×ߤÕYé ‰©. ŠÍؤéد›Ø

’¯Û×é

•¯Û×éد¤é›Ë×ߤÔé±×é ‰©ı

OOÂ

=844# H  y8ƒ

'+ '+ !+

…

=44Kƒ °×

5R* kË×ߤÔZé  ŒÍؤéØ®›Øé ı      ı,ðŒı }ı 5&Hı7&Hı8&H !ıı9&H       ı,ı     ,  5 ı ıııı,ıııı ıı ıı,ıııı ıııı ,ııı ıııııı ı   ı ıı ,ıı9ı9ı9ıé* ‘°ËŸ¤é        , ı~ı BÐ      , ı Iı ×Íé       ,ı ı+ !h ‘°Å°À›ÔÀãé ߤ鬤Øé &#ı+ !#ı+ !ı›Ë¡é+ ,!ı •®Û×é ߤ鮛ޤé !ıı !ıı!ıı¬§ıı,&ı { ıı ı  ,  rr 9œı ×Íé 5.'H ı !ıı!ıı!ıı 9&H ı ı …  ıgé . =4$ kË×ߤÕ_é 54, ™¤é®›Þ¤

" 6H "6H R G^x " 6= 
d )

"H 6=H

•®Û×é 5#@@DH ı__Nı ~ı 9ı9ı9ıı Nı ı %+ <5* kË×ߤÕYé 5KKOL* ™¤é śãé ÏÕ͟¤¤¡é œãé Ûװˬé Ø®¤é Ïհ˟°ÏÀ¤é Í¥é °ËŸÀÛ×°ÍËé ›Ë¡é ¤àŸÀÛ×°ÍËé ›×é ¥ÍÀÀÍß×$é …¤Øé Ø®¤ ÛË°Þ¤ÕכÀé פØé Gı œ¤é Ø®¤é פØé Í¥é O ¡°¬°Øé °Ëؤ¬¤Õ×é ŸÍÅÏհװˬé Ø®¤é Oé ¡°¬°Ø×é ¥ÕÍÅé 5é ØÍé O*é …¤Ø Zĝı €Â Xı €Â :ı œ¤é Ø®¤é ÏÕÍϤÔØãé Ø®›Øé Xııı °ÅŤ¡°›Ø¤Àãé ¥ÍÀÀÍß×é Xı °Ëé ›Ëé ¤À¤Å¤ËØé Í¥é G ~¤ËŸ¤é Bı ı †Dı Xı ı ąÈ+ X†ı ¥ÍÔé ı ·Â Xı €Â :^ı •®¤Ëé Ø®¤é ›Ë×ߤÕgé Bıg »´Bı  »´ııı 9ı9ı9 ı @ı ı z :.ı •®¤é ›Ë×ߤÕé Ÿ›Ëé ›À×Íé œ¤é ͜؛°Ë¤¡é ¥ÕÍÅ ¤°Ø®¤Ôé Íˤé Í¥é Ø®¤éØßÍé¤àÏÕ¤××°ÍË×鲍Àãé #+ íı ÍÔé ëı ı ,é ß®¤Ô¤é dı°×éØ®¤éËÛŜ¤Ôé Í¥ ¡¤Õ›Ë¬¤Å¤ËØ×é Í¥éØ®¤é°Ëؤ¬¤Ô×é¥ÔÍÅé5éØÍé +0 ›Ë¡é dı~ı†ıĖz+ Ùıı $++ "+ı9ı9ı9ııızıdı* ==$ kË×ߤÕZé :ı.

nÆO Éı

E# )HE# *HE# +HE# H"H#H_2 Â)H …E# A E# *E# +E# & "H"H#H …E# @2(H/B# )HE# +H #"H"H#H *E# ı0C# &E# )E# "H E# ı0C# 1(H )&#"H

,H#)H -H+ ) ,H+ +#)HH

•®¤Ô¤¥ÍÕ¤é Ja ı 4… :ı =B* kË×ߤÔYé My ™¤é ×®›ÀÀé Ûפé Ø®¤é DCa LI_aJ\RH[MTRa ß®¤Ô¤é DCa ×؛ˡ×é ¥ÍÔé Í¬¤Õé Í®é ØÍé ¡¤›Àé ß°Ø®é Ø®°× ÒÛ¤×Ø°ÍË$é …¤Øé L4#ı'Jı=ı ¡¤ËÍؤé Ø®¤é ËÛŜ¤Ôé Í¥é ϤÔÅÛ؛ذÍË×é Í¥é 4ı ŸÍÏ°¤×é Í¥é 3H 'ı ŸÍÏ°¤× Í¥é ı ›Ë¡é =ı ŸÍÏ°¤×é Í¥é ı ×۟®é Ø®›Øé ËÍé ØßÍé ›¡»›Ÿ¤ËØé À¤ØؤÔ×é ›Õ¤é °¡¤ËØ°Ÿ›À$é kÅÍˬé Ø®¤×¤ L4Jı'Jı=ı ×۟®é ϤÔÅÛ؛ذÍË×é À¤Øé L4ı' #ı=ı ¡¤ËÍؤé Ø®¤é ËÛŜ¤Õé Í¥é Ø®Íפé ϤÔÅÛ؛ذÍË× ß®°Ÿ®é¡ÍéËÍØ霤¬°Ëéß°Ø®é 3H ~¤ËŸ¤é ߤ鮛ޤ

L4#ı'Jı=ı L4'J=ı

Lı4ı+ |ı'Jı=ııLıı'ı+ zı4Jı=ıı $ ıp…+ |ı4Jı'ı L'  |4Jı=ııL= |4Jı'Eı

OSÂ

†®Â

yÍÔéÙ®±×éÒܤ×Ù±ÍÈé ߤé›Ô¤éÔ¤Òܱդ¡éÙÍé¤Þ›Àܛ٤é ıı ı ı

  ı  ı ı ı ıı ı ıı ı ıı ı ııı ııı ı ııı ıı ı  ıı ı ıı ı  ıı ı ııı

 ı ı ıııı ı ı #ı ı 8x °ıı ı 8x  ı ıı ıÃı ıııııÄıı=a (ıı ı=a (ı ıı ııııı 8x ıı@ı 8x (ı ı ııııı 8x ııı=a ı ıı 8x ı ıııııı=a ııııı 8x @ı ıııııı 6x ıııı=a /ı ı™ıı=a ııı=a  ı  ıı ııııı 8x /ııı=a ı ı ı ııı ıı 8x ıı ııı ıı ı ıı°ııı 8x ıı:ı=a ı  ıı(ı 8x /ı  ııııııı=a ı ıııııı ııııı 8x ıı ı 8x ı  ıı ı 8x Sı

ı ı ı


~¤ÈŸ¤é

?:aı :a 8x YP

ı

  ı=a ı

ı

oãé <é ß¤é ®›Þ¤é ı 6x+ı ‘Üœ×Ù±ÙÜÙ¤é Ù®±×é±ÈÙÍé 5é›È¡é×±ÅÏÀ±¥ã ?;aıP O?:- ı 8x Ú ~¤ÈŸ¤é : ? =

=a





0'KÂ

•®¤Ô¤¥ÍÔ¤é ı=a

7ı .?- ı=a

Cx



*

+

*

~¤ÈŸ¤é

$ * 5:a ı

# * 5:a ı

•®Ü×é ı 8x ı ı6x ıı=a /ı ›È¡é ıoB ıoBBĘı 8x /Čı

OUÂ

#

5:a

#

5:a


ÿ"lGi& - +

+

+

&„ 3„ ÅFı ı &ĕ F F– 3„ &ı –ı ı1ıı „ı1ı ı–ı ı1ı ¹ı Ć&ıı& &ı0ı1ı <ı <ı1ıı F&ı& &ı ı 0ı1ı <ı <ı1ı<& 1ı0ı <ı1ı 0ı& 1ı 0ı 0ı3ı 1ı ı <ı<ı ı3ı3ı ı 0ıı0ı3ı 3ı ı ı ¬Â 0ıı0ı1ı ı1ıı3ı 0ı & 1ıµ3ı 0ıı0ı3ı 1ı &&ı ı 3ı 3ı ı ‰Â¬Â &&ı1ı ı1ıµ1ı 0ı 1ı ı3ı‰Â­Â 0ıı0ı1ı ı3ı‰Â¬Â 0ı 0ı3ı 1ı ı ‰Â­Â ć&ı ı  aııp  aı  aı  aı p aı p aı p €ıı ı K… Ġ €ıı €ı p ê€ ı :¡ı

OWÂ

+

 +   + +

    

966
$ !$ "$ $ !"#$ QÂ "#$ 

 

$

 !"! $ !$ ! !! $ TÂ !(#'#.

("

#+". .



+. . '. #' #. . ,(". +".



 . !(#'.  ""#. (%…  ".

. .  ( '"#. ". +.

5, y°Ë¡é›¿¿éϛ°Õ×éÍ¥é ÏÍ×°Ø°Þ¤é °Ëؤ¬¤Õ×é Ûı¸ı ×Íé Ø®›Øé ııc ı 6x + <, ‚Ëé Ø®¤é ›ŸÛؤé ØÕ°›Ë¬¿¤é  ` rel="nofollow">ı k” °×é ›é ÏÍ°ËØé °Ëé د¤é °ËؤհÍÕéÍ¥é د¤é פ¬Å¤ËØé `ı ›Ë¡é $ °×é › ÏÍ°ËØé ÍËé Ø®¤é ¤àؤË×°ÍËé Í¥é د¤é פ¬Å¤ËØé `ı ×۟®é Ø®›Øé ¨•Â$ `ı …¤Øé ı ›Ë¡é ı œ¤é Ø®¤ ¥¤¤ØéÍ¥éد¤é ϤÕϤˡ°ŸÛÀ›Õ×é¥ÕÍÅé kr ›Ë¡é ©Â ÍËØÍéد¤é¿µË¤×é `ı ›Ë¡é  ı Õ¤×ϤŸØ°Þ¤Àã-é ÕÍÞ¤ Ø®›Øéد¤éÍÕد͟¤ËØÕ¤éÍ¥é ¨ `ı ¿°¤×éÍËéد¤é Ÿ°ŸÛş°ÕŸÀ¤éÍ¥é ¨  . +

B* …¤Øé /x Bx œ¤éÏÍ×°Ø°Þ¤é°Ëؤ¬¤Õ×é߰دé q… 9… bx A Uı ›Ë¡é߰دé qı ¡¤Ï¤Ë¡°Ë¬éÍËé wı tÍË×°¡¤ÕéØ®¤ פÒۤ˟¤é2ıARı2ÍıE E E #ıı2‚ı ß®¤Õ¤

2d˜“ı

¥ÍÕ·gé #ıE E E eııı gé 6éxxxı#ıq+ + 2!“ı2 ı !“fı ¥ÍÕé bı ı æОAı2

߰دé m_x+ qı+ ıgé ı ÍÕé <é °,¤,é 2ĥıgé 2‚fcı2 ı ‚fcžı ß®¤Õ¤é ¸ıgé  ı ĦfcfAı2 ÍÕé <$é •¯Û×é °¥é .+x 6x UDı د¤é פÒۤ˟¤é °×é Rı#ı #ıeıU#ı #ıUı ›Ë¡é °¥é ıgé >ı Ø®¤é פÒۤ˟¤é °× 6é#ı Rı5ıUeı Rı:Rıeı #ıU#ı.Qı y°Ë¡éGıgé2Aı2 ı ırrr2‚ı °¥é °Qı ıgé  :Cı °°a +  ^ı +

Cé …¤Øé ı 3… 2eı ı 3… 0x.ı ‘®Íßé Ø®›Ø

Uı 9… :‘•ƒ2ıı ƒę”2xı ı ‘•ƒı2ı ƒĚ”ı2ıĬě”ı ı‘•ĭı ı+ I* tÍË×°¡¤Õé ›é  ı ~…  ı Ÿ®¤××é œÍ›Õ¡(é …¤Øé k/ œ¤é Ø®¤é ל¿¿¤×ØéËÛŜ¤Õé Í¥é Õ¤ŸØ›Ë¬¿¤×é Ø®›Ø Ÿ›Ë霤é¡Õ›ßËéÍËéد¤éŸ¯¤××éœÍ›Õ¡é×Íéد›ØéØ®¤é×°¡¤×éÍ¥é¤Þ¤Õã韤¿¿éÍ¥éØ®¤éœÍ›Õ¡é°×éŸÍË؛°Ë¤¡ °Ëé Ø®¤é ×°¡¤×é Í¥éÍˤé Í¥é د¤é Õ¤ŸØ›Ë¬À¤×*é y°Ë¡é Ø®¤é ޛ¿Û¤é Í¥é ls!  yÍÕé ¤à›ÅÏÀ¤ é ¥ÍÕé د¤é ı ~… Ÿ¯¤××éœÍ›Õ¡ é د¤éޛ¿Û¤éÍ¥é  °×éB$

OYÂ

 +   + +

    

966
$ !$ "$ $ "! $

 ‚¥éWa°×é›éÏԱŤ饛ŸØÍÔéÍ¥é +0 0 °Øé°×雿×Íé›é¥›ŸØÍÔéÍ¥é +%00 $ ~ÍߤޤÔé ÈÍȤéÍ¥éØ®¤éÈÛŜ¤Ô×  00 0+0 °×é ›é ¥›ŸØÍÔé Í¥é +%00 $ ‘Íé ߤé ÅÛ×Øé ®›Þ¤é Wa 8x +0 0 $ °$¤$é +0  0 ÅÛ×Øé œ¤é ÏԱŤ$ yÍÔé +0  0 6x ; é ı/Cı ß¤é ®›Þ¤é×Í¿ÛرÍÈ×é + 0#0 8x  0 0 ; é 0 ›È¡é ı; $é …¤ØéÛ×é×®Íßé Ø®›Ø Ø®¤Ô¤é ›Ô¤é ÈÍé ÍØ®¤Ôé ×Í¿ÛØ°ÍÈ×$é ‚È¡¤¤¡é ×ÛÏÏÍפé Ø®›Øé +0  0 °×é ›é Ï԰Ťé ÈÛŜ¤Ôé rel="nofollow">?é :ı ’®¤È + 6x ;Æé Æé ;… ;$é ŠÍßé ;é 3… Æé 3… +0 ›È¡éØ®Û×é +0 8x ;ÆÉé¡°Þ±¡¤×é +%0 ’®Û×é +0 ¡±Þ±¡¤× +0 $ 0

+

+$0$#+$&0 Â

+

+

+

##0 0 +0Â +# ;é

$

’®Û×é +0 4H #+0 ›È¡é °ÈéϛÔرŸÛ¿›Ôé #0 CD +0 ‚Øé¥Í¿ÀÍß×éØ®›Ø ,0 +%0 8x +0 $ $ 0 ;… +$0 EF + >?é +%

›éŸÍÈØԛ¡±ŸØ±ÍÈ%é

<$ …¤Øé„霤éØ®¤éÏͱÈØéÍ¥é°ÈؤÔפŸØ°ÍÈéÍ¥éˆué ›È¡é‹ é ı ‚Øé°×餛×ãéØÍéפ¤éØ®›ØéØ®¤éŸ±ÔŸÀ¤é߱خ ¡°›Å¤Ø¤Ôép„é°×éØ®¤éŸ°ÔŸÛş±ÔŸ¿¤éÍ¥é W  Üı k×é lé±×éϛԛÀ¿¤¿éØÍ ˆé „é›È¡ér°×éϛԛ¿À¤¿ Ø͋„é ß¤é ®›Þ¤é †lré 8x †„ˆ‹é ›È¡é † lré6x †ˆ‹„$é ‘±ÈŸ¤é lré 6x ˆ‹é ߤ Ø®Û×鮛ޤé w lé ré°×éŸÍȬÔÛ¤ÈØéØÍé wˆé „é ‹$

’®¤Ô¤¥ÍÔ¤é Ø®¤é ¡±×؛ȟ¤é ¥ÔÍÅé „é ÍÈØÍé lré ¤Òۛ¿×é ØÍé Ø®¤é ¡°×؛ȟ¤é ¥ÔÍÅé éÍÈØÍé lr$é oÛØé ›È¡é„é›Ô¤éÍÈéØ®¤éכŤé×±¡¤éß°Ø®éÔ¤×ϤŸØéØÍéØ®¤é¿±È¤é lré ±Øé¥Í¿ÀÍß×éØ®›Øéé „é°×éϛԛÀ¿¤¿é ØÍé lr$é ’®¤Ô¤¥ÍÔ¤é é „é ±×é ϤÔϤȡ°ŸÛÀ›Ôé ØÍé pé›È¡é éÀ°¤×é ÍÈé Ø®¤é Ÿ±ÔŸÀ¤é ß±Ø®é ¡±›Å¤Ø¤Ôé p„韰ԟÛÅןԱœ±È¬é›œÍÛØé W  ı

B$ ™¤é ×®›¿Àé ×ÍÀÞ¤é Ø®¤é ÏÔ͜À¤Åé ¥ÍÔé ¬¤È¤Ô›¿é +0 ‚ȱر›ÀÀãé À¤Øé ! 0! 00 0!,0 œ¤é Ø®¤é FGZM]Ia פæ Òۤȟ¤$é kÈéTWIXFZMTRaÔ¤ÅÍÞ¤×éØ®¤é¦±Ô×ØéخԤ¤éؤÔÅ×é›È¡é›ÏϤȡéØ®¤±Ôé×ÜÅé›×éØ®¤éÀ›×ØéؤÔŠͥéØ®¤éפÒۤȟ¤$é ’®Û×雥ؤÔéÍȤéÍϤԛذÍÈéÍÈéØ®¤é±È°Ø±›ÀéפÒۤȟ¤é Ø®¤é›ŸØ°Þ¤éפÒۤȟ¤é°× ! 0!. 00 0!,'/0 ‘ÛŸ®éÍϤԛرÍÈ×éß°À¿é×ØÍÏéß®¤ÈéØ®¤é›ŸØ±Þ¤éפÒۤȟ¤é±×éͥ鿤Ȭخé›ØéÅÍ×Ø <$ ‚Èé Ø®±×é Ÿ›×¤é Ø®¤é À›×Øé ؤÔÅé Í¥é Ø®¤é ›ŸØ°Þ¤é פÒۤȟ¤é ±×é !*0 ’®¤é¥Í¿ÀÍ߱Ȭé ͜פÔޛذÍÈ× ›Ô¤é͜ްÍÛ×$é

OZÂ

^ m§Ø¤Õ餛Ÿ¯éÎϤ՛ذÎÈ é د¤é¿¤È¬Ø¯éΧéد¤é›ŸØ°Þ¤éפÒۤȟ¤é¡¤ŸÔ¤›×¤×éœãé <é›È¡éد¤é×ÛÅ Î§é °Ø×éؤÕÅ×é°×é ÏդפÕÞ¤¡& <- ‚§é د¤é ¿¤È¬Ø¯é Î§é ›Èé ›ŸØ°Þ¤é פÓۤȟ¤é °×é ?nƒ ›È¡é د¤é ¿›×Øé ؤÕÅé °×é Nk … د¤Èé ›§Ø¤Õé ¡ ÎϤ՛ذÎÈ× é د¤é ¿¤È¬Ø¯éΧéد¤é›ŸØ°Þ¤éפÒۤȟ¤é°×é " ›È¡éد¤é¦²Õ×ØéؤÕÅé °×é Pl`ƒ… “ÎéŸÎÅÏÛؤé XÂ ß¤é ¯›Þ¤éØßÎ韛פ×W s›×¤é ² 'é dx Ρ¢(é …¤Øé ŸÂ ›È¢é (, œ¤é°Èؤ¬¤Õ×é×۟¯éد›Øé [x 8x ?kƒ B=zƒ 3… ?ktƒ °*¤* é , {ƒ 3… ?k_ƒ ŠÎؤé د›Øé [»±Â 3… [%x “¯¤é ¦°Õ×Øé w±Â ؤÕÅ× é $ B… P2/ … °×é Ÿ›¿¿¤¡é د¤é !" " ›È¡é د¤é ×ÛŠΧé°Ø×éؤÕÅ×é°×é ‚’ 8ƒ B, B B?zƒ 8x ?|?{ƒ B ,3 B ŠÎßé ›ÏÏ¿ãé د¤é›œÎÞ¤éΜפÕޛذÎÈ×( ’¯¤é ×ÛÅé Χé د¤é ؤÕÅ×é Î§é ›Èãé ›ŸØ°Þ¤é פÒۤȟ¤é °×é 6 7a B , B B Wx 8x \  [x B 3( m§Ø¤Õé kx ÎϤԛذÎÈ× é د¤é ¿¤È¬Ø¯é Χé د¤é ›ŸØ°Þ¤é פÒۤȟ¤é °×é ?kƒ ›È¡é °Ø×é ¦²Õ×Øé ؤÕÅé °×é $A9 0@ “¯°×雟ذޤéפÒۤȟ¤é°×韛¿¿¤¡éد¤é ¶¹½ª«´¦Â " m§Ø¤Õé % &, ÎϤ՛ذÎÈ× é د¤é¿¤È¬Ø¯éΧéد¤ ›ŸØ°Þ¤éפÒۤȟ¤é°×é ".0#B ”¯°×雟ذޤé פÓۤȟ¤é °×韛¿Á¤¢éد¤é ¸–™³ŒÂ  " ¤Ï¤›Øéد°×éÛÈØ°¿ د¤é ¿¤È¬Ø¯éΧéد¤é ›ŸØ°Þ¤éפÒۤȟ¤é°×é . ’¯°×é °×éÈÎßé د¤é ¼é B+', œ¿ÎŸ½)é “¯Û×é ß¤é ¯›Þ¤é› œ¿ÎŸ½é ߯Îפé ×ÛÅé °×é uv ›È¡é ¡Â B 7a œ¿ÎŸ½×é ߯Îפé ×ÛÅé °×é ¼ ~¤ÈŸ¤é Gı š/  " B , ‘°ÈŸ¤é d “˜Â 8x J=Q!ƒ Yx 3x , 8x fgƒ B=rhUCM !ƒ ߤ鯛ޤéGı 8x <2HRD?FRM&ƒ s›×¤é ²°& Xx ¤Þ¤È)é …¤Øé ¢Â ›È¡é (, œ¤é ³Èؤ¬¤Õ×é ×۟¯é د›Øé cx 8x =?lƒ B (, 3… =?m; ƒ ²*¤$ +

+ + +

+

+ + +

+

( } jx 3… %, ŠÎؤéد›Øé ]J²Â 3… Z$x ”¯¤é¦²Õ×Øé \I±Â ؤÕÅ× é P' …… O1n… °×韛¿¿¤¡éد¤é  !" " ›È¡é د¤é ×ÛÅé Î§é °Ø×éؤÕÅ×é °×é t  m×é °Êé Ÿ›×¤é ° é ›§Ø¤Õé +é ÎϤ՛ذÎÈ×é د¤é ¿¤È¬Ø¯éΧé د¤é ›ŸØ°Þ¤ פÓۤȟ¤é °×é =VCo ƒ ›È¡é °Ø×é ¦²Õ×Øé ؤÕÅé °×é cW}s€ƒ “¯²×é ›ŸØ°Þ¤é פÒۤȟ¤é °×é Ÿ›¿¿¤¡é د¤é   "  " m§Ø¤Õé =ƒ% , ÎϤ՛ذÎÈ× é د¤é ¤ȬدéΧéد¤é›ŸØ°Þ¤éפÓۤȟ¤é°×é =ƒ[Cm%^ -ƒ “¯°×雟ذޤ פÒۤȟ¤é°×韛¿¿¤¡éد¤é ! "  " ¤Ï¤›Øéد°×éÛÈØ°¿éد¤é¿¤È¬Ø¯éΧéد¤é›ŸØ°Þ¤éפÒۤȟ¤é±×é<(é ’¯°×é²×éÈÎßéد¤é Na B)#, œ¿ÎŸ½(é ’¯Û×éߤ鯛ޤé›éœ¿ÎŸ½é߯Îפé×ÛÅé°×é jq ›È¡é  Â B, œ¿ÎŸ½× ߯Îפé ×ÛÅé °×é œ ~¤ÈŸ¤é Gı x¥Â B / B  , ‘°ÈŸ¤é ‚C gƒ 8x  , *$, “¯Û×é x 8x , ›È¢é žÂ B›È¡é×Îé ~ " ,

+

+

+

+

E0 z°Õ×ØéÈÎؤé د›Øéœãé k‰!|‰ '3:B tx µ—1 +… ƒÂ  ;+1Bcƒ:,2B%B(4<B&B '5=BP…

7… 

=ƒ =,1B%B'6=B"

+

“¯Û×é yoÂ

B r*] ux r{D S…Lhq S… '6=B „ p + V +…P 

8x +

, B,

!, , 

7… 





#,x :=?x x

x

x

F\]D…)… Ex s\f S…Xet

Ox



 B !, ,  , B

+ ,x >TUKx x

Mis-x gx

!"x

t\f*…S…Wfy […

 x … _…… B ze-… x x x v x ydTXjyT… [Wiu,…T… 

+ , , B  @QQPx

+



<x



 #H

 x *+





, 8… = '6> cƒ B >,2$ 

“¯¤é¿›×Øé°È¤Òۛ¿°Øãé§Î¿¿Îß×霤Ÿ›Ûפ鰧é ;+1?B8x ?3Fƒ د¤Èé'6>? =,2B1lx )6>B$B B'7=B?B>+2B$B

S)Â

+

D 4 , ›È¡é - 7 =+2$ ?B 8x

I* ’¯¤é ›È×ߤÕé °×é j‘  $ m¿Áé د¤é ¯ÍÕ°åÍÈ؛¿é ×°£¤×é Ÿ›Èé ¤é ŸÍÞ¤Õ¤£é äé 544Eé Ï°¤Ÿ¤×é ͧ + }… $ Õ¤ŸØ›È¬¿¤×é ¤àŸ¤ÏØé د¤é ÍÝÈ£›Õäé ͧé د¤é Ÿ¯¤××é Í›Õ£é ߯°Ÿ¯é Ÿ›Èé ¤é ŸÍÞ¤Õ¤£é ä د¤é ÍÝÈ£›Õäé Õ¤ŸØ›È¬¿¤&é ’¯¤é դś°È°È¬é Þ¤ÕØ°Ÿ›¿é ×°£¤×é Ÿ›Èé ¤é ŸÍÞ¤Õ¤£é äé 544Eé Ï°¤Ÿ¤×é ͧ $ }… $ Õ¤ŸØ›È¬¿¤×*é ’¯Ý×é ip ¾Â 

+

ŠÍßé×ÝÏÏÍפéد›Øéد¤éŸ¯¤××é Í›Õ£é¯›×é ¤¤ÈéŸÍÞ¤Õ¤£é äé hÂ Õ¤ŸØ›È¬¿¤×é°Èéد¤é£¤×°Õ¤£éߛä$ …¤Øé $ ͧéد¤éÕ¤ŸØ›È¬¿¤×鯛ޤéد¤°Õé ØÍÏé›È£é ÍØØÍÅéÍÈé د¤é ØÍÏé ›È£é ÍØØÍÅé ͧé Ø®¤é Í›Õ£ $ ͧéد¤é Õ¤ŸØ›È¬¿¤×é ¯›Þ¤éد¤°ÕéØÍÏéÍÈéد¤éØÍÏéͧéد¤é Í›Õ£é V… ͧéد¤éÕ¤ŸØ›È¬¿¤×鯛ޤéد¤°Õ ÍØØÍÅé ÍÈé د¤é ÍØØÍÅé ͧé د¤é Í›Õ£é ›È£é $ ͧé د¤é Õ¤ŸØ›È¬¿¤×é ¯›Þ¤é Ȥ°Ø¯¤Õé د¤°Õé ØÍÏé ÈÍÕ ÍØØÍÅéÍÈé د¤éØÍÏéÍÕé ÍØØÍÅé ͧéد¤é Í›Õ£( ‘´ÈŸ¤éد¤Õ¤é›Õ¤é <44Lé°ÈؤÕț¿é¯ÍÕµåÍÈ؛¿é¿°È¤×é ߤ鯛ޤé ııN x $ $ < $ ‘°ÈŸ¤éد¤Õ¤é›Õ¤ =44RéÞ¤ÕØ°Ÿ›¿é¿°È¤×é°ÈؤÕפŸØ°È¬éد¤éØÍÏéͧéد¤éÍ›Õ£ é ߤ鯛ޤé$$ A $ hm… $$ >  $ ‘°Å°¿›Õ¿äé  Nx$ A  $ “¯Ý×é !$ $ $$Jx ?  $ ~¤ÈŸ¤é $$$$V…$ = $ +* B jÂ  $$Nx $  °¤Â @ 

+

P,Â

8^oSBws|OŠ2B€XOkB€^IBfŠ 8sI^O€„Š h›‰~¢Ÿ¥†¶b~ª‹†˜~ª~–¶e–¯˜¢~ƒ¶hbe¶ D33W  !" "

%?2%).%%Ã

u²ƒ®‚z»Ã.?Ãiz»Ã.%%;Ã

  " (-1,4 4 FL± +/-.")(-

(.,4 2)/,4 (-1,-4)(4.!4 (-1,4 -!.4 *,)0" 4

),4 .!4 '/&."*&4 !)"4 +/-.")(-4 (.,4 )(&24 .!4 &..,-4 4

VÃ i£ k£ ),4 4 ),,-*)("(

4 .)4 .!

),,.4 (-1,-4 "(4.!4 (-1,4 -!. 4

),4 .!4 ).!,4 -!),.4+/-.")(-4 1,".4 2)/,4 (-1,-4"(4(-1,4 -!.4 (4 -!4 .!4 **,)*,".4 /3 &-4&)142)/,4 (-1,- 4

)4-.*-4,4 (4 .)4$/-.#4 2)/,4 (-1,- 4

!4+/-.")(4,,"-4 6± ',%

)4&/&.),-4,4&&)1 4

" " " " "  " ""  " " " "



)" S¡¥¢Ã¯‘ƒÃŠ¥²©Ã®¯z¯ƒ¡ƒ¢¯®Ã¥¢Ã•¢¯ƒƒ©®Ã|ƒŸ¥¶ cŠÃ 4 4 4 ¯‘ƒ¢Ã  4 4  4

 4 kà %Õ®ÃzŸ¶z»®Ã¯©²ƒL

 4 4 %Õ®ÃzŸ¶z»®Ã¯©²ƒLÃ

cŠÃ  4 4  4 ¯‘ƒ¢Ã4 4 4

‘¥¶Ã¡z¢»Ã¥ŠÃ¯‘ƒ¡Ãz©ƒÃ¥©©ƒ¯Rà S Ã%Là W Ã)LÃ

j£.LÃ

[ Ã2LÃ ] ÃG*£

. x‘•‘Ã¥ŠÃ¯‘ƒÃŠ¥ŸŸ¥¶•¢Ã¢²¡|ƒ©®Ã•®Ã¥‚‚Ê¥©Ãz¢»Ã•¢¯ƒƒ©ÃµzŸ²ƒ®Ã¥ŠÃœR S à .%%;à £œ3Là W Ã.%%;à £;œLà Z Ã.%%;à £.œ Là [ Ã.%%;à £.%%;œLà ] Ã.%%;œ 2 c¢Ãzî‘¥¥ŸÃzŸŸ 2%%Ãrƒ¥¢‚z©»Ã2‚ƒ¢¯®Ã®¯²‚»Ãƒ•¯‘ƒ©Ãaƒ¥©z§‘»ÃW•¥Ÿ¥»Ã¥©Ã|¥¯‘Ãaƒ¥©z§‘» z¢‚ÃW•¥Ÿ¥»à cŠÃ >%à ®¯²‚»Ã aƒ¥©z§‘»Ã z¢‚à 8%à ®¯²‚»ÃW•¥Ÿ¥»à ‘¥¶Ã¡z¢»Ã®¯²‚ƒ¢¯®Ã ®¯²‚» |¥¯‘Ãaƒ¥©z§‘»Ãz¢‚Ã|•¥Ÿ¥»R S Ã2%Là W Ã:%Là Z Ã>%Là [ Ã?%Là ] Ã)9%à 5 S¢Ã²¢|•z®ƒ‚®•‚ƒ‚•ƒÃ•®Ã¢²¡|ƒ©ƒ‚Ã)ï¥Ã:à u‘ƒÃ‚•ƒÃ•®Ã¯”¬¥¶¢Ã¯¶•ƒÃz¢‚ﶥƒ® z‚‚ƒ‚à x‘•‘Ã¥ŠÃ¯‘ƒÃŠ¥ŸŸ¥¶•¢Ãƒµƒ¢¯®Ã‘z®Ã¯‘ƒÃ‘•‘ƒ®¯Ã§©¥|z|•Ÿ•¯»Ã¥ŠÃ¥²©©ƒ¢ƒR S Ãu‘ƒÃ¯¥¯zŸÃ®¥©ƒÃ•®Ãz穕¡ƒÃ¢²¡|ƒ©LÁà W Ãu‘ƒÃ¯¥¯zŸÃ®¥©ƒÃ•®ÃzᲟ¯•§ŸƒÃ¥ŠÃ5Ià Z Ãu‘ƒÃ¯¥¯zŸÃ®¥©ƒÃ•®Ãz烩Šƒ¯Ã®¨²z©ƒLà [ Ãu‘ƒÃ¯¥¯zŸÃ®¥©ƒÃ•®Ã;Là ] Ãu‘ƒÃ¯¥¯zŸÃ®¥©ƒÃ•®ÃzÊz¯¥©Ã¥ŠÃ).à 8 u‘ƒÃ z©‚|¥z©‚Ã|ƒŸ¥¶Ã z¢Ã |ƒÃ ²¯Ã¥²¯Ãz¢‚Ê¥Ÿ‚ƒ‚ï¥Ã ¡zšƒÃ zà ²|ƒà x‘•‘ÊzƒÃ ¶•ŸŸÃ¯‘ƒ¢Ã|ƒ ¥§§¥®•¯ƒÃ¯‘ƒÃŠzƒÃ¡z©šƒ‚ÃTR



i£ TÃ



S ÃV JÃ W Ãi[£ Z Ã\IÃ [ Ã^IÃ ] Ã





:" b¥¶Ã¡z¢»Ã¯©•z¢Ÿƒ®Ãz¢Ã»¥²Ã¢‚բÊ¥ŸŸ¥¶•¢Ã²©ƒR 4





ˆ£

g£ n£ f [£ h 8[£ j B[£

[ 5IÃ ] 8



W$

r²§§¥®ƒ )à l±



8£ B£ –= ™›£ W± W± ™ ›£ › –£ – ™ ›£

›™£ x‘z¯Ã•®Ã¯‘ƒÃµz²ƒÃ¥ŠÃ c£



f£)Jà h£8Z£ j£BZ£ )%à !Ãr²§§¥®ƒÃ –=

[ Ã5IÃ ] Ã8

 6B–£ £)à W±/$£ x‘z¯Ã •®Ã¯‘ƒÃz®¯Ã‚••¯Ã¥ŠÃ–F£ £– Fc

f£)KÃ h£BZ£ j£8IÃ

[ ÃOZ£ ] ÃW$£

))" c¢Ãz突z¢ƒÃTVYà •¯Ã•®Ã •µƒ¢Ã¯‘z¯ÃTVà 8T )Ä¡à VYà W± 8//O£„¡Ãz¢‚ÃTYà W±x± „¡à ¶‘ƒ©ƒÃ x±•®Ãz¢ •¢¯ƒƒ©à [ƒ¯ƒ©¡•¢ƒÃ¯‘ƒÃµz²ƒÃ¥ŠÃ x!

£)8£

). _•¢‚õz²ƒÃ•¢Ã¯‘ƒÃ®•¡§ƒ®¯ÃŠ¥©¡ Ã¥Š )2 _•¢‚õz²ƒÃ¥Š



68£2

8//O=£ £8//V=£ 6WWB=£6WW8= EU

5à )5 _•¢‚Щƒz¯ƒ®¯Ã•¢¯ƒƒ©Ã4®²‘ï‘z¯

8//O/£ƇB)

 4

)8 r²§§¥®ƒÃ¯‘z¯Ã–£z¢‚ºÃz©ƒÃ¢¥¢¼ƒ©¥Ã©ƒzÃ¢³¡|ƒ©®Ã®²‘ï‘z¯ k±

l±  W™±=

z¢‚à A¶ W±?º



_•¢‚õz²ƒÃ¥ŠÃ –£ º £ à ): ]µz²z¯ƒÃ¯‘ƒÃ®²¡

8//O£ 8//T£ 8//O£ %£ ! ŸŸŸ £ )à ”Ã 8£ 8£”à B£ 8//K£“à 8//O£



6O$

_•¢‚åŠÃ¯‘ƒÃ‚••¯®Ã¥ŠÃ¯‘ƒÃ§©¥‚²¯ ) à )) à ”Ã 8//O£

= 22U£,

QT



)> u‘…•z©z¡Ã®‘¥·®Ã¯·¥Ã•‚…¢¯•zÃ®¨²z©…®à dgik£z¢‚ÃmopsÃ¥µ…©z§§•¢Ã…z‘Ã¥¯‘…©Ã•¢Ã®²‘ z÷z»Ã¯‘z¯Ã¯‘…•©Ã…‚…®Ãz©…çz©z…Ãz¢‚ÃzÕ©ž…Ã¥‹Ã©z‚•²®Ã  . à j¶ „¡Ã¥µ…©…‚÷•¯‘•¢Ã¯‘…®… ®¨²z©…®à _•¢‚ï‘…Ý…¢¯‘Ã¥‹Ã¯‘…z©…à dgik$

6\) x‘…¢Ã.%%<Ã|z©®Ã¥‹Ã®¥z§Ãz©…çzš…‚Õ¢¯¥ÃcƇ|¥¸…®Ã¥‹Ã…¨²zÃ®•½…÷‘…©…ÃcƇ•®Ãz¢Ã•¢¯……©Ã®¯©•¯» |…¯·……¢Ã.%%Ãz¢‚Ã2%%©…Ãz©…Ã…¸¯©zÃ8Ã|z©®Ã©…¡z•¢•¢à _•¢‚à c‚Ƈ ĚĐ r²§§¥®…ï‘z¯Ã ƒ £»Kà W±.%%:à …£ £»Kà W±.%%+Ãz¢‚Æ£ £»Là 7T .%%>Ãz¢‚à ƒ…†£ W±2à _•¢‚ï‘…õz²…Ã¥‹

ƒ£ …£ †£ )à )à &à   $£ …‰£ †ƒ£ ƒ…£ ƒ£ …£ ‡£ .) u‘…•z©z¡Ã|…¥·Ã®‘¥·®Ãz突z¢…à dgi£•¢Ã·‘•‘à qd£ W± :%¾ÃVmÃz¢‚Ãgm£¯©•®…¯Ã qdgiZ£z¢‚ YmÃz¢‚à im£°©•®…¯Ã qdig$£ f…¯Ã¯‘…Ãz¢…à qgxm£|…à k,Ÿ± _•¢‚à »&Ã



¤Ã



88*

r²§§¥®ƒÃ¯‘z¯Ã–£ ™£ 7T

8B*

b¥¶Ã¡z¢»Ã¥©‚ƒ©ƒ‚çz•©®Ã ¥ŠÃ •¢¯ƒƒ©®Ã  ’ ££ ö‘ƒ©ƒÃ /£ ]£ 8//V9£ £’9£^£8//O9£ ££-Q

8G*

cŠÃ–£ £ %¶ £™£ 6T W£z¢‚Ö9£ £–™£ £Š£W±8O £Ž¢‚õz²ƒÃ¥ŠÃ–

8I'

S§§ƒ¢‚•¢Ã¯‘©ƒƒÃ‚••¯®Ãz¯Ã¯‘ƒÃƒ¢‚Ã¥ŠÃ8//O £¥¢ƒÃ¥|¯z•¢®Ã¡Ã •¢¯ƒƒ©ÃÑƇ¥ŠÃ®ƒµƒ¢Ã‚••¯®à c¢Ã¥©‚ƒ© ¿¯¥Ãƒ¯Ã ÑƇ¯¥Ã|ƒÃ¯‘ƒÃ¡•¢•¡zÃ¢²¡|ƒ©Ã¶‘•‘ծ•µ•®•|ƒÃ|»ÃB £I£z¢‚à O£®•¡²¯z¢ƒ¥²®»ö‘z¯Ãz©ƒ ¯‘ƒÃ¯‘©†ƒÃ‚••¯®Ã¯‘z¯Ã¥¢ƒÃ¶¥²‚Ãz§§ƒ¢‚Q

8K*

_•¢‚Ýz©ƒ®¯Ã•¢¯ƒƒ©Ã£Ã®²‘ï‘z¯Ã “L19 5£ `à 8//O911S*

8O*

_•¢‚õz²ƒÃ¥Š

¶‘ƒ¢Ã–£ W±

3*£_•¢‚õz²ƒÃ¥ŠÃ–H£ ˜A£ –A™£ B”™£ £B˜9£ £* ’£ ]£ £Ã ]£ 8//V£®z¯•®Š»Ã¯‘ƒÃ ƒ¨²z¯•¥¢ %¶ £™,

–H£ K–A£ 8–9£ £3V–£ £8B£ –9 V–£ £3I£

3W V£u1

8V*

_•¢‚õz²ƒÃ¥ŠÃ ƒ£®²‘ï‘z¯Ã¯‘ƒÃ¯¶¥Ãƒ¨²z¯•¥¢®Ã–9£ ƒ– £3£ W±/£z¢‚Ö9 – ƒ^£/£‘zµƒÃ¥¢ƒ ¥¡¡¥¢Ã©ƒzÃ©¥¥¯

8W,

k‚‚Õ¢¯ƒƒ©®Ã®¯z©¯•¢ÃŠ©¥¡Ã 3£z©ƒÃ©¥²§ƒ‚Ãz®ÃŠ¥¥¶®Aà  3 £B £I £O £W £3 3 £3B £3I £3O £3W £# * *  ¶‘ƒ©ƒÃ ¯‘ƒÃ £¯‘à ©¥²§Ã ¥¢®•®¯®Ã ¥ŠÃ £Ã ¥‚ §¶ –¢¯ƒƒ©®#à b¥¶Ã ¡z¢»Ã ¥‚‚à •¢¯ƒƒ©®Ã z©ƒÃ •¢Ã ¯‘ƒÃ ®z¡ƒ ©¥²§Ã¶‘•‘à 8//O£|ƒ¥¢®Ã¯¥Q

B/*

c¢Ã tgi£rgdi£ 6T G{v*£k£•®Ãz祕¢¯Ã¥¢Ã gi£®²‘ï‘z¯Ã dk£•®Ã§ƒ©§ƒ¢‚•²z©Ã¯¥Ã gi$£ cŠÃgk£ 7T B ‡¡Ãz¢‚Ãki£ W± 8£‡¡Ãz¢‚Ãz©ƒzÃ¥ŠÃ¯‘ƒÃ tgi£•®Ã–£¡9'£`•¢‚õz²ƒÃ¥ŠÃ–* tÃ



¥Ã



2) c¢Ã hVYÃ|ƒŸ¥¶ ÃTVÃNà TYÃNà µ{Ÿ²ƒÃ¥ŠÃV\ÃÀ\Y

¶{¢‚Ã\ •®Ã{à §¥•¢¯Ã¥¢ÃVYî²’ï’{¯ÃT\Nà )à `•¢‚ï’ƒ







ˆ£

2. `•¢‚ß{®¯Ã‚••¯Ã¥ŠÃ 89:-.N£ £) 22 `± ¯’ƒÃŠ¥ŸŸ¥¶•¢Ã‚•{©{¡à TVY\Õ®Ã{{©ƒà {¢‚Ã^֮⯃©Ã¥ŠÃ¯’ƒÃ®¨²{©ƒÃTVY\à mÕ® {à §¥•¢¯Ã¥¢Ã{••©ŸƒÃ¶•¯’•{¡ƒ¯ƒ©ÃTVà oà •®Ã{祕¢¯Ã¥¢Ã{••©ŸƒÃ ¶•¯’•{¡ƒ¯ƒ©ÃT\ i¥©ƒ¥µƒ©à o Tà {¢‚à m{©ƒÃ ¥ŸŸ•¢ƒ{©Ã¯’{¯Ã•®à ¯’ƒ»Ã {©ƒÃ ¥¢Ã¯’ƒÃ ®{¡ƒÃ Ÿ•¢ƒ à r²§§¥®ƒÃ oTÃNà )5 „¡ÃTmà Nà 5:Ä¡Ã{¢‚ÃT^ÃNà »Ã ˆ¡à `•¢‚õ{Ÿ²ƒÃ¥ŠÃ º%Ã

ST



¥Ã



25 `•¢‚î¡{ŸŸƒ®¯Ã§¥®•¯•µƒÃ•¢¯ƒƒ©Ã£Ã®²’ï’{¯Ã££Ã £) £Ã £8£•®Ã‚•µ•®•|ŸƒÃ|»Ã.5; 28 `•¢‚ß{©ƒ®¯Ã•¢¯ƒƒ©ÃcƇ®²’ï’{¯Ã|¥¯’ÃcƇ £5?:Ã{¢‚ÃcƇ £..5Ã{©ƒÃ§ƒ©Šƒ¯Ã®¨²{©ƒ®



8^oSBws|OŠ2B€XOkB€^IBfŠ 8sI^O€„Š h›‰~¢Ÿ¥†¶b~ªŒ†˜~ª~–¶e–¯˜¢~ƒ¶hbe¶ D33W  !" "! "

) S¢®Aà W k¢»Ã ±‘…à z®±Ã ®±z±…¡…¢±Ã •®Ã ¥©©…±Dà y€-à ^à }…0à •¡§•…®Ã €0à kà %à ‘…¢…Ãyà ^à €à _¥©Ã ¥±‘…© !î±z±…¡…¢±®Ã¥²¢±…©…¸z¡§…®Ãz¢Ã|…ñz›…Ãz®Ãyà W±.Ã}à W±)IÃyà W±%Ãz¢‚Ãyà PÃ%酮§…±•µ…» . S¢®Bà j W…z²®…Ã.œ0Õ®Ãz¶z»®Ã…µ…¢ñ‘²®Ã.%%;à Ã.œ0Õ®Ãz¶z»®Ã¥‚‚à _¥©Ã¥±‘…©Ã®±z±…¡…¢±®౑…©ÃœÃ W±) ¥©ÃœÃOÃ%Еµ…®ÃzÃ¥²¢±…©…¸z¡§… 2 S¢®Aà [ w®…Ãc¢²®•¥¢Ãz¢‚Ã]¸²®•¥¢Ãn©•¢•§… 5 S¢®Aà  S u‘…ⲡ|…©Ã¥ŠÃ¥²©©…¢…®ÃŠ¥©Ã…z‘Ã…µ…¢±Ã•®Aà )8ÃW £;Ã:Ãz¢‚Ã).酮§…±•µ…» 8 S¢®Aà [ e²®±Ã•¡z•¢… : S¢®Aà [ e²®±Ã ¥²¢±Aà fz|…Ã ±‘…à …¢±…©à +"± u‘…©…à z©…à :à ±©•z¢…®Ã •›…à tnw[£ 2à •›…à twg\£ :à •›… tgk£z¢‚Ã) •›…à tgi%£ u¥±zAà ): ; S¢®Aà W g…±yOà ))¹ z¢‚ }à W±)2¹à x… ‘zµ…y3 }3à W± y } 3à r•¡§•Š»Aà 2y}y } à W±%à q…§z•¢ yÃ}Ã|z›Ã•¢Ã±…©¡®Ã¥ŠÃ¹ö…Ê¥²¢‚ñ‘…ñ‘©……饥±®Ãz©…Ã))Ã).Ãz¢‚Ã)2à u‘²®Ã¹,ÃƇ¹-à ù4à PÃ2: > S¢®Cà W r•¢…ñ‘…Ãz©…zÃ¥ŠÃ®…±¥©Ãdkn£z¢‚à tgi£z©…Ã…¨²zö…Ózµ… )à l± = à 0

DU



:¶ no¶ < :¶ u‘…é…®²±ÃŠ¥¥¶®Ã



W$£ S¢®Aà W à B£ 3£ 8£ 3£  _£ z¢‚à _£ –£ › –£ –£ ™ ›£

¯ƒ®Ã²®Ã



›£ ™£ ›£  £ 8£z¢‚à £3£_£B$£ 7T







u‘²®Ã ™£_£/£z¢‚Mà _£>$£ j¦¯ƒÃ¶ƒÃ‚•‚¢¯Ã²®ƒÃ¯‘ƒÃŸz®¯Ãƒ¨²zŸ•¯»Ã|²¯Ã –£_£Z™£_£/£z¢‚Û£ 8T 8£ HT ®z¯•®‹»ÃzÃ¦¢‚•¯•¦¢®Ã

3/$£ S¢®AÃ  [ Ã &£



W»Ãz®®²¡§¯—l¢Ã –£ 



3£ _£3B$£ u‘²®Ã 9 £ A±

7T



¶‘¦®ƒÃŸz®¯Ã‚••¯Ã•®Ã O$£

F 3B 8£ 3KO$£ r•¡•z©Ÿ»à –£ £ :T

:T

8T



_£3KO 8 ;T

3 3$£ S¢®Aà 8//O$£ w®ƒÃƒ£ ^à 8//O£ £3£z¢‚à 8//O£^à ƒ£ £3$£ 38$£ S¢®Aà K'£ w®ƒÃ 8 3 £¿Ã 38£34¶_£¶!¶¾Ã B9  3B'£ S¢®Aà B////$£ w®•¢Ãƒ9 …9£_ ƒ £…ƒ … £8//V9£Q± 3WWB9£ G0/3£¹Ã 3I£z¢‚à 8//O9 3WW89£_£BWWW£“à 4I$ 5T

u‘ƒÃ©ƒ®²Ÿ¯Ã‹¦ŸŸ¦¶®Ã

3G$£ S¢®Aà 8//8&£ W»Ã¦¡§Ÿƒ¯•¦¢Ã¦‹Ã®¨²z©ƒà 8//O9£ 8//O/£ £B 3 £_£8//O£

I9£ £K$£u‘ƒÃ©ƒ®²Ÿ¯Ã‹¦Ÿ¦¶®Ã

4I$£ S¢®Aà 883G$£ ]Ÿ•¡•¢z¯•¢Ã – £ ¶ƒÃƒ¯Ã B™9£ _£V™'£ u‘ƒ¢Ã ™£ _£8O£®•¢ƒÃ ™£ X± ‹¦ŸŸ¦¶®Ã

/$£ u‘²®Ã –£ _£8 3VO$£ u‘ƒÃ ©ƒ®²Ÿ¯

3K$£ S¢®Eà 8//K$£ 3£ 3£ 3£ ¶ƒÃƒ¯Ã w®•¢Ã _ ŽžŽ £ Ž£ Ž £ 3£ 3£ 3£ £ 3£  3£ £ 8//K$£ 8//O $£ 3 £   £$ $ $ £ 8£ 8£ B£ >//K£ 8//O£







8T

3P(

S¤®Aà 3V/KB$ r•¤ƒÃ¯‘ƒªƒÃ•®Ã¤¥Ãz«ª»®Ã•®Ã®•¡§»ÃX£¹Ã 8//O)£ kªÃ¥|®ƒªµƒÃ•¯Ã•®Ãz¯²z»

888£XXXXXXXXX£ XXX£PPP£ +U R

R

U

+U

K??QÃ X ÃPT

3V$

S¤®Aà 3$ $S®Ã¯‘ƒÃ‚•zªz¡Ã|ƒ¥¶Ã®‘¥¶®Ãx£`£ $ž¡¶z¤‚Ã}£ 4T wx

x}£ x£ £}£`£ 3T

8 "£ 8 "£ wx£`£ 8£ 8£ 8£

`£3)

‘Ã

±Ã

3X$

¡x$£ u‘²®

S¤®Fà 8VK) X»Ãz®®²¡§¯•¥¤à 8//O I£`£c Ž£‹¥ªÃ®¥¡ƒÃ•¤¯ƒƒªÃŽ$£ _z¯¥ª•¼ƒÃ 8//8£`£8£ 8//£^à cƇ^à B// £¯‘ƒÃ¥¤»Ã§¥®®•|••¯»Ã•®ÃcƇ`£8£ 3 3 £ 3B£`£8VK£z¤‚à Ž£a£P$ PU

OU

8/$

PU



QU

S¤®Cà 3* qƒ¶ª•¯ƒÃ¯‘ƒÃƒ¸§ªƒ®®•¥¤Ãz®

ƒ9£ £…9£ £†9£ …‰£ †ƒ£ ƒ…£ ƒ£ …£ Kà ££… £ †£9£ ££†£ ƒ£9 `£ ƒ…†£ 8ƒ…†£ r•¤ƒÃ ƒ …£`£ 3 £… †£`£ 3 £z¤‚Æ ƒ`£8 £¯‘ƒÃªƒ®²¯Ã‹¥¥¶®à 3/£

QU

 Ã

NU

3B$£ r•¤ƒÃ

8 3)£ U¢®Bà I/$£ 8£ 8£ pgxi£`£3V/œ pxgi£ £pxig£`£3V/  pdgi£ pdil`£3V/ D 38/£`£3//* k|®ƒ©µƒÃ¯‘{¯Ã xm£|•®ƒ¯®Ã pgxi £¯‘ƒÃ©ƒ®²Ÿ¯ÃŠ¥ŸŸ¥¶®Ã

88$

U¢®Bà 3$ xƒÃ¡{¢•§²Ÿ{¯ƒÃ¯‘ƒÃƒ¸§©ƒ®®•¥¢Ã{¢‚郧Ÿ{ƒÃ–£ ™£|»Ã 3£¶‘ƒ¢ƒµƒ©Ã¢ƒƒ®®{©»B

–F£ ˜C£ –A™£ B–= ™£ £B˜=£ ££ –– ™ ™A£– ™ B–™– ™£ `£ – ™;£ £—™£ £• B—™ `£ –  ™=£ 3$£ *U

3T

3T

8B$

U¢®Bà B$ r•¢ƒÃ 8//V9 8//O9£ 2T “=£ ’= £¶ƒÃ‘{µƒÃG/3I£3T “ ’“ ’ £•ƒà I£Ÿ 33ž£OB£`£“ ’“ ’$ u‘ƒ©ƒ©ƒÃŠ¥²©Ã§¥®®•|•Ÿ•¯•ƒ®Bà “£ £ ’£ b£ G/3I £“ ’£`£ 3Z£“£ £ ’£`£ V/B £“ ’£`£ I\£“£ £ ’£ b BKI “ ’£ 3T 3 3 £{¢‚à “£ £ ’£`£ OB “ ’£` II$£ W²¯Ã ¯‘ƒÃ Œ˜¬®¯Ã ¥¢ƒÃ “£`£ 8//V £ ’£`£ 8//O£•® ©²Ÿƒ‚à ¥²¯Ã|»Ã{®®²¡§¯•¥¢à ©ƒ¡{•¢•¢Ã§{•©®Ã {©ƒÃ “£`£ G/G £’£`£BWW[£“£`£ 3VV £’£`£ 3OO£ {¢‚

“£`£KG ’£`£W' .5 U¢®Bà B$

r•¢ƒÃ– £š ™– u‘ƒÃ©ƒ®²Ÿ¯ÃŠ¥ŸŸ¥¶®

8I$

š ™£`£– ™=£š=£`£–= —™ Š £¶ƒÃ‘{µƒÃWŸ– š ™£ 8O& 2T

U¢®Fà /OI$ d¯Ã®²ƒ®Ã¯¥Ã¡{›ƒÃ•¯Ã‚•µ•®•|ŸƒÃ|»Ã 3/I£{Š¯ƒ©Ã{§§ƒ¢‚•¢à U®Ã8//O///£`£3/I£¹Ã 3W3 3G£ £B/ £¯‘ƒ Ÿƒ{®¯Ã¢²¡|ƒ©Ã¯‘{¯Ã¶ƒÃ¢ƒƒ‚ï¥Ã{‚‚ծà OI$

8K$

U¢®Aà 38+ xƒÃ ¢ƒƒ‚à ¯‘ƒÃ¡{¸•¡{ŸÃ “£®²‘ï‘{¯Ã “E?11Q£ ^à 8//O?11R$£ xƒÃ ¢ƒƒ‚à “E£ ^à 38A£`£ 3O8V£^à 8//O£^à 8 3WO£`£3BA$£ u‘ƒÃ­ƒ®²Ÿ¯ÃŠ¥ŸŸ¥¶®

33£

8//O$£W»Ã{Ÿ²Ÿ{¯•¥¢

.; S¤®Aà 8 k~®ƒªµƒÃ–£ 2T Ÿ¥¤Ã‚•µ•®•¥¤Ã

#9£ W±G ¶z¤‚Ö9 V–£ £)2ÃW±)? V£ 2.Ãà V£¶à )2à W±%à w®ƒÃ –F K–A 8;£ £3V–£ £.2Ã

2T

V–£ £)2 C¿à .–£ £) à £)%

C

v‘²®Ã¯‘ƒÃ¥ª••¤zŸÃƒ¸§ªƒ®®•¥¤Ãƒ¨²zŸ®Ã 1 V–£ £)2 Âà £8–£ £) à £)%Ã

–9£ V–£ £)2à £.

)%Ã





W±8

MU

.> S¤®Aà . w®•¤Ã ƒ– £)à W±– w± ¶ƒÃ‘zµƒÃ –£ 6T )Ã¥ªÃ ƒ£ W±)ÃW²¯Ã¶‘ƒ¤Ã ƒ£ W± )媕•¤zŸÃƒ¨²z¯•¥¤ ‘z®Ã¤¥ÃªƒzŸÃª¥¥¯®à v‘²®Ã –£ W± )öƒÃ‘zµƒÃ ƒ£ W±. .? S¤®Aà 58 .%%;ծÃ)Ã%%5¯‘Ã¥‚‚䲡~ƒªà cŠÃ.%%;ծդЪ¥²§ÃŽ£ £)¤Ã)à £.à £ )à £.à £*   ÃŽ£ £) à v‘²®

MU

ŽŽ£ £) Ã

_Ã )%%7Ã

£Ž£_à )%%5à GHÃ

MU MU

Ž£ £4¥’ƇƇ.

 .à .à xƒÃƒ¯Ã ’Ƈ W±55à r¥Ã.%%;ծդЪ¥²§Ã58ö”™‘Ñz®Ã58Ã¥‚‚Õ¤¯ƒƒª® _Ã

2% S¤®Aà )8 Z¥¤®¯ª´¯Ãim£¶‘•‘ծ烪§ƒ¤‚•²ŸzªÃ¯¥Ã eg£¶‘ƒªƒÃ m£•®Ã¥¤Ã eg*£ r•¤ƒÃ pgei£ 8T 5| DT •µƒ¤  em£W±im'£ v‘²®Ã¯‘ƒÃª•‘¯Ã tmo£•®Ã¥¤ª´ƒ¤¯Ã¯¥Ãª•‘¯Ã 7img£r¥Ã eo£W±ig£W±8 jƒ¸¯Ãtkg£•®Ã®•¡•ŸzªÃ¯¥ÃMiko £¯‘²®

gk£ ok£ ek£ eo£ 2à ek£ J£ W± ¶‘™‘Õ¡§Ÿ•ƒ®Ã W± W± . ek£ ek£ ik£ ¶ƒÃƒ¯Ã ek9£ |ek£ K£W±%Ãt¥Ÿµ•¤Ã ek£ K£¥¤Ÿ»Ãz®Ã ek£kà %à v‘ƒÃªƒ®²Ÿ¯ÃŠ¥ŸŸ¥¶® 

8T

T T T

à ~à à à à à T

à T T



).Ã

\!™ƇƇ! ƇƇ $¹$@Ƈ aƇ@! !k@*ôƇ! Ƈk$*Ƈa$Ƈ$ Ƈ $Ÿ$Ƈ!! Ƈ$Ƈ!!`Ƈ ÎƇ  Ã $ ċƇ  !Ƈ pldk£$ Ƈ pkdi£@mQ*ÃƇ r Ƈ$ Ƈ„ £ž¿ W±$ "ńĿƇ 7T 4™Ƈ! Ƈ


)T CT

9T 

W±4&

mynyÃ

Q! ? *Ƈdk£ W±K' «4 ) ĥƇ<$ ®! @žQƇ$ƇQ@QƇaƇd£$ƇaƇQ @ƇŐ Ƈdg£W±di£ W± m¶$ƇaƇ@$ ƇijR Ƈdk£!Ƈk aƇQ@Qk@ QƇ$Ƈ s£$ Ƈu£$Ƈa! Ƈ

(T ' &T %T @AT >T ?T

…à †Ã ‡Ã €Ã

Ã  0123à ˜Ã ˜Ã ˜Ã ˜Ã ˜Ã ˜Ã ˜Ã ˜Ã ˜Ã ˜Ã

'()Ã*+,Ã

(U

 T

à à KT JT

™Ã ˜Ã ˜Ã —à ˆÃ

tF¶





‚à „à ŠÃ ‹Ã ‡Ã ŒÃ …Ã

Ã ƒÃ ‰Ã ˆÃ šÃ ˜Ã ˜Ã ™Ã

?T >T

JT

tà 

T

*T 'T

xÃIT T

IT

*+,-./Ã  



' (T

4567Á:Ã 



 

ŽÃ

r ƇaƇĸ @Q Ƈ ®a!@ ƇUa!@kƇ

gk£žki^£skž£uk£ W±Ƈ# 4¥Ƈ"¶ £4¥Ƈ W±<$ « ) `Ƈ O$ P@Ƈa$Ƈ!@Ƈ ¬Ã @±< £<=£$$*Ƈ Ƈ aƇ$Ƈ K$£ UaƇ@Ƈ!!

4«Ƈ

22 ) ĦƇ 25

ĺ! Ƈ yk£ $ Ƈ xg£  Ƈ Ƈ $*Ƈ xg£`£yd£`£ & 5

!Ƈ  !Ƈ $ Ƈ @ Ƈ ¢kyd£ Ƈ Q! @ Ƈ !Ƈ @ Ƈ txg%£  

)mmZ*Ƈł* $!@$åƇ  !@kƇ !Ƈ txg

£€–9£`£5:-à £ & G 9 ²!Z™Ƈ –£`£25



–Ã –Ã –Ã –Ã –Ã –Ã –Ã –Ã –Ã –Ã –Ã –Ã

ST



¤Ã



25 ) `Ƈ 2;

² QƇ .5;Ã`£ &2 & ?à ! Ƈ!Ƈ ¬à ¬Ã £ &ìà £8£Ƈ ZƇ*Ƈ &2à Q$ZZƇ Ƈƒ£$ Ƈ! Ƈ*Ƈ & ? ÃQ$ZZƇ Ƈ…$ ®Z$@Z*Ƈ ‘…£ ƒ‹£ £± . LU

ľ Ƈ …£`£ & ? …à ³  Ƈ …¿`£ &à  QƇ ‘… ƒŒ£ £± .à ƒ£ Ƈ $k! Ƈ & ;Ã& >Ã& ?Ã.%Ã. &  à ; Ƈ ! Ƈ

ZƇ*Ƈ &2à  QƇƇ @*Ƈ …¿`£ .à \!Ƈ …£`£ 2>à $ Ƈ ƒ£Ƈ $k! Ƈ 2:Ã2;Ã2>Ã2?Ã5%à  Q ƒ`£2?à I Ƈ ƇZ$ Ƈ¬Ã`£2; 28 ) `Ƈ 5.:8

Î Ƈu£à 5?:à 0T ƒ9£$ Ƈu£ £./5à 1T …=£!@Ƈ!kƇm! Ƈ Ƈw±$ Ƈ…$£   Ƈ ƒ@  …@£`£5@:à 7T ..5Ã`£ .;.Ã`£ .6à & ;à I Ƈ &;'yà £…ƒ …$£ {Ƈ & ;'y y±  Ƈ ƒ …£ BC± & ; Ã$ Ƈ ƒ £…£ £± & : km!ZÃƇ  Ƈ & ;(y à £…% OU

³Ƈ $ƇřťƇm!Z Ƈ!@Ƈƒ £… £ƒ …Y£  &;Ã& : Ã25Ã> Ã:>Ã5 à &2:Ã. Ã.;.Ã& à ²!Z $ Ƈ Q$@ Ƈ ! Ą @Ƈ!Z ! ™ƇƇ $Ƈ ƒ £…£ 5T . &  Ã&2 Ã2:Ã2. à $ Ƈ :?Ã:; à  Ƈ  Z$@ Ƈu£Ƈ:?-5?:Ã`£5.:8 &5Ã

5\mQ@uqyMŠ 0@~UMh@~\G@dŠ 5qG\M~‚Š 6]nRAvrzNŠ1AVNiA]HAeŠ3eƒiv]AKŠ 613Š B22V  " "  " "  !" "  "  "

  "

 " "" 5 ";AD1?M"((M K± >C1AB5<;A 9) *4C1AB5<;M.,??51AM 5*± :,?8  )<M .,9.C9,B
 G…¢’vo¢ q €€‡–{…u¢tu”o¢ EN¶ ]^¶ RPµ¶ EN¶D¶ i¢ PRD¶ ŸÃ g…m¢f4¢ i¢ So’¢^¢ ho¢’wo¢gog ‡q ’vo¢ ’gŠo{”ƒ¢ENPR.¶ _”ŠŠ‡o¢ ’vo¢gog¢ ‡q¢ =NdP¶ {¢  # C{…m¢ ’vo¢•g€”o¢ ‡¢q f}i wÃ

£Ã





 BŒ”{€g’og€¢’{g…u€o¢ENT¶ g…m¢NPW¶ gŽo¢ ook’om¢ o˜’o…g€€š¢ ‡…¢ ’wo¢ {mo¢EN¶ g…m NP¶ ‡¢q g¢Šgg€€o€‡ugƒ¢ENPR.¶ [‡•o¢’vg’¢=RUW¶ {¢oŒ”{€g’og€¢  So’¢# jo¢ g¢ Š‡{’{•o¢ {…’ouo¢ g…m¢~± hp¢ ’vo¢ uog’o’¢l‡ƒƒ‡…¢ m{•{‡¢ ‡¢q # ± M g…m ##¢M  ± # C{…m¢g€€¢’vo¢Š‡{h€o¢•g€”o¢‡q¢~± J”’{q š¢ š ‡”¢g…–o  avo¢m{soo…ko¢ ho’–oo…¢ ’vo¢ Š‡m”k’¢ g…n¢ ’vo¢ ”ƒ¢ ‡q¢“–‡¢ m{soo…’¢ {…’ouo¢{¢ oŒ”g€ ’‡¢ ’wo¢ ”ƒ¢ ‡¢q ’vo{¢D?@¢  uog’o’¢ kˆƒƒ‡…¢ m{•{‡¢ g…m¢S?U¢  €og’¢ k‡ƒƒ‡…¢ ƒ”€’{Š€o  ¢C{…m¢g€€¢’voo¢Šg{¢‡q¢ …”ƒho¢ K”’{q š¢ š ‡”¢g…–o¢ ,  ¢ C‡¢g…š¢ Š‡{’{•o¢{…’ouo¢# €o’¢# mo…‡’o¢’wo¢# Š‡{’{•o¢…‡…Œ”go¢{…’ouo¢ { o¢  #5¢# #5¢# #5¢,¢ #5¢-¢ o’k¢ [‡•o¢’wg’¢ #### #¢ž¢k³¶ –voo¢ž¢¸³¶ mo¡‡’o¢ ’vo¢ {…’ouo¢ k€‡o’¢ ’‡¢¸$à C‡¢ o˜gƒŠ€o¢ ž¢i³¶5¢ M ž¢_³¶D¶ 

ž¢­³¶D¶ # ž¢6±¶ # AU

M

5\mQ@uqyMŠ 0@~UMh@~\G@dŠ 5qG\M~‚Š 6]nRAvrzNŠ1AVNiA]HAeŠ3eƒiv]AKŠ 613Š B22V  " "  " "  " 

ec{’v‡”’¢ €‡¢ ‡¢q uo…og€{’š¢ €o’¢^¢5¢ # `vo…¢qNdZw¶# #  ¢ _{…ko¢rEORx¶5¢rENQx –o¢vg•o¢qEdRw¶5¢rOdZw¶5¢#  ¢ So’¢rRdQw —¢ g…m¢qEdNw¶# ™ ¢ `vo…¢—#¢5¢#™ { o¢ —™¢# *¢ :€‡¢—¢± ™¢# ,¢ `v”¢—, —¢5¢*  ¢ _‡€•{…u ¢ –o¢uo’¢—# ¢  ¢ g…m™5¢*  _{…ko¢=RdQ¶ =NdE¶ –o¢vg•o¢—}›¢# f$¢}i$¢ `v”¢fi¢# # ?U

JU



#  ¢ do¢vg•o¢

¢Ã

`TOX¶ #*Ÿ¢ `EOQ¶5¢#*Ÿ¢  0Ÿ¢ `OQR¶ 5¢-Ÿ¢¢`NQR¶5¢`RQW¶ @U

:€‡¢WV¶# XQ¶ g…m¢NT¶# OE¶5¢QR/¶ `v”¢=WNT¶ >XQR/¶ `voor‡o XT¶ X¶R/¶ _{ƒ{€g€š¢=T¶ER¶ =RQ¶X/¶ `voor‡o¢TR¶5¢R¶W0¶ `v”¢=R¶T¶W¶ { oŒ”{€g’og€¢ JU

AU

IU







 C‡¢†¢# ¢ g…m¢†¢# #  ¢ ’vo¢ ukm¢ go¢ ¢ g…m¢, ¢ oŠok’{•o€š¢ :…š¢ k‡ƒƒ‡…¢ m{•{‡ ~± ‡q †&¢ ± ¢ g…m¢†¢± $¢ ± ¢ m{•{mo¢ ’vo{¢ m{soo…ko¢ #†¢¢  ¢ Eo…ko¢~± m{•{mo *†%¢¢  #†¢± #† ¢# ,¢ `v”¢’vo¢‹‡{h€o¢•g€”o¢go¢ ¢ g…m¢,



*¢

9V~ ~\V ]o~V[Vx| OV † MoTˆ MoT M||ƒlV ~\M~ †

jȍ

.]x|~ …V os~V ~\M~ † MoTˆ MxV

Os~\ osoŠVxs MoT ~\M~ ~\V]x 0*, MoT 9+; MxV Os~\ us|]~]„V O‰ TVZo]~]so 9V~ 2 OV ~\V 0+,  ~\Vo f†g

>U

<N MoT

[l\¶"=^P  …\VxV N MoT P MxV Rsux]lV ]o~V[Vx|

E\ƒ| ~\V 9*; sX † MoTˆ ]| <NQ 4Xˆ"   ~\Vo <#  MoT ]~| VM|]h‰ R\VRdVT ˆ# < < ˆ E\ƒ| †ˆ †   NP ~\M~ ~\VxV ]| os |shƒ~]so I\Voˆ jà   †ˆ jà †  'X~Vx |ƒO|~]~ƒ~]o[ Xsx † MoTˆ MoT |]luh]X‰]o[  …V \M„V

NP< " N P

 ]à < *  ¢

YZÃ

5X <"  ~\Vo NP N P#   ]V  N P #  E\ƒ| N#   P#  sx †".¢ ˆ"*  ¢ D]l]hMxh‰  …\Vo 3

>U

  …V [V~ NP N P" ;ƒh~]uh‰]o[ ~\xsƒ[\sƒ~

O‰  MoT ~\Vo XMR~sx]ŠV  …V [V~ N P #  …\]R\ []„V| † # .¢M oTˆ" Lb\Vo <# *¢ …V [V~ †" ˆ"*¢ …\]R\ ]| xVcVR~VT M| † MoTˆ MxV T]|~]oR~ >V‡~ …V Rso|]TVx ~\V RM|V †



¢jȍ

Mx[ƒlVo~|  …V [V~ † ˆ †ˆ

>U

E\Vo † #
‰Š …\]R\



< 
‰]VhT|  3  I\Vo <"  …V [V~ N" P E\ƒ| ~\V |shƒ~]so| MxV P"  N$   sx †#    ˆ"    …\VxV ©¶S¶c+¶ I\Vo #   ~\V VwƒM~]so |]luh]ZV| ~s N P  "¢ E\ƒ| …V [V~ N" MoU P MxO]~xMx‰ M| ~\V soh‰ |shƒ~]so E\V |shƒ~]so| MxV †" ˆ"    …\VxV S¶c*¶ .]oMhh‰  …V Rso|]TVx ~\V RM|V
>U

¢ jà à jȍ

1VxV † "„±N ˆ"
< 


<  ~\VxV ]| os

|shƒ~]so E\ƒ| ~\V |shƒ~]so| MxV

. ¢  . ¢*¢   ± ©¶  MoT    …\VxV ©Sc)¶

,  ¢ Fr¢#$ # e  ~\Vo ~\VxW MxV V‡MR~h‰ e |wƒMxV oƒlOVx| hV|| ~\Mo  # e ]à # ]à e  >s… …V |\s… ~\M~ e# °¶‡±-¶ IV \M„V  e  ### # e #e   1VoRV

%   e  

E\VxVXsxV

|s ~\M~ °¶‡²¶

   

*

>U

 T  ]à # ]à e e e LM   =

 e   >U



e





NO
e#

 e    

  ? ± 4 T e LM

% 





E\ƒ|

4[lP?tpxLŠ/?}TLg?}[F?cŠ4pF[L}Š  # !!# "#  # @00T¿  # !#

}¶±Š»¿A]¿s¼¿A11U¿

1]N1 8A11¿•­±¿

 !!#

"(,'. . MR¿ &*()$"(. ")'. -$*'. "(,'(. $".). "(,'. ().%'$+. $'. ). !* )% . $. &*()$"(. ")'. -$*'. "(,'(. ". ). "(,'. (). -. (". ). * (. $")"". ). ))'(.  . . i¿ j¿ $'. . $''(%$"". )$. ).$'')."(,'(. $'. ). $)'. ($'). &*()$"(. ,'). -$*'. "(,'(. ". "(,'. ().". (. ). %%'$%').* (. $,.-$*'. "(,'( . $.()%(.'.". )$.*(). -$*'. #(,'( . .&*()$". ''(. 7¿ !'. $. * )$'(.'. $,.

#  # #  #  # # #  #  #  # #  #

6Z¿

rµ¡³š« ¿h”©š†¿xµ°³š©§°¿

#J

#J @6$JƇA6#JƇ?JšƇ # £J

F Ƈ ƇƇ!JƇ Ƈ Ƈ!JƇ Ƈm! Ƈ Ƈ )Ƈ f¿

g¿ [ Ã l¿

#Ƈ



#!J 6L¿

)Ƈ$Ƈ! $ Ƈ»Ã  Ƈ$ 9Ƈ ƇƇ )ƇƇƇ Ƈ$Ƈ$ !ƇJ!Ƈ Ƈ $Ƈ$ ƇƇ!!Ƈ ! Ƈ …ƇƇ  Ƈm$ Ƈ !Ƈ Ƈ$Ƈ! Ƈ Ƈ $

! $ƇƇ!JƇ Ƈ$Ƈ!!šƇ )Ƈ! ƇƇƇ RƇ$ !/*Ƈ $ šƇ F Ƈ Ƈm!$*Ƈ $Ƈ Ƈ! ƇƇƇ šƇ

#!J

)Ƈ f¿

g¿ [ Ã

l¿

"

Q¿ S¿

(""& *J



½¨Ã ·½¨Ã ½Ã ·½¨Ã ½Ã ·½Ã

·½Ã ·½Ã ·½¨Ã

³ $ƇƇ Ƈ$ Ƈ  Ƈ Ƈ 

@@@¶´@@¶@@@¶(((¶@@@¶*#Š """Š"""Š"""ŠŠ"""Š*$Š

(""+J

(""+J

W¯

N°

##-J

Ƈ   Ƈ*Ƈ )Ƈ 8£ f¿

g¿ "Ƈ S¿ [ Ã \! Ƈ!J  Ƈ$!¡Ƈ k¿

!J

#@

WAƇ

]=Ƈ Ž ¿ Ƈm±  Ƈn± Ƈ(Ƈ %Õ EƇƇ(ƇƇE EƇƇ%Ƈ);3OƇ qEƇ % Ƈ Ƈ(Ƈ ƇE Ƈmn± ](Ƈ% ƇƇ(ƇƇƇ%Ƈ);3OƇ EƇ ƇqŽ ]‹Ƈ •±



)Ƈ ;Ƈ 3Ƈ OƇ l¿



, Š W,Š  Š ,Š  Š Z±

,_



—±

FƇ EƇƇ  Ƈ(Ƈ·ƇŘŪ¢Ƈŧ EƇƇ(Ƈ EƇƇ‘Ƈ ? ƇEƇE( AƇ ° EƇj¯Ƈ, W Ƈ¶=Ƈ±ŒƇc¿ Ƈ¶=ƇjŒƇ, + Ƈ¶Ƈ †¯jŒƇ †±ŒjƇ + n_Ƈ F Ƈ †¯±Œ ‚ =U

)Ƈ ;Ƈ

 nƇ  ,nƇ

OƇ

 nƇ ,nƇ

l¿

, nƇ

3Ƈ




I)± >)± M)›± ±

M)›± M)±

±

+_

]( Ƈ 2Ƈ EƇƇE%ƇƇ%‘ Ƈ Ƈc¢E¿Ƈ·ƇcƇEƇƇ%EƇ = (ƇEƇƇ ƇƇ,Ƈ DƇ *Ƈ%EEƇEƇƇcƇƇ(‹ W )Ƈ + ;Ƈ 3Ƈ 7 OƇ   l¿

2_Ƈ

>%%EƇ$,  EƇ(ƇEƇƇ EƇŷAƇ FƇ¢%=Ƈ,, W<&ƇžMà , W_Ƈ F Ƈ (ƇƇƇ ,Ƈ,gƇžMà à 'AAAA((__ƇgƇ'&  )Ƈ , 7<Ƈ , <<Ƈ ;Ƈ 3¿ ,  <Ƈ O¿ , <Ƈ , <Ƈ l¿

 Ƈ

7A

U(Ƈ ƇƇ(ƇƇ? Ƈ);3OƇ(Ƈ);Ƈ,  &Ƈ;3Ƈ +&Ƈ3OƇc¿7  ƇO)Ƈ, AƇ U(Ƈ  Ƈ)3Ƈ Ƈ;OƇ ƇƇ(Ƈ% Ƈ+¿ Ƈ(Ƈļ3P;Ƈc W,:AƇ  Ƈ(ƇƇƇ(Ƈ? Ƈ);3O#Ƈ ;U



)Ƈ ;Ƈ 3Ƈ OƇ k¿

<_

7Ƈ L<Ƈ  Ƈ Ƈ Ƈ

{ Ƈ(Ƈ Ƈ vƇ8epIJƇƇƇ?Ƈ(ƇjƇ8  Ƈ†8je

;U

-&ƇjeƇc¿-Ƈ ƇjpƇ , -#Ƈ



ˆ£

eƇ )Ƈ ;Ƈ 3Ƈ OƇ k¿  _

 :Ƈ  :Ƈ ,:Ƈ W :Ƈ W,:Ƈ

](ƇƇ(ƇƇ%Ƈ%Ƈ¡ƇƇN 56NƇƇ2 &Ƈ(ƇNƇ  Ƈ¡Ƈ %¡Ƈ ‹ < )Ƈ 2 ;Ƈ 3Ƈ  2 OƇ 2 k¿

Ƈ

|”©¬³¿xµŽ°³›©§°¿ 44

>%%Ƈ Ƈ5´MVƇ -}ƇlƇMƇ ´VƇ  C}Ƈc¿VƇ ´ M }Ƈc¿ Ƈ F 5 5Ƈ  ƇƇƇ-ƇƇCƇƇ#Ƈ

4

F Ƈ Ƈ Ƈ ƇƇ4Ƈ

4ñ Ƈ

Ƈ '¶ ž Ƈ Ƈ–±Ƈ ˜± bà 4Ƈ Ƈ–˜±Ê, &Ƈs‰ Ƈ ƇRƇƇƇ–5ƇŽ

4W

>%%Ƈ Ƈ4WŻƇc¿ # ęWêŒƇc¿4 Ƈ F Ƈ ƇƇ

K,_

F Ƈ ƇƇƇ K  ƇBHƇ Ƈ4HƇ  Ƈ4+HƇ ƇH é

4+

U ƇƇƇ Ƈ ƇÏ)UDzÏ)U…3>ƇƇŦ  Ƈ Ƈ ƇƇ£Ƈ Ƈ  śƇƇƇƇ Ƈ  ƇƇĵƇ_Ƈ F Ƈ ƇƇ Ƈ

 Ƈ  Ƈ Ƈ ƇƇ Ƈ Ƈ Ƈ£_ \`ƇU ƇƇƇz  Ƈ ƇƇ)&Ƈz&Ƈ…òƇ+¿~

42 Ƈ

ž ƇƇ >ÊƇ—4&Ƈ&Ƈ&Ƈ# #  &Ƈ4<<& Ƈ  Ƈ€AƇ U ƇƇ)ËƇ —-&Ƈ C&Ƈ€ƇƇ > Ƈ  ƇƇ ¿ Ƈ¥—…Œ Ƈ -gƇƇc¿CƇDƇ ƀƇ¥˜…Œ¿Ƈ Ƈ> ‹Ƈ \`Ƈ U Ƈ ƇƇ Ƈ Ƈ  Ƈ ƇƇ Ƈ Ƈ#ƇFƇR%&ƇƇ   Ƈ —-¾Ƈ C§Ƈ €ƇƇ —-¾Ƈ§Ƈ C€ƇƇ —ƇóƇ C§Ƈ -€ƇƇƇ ƇƇ_Ƈ

47 

F Ƈ Ƈ Ƈ  Ƈ „„ƇƇ4ƇƇ   Ƈ£Ƈ

4<

ž Ƈ Ƈ Ƈ  Ƈ ƇƇă Ƈ ƇƇ,7Ƈ  Ƈ ƇƇ

ƇƇ Ƈ•ƇƇ ƇƇ ¹ƇƇ Ƈ&Ƈ s‰ Ƈ ƇƇ ĉ

 

zƇB+Ƈ Ƈ4 »Ƈ Ƈ »Ƈ Ƈ, :Ƈ Ƈ2 »

4

F Ƈ ƇƇ ƇƇ  ƇƇ•ƇƇ ƇƇ¶

Z#ŠRƇ4


Ƈ

3J

z

DU

;<Š

-Ƈ CƇ

,

,  ,K6) , W6) #Ƈ

#Ƈ

>%%Ƈ Ƈ), , Ƈ), Ƈ Ƈ ƇƇƇKK6) F Ƈ Ƈ/ƇƇ ), ,) ,,),

#Ƈ

… Ƈpfg¿ngef¿,  HƇ Ƈoefg¿c¿7 HƇ U Ƈ% Ƈq¿/Ƈ  Ƈ Ƈ /Ƈ  Ƈ Ƈnqeg¿, K HƇ Ƈ nqge¿c¿ H #ƇF Ƈofqg¿ Ƈ ‚Ƈ

W#Ƈ

DƇ *Ƈ%Ƈ Ƈ¬Ã /Ƈ  Ƈ 2Ƈ  ƇƇs‰ Ƈ Ƈ Ƈ

&U

%!% % % %"% ,

,

dà ¬Ã

 Ƈ p¶)v¶ Ƈ ƇƇ Ƈ/Ƈ  ƇƇ•/ƇƇ),

FƇR%/&ƇŸ#B}Ƈ ›Ƈ ŸB}Ƈ , B›Ƈ ŸÀ#B}Ƈ , À ƇAƇ :U

B#Ƈ

… ƇÐep&Ƈ/ 8eƇb !, epƇ 

  8pƇ , Ë

CƇ >%%Ƈ 

,

,

6ą ­ƇKvƇs‰ Ƈ Ƈ



!,

!,

/ƇƇ ƇƇ /ƇƇÐepƇ Ƈ Ƈ

+#Ƈ

F Ƈ Ƈ ƇƇ żƇc±ŗƁŤŴŝƇ’‚± €Œ‡ˆ‰“ƒ‹±‘o”Œ±zŒ‹}ƒ‘ƒŒ‹P± 6¿`¿t¿`¿ ;((OV± v‹}± Ƈ  Ƈt¿Ƈ /Ƈ*Ƈ 2(± ŒŽ± 3<± æŵƇ ‚¨²ƒ Ƈ

2 #Ƈ

>%%Ƈ ,  ƇCƇƇ ƇƇƇ)

, ,),  ƇÃ ,KƇ

, ,), Ƈ Ã

 Ƈ• Ƈ)



Ƈ /Ƈ!, ƇŕƇŽŞƇƇƇ KƇ KƇ KƇ KƇ AƇ Ƈ, #Ƈ F” Ƈ Ƈ/ƇƇ ,,+T , ,, +T   :U

,

7 #Ƈ

e¿• Ƈ Ù-ų€ƇƇ s‰  Ƈ*Ƈ v Dà b¿&Ƈ %,

K  ,%

,

KƇ % 



!,

"

¿ Ƈ¢± #ƇF Ƈ Ƈ/Ƈ

6 7Ƈ 

<#

ĽƇ), ˜±  Ƈš±Ƈ·Ƈ/Ƈ Ƈ Ƈ Ƈ%± à ˜š± ,»š± b¿W#Ƈ F Ƈ Ƈ/ , %/Ƈ/ƇƇ) ,’¶,™&

ĝ #

q Ƈ ƇƇ Ù4=Ƈ&Ƈ&Ƈ# # # &ƇƇKW&ƇKBƇ€#Ƈ…Ƈ ed¿ * , , +, ƇƇƇƇ qƇ  œƇ , Ƈ Ƈ , ,+Ƈ`¿ ,,Ƈ U± , DƇ *Ƈ ƇƇƇ Ƈ , 4 , q‹Ƈ

Ƈ

®­©¦Ã



IJ"¿

`b 

ªÃ

¸’–„“¿



1H±

.5Ã

!"#$%U

f™ˆ} œ£„¶a}¨Š„—}¨}‘¶fœ„¨®¶ |š§‘€«ª¬¿r€³”£€³š†€ ¿u º£«œ€‰¿ |ru¿ :''N± |§šª¬¿|†³šª§¿|ª µ³šª§¿ 4#

)

4Ƈ ¸ >4Ƈ ¸>4Ƈ ¸ J KƇ ,











U

,



G>>SÃ

,

H>>UÃ

'Ƈ †±FT ±ET …±ƇG>>Và bƇ G>>Và bƇ4Ƈ "Ƈà GÃ





G>>UÃ



Ƈ(ƇƇJƇ(Ƈ ƇƇBƇ L

3Ƈ q/£'Ƈ

RT

µ¼Ã

•Ã µ¼à ª >

  Ã

RT

µ¼Ã

%

¼Ã =à C µ¼B=Ã

&Ƈ((Ƈ % Ƈ%/sŢ

*/ Ƈ3ƇƇ(Ƈ  

) PƇ(Ƈ(ƇJƇ(ƇƇ ƇƇ /Ƈ*Ƈ44&Ƈ( Ƈ(Ƈ JJ Ƈ /Ƈ /Ƈ*ƇKƇ D Ƈ(Ƈ ƇƇƄG

"AƇ

O q///Ƈ] ŽŇʼnƇƇ(/JƇJƇİ;3OAƇ U ũƇŀņ ]ƇƇ(/JƇJ ] ŽňŊƇ U(& qŽ ]ƇƇƇ?ƇJƇy;3O‚Ƈ D ƇO

BG

z

=)±

±

=)±

M)±

±

]ƇR Ƈ(Ƈ ƇsţƇƇ(Ƈ=Ƈ Ƈ Ƈ?//Ƈ /Ƈ{ƇƇ Ƈ %/ƇƇ(Ƈ(Ƈ nvwy¿ dà h n ƇD vƇ nwy{¿bƇ nƇ,KL nAƇ +

) L 2Ă BƇ'ƇL LƇ t¿ƇƇJƇJ L LƇ ƇL LƇlƇLƇRƇ2Ƈ RƇ44ƇRƇ4#

LBƇ

I(ƇƇ/(ƇLRLRLRLƇ , +ƇJƇJƇd± DvƇu£ƇR Ƈ,GƇ >v u£ ƇƇƇƇL#Ƈ D Ƈ(ƇƇ"Ƈ%“Ƈ(ƇJƇd±

Ĵ(vƇ-ēƇƇ-5ƇƇ-MƇƇ-ƒƇb¿ G Ƈ #Ƈ \Ƈ(Ƈ 2Ƈ , "ƇRƇ, Ƈ^ D Ƈ(ƇƇ?“Ƈ, ƇRƇ ƇƇ2ƇƇ"<ƇƇ"Ƈ , <
7G



O

ıƇJƇ? //ÉƇ Ō . tƇ9 tƇ Ƈ 9 u Ƈí. uƇ˜  Ƈ", :G Ƈ r Ƈ3 Ƈ°“vƇƇ(Ƈ . 5Ƈ t5Ƈ . t ", :Ƈ b  5 ðƇ t5ƇƇ9 5Ƈ ú  t9ƇƇ",:Ƈ a¿+5Ƈ 9 5Ƈ^Ƈu5Ƈ^Ƈ 9 u ",:Ƈ XƇ7 5Ƈ . 5Ƈ^Ƈu5 T. u ",:Ƈ XƇ5Ƈ

Ƈ Ƈ Ƈ "Ƈ

6L¿Ƈ6"oƇ . t^Ƈ9 t^Ƈ9 u^u. ",:Ƈ X  5Ƈ6+ 5Ƈ^7 5ƇT 5Ƈ  T+^+"6"Ƈ #Ƈ bƇ#Ƈ D ƇƇJƇ? Ų““Ƈ, "Ƈ

<#

3 ĻƇ8eƇ ,.Ƈ Ƈ†jƇ8eƇ, ~GƇ ;*Ƈ Ƈ3 Ƈ°/Ƈ ƇŁ*(äƇI(Ƈ ( . 5Ƈ^Ƈ- 5ƇT-.  ~Ƈ­Ƈ- 5Ƈ  .T-  ~˜ 5Ƈ .6- 1 ~˜ 5Ƈ X -5 #Ƈ ]Ƈ ƇŜƇ(ƇƇƇ? Ƈ(Ƈ

L+Ƈ

.0ƇT-0Ƈ

 -.Ƈ r Ƈ0Ƈ~Ƈ0~Ƈ'Ƈ=ƇƇ Ƈ

-.Ƈ

.0T-0Ƈ 9J .0T,-0Ƈ 9JbƇ"-0.0 ƒƇ ƒƇ Ã

I  . 6 -0.0ƇƇ2 Ƈ > =ƇƇ Ƈ.ƂƇbƇ ,Ƈœ± © 0 = ƇƇ Ƈ.Ƈ į-&Ƈ Ƈ Ƈ .0Ƈ XƇ ,ƇƇ © 0  9U



D ƇƇ†8jeƇ'Ƈ û  

ˆ



ˆ

© =ƇƇ Ƈ? Ƈ ƇƇ,: Ƈ

z \Ƈ Ƈ¥?NƇƇ2 ¥[¥Ƈ FƇ Ƈ ƇƇƇƇ%&Ƈ Ƈ¥¿'Ƈ  =Ƈ Ƈ  ƇƇ¥?NƇƇ2 2 › ƇƇ¥¿'Ƈ&Ƈ Ƈ  ƇƇ¥?NƇƇ2Ƈ'Ƈ2  Ƈ > ƇƇ%Ƈ Ƈ ƇƇƃ Ƈ Ƈ Ƈ*&ƇƇ‘ Ƈ Ƈ2Ƈ Ƈ   *Ƈ  ;U

8U



y xƇ Y[  ŸMƇ ƒƇ-}Ƈ'Ƈ Ƈ %Ƈ MƇƒƇ-lƇƇ D ƇƒƇ-'Ƈ=Ƈ  Ƈ-'Ƈ+" 0Ƈ >¹*=Ƈ CƇ'Ƈ+Ƈ Ƈ…¿'Ƈ[$¿ I ŶƇ-Ƈ ± CƇ ± …¿ b¿Y[%¿



y xƇ  2dÅƇ/T 2 dÅƇ/T 2Ƈ  Ƈ   Ƈ2Ƈ ´Ã <Ƈ´Ã 2Ƈ'Ƈ""Ƈ.T ƇœƇ ç&Ƈ Ƈ Ƈ ÔƇƇ



) xƇ ""Ƈ rƇ  Ƈ  *Ƈ.0ƇƇĊƇ'Ƈ.ƇƇ90TſƇ ""6 Ƈ'Ƈ"" \xƇ { Ƈ Ƈ&Ƈ Ƈ*Ƈ ƇƇƇƇÔ Ƈ  Ƈ Ƈ*Ƈ¨ ¨ ‘›Ƈ  ƇƇƇ.Ƈ Ŕ9ƇƇ Ƈ ƇƇ Ƈ ;U

"

ÌxƇ  ] Ƈ

Ƈ ĔČƇ"

 Ƈ CƇXƇ

Ƈ ĕčƇ‚"T"Ƈ

*  Ƈ {| y¶XƇ -Ƈ CƇ ,

) ġƇ , Ƈ|| Ƈ,HƇ ƇHƇ  Ƈ+HƇ ƇH  'Ƈ  Ƈ2 HƇƇ"2H6 Ƈ2HƇ Ƈ"HƇ 'ƇƇ Ƈ Ƈ"Ƈ2 H Ƈ Ƈ2HƇ6Ƈ"Ƈ2 H Ƈ  Ƈ2HƇ 'Ƈ ƇƇ"2 H62 HƇ'Ƈ Ƈ Ƈ HƇ ' , Ƈ

+

) xƇ ,Ƈ 2Ƈ

D Ƈ Ƈ% Ƈ

"áƇ 2 ĖƇ I 1  11ÉƇ ōŎºŋƇƆ b º+ b , G ¼Ƈ ¼ÛƇ Ƈ2 # Ƈ

yŰoƇ << Ƈ >1ƇžÃƇµƇƇ1&Ƈ žÃ 1 ƇµƇ Ƈ ƇƇ%*Ƈ Ƈ Ƈ

ƇƇ Ƈ1GƇ I ƇƇ Ƈ

Ƈ1 Ƈ Ƈ1Ƈ1Ƈ1 >#ƇP1ƇžÃ 1 ƇµƇ 1&ƇCƇƇ

:;=JbƇƇ <<ƇbƇ<<

 GƇI Ƈ1ƇƇ¥—…Œ¿ƇƇ Ƈ 7 ĢƇ

)1oƇ Ƈ \Ƈ Ƈ»EEÃƇƇƇ Ƈ*Ƈ»ÃƇƇ*ƇFƇI ÁƇ D1Ƈ „„ƇƇƇ'Ƈ ėĘƇ ½ ƇƇ Ƈ*ƇƇƇƇ'Ƈ&Ƈ#ÁƇ  „„ƇƇƇƇ Ƈ*Ƈ#Ƈ

<#

)1oƇ  ÍƇ Ƈ1ƇƇ¥¿1 Ƈ ¢¿ Ƈ¥¿ià ¢$ ¥¢¿' ,7 ¥>¢>¿'Ƈ¥¢ ¥¿½Ƈ¢ ¶lƇ,7NƇ îƇ¢ ¶ƇƇ%ƇƇ &Ƈ Ƈ& ,7NƇƇ¢ ¶' ||’Ƈ ƇƇ1Ƈ’GƇ I Ƈ%ŖšƇƇ <NƇ Ƈ¢ ¶'Ƈ, ’Ƈ >1Ƈ <&Ƈ, 'Ƈ&ƇƇƇ Ƈ ¥¿Ƈ¢¿' Ń |ÖƇƇƇ%Ƈ1ƇÖG >1Ƈ ƇƇ1*Ƈ Ƈ¥¿,-T ¢¿l ,7Ƈ1 Ƈ1Ƈ1Ƈ 1 Ƈ Ƈ

¢¿1 Ƈ¥¿ƇƇ Ƈ1&Ƈ1*Ƈ¥¿'Ƈ2 < Ƈ1 Ƈ¢¿lƇƇ Ƈ Ƈ?1G I Ƈ ƇƇ1ƇƇ#

 G

)1oƇ + ,+Ƈ1Ƈ  :Ƈ1Ƈ :Ƈ1Ƈ , :Ƈ1Ƈ2 : â Ƈ" :ƇƇ :Ƈ ƇºƇ,+Ƈ1Ƈ :ƇƇ :Ƈ1Ƈ ĎƇ   :Ƈ 7 1  :Ƈ  :Ƈ " :Ƈ  :Ƈ +"1 " :Ƈ " :Ƈ    :Ƈ 1 7 :Ƈ    :Ƈ +#Ƈ

#

ÌoƇ , r1Ƈ1Ƈ ƇƇƇ*ƇƇƇ 

 Y 8J TY 8JX ,& 1  Ƈ œƇ 6Y¶œ &Ƈ Ƈ ¬ TY 8J¬Ƈ      ×Ƈ¬Ƈ Y 8JœƇ×G 7Ƈ

G

) xƇ + ÍƇ-Ƈ'ƇƇ6¶Ã  ƇCƇ'ƇƇ6¶#à D Ƈ Ƈ? Ƈ -MƇ ¦ƇCMƇ -ƇƇCƇMƇ ƇƇ r Ƈ Ƈ ƇR%  Ƈ -ƇCß'Ƈ-ƇMƇƇCƇMƇ ¦Ƈ-CƇ- ƇC&Ƈ z? ƇƇ Ƈ -C -  C' |Ƈ I Ƈ Ƈ ƇƇƇ Ƈ -Ƈ'Ƈ &Ƈ CƇ'Ƈ Ƈ Ƈ-ƇƇC Ƈ Ƈ? *&Ƈ Ƈ ƇƇ Ƈ Ƈ? ƇƇ¶Ã'Ƈ&Ƈ ƇƇG D Ƈ ³AÃƇ¶ JÃƇ¶Mà Ƈ¦ƇƇƇ Ƈ +GƇ 5U

)U

6U

3U

A

2U

yűģƇ  :G 3 ƇƇƇƇ Ƈ Ƈ);3&Ƈ Ƈ*¿Ƈ Ƈ ƇƇ  G

-UU,U

\Ƈ Ƈ[)3;Ƈ'Ƈğ SƇ > Ƈ P3Ƈ P;Ƈ Ƈ[ 3P;Ƈ'Ƈ+ SƇfà [P3;Ƈ'Ƈ+ SƇ Ú3P;ƇƇ Ƈ?Ƈ GƇ I Ƈ[P3)Ƈ'Ƈ2 S6+ SƇ 'Ƈ SƇ'Ƈ[P)3Ƈ ;Ƈ[i)3Ƈ'Ƈ SƇ ÂƇ >Ƈ)PiƇƇ ƇƇ ƇÕ #Ƈ [)P3Ƈ + SƇeà [3PiƇ'Ƈ S#Ƈ > Ƈ[i3)Ƈ'Ƈ SƇ Ƈ Ƈ[P3)Ƈ'Ƈ ƅ&Ƈ  Ƈ[i3PƇ  SAƇI Ƈ Ƈ Úi3PƇƇ ƇƇ Ƈ Ƈ[i3;Ƈ 2 ü Ƈ " SƇ >Ƈ;iƇƇ Ƈ%% ƇƇƇ Ƈ?Ƈ Ƈ P;3#ƇI Ƈ [P;iƇ'Ƈ+ SĆ Ƈ'Ƈ  SƇ [;i3Ƈ'Ƈ7 S6" Sý SƇ 'ƇďSƇ 4U

4U

4U

5U

"#

) ĤƇ "

<Ƈ

7U

7¶% Š ;¶ Š ¶ &Š % % %'Š %%  %%(Š  %'Š

$¿ žķŅ Ƈ >Ƈ

¦

¦

¦

¦¿ ¦¿ ¦ ¦¿ ƇƇ Ƈ 8¶ ;¶ ¶ ¦

Š Š 'Š



T

T

I Ƈ¥¿ƇƇƇ%ƇƇ+Ƈ  w I ƇƇ "Ƈ Ƈ #

,#

) oƇ  :# {ƇƇ Ƈ

E =Š C EŠ

ÂƇ I Ƈ ƇƇ  Ƈ Ƈ

'ŠC Š Š=  Š E Š(Š !=CŠ

;*Ƈ Ƈ ƇƇƇ &Ƈ ƇpƇ  ƇƇƇ ƇƇ #

+#

CŠ Š= EŠ

1U



=CŠ

Ƈ D ƇpƇlƇ : Ƈ

) ÇƇ " \ƇƇ Ƈ Ƈ*Ƈ Ƈ

%

&Š Ƈ‚ 'Š+wƇ

\ƇƇ Ƈ Ƈ*ƇƇ

\ƇƇ Ƈ Ƈ*Ƈ ƇƇ Ƈ Ƈ  ãƇ

%  w

XƇ# + Ƈ ;*Ƈ Ƈ% %ƇƇ  Ƈ Ƈ¢ &Ƈ Ƈ ƇƇ Ƈ Ƈ*Ƈ  Ƈ ƇƇƇƇ lƇ Ƈ +w6ƇlƇ"#

Š

w#

y oƇ ]Ƈ Ƈ )&Ƈ

=CŠ

EJŠ 6Ĉ 9U

=Š Š&Ƈ  Ƈ CŠ' Š Ƈ CŠŠ =Š èƇ Ƈ D Ƈ J &Š  Ƈ EŠ'Š Š  EŠ Š J.Š

# D Ƈ

TU

U

=Š ŠC)Š !=CŠ ŠŠEŠ ŠJŠ Š !EJŠ  Ƈ Š Ƈ ŠƇŠ>Š Š œÃ



 Ƈ

L7AƇ

) `Ƈ  "Ƈ Ƈ AƇ > ”Ƈu-± , Ƈ u , ¨ƇàƇuMƇ , uV 0Ƈ ¤Ƈ ¤Ƈ ƇƇuŠ 9± ƇƇ Ƈ u¡ . uŠ 9± , u±Š G± Ƈ ƇuŠ 9± T-ŮìěƇ Ƈpq± 6uŠ 9 

BU

GU



0

¤Ƈ

uJ±

2U

 = ƇƇ ƇƇƇ đƇÜ ƇƇ

'U

 uŠ



.u±

«Ã c ž Š± Ã

 -Ƈ ö÷ÝƇ 6-ƇŠ±

 , , 0;;TÃ uV (R; #, TMƇ uD±

u

 



õ  7Ƈu & ;@ ,  " 0 <ćƇ

) `Ƈ "Ƈ rƇ Ƈ  *Ƈ

FJ & Ƈ Ƈ 



.0Ƈ ± 90Ƈ– 0Ƈ6.9 Ƈ9– .–  ƇÈū .69 0Ƈ96– 0Ƈ.6– 0Ƈ Ƈ Ƈ Ŭŭ *AƇ  AƇ

) `Ƈ +,Ƈ >%% Ƈ ƇƇ.dƇ ƇŹƇ  Ƈ-d&Ƈ. Ƈ Ƈ-dƇ Ƈ- &Ƈ‡ M 0Ƈ 0Ƈ Ƈ Ƈ-  Ƈ-ÄƇ  Ƈ‡ VƇ ƇƇ  Ƈ-MƇAƇ 0Ƈ F  Ƈ Ƈ ƇƇ%ƇƇƇY± Ƈ? ƇƇŚ  Ƈ Ƈ  Ƈ Ƈ ƇƇ Ƈ? Ƈ .dƇƇ. ±.ÄƇ Ã.VƇ Ƈ  Ƈ Ƈ  Ƈ Ƈ 0Ƈ ‡ĹƇ Ƈ&Ƈ‡  ± &Ƈ‡ MƇ Ƈ  Ƈ‡ VƇ AƇ 0Ƈ I Ƈ ƇƇ ƇƇ ƇƇ? ƇƇ? ŏ ƇƇ Ƈ ƇƇ  ƇƇ Ƈ? 9dƇ ±ˆ ÈĜƇƇˆ VƇ , 7&Ƈ Ƈ9d&Ƈ9 &Ƈˆ MƇ  9VƇ  0 0Ƈ  ů Ƈ AƇ

FJ

FJ

D Ƈ Ƈ ƇƇ AƇ

0U

FJ

2<J

, +,A

) `Ƈ "Ƈ ;*Ƈ Ƈ &Ƈ ƇƇ Ƈ VƇ V  .ƇˆØƇ .6ˆØƇ 6T . +.090Ƈ ëƇ 9  I Ƈ Ƈs‰Ƈ? ƇƇ V VƇ . Ƈ+.0Ƈ90Ƈ à 9  "Ƈ VƇ VƇ . Ƈ+.090Ƈ ± 9 ÈƇ ,+ 2U

.0ƇT90Ƈ0Ƈ

±7.0Ƈ90Ƈ XƇ ,+

 +˜0Ƈ ±7.0Ƈ90

2U

 ,+Ƈ

Ƈ

U =Ƈ

. 5 ^9 5 5Ƈ ­ . 569 5 5Ƈg ". 59 5Ƈ X B+Ƈ

D Ƈ . 5Ƈ^Ƈ9 5Ƈ



y `Ƈ B  78Ƈ > Ƈ 78Ƈ

XƇ"Ƈ

=U

=U

 Ƈ78ƇXƇ

8T 8Ƈ #Ƈ #Ƈ =Ƈ *Ƈ Y”YƇƇ ” Ƈ8 ^Ƈ Ƈ8Ƈ

 Ƈ78ƇƇ8g Ƈ Ƈ8'ƇƇ78ƇƇ8ø  Ƈ8Ƈ  Ƈ78ƇƇ8gƇƇ78Ƈ Ƈ8Ƈ'ƇƇ78ƇƇ8Ƈ Ƈ 6 Ƈ78Ƈ Ƈ8Ƈ  Ƈ78Ƈg8Ƈ'Ƈ Ƈ78Ƈg8Ƈ  Ƈh8Ƈ'ƇƇh8Ƈ   Ƈ Ƈ Ƈ  Ƈh8Ƈ, Ƈ U Ƈ// Ƈ%/Ƈ h8Ƈ'Ƈ"B:=Ƈ  ÞYƇ.Ƈ'ƇBƇ



) ÇƇ B { ƇƇY Ƈ  ƇNƇ ƇƇ% Ƈ Y# A<<Ã

ƇNƇ 8±  =Ƈ

, YN 6_a

9U

 Ƈ¥ B Ğ =ƇƇ  Ƈ Ƈ Ƈ/ƇƇ"h B Ƈ  Ƈ

§aAÃ

Ƈ ƇNƇ'Ƈ Ē Ƈ A <<Ã

ƇNƇ [à  =Ƈ

, YN T_a

;U

N 56 Ũ|ş NgƇB B Ƈ { NƇ ƇƇ% Ƈ Y=Ƈ Ƈ Ƈ

§aAÃ

/Ƈ ƇƇ Ƈ ƇNƇ /ƇƇB #Ƈ

"G

:U

B ƇƇNƇ'ƇBƇ /*Ƈ ;*Ƈ  Ƈ ‘ Y=Ƈ Ƈ Ƈ

) `Ƈ " .ƇgƇ9Ƈ'Ƈ 2Ƈ Àà .Ƈ'Ƈ 2 69Ƈ'Ƈ++h 69 > ƇƇ ƇƇƇ/ /*Ƈ%=Ƈ Ƈ//Ƈ  Ƈ. Ƈ /Ƈ*Ƈ#Ƈ ] Ƈ. f=Ƈ  ƇfƇƇƇ% Ƈ YƇ U Ƈ?  Ƈ Ƈ  9Ƈ ++hƇ ù fƇ > Ƈ++h 6fƇhà =ƇƇ ƇfƇ[à "BïƇ Ƈ Ƈf ƇƇ% Ƈ Y=ƇƇ Ƈ ƇħĨƇ fĩĪƇ "#Ƈ 3 /*=ƇƇ *Ƈ% ŠƇ YƇfƇ   YƇƇīĬƇ fĭĮƇ "=Ƈ ƇƇ/*Ƈ Ƈ  Ƈ f=Ƈ++hƇ6fƇƇƇ%ƇƇ% Ƈ YƇ  Ƈź Ƈ ƇY Ƈ?  Ƈ U Ƈ ƇƇ"Ƈ%ƇƇ% Ƈ YƇ   YƇ ƇY Ƈ?  #Ƈ /U

1U

Ƈ

,

 '

'' ' ' '   '   ' '  ''   '' % ''' ''' '  ' ' '  ' ' ' !" #" ' $' %  ' 

& 

'   '   '  '

'

) `

\Ƈ(

U





.

Ƈ/Ƈ”Ƈ



D Ƈ(Ƈ? ƇƇ ƇƇ Ƈ*Ƈ(Ƈ( ƇƇ  Ƈ

SU

D Ƈ

´Ã {¿,

Ƈ

5\mQ@uqyMŠ 0@~UMh@~\G@dŠ 5qG\M~‚Š 7]nRAvr{NŠ1AWNjA]HAeŠ3eƒjv]AKŠ 613Š B22V " "  " "   "

 !" "  "  "  " ""

";@D0?M"((M K± >C0@B5<;@ 9) *4C0@B5<;M -,??50@M 5*± :,?7  )<M-,9-C9,B
 ¢ 4~ ]| []„Vp ~\M~ E MG MKM MxV MxVMh pƒlOVx| |ƒR\ ~\M~

G KM K EM ± M#EGM M GKM M± KE ±

>U

¢

4| ]~ ~xƒV ~\M~ M~ hVM|~ ~…s sX ~\V ~\xVV pƒlOVx| lƒ|~ OV VwƒMh&

8ƒ|~]X‰ ‰sƒx

Mp|…Vx



/sx Mp‰ us|]~]„V ]p~V[Vx # hV~ # TVps~V‹~\V q~\ us|]~]„V psp|wƒMxV ]p~V[Vx  ]V

2M" M 2M  M 2M",¢ 2 M -¢ V~R @xs„V ~\M~ >U

>U

 !"#

±¶ TVps~V| ~\V ]p~V[Vx Rhs|V|~ ~s ¸$à  /sx V‡MluhV  !# "#" M !#"#" !#"#" M!#"#  M …\VxV !#†

FU

AU



9=> ¢ ¢ W¢ MxV ~\V m]Tus]p~| sX ~\V |]TV| 9=¢ 9>  xV|uVR~]„Vh‰ E\V h]pV T¢W¢ ]p~Vx|VR~| ~\V R]xRƒlR]xRhV sX /9=>¢ M~ M¢ MpT b± MpT ~\V h]pV| >¢M¢ MpT >¢a± lVV~ ~\V h]pV 9=¢ M~ Y¢ MpT \  ¢ xV|uVR~]„Vh‰ @xs„V ~\M~ e±9 $¢ \=¢"\9$¢ e±= 4p ~\V Vwƒ]hM~VxMh ~x]Mp[hV

QU



MU

E\]x~‰ ~…s uM]x| sX ]TVp~]RMh ~…]p| MxV h]pVT ƒu ]p Mp 0¢ ²Ã 0¢ XsxlM~]sp @xs„V ~\M~ ]~ ]| us||]OhV ~s R\ss|V

MuVx|sp|  spV Xxsl VMR\ uM]x sX ~…]p|  |s ~\M~ ~\VxV ]| M~

hVM|~ spV R\s|Vp uVx|sp ]p VMR\ xs… MpT ]p VMR\ Rshƒlp

,

.]pT ~\V lM‡]lƒl MpT l]p]lƒl sX

EM± G

AU

EM#GM|ƒR\ ~\M~ M± GM± 

M

5\mQ@uqyMŠ 0@~UMh@~\G@dŠ 5qG\M~‚Š 6]nRAvrzNŠ1AVNiA]HAeŠ3eƒiv]AKŠ 613Š B22V " "  " "  " 

KV} ;ƒh~_uh‰_o[ Os~\ }_TV} O‰ #¢¢—™#¢¢™œ#¢¢œ— ¢ …V [V~

W¶5 1 ¢— ™#¢™œ#¢¿œ—¢¢ ™ œ#¢—™#¢¢œ—¢¢ œ —#¢œ¢ —#¢—™¢5¢  >s… xV[MxT W¶ M} M ush‰osl_Mh _o —¢ D_oRV W¶5¢¢…\Vo —¢5¢™¢

W.¶ D_l_hMxh‰  ™¢ œ¢ MoT œ¢ —¢ MxV Mh}s XMR~sx} sX _±

— ™¢_}M XMR~sx sX D_oRV W¶ _} sX TV[xVV '

W ~— ™™ œœ — Xsx }slV Rso}~Mo~ ~  ¢ )‰ hV~~_o[ —¢5¢

 ¢™¢5¢ ¢ œ¢5¢¢ …V \M„V ~¢5¢#¢ E\ƒ}

W #— ™™ œœ —

# '

DVV 8ƒo_sx DVR~_so CsƒoT #  ¢BƒV}~_sr , 

9V~~\VxMT_ƒ} sX~\VR_xRƒlR_xRhV OV h±5¢*‘¢MoT€\V SVoyV OV &± 9V~]¢ OV~\VXss~ sX ~\V uVxuVoT_RƒhMx Xxsl &± ~s TV¢ E\Vo X]5¢XW#¢5¢‘¢ M]¢5¢

‘$¢

@U

‘$¢5¢I(‘¢ E\ƒ} MT¢5¢M] T]¢5¢IH I(‘¢ D_oRV L‘¢ ‚x]¢5¢ NYTM¢ NYA>¢ MoT A>¢5¢‘¢ …V \M„V YTYA¢5¢TMA>¢5b;¢ * _V  YT5¢b; YA*¢ E\ƒ} Y8¢5 YT¢ T8¢5YT¢AT5¢#YT¢ YA¢5 b;¢ YA#¢ MoT A8¢5¢Y9YA¢5b<¢'YA}# ¢ E\VxVXsxV Y8$¢5¢YA¢Œ-( D_l_hMxh‰  \9$¢5¢\A¢ A9¢ F\ƒ} Y8$¢ \A¢5¢\8$¢ YA JU

MU

¢Ã

Ã

',¢

KU

g…€š‚¶¦“«©šB¶ >s~V ~\M~ O=>P¢6¢ Q>=M¢ MoT O=M>¢5¢-Ÿ¢

E\ƒ| Q=>P¢ Q=>Y¢6¢ Q>=M¢  Q=>M¢7¢ 0Ÿ Q=M>¢5¢ #Ÿ¢ E\]| ]luh]V| ~\M~ Q=\>¢7¢ 0Ÿ Q>=\ Q=>\¢6¢ #Ÿ Q=>P¢6¢ Q=>Y¢ MoT~\VxVXsxV |=>\¢ |=Y>¢ %¢ Œ †‡Š 7¢ >s… |]oRV 7¢ E\ƒ|  >\¢Y>6¢ =\¢=>¢5¢=>¢¢=Y¢ ]V 

JED J BDJ

HU

BCJ

MP¢ 66 =>¢ 8¢ ]| Mh|s l]Tus]o~ Mhso[ ~\V MxR sX MP¢ E\ƒ|  QM>8¢7¢ Q8>R¢ )‰ ~\V Mo[hV O]|VR~sx ~\VsxVl so |Y>\¢ …V \M„V >\>Y¢5¢8\¢8Y¢ *slO]o]o[  %¢ %¢ ¢ %¢ %¢ …V [V~  V  Y8  \= \8  \=¢

15J  05J .5J /4J  41J 04J



7

*

Dƒuus|V so ~\V Rso~xMx‰ ~\M~ ~\]| ]| os~ us||]OhV IV |M‰ ~\M~ M xs… sx M Rshƒlo ]|

€¬…¤…‚¶ ]X ]~ Rso~M]o| soV sX ~\V R\s|Vo uVx|so|

*\ss|V '#¢ uVx|so| |s ~\M~ ~\V

~s~MhoƒlOVxsXRs„VxVTxs…| MoTRshƒno|]| lM‡]lƒl I ]~\sƒ~hs||sX[VoVxMh]~‰ M||ƒlV ~\M~ Rshƒlo

¢ ]| os~ Rs„VxVT

E\Vo os uM]x sX ~…]o| RMo OV ]o Rshƒno



E\V Rsƒo~VxuMx~| sX ~\V|V uVx|so|lƒ|~ OV R\s|Vo )‰ ~\V lM‡]lMh]~‰ RsoT]~]so ~\V|V  uVx|so| lƒ|~ OV ~\V soh‰ R\s|Vo uVx|so| V]~\Vx ]o ~\V]x xs… sx Rshƒlo 4X M

¢ R\s|Vo uVx|so  ~\Vo ~\VxV MxV -¢ s~\Vx uVx|so| V‡RhƒT]o[ ~\s|V ]o Rshƒlo ¢]o ~\M~ xs… …\s MxV os~ R\s|Vo D]oRV '#¢ uVx|so| MxV os~ R\s|Vo  …V \M„V M~ ls|~ *¢|ƒR\ xs…| D]l]hMxh‰  ~\VxV MxV M~ ls|~ '¢ |ƒR\ Rshƒlo| MoT …V \M„V xs… Rso~M]o| soh‰

M Rso~xMT]R~]so OVRMƒ|V …V \M„V  |ƒR\ xs…| sx Rshƒlo|

9š‚¶¦”«©šB¶ IVhh |\s… ~\M~ M~ ls|~ ¢vVx|so| MxV oVVTVT >s~V ~\M~ ]X M |V~ sX uVsuhV Rs„Vx Mhh ~\V xs…| MoT Rshƒlo|  ~\Vo ~\V‰ …]hh |~]jh \M„V ~\V |MlV uxsuVx~‰ ]X ~…s xs…| sx ~…s Rshƒlo| MxV ]o~VxR\Mo[VT E\ƒ| ]X ~\V RsoZ[ƒxM~]so sO~M]oVT O‰ ]o~VxR\Mo[]o[ |slV uM]x| sX xs…| MoT |slV uM]x| sX Rshƒlo| RMo OV Rs„VxVT O‰ M |V~ sX uVsuhV  ~\Vo ~\V |MlV |V~ sX uVsuhV …]hh Rs„Vx ~\V sx][]oMh RsoZ[ƒxM~]so IV Mh|s hV~ Tz¢ TVos~V ~\V RsoZ[ƒxM~]so sO~M]oVT O‰ TVhV~]o[ ~\V Zx|~

¶ xs…| MoT

Rshƒlo| sX ~\V sx][]oMh RsoZ[ƒxM~]sz{ :V~ ƒ| TVos~V M uVx|so O‰ N P ]X \V ]| ]o ~\V N~\ Rshƒlo P~\ xs… *\ss|V

 ¢ ¢

 ¢ ¢

4o T"¢ ~\VxV ]| M uVx|so …\s ]| os~ ~\V

I]~\sƒ~ hs|| sX [VoVxMh]~‰  hV~ ~\]| OV #¢#¢ DVhVR~

#¢#¢ ¢ ¢ #¢#¢ + ,+¶¶ ¶ ¶ ¶ \à -¢ MxV ~…]o|  ~\Vo ]o ?a` ~\VxV ]| M uVx|so …\s ]| os~ Mlso[ ~\V]x Rsƒo~VxuMx~| 9V~ ~\]| uVx|so OV ¢ ¶  ¢ ¶±  MoT R\ss|V ~\]| uVx|so 4o ~\]| …M‰  …V RMo R\ss|V ¶ ¶ ¶ 7¢ ¢#¢   ¢¢-¢ ,Vos~V ~\V Rsƒo~VxuMx~| sX ~\V|V -¢ uVx|so| O‰ J MoT ~\V Zx|~ -¢ uVx|so| sX ~\V ~\ MoT ~\ xs…| MoT Rshƒlo| O‰ 8!¢ 8%¢ 8)¢ 8+¢ xV|uVR~]„Vh‰ 9V~ 6J „8y¢5¢N` Œ E\Vo f"¢ f%¢ f)¢ f+¢ 23¢ -¢ '||ƒlV ~\M~ f"¢ :¿ f%¢ :¿ f)¢ 9¿ f+¢ Dƒuus|V f!¢5¢-¢ *\ss|V   MoT R\ss|V M uVx|so ]o 8%¢ MoT M uVx|so ]o 8+¢ …\s MxV os~ ~…]o| MoT MxV os~ ~\V Rsƒo~VxuMx~ sX   ? &¶ E\V|V # uVtuhV …]hh Rs„Vx Mhh ~\V xs… MoT Rshƒno| 4X f!¢ \à -¢ ~\Vo f%¢ ^_¿ '¢ f)¢ 23¢ #¢MoT f+¢ 23¢  ¢ E\Vo ~\VxV V‡]|~A`S¶8y¢|ƒR\ ~\M~A` ‚ J ¶5¢ ¢#¢'¢*¢MoT os~ ~…s sX Ai Z%¢‰)¢A MxV ~…]o| *\ss|V ~\V|V *¢ uVx|so| M| …Vhh MoT ~\V ¢R\s|Vo uVsuhV …]hh Rs„Vx Mhh ~\V xs…| MoT Rshƒlo|

Rsƒo~VxuMx~ sX

4o [VoVxMh  ]X os ~…s sX 

'-¢

,

IV \M„V

EM¢GM ¢ @U

M¢

¢M±



 M¾¿

MGM¢

MoT

M¢ EM± GM% 9V~ % EMMGM G\Vo …V \M„V

¢MM

B B M ¾¿ ¢ MoT BM BM MG¿ ¢ D]oRV

MGM¢

MMF¿

 +Š¾¿ BM ¾¿ M¢+Š  E¿ MM' M  ½¿ M %!M

à gÃ

BM G¿ ¢ ~\V lM‡]lƒl „MhƒV ]| MM,ŠMoT ~\V l]o]lƒl „MkƒV ]| M± -Š

>s~V ~\M~ ~\V lM‡]lƒl „MhƒV ]| M~~M]oVT

EMMGM MM%6M @U

…\]hV ~\V l]o]lƒl ]| M~~M]oVT

hš¢~\V |shƒ~]so sX

MoT

E M5¢

M M

hš¢~\V |shƒ~]so sY

EMMGM MMLM >U

 M

MoT

GMMM5¢¢

'1?-)BAE+Q $)G.+;)G1*)8Q 'A*1+GIQ #  #   # # 

Z

# , &! , !) , ,   ,

 %+, , +,  ,  %&$)&! %, &!,! &%& &%,

#)0'5 5 \lÞ &.),$#) ',52$.'5 #)0')5#5,5#)0'5),5%'$/5 #5 )5 ,5%%($%',5.!) !$05 2$.'5 #)0') d8Þ $5 ),%)5'5#5 ,$5.),25 2$.'5 #)0')

 5 &.),$#5 '')5 OÞ "'5 l7 $5 !.!,$')5 '5 !!$0 O9

\6

O - ơ Jơ ' ơơ îo$ơ*ơơơơ 3<ơơBơY“ơ 5ơO €€U-ơ Ù ơơ   ô *ơ$*ơơîơ* ơơơ ơ ơ*ơơƑ  ơơơ*ơ5 ơ Š ơ 5 * ơơ  5 $%Z Y#ơ 





- Ù ơ ơ *ơ ơ ơ €o<ơ  ơ 8* ơ ơ ơ €Ãơ ơ ơO þơ   ơơ  ơơ€ÿơB- Jơ 4Z ơơơơơơ v ơơơ  * ơ ơơ  $-ơ ň  ơơ v ơ ơ ' v* ơ ơ ơơ ơ  $Ģơ ŕ*vvơơ ơ* ơ ơ ơ  *ģơ A ơơ*ơơ ơ   ơơ  ơ  ơơ  ơ $$ơơ Y-ơ áơ  ơơơ   ơ dĐ  ơ *$ơ *IJ O <
Þ 25 Þ \’ Þ Q’ "#Z -ơ

B~ơ

e # ơáơơ%ơ ›ơ ơ ơeơ  ›ơ ơơơơqơ 0ơ ơ0 ơ ơ ơơ ơ0ơơơ ơơ  Ķơ ~- Jơ:>2ơơ ơ   )ơ JơFơ ơIơ ơơ ơ   ơ ơơ% ơ :>ơ ơ>2ơ0 ơ ơ :Fơ–ơe F>ơ ơ>I–ơ O I2#ơ +0%ơ ơ :Iơ  2Fơ %ơ ơ ơ¯ # ơ˜%  •Žo » ơ Ž» ơơ$ơ0%ơ 0 ơ ơ:¯ơ — Ž Y’¯IËơ 7Í J k—ơ =O5 5 BM Y5 ƞŽ ƟM q > Ìơ +ơBÉ$%ơ 0ơ =2 3 4>@ ơ k ơ  ơ ơ ơ ¯ÅÅ©Þ  2@@3@@4@ ơ     ơ ơ B-ơ ˜% ơ ơ 0%ơ ơBÉ %ơ ‰ơ kơ ơ  O #ơ Jơ <8 @< @ #@Þ ™‡°q?Þ ơ   ơ ơ   ơ <8@@< @ŽơŽơŽơ @  ˜% ơơ ơ  ơ 0ơơ Ó ;@1?@@@Ó ;sq?ÜÞ

™ˆsq?Þ

—ơ e¨

OO¨ ˜% ơơ ơ 0ơơ2@0 ơơ2@ œơơ 0 ơ2@ 37 02@ eơ ĵơ ơ 3@ Þ4@@37@ 2@@@ –ơơơ%ơ ơ0%ơ 3@ơ4 Oĩ¨ơ Ú% ơơ0%ơ ơ   ơ ơ%ơơ0%

' e b eơ



1BĤ Ú% ơ ơ 0%ơ ţơ ơ 2 @3@ơ ơ  ơOơ /Z 3@ 2’ 2@ Z ơ 0 ơ  ơ0%ơ 2 @3@2 3@23@@293@ ơ ơ 0ơơơ0%) 1Ĭ-  ơ 0 ơơ ơ$1q- ńơ ơ  0o ơ   ơ :>25ơ  ơF5ơI5ơ ơ ‚Þ ơ ơ ơ ‰ơ ơ Ɯ  0żơ%ơ :5>5ơơ2ơ ơ>25 :2ơơ :>5ơ   $)ơ µ0ơ   :ơ —ơ O ơ> 2 ơ8’ B75ơ œơơ ơơ :ł5ơ ơ™ơ ơơ   ơ :Fơ  ơ>I#

 Jơ ' ơơ ơơơ  0%  $ơ‰ơĺ :>25ơ .@ ơ /@ ơ%  ơơ !ơ>25ơ    )ơ µ0ơ ō 2> :–ơ ơ +-./@ ơ Ŏ :2sơ 8’ +-./@ ľ  ơ ơ‰ơ ,-./@ ơƀ-

B7ơ

'H# ơ '"#% @ "%@ ?’ "# @ ơ  ơ  ơ 4  ơ ơ *"@ ơ #%@ ơF ơ ơ  ơ ơơơơ   ơ ơ #@ ơ "&@ Uơ "#@ qơ#( @Yơ "(@ 9wơ A  ơ ơ  ơơơm 

 





'=ę A  ơ ơ ơ

’



Q žÈ '  w

j7p’

'7# [  ơ ơ ơơơ  ơ   4 ơ ơ ơ  ơ ơ 

ơ ơ qơ'ơ'Ī# # Uơ ơ ơ2@3 @ @5 @ @6@

2@

3 2@

6 2@

@2@ 



XZ

YZ

5 3@

6 7@

235

A  ơ 376@2@ 'w J ƒ 4ơơ  ơ ơơ



ƒ =) º» ZÞ



@ Ɛ

ơ  ơ ÓÞ ?’ ơơ ơ ơ  ơ4ơ 'wơ A  ơ ö  <  # Ě Jơ ' 4ơ ơ  ơ  ơ ơ '"#%@ Ûm ơ "-@ ơ ơ ơ 

ơ #%@ ơF# U   ơm ơ $-@ ơ %-@ ơơ %"@  ơ "#@    ơơÜơ  ơ ­Ėơ  "-@ @9ơ)-@ @ơ$-@ @Gơ&- @'ơ ơ %- @ơũ ơ ÜQě 9# [ơ ơ  ơ ơðơ ơ2:@ ơơ4ơơðĎ  ơ ơ ơ4 ơ ơ  ơ ơ ơ  ơơ'ơơ ơ 9ơ  ơ  ơ  ơ ơ 4  ìơ ơ 'ơơ  4  ìơ ơ '#ơÛ ơ 2 Y( Jơ ˜Þ 4ơ ơ ïơ ơ ơ žơ #ơ A  ơ ơ  ơ  ơ ơ žơơ   ơ  ơ m ơ ơ   ơ ơ ’  ơ ÖÞ ơ ˜Þ  ơ ơ ÓÞ@ÖÞ ơ ÓÞ ÖÞ 

  4 ơ4ơ H# q wơ Jơ .@4ơơYo  ơ mơ wơ A  ơơï4ơơ  ơ ˜Þ  ơ4ơơ  ơ ơ .@  ơ ơ ơ ơ  ơơ ˜Þ ơ  ơ 4ơ ơ ơơ ơ  ơ ơ .@ Yơ

'1?-)BAE+Q $)H.+<)H1*)9Q 'A*1+HIQ #  # ! # # 

Z

# , &! , !) , JÞ !)(! %, X{ +ªơ ' ²ơ 1\ ơơơơ  VơơCơ"ơ ¡'ơơ9ơơơ '¢ơơ 919\ ơ ơ9B CN,-.\ Cơ ơơơ ơ 1\ ơ ơ :19\ ơơÁ" ơ ơơơ  ơ §ơ Þơ ơ 919\ <ơơơ »ơ   ơơơơ ơ B\C\ ơ ơ ơ ơ  "ơơ ı 









¡'9 cC¢¡G Ó=7X¢ ¡7G7 Ó7=77X¢“ơ  ơơ 9BCN\.Z c”ơ ơơơ"ơ 'Ğơ Ì

+Õơ  cG= ²ơ ơơ ơ8 ơxÃơ"ơơ Vơơ ơ  íơơĜơ ²ơơ8  ơ Á Vơ ]ơ V íơơ  ơ xý“ơ ơ ơơ ' 3]ơ" ơxħơ ơ ơ ơ  ơ8 ơ 

''7 ''ơ3 ]ơ §ơ Œ’



ơơơ ơ

XÈ ]ơ

XX7 ''ơ3 ]ơ  #Q ’

Œ’

s6O’



X 3]ơ



 cG={ơ

9{ +ªơ C9G ¬

ơ ơơŠơơơ ơơ ƒ1Þ ơơơ"ơ8Vơơơ  ơ  ơ  ơ ơ ƒ1Þ ơ ơ ơ  ơ ơ ƒÞ ơ ơ Þ  %' ơ ơ ơ VV

ơ  õ ơ W’ XơÞ cơ W’ c

7 7

 ơơơơơ ơ"ơ ƒÞ ơ 'ơ W’ XơÞ ơ W’

7 õ7ĝ c

Þ ơơơ ơ "ơ “ơ ơ Ļ»Äơſ ơ ơ

7

Þ L=Þ V’ YMÞ  X<Þ Þ KÞ

c'ơ



k>bn2Þ

cz…ơơ&ơơ ơ&"ơ"ơ{…"ơơơ "& ƛ & Íơ ơ c)…N…* …c2…N…3… ơ"ơ& ơ&ơ 3…cz u…N…cz… { …q¹XCL»‰š­ ‰7» "ơ ơ ơ " ơ ơ  V(ơ Uơ &ơ &ơ {… …+…"ơ z@»666»»z™» " ơ& &ơơơ "& ‰& (ơơ & ơ& ơơzD»666»»z™» &ơ cz…&"ğ ơ & ơ z@zC» 

ơ ơ &"(ơ ơ & ơ z@zC» 

ơ+…  3 &"ơ  … * … … { …

… "ơ & ơ "& ơơÁ""ơ & ơ"ơ*… … N»"{ơ U"" & ơơ & { … z@zL» 

ơ ,…

…3… {6…&"ơ (ơA&ơ"ơ"ơ"ơƃơ ơơ ơ czN* …/7 …š  N7 3{‚6…  {6 ƒ N , …/7 …{J *… +{‚6… € {8ƒ… NO…* … 37 …{J *… …{J 3…  …-7N * … 37 …. s}›š€»s›“£Ÿ‰›šY» „ơ



Z

‹’

‹’

‹’

8

3A88…

+P ơ Vơ ơ ""ơ &ơơ &ơ &  ơ &(ơ àơ ơ ơ ""ơ ơ &ơ &ơ ơƗ&ơơơ&ơ ơ ơ &ơ"

B

3 07 17 3;3" … àơơơơ ""ơơ&ơ"ơ "…ơ ơ&ơơ ơ &ơ" 7"… … 47 27N…6*; ŀ ơơ &ơ "ơ B… …3;3"…

…6*;…O…3A88… ; +P 3">B*… „ cV» ©»Ž» c» <…i»Ž«^» c»L»N…©»Ž» c» 3…h» Ž!»<…¡»Ž«^» cR»N…ª»Ž» c» 6…i»Ž»<…¡»Ž«:» 







…ơơ





€Þ







N…v-(=#$…q…N…*<< K… icME»N…w *"3""…$7x *%=&'…%7N…6 6 8K… %7 *<>… icRE» y *"""…#7 6 83…

icVi»





<…

Aơơ   ơ B\ ‰’ C\ơ4ơ   4?ơ4ơ  ơ  » ơơ4ơơơơ ơ 0& ơ 

» ơ ơ ơ 0#\ ơơơ ơ ơ 0$\ ° ơ «ơ



<

*<<3…7 \,<<… 6&&& ,<< … \669… ,


,"<3<… P…3"

+P ʼnơ …’  Š’ \ÖÞ \’ \S \ ơ …’ VZ ơ ơ?ơ ơ Š’ \ÖÞ \’ \U&'Z °  ơơ ơò?ơơơ4ơơo¹ ơ ¹ơ? ơơơ?  ¹ ò P

3" …



\ÖÞ \[’ \P’ \R’

™Þ

 ơơ ơ ơ Q

?

P…,"<3<

P…3"…

*
+P ļ ơơ ơ  ơơ Ĵơơơ ơơ ơ  ơơ Ġ ơơơ 

,7 ;… \+7 *7 *…K» ;…Q…*
A







+Pơeeơ +ơ ơ

fZ\^g…P…,!…ơfV^Zh…«ơ Ž»  ơ fY^Zi…Q…,,… U  ơ U[R…?[Y Ž»R…fV^Zg…R…@fY^Zg…P… pp.Þ





B

<;99……  3…  "* 0K\ CP…T*…Z[» Ž» ;"…C…6…9\ Ž*y»U ơ 90QN\P…*<…;0!9\ *?… ơ 90#9\P…,? …Ńơ ơ4ơơơ 6.…B C D \ ơ ?ơ ơ  ơ TBCDU\ 0K\ ơ ơyơơ TBCDUP0K\ ab ơ  ?ơyơ Q…, 3 6… ơ ơ ơ +P Ay«ơ

&'Z



WZ

Z

96…

;
'( + ·ơ ơ  ơ

!Z





šˆtq?ÛÞ



Þ



**





šŠuq?Þ

›‰totÞ

+İơ 7 U l   ơơœơi  ơ ơ ơ ơ i   ơ i  ơ







Aj…F7»…Hơ ơơ  ơi şƅ ėơ ƊæơˆƔæ k……l 2…N…j * 2… k…kmO…j2 … ~j ơ ˆơ k…ˆ ơ ˆơ ƆƂơ  }2…j …j2Aj… …W» S…(ơ…jHI ŗ ơ ơ   ½  N…fj * g2 :j2Aj… …H‘ơ* }…5ơơi

Œ$ 5ơ F» FG …7(ơ [ơ k…N…l…N…:… j…N…7ơ œơơơi  (ơ ơơ ơŒ ơơ j… ơ 7(ơ +ơ *;"<… S…L M… FG…*… ơ  ơ  ơ ơ {… N…C'ơơ ƒơ jo…S…y ź a©HŅ(ơ Őơ ơ G©ơ S… ơơ jp…  6 (ơ U  ơj0…N…ơ ơ * $$ơ ơ Œ ơ ơ ơơ C'(ơ ·ơ $ơ  ơ ơ ơ    

  ơ (ơ5ơ ơ ơ ơ Cơ* …"":…N…*;"<… c’





Z



'(





343ÞÞ¢N?ÆcÞ

¢N?ÆcÞ

¢N??gÞ 544ÞÞ ¢[ÆÆqÞ

* 6

+ FF N» U  ơ ơ  lơ

ơ ơ  ơ i   ơ  ơ F»   Œ ơ  ơơ  lơ  ơ lơ ơ  ơ i (ơ ƒ +ơ ơ  lơ Z ơơơ (ơ [ơ  ơ  lơ   ơ ơ  ơ ơ (ơÝ Ĉơ ơ ŧ ơ ơ  ơơ ½lơơ  ơ ơ   ơ ơ  ơ P» F»  ơ Z (ơ Ý ơ ơ ơ  (((ơ Ż©aơņơ Þ ” NAA rel="nofollow">FUO»m» ''(ơ

j…k …jk …jk……jsk…S…jsk k…*… 2… k… 2 k…… jtk… 2… jsk…N…{ … jT…k… jN…k{ HI… {… k…{ … k… k{{…2…HI… k… {… …





*:  ơi  ơơ lơ $ ( XC( +ơ C [ ơ ơ ŵơơ/Iơ /sơ ơ/ơ  ơ /­ơ /Sơ ơ/(ơØơ  ơ  ơIơ /®aơ 3 /Iơ ˆ’ /­ơ ơ/ơ aơ/ z/saơ Þ /Saơ 3 /sơ ˆ’ /Sơ ơ/‘ơ aơ aơ sS  /(ơ ơ I ® ơ sSơ ơ/(ơ ơ ơ ơ ơ ơ ơ /ơI5ơ™ơ®ơ ơ ơ ơ  ơ  ơ ơ /™(ơ  ơ/™ơ sSơ ơ/ơ ˆ’ IaÞ

X]2……







`2…N… ;5… 



N… N… 



ffÞ

S…6B…





+Pơ ' ƒơ uơ gơ ơ 

 ơơ QS(ơ ‡ơ †Q!ơ uQơ!ơ ut(ơ Jơ \Q†tơ!ơ " $ ‡ ŊœQuơ!ơ \Rơ!ơ =6ơ  ơ\RQ/ơ!ơ\Sơ!ơ '6Îơ +?ơ\uQtơ! \/!ơ Ò=Wơ!" †ÏÞ ??ơơ\†tQơ!ơ Ò=Wơ36 3=6 3$'=Wơ HÞ"$ "$ ơơ†Qơ!ơQt( åơ ŌQtuơ ơ8 V?ơơ\uQtơ!ơ '=W 3!"$GWơ ơ "$!ơ 'W( 





+Pơ  JơFI!ơ "$ /Sơ$ $ ơ SF!ơ „’ ‡ơ ĹFơ! $"$  ơ  \S/Fơ! ơ\Ľ/F!ơ $ róơơ  ơ ?ơ?  ơơ Į/FSơơ ơƇbơ!ơ$Þ "$ !ơ 5» ‡  $$" $ ř ơơVơg  ơơơ $$"$Þ  $Þ "$!ơ$_ơ" 



åŰơ ơ gơ ? Ū ơ ơ "$$ "   $!ơ (ơ U  ơ $ >’ $ ơ ơ  ?ơ ơ ơšơ $ơ ơ $"$$"  $!ơ ơơ 8 ??óơ "$$$ \"  $!ơ (ơ ‡ ơ "$!ơ  $ \ $ Aơ  ơ’ ơ g ơ Ĩ=Î



@ F @C  @F @C’

 IF!ơ  "$!ơ  `  ` ’

+Pơ  őơơL 7W 3¸‘ơ!ơ$¸ơơWơ 1’ ¸ơ1’ 7W ơơơ ơ ơ  ( U ơëơ!ơ$3 Ž!» ’

’

QL žď fž»! QL 7 3L ë 3 fž»!

`g»

‹`g»

$

$ !ơ $ u‹`’»L Ň 3'‘ ’





1Ô# +Pơ  ơ  ơ ơơ ơ  ơ ơơ 0/\ Cơ‡’ 1Ïơ/\ 9#ơơ ¾ 

ơ ’/\ MQ ‡’ ĀC _ơ1ơ \19Æơ/\ 1CEơ ± ơơ  ơ ơ 0\’/\ E # +Pơ  K ƒ5’

D\ B\ C\  \ \ ³´µ» ơ ¢ ¤» ¢¤‘»  ơ ơ ¢»/\ œ¤»çơơ œ» Z #ơ  ơ ¤_





CB\ B\

DC\ C\

EF\ D\

± ơ CB\/\DLC\/\ELD\ ””5ơ B\C\D\Ţ ơơ ơ¾ ơ B\RB\R"B\R$B\ …ơ CDELB$\/\R&\/\ R$"\/\  1# +ªơ  ä ơ 1Ïơ6ơ ơ¾  ơçơ6ơ ơơ  Tơ  ơ ƒ  

K T ơơšL6¤ơơ. ơšL6¤ơ E

+Pơ  śơT

ơ

<5\ 05\

NQ 6”ơ ± ơšLŸxxx¤ơ/\ # =0<2@\ >024?\

=2<4?\ <6\ =0<4?\ <8\ =024?\ 26\ =024?\ 48\ <5\ <7\ <8\ /\1ơ \ \ 05 26 48\

$



<6\ Eơ 

B

9)

+Õơ }9== [ơ w’ ÝÞ %\ ơơ BO\ ơMơ ơ CO\ ơ ơ.ơ Dơ ơ   ơ ơ @)ơ rơ ƓMơ ơ .ơ Dơ ơ ơ  ơ ơ ơ ơ  ơ ¿ ơ  CO\ + ơ ơ.ơ Dơ ơ ơ  ơ ơ ơơ %\ơ.ơ BOMV\ ¶

+ơ ơ CO\ơ ơ  ơ ơ <;ơ ơ .ơ Dơ ơ ơ  ơ ơ <<ơơ COA\  ơơ.ơDơơ ơ ơ ơ<ơơ 9<ơơ.ơ BO#V\ ơ £ÁÞ  ¤Â GÞ Þ V¡Ã!WÚÞ

ƝƁ Mơ ơơBO/\%BOM\Çf‹ċaŽơ K ơ B \ |Ĕ



%\ ơB#\ @;ơ ơơBJ\ H99G=) 



+įơ @} ´ơ ơơ. ơ ơ ơ   ơ .ơ GơơD Ö z;ơ z<;ơG;ơ z;ơ9} ;ơ ) ) ) ơ üơ z<9;ơ<ī)ơ Kơ kơơ@}ơ Eơ ßDơkơơơ ơ ơơ › ơ . ơ .ơ H;ơ ơơ Eơ Œơơ ơDơ kơ  ơ ơ ơ

  ơ .ơ H)ơ rơ ơ  ơ   Mơ kơơ ơ ơ ơ ơ ơ

. ơ.ơH)ơ ơơƙ@;ơ;ơ) ) ) ;ơơ<|ƚơ ơơơơ Mơ Ó  Þ×Þ  ơơ ÓÞ ×Þ ơ ÓÞ & ÕÞ ơ . ơ.ơ H)ơ

})

+Öơ H| [ơ .ơ   Mơ ơ  ơ ơ  ơ ơ ơ †ơ ơ w’  M ơ v’ ÝÞ Ô#ơ Ŝơ Ũơ ơ ơ .ơ Dơ  ơ  ơ kơºơ ơ  Ź O ƍơ: ) ơơ.ơơ  *) \  ơ ơ.ơ. ơơD Eơ âơơ  .ơ :>2ơ   ơ  ŴEơ ¶ơ>ơ ơ ơ ÷ Døơ .ơ  ơ2ơ ơ ơ ùú .Eơ ơ  Žơ Dơ Dơ>ơ  ơ2ơ ơ ¿ ơ  ơ .ơ ơ ô .Eơ r ơ : M ơ> M ơ2ơ  ơ ơ ơ  ơ ơ ơ Dơ  ơ ơ  ơ ơ  ơ Dơ : ą ơơ ơ .ơ ơ .ơ ơ Dơ ơ ơ w’ & Hơ  E O ¶ơ ơ¿ ơơơ W\ +) \ H|) 

|Hơ

'1@-)CAE+Q $)H/+;)H1*)8Q 'A*1+HJQ #  # " #   Q 

 " , &! , !) , , &)$*, , ) ,  ,

  ,

 %&$)&! %, &!, ! &%& &%, O: 3JPSANZ 399Z iÞ MRAPQEKJP


 :KZ ?=H?RH=QKNPZ =NAZ =HHKSA@

@E ‚ơ fp`ơfb;ơ) ) ) ơ`ơf‹ơơ w’  ơ Ÿơ ơ žơ Ÿơ ơ  $ ´Tơơ ơ  ơGZ VZ `ơ ơÀơw’ ơ6 p`ơ6 b`ơ) ) ) ơ;ơŝ‹`ơ ơơ ơT ×ơ GZ…  ơ ơ ơ$  ơ

"=Q

_gSh’

i6o’

-Z Œ% ¯» 

ºÞ







) ßơ fħMơ = 44 Z f‹ơơ  ơ ;ơ Tơ ơ 6 Č fp6 č fb») ) ) ơ6 3 f‹E  ơ ơÀ ơơơ  ơơơ Ÿ Mơ ơơơ Ŧ   ơơ ơơ ơ@E 9ġ ‚ơ Zp;ơ Rpơơơơơơơ ZRơơơ  ơ ơ ZR2 ;ơơ ³2ơ =’ #$  ơ ơ ³Zpơ2 Rpơ8’ ³ZR2)ơ Þ  ơ À ơ ơ ơơ  Ÿ   ơ ĭZÑRÑ2ơ ơơ ơơơ2 Z ơ ơ2 R ơơ ơ Zbơ ơ Rb;ơ  T ) ´Tơơ Zb Rbơ ZR) 

 i

‚ơ ‘Þ ơơ ơ ơTơ`ơ){`ơ ‘Þ ƘN¥ơ `ơ 4 Z«;» A ơ ơ  ơ *E ‘Þ ±» ‘Þ  ơ !*6E$ *7E 6E$ w’ ơ ơ q’ w’ € ’/Þ 



j0

A ơơ ơTơơ6ơ ơơ6 ơ T ơơ ơơTơ

\» ²®»

Y=ơ

'1?-)DAE+Q %)H0+<)H1*)8Q 'A*1+HKQ #  #   # # 

Z

" , '! , !) , TÞ !)'! %, @ĕ Ş  ơ ơơ ơơơ  ơ ơ$ ơơ &/Eơ  ơ ơ ơê  ơơ ơơ
8E

ơ ơ CE&/E *Z l˜3» ơ ơ 2 ơ 8  ơ ; E; EE E;8E    E (Z ;/EE 2E31ơ ơơ ơ ơ8  ơơơ E (Z "D 4E&/;/EEE2 1^ơ‡2» …ơ



ơơ $ơ$ EE2E¦1^ơI?\ ơ28E31ơ8  ơ#ơrơơ $ơ  $ ơ  ơ m ơ ơ  ơ 8  ơ > > EE >8E  ơ @ @ EE A8E  ơ Êơ ơ ơơơ ơ ÂÂơ ÎÞơơơ •ơ •ĥ# ĉ #/ A/EơŶơơ &/#/ &/@/E E 335 ?:E 9%E #ơ Kơơ


U+



Ŕ ơơơ ơơ = &= &E = &8E31ơ!ơ *=+=Eơ¼ $  ơ *=E ơ +=E  ơ ơ è ơ  ơ ơ*= E ơ+=E£ơN#ơ Ŗơ *&/+&/E!ơ ¦@ơ Ů!ơ N’ơ¥ơ#jjơhơ $ ơ ơ *&/E!ơ Nơ  ơ +&/E!ơ 3Nơ ơ *&/E!ơ ¦1ơ  ơ +&/E!ơ 1#ơ ơ ơ ơ ơ -=E!ơ *=E_ơéL6^ơơ -&/E!ơ E ơ  $ơ DZ!ơ 1¥ơj##ơhjơ +ơ  -= +Z mÅ LšL6^^ LéL6^^^ơ 1’ hơ Ƅơ   ơ8  ơ -=E!ơ E ơ hơ   ơ#ơơ $ơơ -=E  ơ ơơ ƕơ $ $#ơ ơ *=E!ơ += E ơơ 





ơ» ơ$ơ$ ơơ6#ơØ ơ ơ  ơơơ   ơ  ơơ ơ ơơ ơ ơ6ơ ơ ơơơ = &0= &EE= &8 @ơ ơ  jơ #5*$!.-$# 5 5 ơŤơơ ơơŠơ  jơơ  ơơ Š ơ -= Eơ   ơơ )ơ âơ *&/E!ơ F»F» )Z D,Z 2E ơ *&/E!ơ (F» 2E_ơN×ơ ‰]» hjơĂ +&/E!ơ 3@ơ1ơ (Z D+Z 2E  ơ +&/E!ơ Năơ 2E_ơ@ơ]» 5+Z h#ơ ơ ơ*= 1^ơ! !ơ °# K  ? ŭÅƋāơ £ƠĊ Ŀ (),*=E £ơm 2h 2E £ơ

ơ*= (),+=E hơ ơ ơhơ ơơ ơ ơ*=E (),+=E 2E °$   *=E31ơ!ơ '= &= &E = &3 E ơ += _ 1!ơ ơ '= &0= &E = &3E ơ¼ơ ' E'EI’ ;,Þ ơơơ *=+=E_ơ*= += Nơ!ơ .'$= &E=B &E = &31E rơ  ơ è ơ ơ ơ =8Eơ''E!ơ N#ơ ơ  ơ  *= @ơ!ơ +=E_ơF8» K ¼ $ơơơ !=E_ơ@ơ!ơ += @Ćơ ơ   Ê 





















Y7ơ





9E ‚ơ ơ  ơ ơ  ,ơ ơ ,ơ ,ơ  ơ gơ !' A ơ ơ ơ ,ơ  ' —Þ  À¨ ' ơ  -ơ ƒơ ,ơ ơ,ơơ !' ,ơ '' ơ ' ơ '' ơ' ªÍÞ ' {» '   ' ' c’  !' ¦» ơ  !' Ô)Þ ơ ¦» \ÔÞ {y—~»G{» \c’ 1=WËơ ·ơơ !' ' ¦»  !'' Ô*Þ  gơ   ơ ! ' !' G¦0» ° ơ !' ¦» ơ ñ   !' ¦» \ÔÞ {1»  ơ $ ơ ơ  ' !' ơ ' ơ $$"-ơ K  $ '!'ơ  ' ơ  -ơ ơ  '   'ơ  '  ' '  '    ơ  ' ' ơ   '  $' ơ    ơ ' ' ķŷơ  '  ' ơ  '   '   ơ &'  '  ' ơ' ơơ  ơơ ' K  ơ' ii» '  ơ ' ' ơ ñ -ơ 















































#5 )$ .+$# 5 ƒ,ơ " ' gơ ,ơ    ơ ơ   ' ơ ,ơ ơ gơ

' ơ

,ơ ,ơ  ơ ÉÞ' ơ ,ơ ' ơ ,ơ  ,ơ ơ  ơ ơ ơ ,ơ  ơ gơ !' …ơ   ơ ' ơ ' ơ ,ơ ơ #' ơ ơ ' ' ơ  ,ơ ' ' ơ ơ ' '  ơ  ơ ơ '  ơ ' ji» ' ŏơ  ' <=W 3 ' '   '   ' ơ   ơ  ơ 

 'ơ  ' ơ  -ơ K  ơ ' ki '  ' ii» ' $ơơ,,ơ ! ' ơơ, ơ Eơ ơơ    !'  ' !' ! ' !''  ơ $ ơ ơ  !'' ơ ơ  ,ơ -ơ K "$$ơ'’!'' ơ ơ , ,ơ -ơ …ơ ơ ,ơ ÊÞ '  ơ '' ơ ' ơ ' ơ '    ơơ $,ơ  '  ,ơ  ' ,ơ ' rơ ä  ûơ ơ ơ # ' ơ ơ ƌơ  ! ' ơ ơ ' ' ơ ' $$ -ơ Ř"ơ,ơ ơ,ơ '  '   ' ơơ  '  '  '   ' K  $ơ  '   '  ơ $ ơ ,ơ ' ơ ơ

ơơ ' ơ ū  ơ ơ ơ ' K  ơ ' ơơ '' ơ, ơơơ ơ ' Ŭơ '  ơ' '



































qơ



„ơ ÉÞ \ ơ ơ  %  ơ ơ '1\2 \4\  ơ ơ ơ ơ  \ ơ ºơ ơ  ơ G#\ơ ơ #\ ãơ  ơ 41#\  ơ 42#\ơơơ H#\ ơơ 41#\ 42#\  ơ % ơ ơ '412\ \ (41#2#\ „ơ ơ %ơ  ơ  ơ ơ 4\ ơ ơ 41\ ‚ơơ %ơơ 1 \1#\3 \3#\  ơ ơ  ơ ơ 1X\1Y\3Z\2[\    n#ơ 1\ĸ ơ 4\ % ơ  #ơ ơ  ơ ųơ nơ ơ ơ ơ ơ 4\  Ēơ ãơ ơ %ơ ơ ơ ơ 11 2 2\ ơ ơ  %  ơ ơ '4\13\ ơ ơ ơ ÉÞ’  ơ ơ %ơ ơ ÉÞ \ ơ ơ ơ 1\ \2\\\ ơ ºơ %ơ ơ G#\ ơ ơ ơ 1 2 \ 14\  ơ 24\  ơ ơ ơ ơ   ơ ơ '124\  ơ  ơ ơ  ơ 14\  ơ 34\ ơ 1Y\  ơ 2[\    n#ơ ơ #ơ #ơ >20Z  >51Z  \  65 Q\ „Þ ơ 1\ ơ ơ 2\ ơ ơơ E(Þ ơơơ 62 Z 72Z  75Z  RÞ ơĦ ơơ •%ơ•ơ 72Z  75Z %  ơơ 1#4\  ơ 2#4\    n#ơơ 1#2#\QQ `È§Þ ËĽÑÌ´Ä¿wÞ





e'

Śơ ơ    ơ

, ơ ơ  nơ   #ơ [ ơ ơ ơ ơ , ơ ơ ű  ơ

,:QQ,>Q

‘’ ‘’ ’ ’

„ơ 5



,:Q Q,>QQ,>Q Q,>Q !,:Q Q,> QQ!,>Q Q,> Q : >> >Q :>Q

 ơ  njơ[%ơơ   ơ8 ơơơơ8 ơ

,,5 7 ,5 QQ5 7 5 QQ5 Q

 ơ

,,5 ,5Q 5 5QQ5Q 

µ  ơ ,Q ơ ê  ’ơ ơ

,5Q @Q Æơ Q,5 QQ,5Q Q,5Q

ơ

,5Q Q  ,5QQ,5 ,5 Q

 ơ   Ɩơ ,Qơ ơ  % ơ  ơơ ơ %nơ  ơ 2>QQ{» Q >  6FQ ơ {»Q,6Q ơ FQ ơ ơ %%ơ › ēơ ơ    ơ 8   %ơ {» Qx{» Q:  7FQ Q{» Q> 7FQ 6(FQQ:QÞ > Q  ơ  % Šơ  Q{" SÌ B» Þ :Q Q>FQ :Q Q>Q ¬% ơ ť ’ơ ơ   ơơ ÌÞ @  ơ {»QQ ơ ,Q> Q QQ> Qơ  % #ơ ¬  n’ơ  ơ   ơ  Šơơ       ơ 8 #ơ 



Q

®$Þ +] ?IVG] /I] %’



 i

M] ^i «ŒØލÞ i,] 72] M-] EG/8I] >bc?[HDBi /?]/?G I/?*] \cBDc i Y] GMBQ9HI] CJMR2M! V] -U"] ‹zÞ 0’

0’ _i (CG] 22]

²Þ



¬Þ M+RK] ]E’ -U#]

0’ _Ni 0’ 9OSi 0’ #`Ni

PÞ V-/-

/:E2/I] N.N] !_L);2iŽÞ ~Þ 8Oi k’ L’

;Z ,» 0’ ÐĄơi iZFZ REECK] ’ )\ci IC; ] FZ G’ #Þ DM$] M-M] ÇyÞ e» )\ci ³Þ FZ Z Þ M-? -» Z ­hÞ NÐ)ơ /?%] NBÞ ;’ $Ti ?] B Nơ ~Þ xPÞ V] -U] %.i 9\ . ’ /?# FZ Z Z GAB^M+E5P8Qi PÙÞ &-i Nơ žÒ»Þ M-RJ] Q_xP•¶¼«Œ¹P e» (’ ./I] :
M] •µÞ









Z

"gI4Ji C–D&PÞ ~Þ

/M/C?3=J] ·Þ 0’

Z ž?¥Þ



'Ti KYaUfK[Gi

~Þ 8Qi

’ 0’

†» 0’ i ]ci &}’ 0’ -*Z

¨/» V-/-] /I] ] C?NG\

i /I] 0U/J/2'] Z] 62] ECI/P/U#] /?M*"GJ] ;Z „…» M-SI .» ~Þ " i M] M-?] )C22CVL] M-M] !z’ /J] /U/K/2] Z] i |Þ Z "i  i PÞ ®Þ "i ,i /?22[] JTEECK#] v» Z 1 i œÞ M+@] -U] $Vi {PÞ Z "i e» €’ /&] ’ &}’ Z (Wi x V-/-] /K] ^’ C@MG/M/C?] I/?] +’ 0’ Av]^B%Þ 5>J Š» 0’ r}Þ CG] /’ 0’ "1 i /?#] i e» -’ ?

XM] †d’ ?CM/ ] M-M] M-] K[dCGDci

€’

Z i :’





$|’ 0’ "<i V] -U#] #{’















}Þ  i

 UMAWM/U fi J/? ] '~’ 0’ *i Ÿ¾¦Þ i 07i #i ! i





d

’ V] C?2Z] eh@Bi MC] -1] M-] IJ] "z’



 !(  $ $ (  $'(   (  " " ( & ( (  % ( #( )' -/ /

-/ /

/

 

!% ( (  #!"!( " (&# ( !% !(("( !% (!"( $(

 ( "( #"( ( #!"!( " ( &( "( "" !( ( & / ( / * /  ( (  !( "( "(  "( !% !(("( !% ( !"(

 ( "(" (! "(#!"!( % "( &# ( !% !(( !% (!"( (! ( "(    "(#' !(%( &# ( !% !( (!"!( (("(#!"(&# ( !% !( (#!"( !(   ( ( # " !( ( %(

  (/ &/(/ '(/(+$/ ('/ $/ /  /  /  /  % / /

' / /

) / / + //









/  /  $/

,//

 (/  / /& /#)!&/  (/'/(/"#")!/* )/$/  ./  # % //

' / /

) /  /

+ /  

-//

  "'/  ) ('/ (/ ')"/$/(/ &'(/( %$'(*/ #(&'/ #/ #'/ ( (/(/ ')"/'/ / /   '/$)#(/$#/#(&/(+/ +/ $#/'/(

% /!/

' / /

) / !/

+ /"/

-//

 / $/ (/$ $+#/'/ /%$''/#)"&/$/ $# '/$/ /$#*,/%$-$# % / !/

' / !/

) /!/

+ /!/

-//

  (/'/(/ &'(/%$'(*/#(&/ ( ' ('.#/  /   % /!/

' / !/

) //

+ / /

-//

/

< 6ę J"ę&ęęPJ  ę" ęÛŴ JJ ęęPę>% Ŵ bęJęę^ ę ęęêŴ ęęĆ&%Ŵ fi

>i

,O

9

Ci

Ri

=i

?i

!i

Cę @ i

dΊ

2Yę 

/ bęJęę^ ę ę ę  žpę ľ ľľľ eiľ W eiľ

îę . 2ęľ

2ęľ

ę eiľ W eiľ ę eiľ 2ęľ i

>i Sľ

Cę ,Oę

Cę @ i ľ

Cę Bi

Cę 2Yę

/ ľ.% ę 6

9$ œęę (ęęęę &J"ęJę2! .ę(r ęęrr^ęr"ę

>i ”•ľ  Kę.ÜÝę  Cę ,Oę Ƙ™ľ / KęSÀÁŴ ڐÕÞę ¬ä°ę ą–—ľ ÏÖ  ´ę SŴßę  //C Bie56i 9 !ę.ęę 9 Cę ­Y±ę ęę Ò µę śŴ àę ُ Ç $ éę^ ę ę 

ęľ ę



8x .%&"#/i

ę ę 9ę ľ  ľ .%iľ ęj . Ôęľ ę 2iľ 





ľ ę

õþ“ľ

>i





^_ę

,Oę







8MM× .%&ZZai  Cę
@ i



{



8 MMx .%&ZZ/i ęCę
Bi



{



8MM‘ .%&""1i  Cę

2Yę šę ęę



 $ œęę[i  ę\i ęJęę ę(ęęęęęPJę e% *ęę ꂵ bęJęę^ ęę Diľ v›ä \i [i

>i Cę ,O

@ i

Bi

2Yęšę ę^‹

ę



ęľ:ę

ęę

 bęęę đę&ę-

ę )ľ -

ę )ľ -

ę ľ -

ęę +  ę(ę;p

- U

ę --ę

  Zę

ľ ľ

P ehv

Aę&ę [ľ Pi  ę Aľ ę+ę"Aę öÿ ęę+ ęęľľPiľAľ



ľ ęę+ę ęę @ ęęþę+ ęę ľľ f ľ

"Yľ

ľ - ę

Œ @ ęę+ ęGęę ęę

"

- ę

< }ęľ ęę

Yęęę <   ęę7zä Y

ę Ø ę

ę   < ę

ęÈä 

-

ęfę

-

ę fę

@ ęę ę&ęľ ę +  ę(ę Ŵ ę  < q+ęędi 7i ę ęęġdc ľ !ęi ęę+ ęędi g $ di di -I

»ž &Q %ęę% (ęę  ę ę%e ęę k!ę &ę%ękę%č"ęę%Œę U &Q ęꉄę%ę%ę ęęęęęę%!ę&ęęę ę ęę

kęę%ęę%e p ¼Ÿ { ęę &"ę¥¦ęi"$

q+ęę G[iľ G \iľ ńŴľ ŋŴľ Gci ľ 

lę  ę Ŵľ ÒŴľ %Ŵľ Ŵľ &Ŵ ľ ę ľ

ĝ \ľ i ęę+ 

$ q+ęęěľ ęę+ę"ę ęIę ľ Ŵľ -ęľęľ ľ ľ ľ ľ€ľ 6ęïòù¢ęęIę %ę ę7ę @ ęęęęę



-  Zę:ę Tv / Wxę / <ę @ ęę' 'ę&ę:ęę +  ę(ę 

<

- Zę?HFDRSęęęs"ę%ęęę " ę?DLęHRę ęFSę'%ęęęę ¥h  ęę'ę*ęę'" ę *' ę(ę (ę 'ę   %ęęę -ę2*'ę s Lę 'ę * ¡HFDęę-<ę @ ęę'ę*ęęs"

-- ZęFę ę ę %'% ę &ę ' ę -

$ę Tę )ęę 'ę  % ę  ę ę %'% ę  ę  %ę&ę(ę&ęęs% ę-

7ę êęęę '"ę ę)pę

-ę Zęę .ęę+ę' ę'ę%ęęwęľÄäľ ­®ä ľ RŴ @ ęę+ ę* -/ę

 Ąľ

-< åŴ ę PLę IęúĈĉľ ę ę 'ę *ę ¡ìíę Uę ę * &"ę i"'Lę *ę DRę–ę–ęHFL Ię#ľ ľ ę ę õŴľ ľ Kęi IęĂèê ľ

éľ

çľ

-< bęęę' %ę*ęę' ę'ę*ęęP Ðę ľ ęľ- -/ę ę



Ñę ,ľ  ę ľ - «ę

- < 6'ę'ę*'ę ę*ęÛę ›ę&ę ęLę &ę&ę9Kę  ęę&ęę @+ę (' ð' ę Kę-Kę³ęę ęęeęĖē!ęęę' Đę*ę'ę'Lę%"ęę*ęę   ę +ó"ęęJę&ę 'ę ę 6ę 'ęęę ('ę&ę%ę ęę Tę 'ę*ęęċ('ę&ę  -/ Zę Ęľ ËŴ )ęñę &ę' ę'ę %ę ę :Wę :) : w½ )w

-9 6'ę'ę ęõ&ę"'ę+ ę*ę ęė%ę ę"'ę+ ę*ę 



Tv

)ęľ -ę ę )Wę

W m  Š Š-ę



:ę ľ -ę @ ę ę + ę *

ę ę"'ę @ ęęę*ę ę

 h&ę(ęę*ę"ęG!ę.ę(ęęP

bäľ Ïä

Tv

  6ę ªfę ęę(ę% % ęę !ę ęUę%ęęę&%ę%ęę " $ ˜+!ę(ę% % ę ęę  (ęę "ęę*ęęęę*ęę&ęę 6ęU %ęęę% % ę ęľ ę ę ęęęę ęę 5 !ęę ę*ę  ("ę ę(ę 9ę ę  ( ęęę ę *ęę ęę ę *ę   ("ę /ę(ę /ęę  ( ęę ę ę*ęę U*ęUę  (ęę+ęu "ęę(ęę+ę&u "ęęę (ę% % ę ęę  (ę Kę&ę(ę ę% ęUę+ę   zę

  6ę* &"ę%" ęę* ę(ęę%ę*ęPęę ÷ę„ę Tę %ę ę*ęęPęYęę*ę "ę/ęôę Zęęę*ęę%" ęęę%8 ę @ ę + ę*ę$

æľ

äľ

I

îľ

ñľ

ìľ

đľ òľ

ôľ

  Tęę ľ ęę+ę"!ę  ęOiPi ę+ę ęę&ęOiiPi Sv $ę @ ę ę ę+ ę* ę ę 

ę OXi





,I

ę iPYi

 bęęę "ę+ę"ęľ *ę&%ę)Eę i

ęę + ę(ę ľ i z



 Tęęę&ęęęęP ę

ę -ę ę  ę ę V[ -ę 0 V[ ę 0 V[ ę ę€ę ęŸę @ ęę+ ęę€ęęŸ<ę $ Tęę !ę1Aęę .ęę ęę3ę ę1.ęAę

0ęę1Ąęę;Aę

Eęę1Ääę ę  ę

@ ęę+ ęę oęę1.o7ę

ę

Óęę1Éęę $

3<.$A?C*Y $F0*9$F3'$6Y ?'3*FIY   & # # & % &  & 8 "Ŵ $ & #& $#"& < 5Nę G5 O3ę 

ęęęęę37

< 5Nę GO aę8ę ęię Tv 2 8ę < 7 5Nę @ aę ę i ę i7 7 ęľ i

ę Tv   $  5Nę A

¶ )2)ęº ę 6ęęę " ęęę)‰ ę ("ęę 7ę h3ę A  ę  5Nę @ O3ę)8ę 0ľ Eę Tv  $

 5ČNę A

}ęOięę "ęę>¢Ŵ 6 ę—DRę ęęøDRFę

VOi Oi ę l 8ę iOi7ę ęËl VOi ľ DI

Tv

CI

Rv

«pä

; 5Nę GO a

ę 2ęi2ęi ę ę 

Liię





ę iii 2ęi 2ęi¯ 2ęi

2ęi





ę ę ę  iii Lii ę Lii ę Liię Lii







2ęi2ęi







Lii

ę

ę T (%

Liię

Tv



Lii Ê ę

Tv

9 5=ę G5 T3ę0ęi¯äęę34ęP!ę4ę 4ę "4ę4ęę !ę!ę!ę!ę !ę$ę h3ęG{ę ę A i 4ę4 ę @ę GOę ę G!ę 4ę 4ę4&ę "4ę ę  ę ę ę ‹ę h&+!ę ęęę  ę&4ę !ę 4ęPę4ę ę&4ę4&ę[¨ę 6ę4ęęęG5ę&ę i.0ę 4Ēę4 4ęę99 ę  5Nę A

T3

c…ą †Ćę  0d   c  0dę i





c  0dę i f

c  0d ę

c…ć †Ĉę  0d »  0MM‘

c 0 d ę

3ęę4 3ęę 6ę4ę 4ęę

ę 

ę

$

FI

?@

A 0¼ Ev ę

  5=ę GO

[i i\i ľ $ę 6ę (ę ęę 4ęę !ę;  5=ę $

ę >ľ

ęľ

ęľ

ę Hv ę >ľ ę ľ ę Hv ęG ę;$

 $ 5=ę  $ Y(ę3 3 4ę&ę44ę  ľ !ę 1ę ľ ę ęĐľ ľ ;  5Nę   Lii ,Ŵ Li 

+=

L  ľ ! Li DI

 ę ľ  ę&ęę ľ ę  5=ę  <

i Yi



Yľ

i ę"0ę i

"0ę ľ  

ę

$ 5=ę  ęęę $ęęę ę

ę

ę

ę <ę$ę$ę

ęÆę

ę ©ę  ę ›+ę44ęę + ęĔę

ęfę



  5=ę  ę ę aę#ę,ęľę  ę8ę Qv ,ęľ b  ę8ęę ľ



; 5=ę ;7 6ę #ęu„ę3ę3ęęęę:ęę 0ę 'ľ ę`ęľ E7ę6ęę #ę  ęękę#ę3#ęę#3e ęęę ľ 8 ę;7 BI

9 5=ę   aę#" ęęę ("! Ìę +ľ ę ľ  l‡ H[i +ľ H]ię H^ię H`ię Hc ię9ę ľ 9 l! ę ę

 5=ę  ),)ę ę ę)ę yę 7ę 6ęďę "ęęIę 3#ę (ęę !ę AęAę!ę !ę9ę 6ęI ę 3#ę ę&ę !ęAę;ęęę 6ęęę 

Ięęę i ę ię 

DI DI DI

 5=ę  'ľę; ę DI

,; 0MMot ęyę; ę ľ ;ęľ HŴęęH›Ŵ 6ę

ę + ę'~ľ

 5=ę   T3ę##ęę—HFę y##ęę£¤S?H!ęSFBęB?H7ę T # (ęSFBęBRD7 â(ęę#ę#!ęSRBęB?DBęBHFę# ?SBęBHRBęBFD7ę 6ę~SDRAę~DHFę# ę~H?Sę# # ę##  "#ę# ę#3ęęüę#ę##ę7ę 6ęę# ę##ęęęk#"ęę  g  5=ę 7 { # (!ę ę ęę ę ę ę ę 3ę ¦œŴ }# ę ę ę ?Î! ? 8 !ę NŴ ę 3 3e& 3gę ™ę #ę ę&ęę ęę ę#ę# 7ę ™&!ę ę#(ę&ęę j\K N0KŴę? M!ę ę #" ę í0ĥŴ 8i ę 6!ę )ę 0ľ ę @ę)ęęAę ę #eęęę  ęę+3ęę#ę # ę" #ęĕ#"²ę ęę& ę#(ę ę 3 7ę 6! )ę ę7 +

F

F

I

 5=ę  a"ęę#3ę#ęÐ,Ŵ q˜Aę &ę"ę`ęę.`ę ľ B ;ę ,Ŵ &ę`ęęB ;ę 6ęęę B7



ęRŴ # ę ę (

$ 7>K  $ Ię#ľ K Ięğ#ľęę K <ę &7#&K ?|ę =ę |•ęę?Dę Nę D”ęęę =ę ę ,@>K ?|ę =ę ?•ęę?DęNę ?”ę ę 7 ę ę!7%K Ięā;ľ ę =ę $ę &7#&K Ię;ľ $ 7>K

$ &?K ]ęę

ľ  ę i $ę ,&7K ]8 t

9ęę]<ę 84C&K ?8K )7%K?,&K98>0?0C&K ;88?K ]ęę;7ę &7#& 8ę ľ  ę ľ

ęę !ę >8K?,&K 9;8%@#?K 8'K ?,&K ;&!4K ;88?>K0>K

$

v 7>K < ">&;D&K?,!?KE&K"&*07KE0?,K ę904&>K8'K >?87&>K !7%K&7%K@9KE0?,K ęi9ęi9ęi ę ęę904&>K8'K 87& >?87&> K ?K ?,&K &7%K 8'K &!#,K ?A=K ?,&;&K 0>K &F!#?4HK 87&K 58;&K 904&K 8'K >?87&>K ?,!7K ?,&K "&*07707* 8'K ?,&K ?BK ,@>K ?,&;&K #!7K "&K &G!#?4HK ęm

ę ľ ;ę 4&*!4K ?@=>K &7#&K 4!H&;K ę 0>K ?,&K );>?

94!H&;K E,8K #!778?K 5!3&K !K 58C&

;$ 6K $ 07#&K :ę ÌÍŴ )ę!7%K:8ę

)Węę2)ę :ęQAę E&K +&?K :ę ľ )ę$ę ę ,@>K :8ę ľ )Węę2:ę ľ )Qę ľ ęęę !7% :)ę ľ æã:ę ,ľ )QWę :8ę )8 žęęm$ę ,@>K :) : Eę · )Eęę:) 2:ę ľ )Q2:8ę ľ )8ę :)Qę ę$  2  2  2QQę

9$ 7>K 9$ 07#&K

8 ‡   ę ę §ę    ¿ !ę ?,@>K ę!ę!ęAę;ę ,&0;K >@5K0>K 9vę  ę 

v 7>K $

]0ę '8;K >85&K 98>0?0C&K 07?&*&;K ] 0504!;4HK ęę

‚Wę '8;K >85&K 98>0?0C&K 07?&*&;K ‚$ę ,@>K ]ę Rľ ‚ęę $ę ,&;&K !;&K ę 9!0;>K 07 ?8?!4K 7!5&4HK 2 !ę Q!ę2!ęQ!ę$ $ $ ęAę2 Aę Q$ 07#&K ęę

ę ľ .ęm

 K 7>K

ę

0>K !7K 07?&*&;K .ę ľ

$

07#&K ?,&K 9;8%@#?K 0>K !?K 58>?K !K ?-;&&%0*0?K 7@5"&;K ?,&K 98>>0"4&K !7>E&;>K

!ę !ę  ę 8;K $ę 07#& ę ę Ŵ ľ :Ŵę Ŵ ľ   ę ę ę ľ  ęę ę ľ ęęę ľ  ęę:Ŵ ľ º Ŵ ę ę ľ ę  ę ę ę ľ  ęę ę ľ  ęęŴ ľ  

ę ę ľ

ę ę Ŵ ľ Ŵ ę ę ę ľ ęę ę ľ  ę ę ęľ ę ę ęľ !ę ę ę ľ ęęŴ ľ Ŵ ?,&;&K!;&K ę 98>>0"040?0&>

Í ę

Uę >,8@4%K *&?K 0>K

I

 7>K    &?K @>K @>&K ĉ>ę ?8K %&78?&K ?,&K 4&7*?,K 8'K 87&K >1%&K 8'K >:@!<&K ę ,@>K ĊR>ę K

/nę &?K >ç>ę "&K ę !7%K >D>ę "&K 1ę ,&7K >S>ę 1>K ę/Kę>H>ę 1>K ęLę>è>ę 1>K  ę ę/Kę>å>ę 1>K   ę ę/Kę>F>ę 1>K   ę ę1ęę/Kę !7%K >?>ę 1>K   ę ę 1ęę/ę &7$&K E&K ,!C&K ,  ę ę 1ęę/ę , ęęK , ęę1ęę/ęę, ęę/ęę,  ę ę/ę ,  ę ę 1ęę/ęę, ęę1ęę/ęX, ęę , ę/ęę,  ę ę/_ę Xęę !7%K 1ęK nę &7$&K?,&K !<&!K8'K ?.&K<&$?!7*4&K 1>K  ę)ľ ęXę  ę

,&<&'8<&K

  7>K  8?&K?,!?K

1ę ’“ę ę !7%K ,ę 1ębi ’“ę  ęę \ęę1 \ęęę ę  ÀXę á_ ęę \ę ęę1 \ę ęę \ęę1 \ęę,ę 1ę\ę ę



,&7K

ęK 1ęK Lę E&K*&?K ę ,@>K ?,&K >5!44&>?K C!4@&K 1>K gę

 7>K g )Eęę

ęK ,)ęę ,)0 )ęę /  / nę 8K1'K )ęę ę %1C1%&>K )Eęę

Lę)ęę ę 5@>? %1C1%&K / ę ,@>K?,&K4!<*&>?K )ę 1>K _

n 7>K   0  V[ ę /&7K 0  V[ ę K .   Lę 0  V[ ę K .  ę ,@> ę ę ę ę  K ę ,&<&'8<&IKK  Åę 8IK 0V  ęK  ę h3 0 ęX .ę .  ę .  ę

 ,&<&'8<&K €ę(KK  g &?K .ę K





$ 7>K  Eęę1.EęK , 0ęę1.0,ęę. , ę1..ę ,@>K E&K *&?K  ęK /,ęę. Ę_ę 2514!<4H  Žę ûýęK , Eęę1.E,ęę. , 0ęę1.0._ę 8K E&K *&?K  ęXV ę ,ęę.ę /ƒę 84C17*K E& *&?K ęę.ętę !7%K ƒęK9ę ,&<&'8<& ę oęę1 āę K , Žęę1 Ă,ęę. , Eęę1ăƒęK  ,  ,9ęK ę

ę

2;,#@>B)Y #E/)8#E2Y >&2)EHY  !& # # & % &  & 8 "Ŵ $!& #&  & $& 5 !ZĨ [Ĩ£Ĩ 7Ĩ

q.À .Ĩ

!# #& JJ1?CJQ +GQ ?+@OQ DK1GJ6B@GQ +GQ OBKQ -+@Q &BQ -+:-K:+JBFGQ +F1Q +::BM1/ Q )5BMQ +::Q J51Q GJ1CGQ 6@Q OBKFQ MBF96@4 Q +-5Q DK1GJ6B@Q -+FF61GQ 5Ŵ ?+F9G Q

$ 1  Ĩ Ĩ Ĩ G YOQ Ĩ :!ĨĨ U  NQ >ĨOQ Q

GYR "2Ĩ,ĨO  $

$ #Ĩ : Ĩ TY Ĩ Ĩ y Ĩ Ĩ Ĩ Ĩ :Ĩ  Ĩ

 Ĩ  Ĩ: ÑĨ SY >Ĩ U 5 ĨĨĨ: Ĩ Ĩ*Ŵ +i FĨ ĨĨ  ZĨ  Ĩ Ĩy Ĩ ĨĨ  :Ĩ Ĩ

 Ĩ Ĩ: $ .$ 5 Ĩ Ĩ Ĩ :Ĩ ľ   Ĩ € Ĩ H !Ĩ Ĩ  Ĩ :Ĩ :  ZĨ  Ĩ  Ĩ ľ  Ĩ Ĩ

”‡Ĩ M!Ĩ  Ĩ  Ĩ Q  Ĩ  Ĩ Q Q  Ĩ Ĩ !  Ĩ€ ‚Ĩ  Ĩ Ĩ  Ĩ Q   ‡Ĩ hĨ ĨĨ   Ĩ :Ĩ Ĩ  Ĩ   Ĩ :Ĩ  Ĩ  Ĩ: Ĩ! Ĩ Ĩ€ Ĩ ! r [œ _Ĩ Q Ĩ   Ĩ :Ĩ "Q Ĩ CĨ Ĩ Q  Ĩ Ĩ   Ĩ :Ĩ "Q Ĩ  Ĩ  Q #Ĩ   Ĩ Ĩ Ĩ Ŵ  Ĩ 'Q Q ' Q _:Ĩ Q šä Q  Ĩ  "Q 7©$ !šŴ

’ Ĩ Ĩ Ĩ Ĩ Ĩ :Ĩ Ĩ U  Ĩ   Ĩ  Ĩ Ĩ Ĩ  Ĩ 

! ĨĨ ĨþĨ ľ :ĨĨ Ĩ Ĩ€ĨĨ yĨU  Ĩy Ĩ  $\‚Ĩ   Ĩ  Ĩ Ĩ Ĩ ©k$Ĩ   !ZĨ ľ  Ĩ Ĩ!Ĩ    Ĩ $Ĩ # uĨĨ:  Ĩ ľ Ĩ7Ĩ Ĩ  Ĩ .$Ĩ 5 ĨĨ Ĩ Ĩ Ĩ:  ĨĨ [Ĩ $ Ĩ

2;,#@>B)Y #E/)8#E2Y >&2)EHY !4<.$A?C*Y $F0*:$F4'$7Y 7I:A4$(Y !Y yssĊ $ & #&  & $& $#"&

5  Ĩ  Ĩ2-Ĩ Ĩ Ò

ª“

2 #Ĩ ."3 kAĨÓÔĨ [Ĩ $$Ĩ

3 kAĨ,Ĩ[

"Ĩ, Ĩ ĨÛä Ĩ

Ĩ Ĩ 0Ĩ ,Ĩ™$Ĩ 3 ™ 0 "ĨV

ÎQ

#Ĩ"Ĩ  „Ĩ -ĨĨ Ĩ"-Ĩ2=Ĩ



Mv

Íä

Ĩ' „ĨĨ k-Ĩ -Ĩ' „Ĩ -Ĩ'Ĩ-Ĩ kĨ-Ĩ'Ĩ Ĩ ĨĨ  $Ĩ

ç ĨĨ SY,Ĩ1Y Ĩ RJY 5 Ĩ SY  Ĩ 1Y ěĨĨ Ĩ Ĩ  Ĩ Ĩ Ĩ 8ĨR : Ĩ  Ĩ xĨ 'i EDĨ Ĩ  Ĩ Ĩ Ĩ  Ĩ xĨ 'i E$Ĩ ·5 Ĩ DY Ĩ QY,Ĩ Y  Ĩ Ŵ8Ĩ9lĨ # hi ĤŴ,ĨJCŴ 0Ĩ ŭC˜Ŵ # Ĩ 9Ĩ0Ĩ FĨ $$Ĩ 9ĨĨĨ ĨĨ Ĩ Ĩ  Ĩ Ĩ  –

Mv

lv

gv

hĨ  ĨĨĨ  Ĩ Ĩ Ĩ  Ĩ Ĩn3  lĨ `ĨĨĨĨ  Ĩ  ģĨ nĨV Ĩ  Ĩ  Ĩ !Ĩ Ĩ  ĨĨ ĨĨ  Ĩ  Ĩ ĨĨ  Ĩ Ĩ ! !Ĩ ¯Ĩ Ĉ:  $Ĩ#ĨĨ ĨnĨV Ĩ Ĩ   $Ĩ ´•

ayĨ Ĩ ĨĨ Ĩ  ĨĨ Ĩ n 3 Ĩ  \Ĩ 5 ĨĨĨ ĨĨUĨ ĖĨ  ĨĨ Ĩ! !Ĩ   \Ĩ `Ĩ Ĩ ĨĨuĨ ėĨ ĨĨ !Ĩ   Ĩ ¦ Ĩ Ĩ Ĩ  Ĩ  Ĩ Ĩ  Ĩ Ĩ Ę Ĩ Ĩ Ĩ xĨ  \Ĩ #Ĩ Ĩ Ĩ Ĩ Ĝ  Ĩ nĨ3 Ĩ  Ĩ Ĩ Ĩ   ĨĨ Ĩ Ĩ  Ĩ  Ĩ  Ĩ *Ŵ  Ĩ Ĩ Ĩ Ĩ Ĩ ü !Ĩ   $ĨB  ĨĨ  ĨĨ  ĨĨ Ĩ Ĩ ĨĨxĨ Ĩ!ĨĨ Ğ Ĩ Ĩ !Ĩ   ĨĨĨxĨ $Ĩ #Ĩ Ĩ Ĩ Ĩ ĨĨn Ĩ Ĩ3 kĨ \Ĩ

Y

93Ŵ



BĨĨ &Ĩ8Ĩ<6Ĩ `Ĩ T WĨ KDĨ T Ĩ Ĩ 9DĨ TdĨĨ ïĨ Ĩ T" > 9ĨĨ LzĨ  Ĩ"Ĩ>Ĩ9Ĩ>Ĩ ľ Ĩ Lz%Ĩ #ĨKĨĨ 9Ĩ>Ĩ ľ  ĨĨT dĨĨ KĨĨ 7z%Ĩ

BĨĨĨĨĨĨ ĨĨ %Ĩ aĨĨĨĨĨU ĨĨ Ĩ U Ĩ Ĩ Ĩ Ĩ"Ĩ Ĩ Ĩ>Ĩ SĨ>Ĩ ľ ľ ľ >ĨĨ"Ĩ3 'Ĩ Ĩ"A6Ĩ #Ĩ Ĩ

!Ĩ H ĨĨĨ ĨĨĨHĨĨ ĨĨU ĨĨĨ Ĩ Ĩ!;6Ĩ #ĨHĨĨ Ĩ !Ĩ ĨĨĨĨU Ĩ Ĩ Ĩ Ĩ Ĩ KDĨ9DĨ%Y Ĩ;8 Ĩ ĨKĨ>Ĩ9ĨKĨ>Ĩ Yľ 9Ĩ>Ĩ .ľ BĨU  Ĩ ĨĨ ĨĨ N AĨ Ĩ" %Ĩ #Ĩ ľ ĨĨĨ  Ĩ !Ĩ  ·’



Ĩ;AĨI KA I 9A I

%Y

ĚĄ Ĩ ;Ĩ8ĨK > Ĩ9Ĩ K > Ĩ Xľ 9Ĩ>Ĩ ľ

#•Ĩ;ĨĨ !1Ŵ KĨĨ SĨ 9ĨĨ %YĨ +Ĩ#Ĩ ľ Ĩ 4Ĩ  ĨĨĨ % Xv X Xv [v`v Xv \v Yv [v v gv gv Xv ]v Xv ^v Xv _v

5I

gv

%ľ &ˆľ



v##

`v [v

?%ľ ?ľ #v



%ľ TUVĨklmľ

aHĨ Ĩ Ĩ Ĩ ľ Ĩ oĨ Ĩ Ĩ  6Ĩ `Ĩ KĨ *i 9Ĩ *i .ľ 1Ĩ uH Ĩ;Ĩ Ĩ Ĩ  u Ĩ Ĩ Ĩ  Ĩ Ĩ |ľ 1Ĩ Ĩ  Ĩ Ĩ !Ĩ Ĩ Ĩ KĨ>Ĩ9ĨĨ;Ĩ3 %Ĩ #Ĩ 9ĨJi ; I ' m%Ĩ fĨ K /ˆ — 3 PDĨ Ĩ 9Ĩ ľ ĨPĨ Ji ; † ' m%Ĩ # ľ

A AĨ *i;A I PA I ; I I P' Ĩ Å i I ' I  SP I P; I ' ' l

5Ĩ Ĩ Ji PĨ Ji Èi3 Ĩ ' mĨĨĨĨĨHĨ ĨĨ SPA3 Pˆ A ĨPĨĨ ĨĨPĨĨÆ ľ *i 4;I 7 ĨWĨĨPĨĨ 'Ĩ Ĩľ *i ; > 7;3 S' m4Ĩ  Ĩ Ĩ PĨ Ĩ ; 3 ' m%Ĩ #Ĩ ľ )i 7Ĩ  ;Ĩ Ĩ SZĨ ľ )i Ĩ Ĩ;Ĩ Ĩ 4  Ĩ ľ *i S ĨĨ;Ĩ Ĩ !1 ľ (i LĨ  ; Ĩ 7%Ĩ 1;Ĩ (i EĨ ľ (i %Ĩ #Ĩ Ĩ !ĨĨĨ•ĨĨĨ ˆiĨ ÇiĨĨ SĨ ĨĨ K  Ĩ9ĨĨ %YĨ %Ĩ 1; Ĩ 4 Z ĨĨ Ĩ cŴ 9DĨ 'Ĩ ĨDĨ Ĩ ' DĨ DĨ Ĩ '%Ĩ #Ĩ ľ Ĩ7ZĨ DĨ S6Ĩ #Ĩ ĨĨĨ%Ĩ 4Ĩ

aÐĨ ’ĨĨĨ Ĩ 4Ĩ Ĩ Ĩ !Ĩ Ĩ Ĩ HĨ Ĩ 4Ĩ  6Ĩ fĨ Ĩ  ĨĨĨĨ ĨĨĨ Ĩ!ĨĨ$Ĩ ĨĨĨ ĨĨĨĨ Ĩ Ĩ Ĩ %Ĩ fĨ Ĩ  ĨĨ Ĩ.-Ĩ Ĩ  Ĩ Ĩ Ĩ 4Ĩ  6Ĩ fĨ 1$Ĩ Ĩ Ĩ Ĩ Ĩ ĨĨ Ĩ.Ĩ ĨĨEĨ ĨĨĨ  Ĩ ĨĨ Ĩ6Ĩ aĨĨ Ĩ !Ĩ -ĨĨEĨ ĨĨĨ  ĨĨĨ ĨĨ6Ĩ fĨĨĨĨĨ Ĩ HĨ Ĩ Ĩ  ĨĨ Ĩ S%Ĩ fĨĨ ĊĨ  ĨĨĨ Ĩ -Ĩ Ĩ Ĩ  Ĩ Ĩ Ĩ 7Ĩ  Ĩ1Ĩ '$ĨfĨ ĨąĨ  Ĩ Ĩ Ĩ Ĩ -Ĩ -Ĩ Ĩ Ĩ Ĩ  Ĩ Ĩ Ĩ .Ĩ Ĩ1Ĩ.=šĨ #Ĩ !ĨĨ ĨĨ -Ĩ Ĩ Ĩ -Ĩ -Ĩ ĨĨĨ Ĩ Ĩ1Ĩ 4 =6Ĩ #Ĩ ĨĨĨ 6Ĩ 'Ĩ

 [v

7vv* *I v56v6 v* : *Ii:vZv89v"v

 =Ĩ

.=Ĩ

'Ĩ



4 =Ĩ

2;,#@>B)Y #E/)8#E2Y >&2)EHY   & # # & % &  &  "Ŵ   & #& q.¾o Ĩ

# !DĨ .Ĩæ!Ĩ7Ĩ  # #& @GM1FQ $$Q ³¶Ŵ DK1GJ6B@G Q @J1FQ OBKFQ +@GM1FGQ B@Q J51Q +@GM1FQ G511JQ CFBL6/1/ Q

BFQ J51Q ?K:J6C:1Q -5B6-1Q DK1GJ6B@GQ 1@J1FQ OBKFQ +@GM1FGQ 6@Q J51Q +@GM1FQ G511JQ ,OQG5+/6@4Q J51Q ,K,,:1GQ -B@J+6@6@4Q J51Q :1JJ1FGQ B¶Ĩ

:ä JDĨ žä BFQ M'Ĩ -BFF1GCB@/6@4Q JBQ J51Q -BFF1-JQ +@GM1FG

BFQ J51Q BJ51FQ G5BFJQ DK1GJ6B@GQ MF6J1Q OBKFQ +@GM1FGQ 6@Q +@GM1FQ G511JQ +@/Q G5+/1Q J51Q +CCFBP CF6+J1Q ,K,,:1GQ ,1:BMQ OBKFQ +@GM1FG Q &BQ GJ1CGQ +F1Q @11/10Q JBQ 8KGJ63OQ OBKFQ +@GM1FG Q +-5Q DK1GJ6B@Q -+FF61GQ Ŵ ?+F9 Q &BQ -+:-K:+JBFGQ +F1Q +::BM1/ Q

l `ĨCQ

Pv

 pQQ΃Ĩ DQ

3I

0Ĩ DQ 0ĨFQ

B'Ĩ

CQ

M'Ĩ

ĻŴ 0ĨFQ

.AQQŸĨ  ĨFQ

6ä

CQ

1I

F‰QQp lĨ hY “YĨ ĨYĨ ĨĨ Ĩr

0ĨFQ 0Ĩ D



°J'Ĩ

DQ

0ĨCQ 0ĨFQ

ž3ä

FQ

0Ĩ CQ 0Ĩ D

0ĨCQ

$ h “YĨ ĨĨ ĨĨĆĨYĨr B'Ĩ .AQĨ

6ä pQ

J'Ĩ . ‰QĨ>ĨAQĨ

ž4ä  .ĨŴ o0 AQĨ

M'Ĩ  .Ĩ ľ 1 ‰Q

.l hYĨ Ĩ YĨ Ĩ

 ĨĨYĨ

B'Ĩ Ĩ

6ä [

J'Ĩ 7Ĩ

ž3ä L

Ĩ [œ `tĨ NQ tĨĨtĨYĨĨ NQ > P Ŵ

B'Ĩ [LĨ

6ä F

J'Ĩ FĨ

M'Ĩ Ĩ

Pv

Ĩ [$Ĩâć ĨtĨtĨĨ N Q >Ĩ ²Š Ŵ

ž2ä F4 7Ĩ

M'Ĩ aĨĨ YĨĨ

. Ĩ Ĩ JĨ  + F+ J  Ĩ Ĩ  Ĩ )Ĩ Ŵ NQ Ŵ 7NQ Ŵ oĨ  Ĩ Ŵ ŴcĨ 1 ĨĨ)Ĩ Ĩ Ĩ ĨĨ ĨĨ Ĩ)÷Ĩ+Ĩ 1I

B'Ĩ Ĩ

] .

Q

1I

 J .ä .Ĩ ž3ä Ĩ M±Ĩ F

7+ Ĩ Ĩ  Ĩ v)DĨ  Ĩ Ĩ Ĩ Ĩ Ĩ Ĩ  Ĩ Ĩ Ĩ ġĨ ™Ŵ Ĩ  Ĩ Ĩ  Ĩ 4Ĩ bDĨ dĨ .Ĩ bDĨ T <Ĩ T< &$Ĩ 5) Ĩ Ĩ Ĩ Ĩ  –)   Ĩ dĨ<Ĩ Ĩ NQ AĨ ĨĨĨ Ĩ Ý
1I

2I

OQ

B'Ĩ EĨ

 ] .ä LĨ

 J.ä 4Ĩ

ž4ä F

M=Ĩ 7Ĩ

ÚŴ

E6 1Ĩ) ĨƒĨ ĨdƒĨ<Ĩ Ĩ&Ĩ ĨĨĨ ĨČĨĨēĨĨF¼ Ĩ !Ĩ Ē 6 ĨĨ Ĩ)ĨĨvĨ Ĩ Ĩ<Ĩ Ĩ)ĨĨ Ĩ Ĩ  Ĩ!Ĩ!ĨĨ ĨĨ Ĩ)ĨĨ !r B'Ĩ E4Ĩ

 ] .ä E7Ĩ  J .ä ELĨ

ž3ä LĨ M=Ĩ L

L+ #Ĩ Ĩ ľ  Ĩ ĨĨ  HĨ  Ĩ ĢĨ  Ĩ Ĩ) Ĩ ĨDĨ.DĨľZľ ľ  !6 èĨ Ĩ Ĩ § Ĩ Ĩ   Ĩ  Ĩ Ĩ  HĨ óĨ Ĩ )Ĩ )Ĩ Ĩ Ĩ )Ĩ Ĩ  )§Ĩ Ĩ Ĩ Ĩ  Ĩ Ĩ Ĩ  HĨ Ĩ F4 L6Ĩ _Ĩ Ĩ )Ĩ Ĩ Ĩ    Ĩ ĨĨ HĨĨ Wľ vôĨĨ )Ĩ Ĩ yľ

œ/ä oĨ

] .

Q

 J .ä .Ĩ

ž4ä 4Ĩ M=Ĩ FĨ



q% 5 Ĩ gŴ fŴŴ Ĩ Ĩ Ĩ )Ĩ Ĩ ŴŴ Ŵ .Ŵ Ŵ Ĩ Ĩ  Ĩ Ŵ Ĩ 3 $Ĩ Ž Ĩ Ĩ Ĩ "ĨĨ˜ .Ŵ r / O %Ĩ hĨ ĨĨ  ĨĨĨr Ŵ

Ŵ

B'Ĩ "8Ĩ



]=Ĩ "   Ĩ J=Ĩ V 0 " 0 

ž5ä V  0 " 0 V

M=Ĩ "Ĩ0Ĩ3 Ĩ Ä ŽĨ Ŵ  Ĩ Ŵ Ĩ ăĨĨĨĨ Ĩ Ĩ º3 ˜ V Ŵ

Ŵ

Ĩ   % Ŵ Ŵ

c 1 ĨĨĨ Ĩ Yâä /Ŵ PY 6 Ŵ

B=Ĩ 4Ĩ

]=Ĩ FĨ

Ŵ

J=Ĩ 7Ĩ

ž5ä kĦ M=Ĩ L

% 1 ĨĨďĨ 

?7cĨV qq4 cĨ 7Ĩ

?% 1 ĨĨĨĨĨ ľ ĨvĨĨ–!

.% 1 ĨĨĨĨĨ  ĨŴ Ò}iä 4 +Ĩ1 ĨĨĨ ĨĨ Ĩ ĨĨ "‚Ĩ2- ĨÿĨĨ–)  ?"ĨŴ 72Ĩ Ĩ?7%Ĩ

F+ ª«¬ CĨ ĨĨ½ ĨĨĨå CĨ Ĩ újĎę BĨĨCžĨ Ĩ ĨĨ Ĩ Ĩ  Ĩ  Ĩ Ĩ CcĨ Ĩ Ĩ Ĩ CĨ Ĩ 6Ĩ #Ĩ Ĩ Cž  ĨC cĨ ĨĨĨ Ĩ Ĩ Ĩ‘%Ĩ _Ĩ ĨĨFĨbĨ CĨ Ĩ ?ĨbĨ Ĩ ‘ĨĨ"Ĩb ?4 Ĩ v ĨĨ)Ĩ   6 'Ŵ

ÏŴ



7% M Ĩ  cĨ

Ĩ Ĩ Ĩ Ĩ Ŵ Ŵ Ŵ ‘Ŵ  pĨ ? ¡ä  Ì ? ¡ä  ÍĨ ? ¡ä  ŸĨ ? ¡ä

E% _(Ĩ Ĩ

Ĩ î -Ĩ  C<Ĩ *Ĩ Ĩ *U›Ĩ #Ĩ  (*Ĩ -Ĩ ( Ĩ~Ĩ Ĩ  ā(% #Ĩ (Ĩ*(*ĨCĨ ( Ĩ<~Ĩ (*Ĩ &-Ĩ ( ĨĨ(Ĩ*(*Ĩ<~Ĩ ( Ĩ C

(*ĨĨ ^$Ĩ 5 *ĨĨ<& /Ĩ |Ĩb-Ĩ&^Ĩ /Ĩ‹Ĩb-Ĩ ( Ĩ^~Ĩ Ĩ"Ĩb%Ĩ 1 ( Ĩ Ĩ Ĩ"%



L% 1 ( ĨĨ*Ĩ ĨĨ ý (*Ĩ ĨĨ !(  

‹%   ”(Ĩ *  Ĩ.…

 Ĩ (*ĨĨ  Ĩ Ĩ ĨuĨ

 *Ĩ Ĩ ?ZĨ .-Ĩ |-Ĩ E-Ĩ (

  (*Ĩ ĨĨ

 *ĨĨ   %Ĩ B*Ĩ(ĨH-Ĩ *Ĩ *  Ĩ.…

 Ĩ (*

( Ĩ.|?-Ĩ |EE-Ĩ -Ĩ %Ĩ 1 ( ĨĨ*ĤĨ úĨĨĨ

* (Ĩ *  Ĩ.…

 Ĩ (*   Ĩ (Ĩ *Ĩ !% ?% 1 ( Ĩ ĨĨ 

Ĩ N 4Ĩ* (ĨEz%Ĩ * (Ĩ zĨ

Ĩ ? % ŽĨ (ľ Ĩ Ĩ áäŴ ÖäŴ ØäŴ ×oä 1 ( ĨĨĨ Ĩ  Ĩ v , pÊ% (ľ

??% 5 *ĨĨ( Ĩ ĨĨ Ĩ(*Ĩ*Ĩ * (Ĩ,Ĩ* (Œ Ĩ Ĩ ( Ĩ  *Ĩ,Ĩ * Ĩ/Ĩ%Ĩ 1 ( Ĩ ĨöĨ Ĩ ?Ĩ *Ĩ?ĨŴ 4Ĩ *Ĩ? %Ĩ ‹Ĩ

% U Ħ ĦHĦ 

%3, %3, %3,Ħ55Ħ %3,Ħ D Ħ@ Ħ ľ ĦĦ Ħ Ħľ vĦĦ Ħ Ħ  Ħ Ħ  Ħ ĦĦĦ  Ħ 'ĦĦ Ħ 'ĦĦĦ %3,ĦIĦ Ħ ĦĦ ľ,Ĩľ

35 ˜ ĦAĦ Ħ 9Ĩ  ĦĦ   Ħ ;  ĦĦ #Ħ Ŵ A#Ħ,Ĩ9ĨĨ ĦIĦ Ħ ĦĦ 9¿ AY

3ÝËĦĦ Ħ Ħ Ħ  Ħ < 

,5 ;  Ħ#ĦĦŴ  Ħ  Ħ Ħ #$ 3& #z S&Ħ/Ĩ V5Ħ IĦ Ħ   Ħ Ħ ĦĦ#Ħ,Ĩ Ĩ

S5 EĦ Ħ Ħ ' CŒ /ĨT ¥yĦ Ü&ċ C -Ĩ & /ĨC Ĩ Œ< Ĩ xä €'Ħ <&,Ĩ CĨ Ĩ Ĩ €'5Ħ;  Ħ  CĨ ĨªrÜä IĦ Ħ ĦĦ# <Ŵ

ÙŴ

k E nā ª*ä Ĩ

Ĩ ,Ĩ ,Ĩ , Ĩ Ĩ

Ĩ,Ĩ

 Ħ Ħ Ħ  Ħ#Ħ@Ħ Ħ ĦĦ û =Ĩ,Ĩô %&ĦŴ † ,&Ħ,›Ĩ $6Ĩ,Ĩ ……„†ä

7ŘŴ ZŴ Ŵ

!

Si ………+mä

H EĦ Ħ@ Ħ ĦOĦĦĦ¢ĦĦ ¤ĨAŴ īćo©Ŵ ÓŴ ġPŴ Ŵ ķijŬĦn øĖ ľ @ŴAŽŴ çø Ħ  Ħ '  \OĦĦ]sĦ  ĦĦ ĦĖĦ øŴ äQŴ ĢPŴ ėģdmďĬŴ ïBŴ BôŴ ĚuĦ ¤ëĨĨ rO 5ĦE  Ħ ĦĦßà\]rĦĦ,Ħ'ÈĦuĦ ŒęĐŴ łđŴ ĴĕŴ Ă@µùŴ ¾ŊŴ Ģľ ďėxľ ÙuĦ Ħ ĦĦ ªhä :I

ĶŴ



0I

 9 Ħ ā Ħ *ā  ĦĦ  (5Ħ ;  Ħ ā āAā Ħ Ħ (Ħ Ħ  Ħ  "'  Pāā* Bāā+ā IĦ Ħ ĦĦ 



+ EĦwĦ$2 Ħ$Ŵ

£M¥ä

1I



0ā @Ħ Ħ ĦĦ wCĦ$2 CĦÓ+



 >Ħ'"Ħ( ( Ħ(ĦĦ Ħ  (Ħ ªā%ā žĦ Ħ <  ā Ýā

úāāŠêāā

1I

 IĦ Ħ( ' (Ħ Ħ Ħ  ( 







ā ľ EĨ ľ 'ā'ā'ā ľ ā ľ  ā

Ħ Ħ"Ħ ā ›Ħ ĦĦ yĦ  ( Ħ ?Ŵ   Ħ Ħ ĦĦ ( Ħ ' (yĦĦ› ā ā   Ħ Ħ ĦĦ(ĝ Ħ ' (Ħ;  ĦĦ nā ā 

 9 Ħ <ā ’ľ ?Ŵ

)*v

Mv

”ā

„ āā%ā

1I

āā ā







< āā āā ā „%āā %āā %āā % %

(Ħ Ħ ( Ħ ' (Ħ ā%ā IĦ Ħ ĦĦ nā   ;  Ħ 0ā . 0ā 0ā'ā'ā'ā ĦĦ <  ĦĦ ' (Ħ ĦĦ Æ

1I

Çā Ħ

ā ā .āāāā'ā'ā'āā59.  5ā »6ā 1I



(Ħ Ħľ ,-i +ā IĦ Ħ ĦĦ ā ā ā Z臇' ā āā āāāāBāā'ā'ā'āāā¹Ŵ' Ŵā#Ŵ · 6āā¸Zā ā āā  + 9 Ħ$ā  ĦĦ  (Ħ ĦĦĦ(ĦĦ Ħ <  ā $ āāā$ā ā ā Mv

( Ħ Ħ  (ĦIĦ Ħ ĦĦ $ +ā

ā

2;,#@>B)Y #E/)8#E2Y >&2)EHY 4<.$A?C*Y $F0*:$F4'$7Y 7I:A4$(Y yssĊ   & #& $#"&

+ .Ħ ž



Ôā

 ø!

ā Eā  !



!



āï  ā   îŴ ā ĹŴ ā 

Qā ā ù!

ā ā  !

ā ā !



  ¡Ħ !

 +ā

Y

 



Mv

D ĦĦ¡Ħ 0Ĩ *Ŵ 0Ĩ ĺ‹ + .Ħ ž  ā ! āEā ! +ā  ā   ! āā! +ā ! āā āEā ! āā !Èā ā! āā ! ā 0Ĩ  +ā ! +ā 0Ĩ  ā ľ ! āā" ! +ā āā ā  āā ! āEā ! ā ā ľ ! āā! + Mv

Ov

Mv

D  1ĦĦ  Ħ ' ĦĦāā āā ! ā + .Ħ M

I ĦĞĦ ĦĦ ā ā ;)Ħ  ā Ħ Ħ"Ħ 0ā BĦ ĦĦĦ  ĦĦ +ā ;'  "1Ħ Ħ ĦĦċĦ 

ā Ħ ā DĦ ĦĦĦ 

Qā Ħ 'ĦĦĦĦ

Qā ;Ħā ľ ā āā Ħ ĦĦĦ ĦĦĦ"Ħ   Ħ ĦĦāĦ+ā ~ ĦĦ ĦĦĦ

Qā Ħā p1Ħ ā ā D ĦĦ ĦĦĦ āā āā

āā

Qā Ħ Mv

 .Ħ U



Ŵh ā Ŵ

ā ā āÊËľ ŴAā ā ā ā āā Ŵ

Mv



ā ā āā YŴ  āāā K āLā Eā  ā ā ā ā 56ľ ā°Ŵā ā ¹āEā  ā

56ľ

BI



ÀÁľ





āā±Ŵ ā   ā ā Ŵ

ā

.Ħ 

Ħ@ĦB ĦB óBĦBĦĦB ĦBĦ  1Ħ Ħ  ĦB  Ħ < BĦ' Bģ    "Y

 1Ħ Ħ Ħ ā  ā Ŵ  ā Ŵ ā ā l ­Ŵ âãľ  Òā -Ħ ā Ŵ āŴ ā ā  ā `aā  ā Ŵ   ā-Ħāā  ā Ĩ àáľ  Ŵ ā ā dĦ  Ħ āŴ ā8 ā `aāā ā g ā ā `a  Av

³ä

ā `aāŴ āvā



 ŠĦB Ħ "ĦBĦĦB  BĦĦ  ~Ŵ





 .Ħ 

K  ĦBBĦ ;#•–ā ;&#ā  1Ħ & āāā rā # ā#&ā >   ĦĦ R8ßāā ā ľ  ĦĦ;#8&ā ; Ħ8āā©ā&8āā& 8āā  ā ā ā  ŠĦ  ĦĦ ;8āāā ľ  ĦĦ;8&+ā āā ľ  ĦĦ;#8&ā  1Ħ

 ĦĦ8ā#ā ā ĦĦ Rā8ā Ŵ  ĦĦ;8ā#ā āā ľ  ĦĦ;#8&āŴ  ĦĦ #8&ā ā kĦ ľ  ĦĦR#8&Âā

ā

E\

.Ħ U D Ħ ' ĦĦ"Ħ ?Tā ‘ā Tā āTā ā ā  ā  ā āā ā," ā

   "Ħ Ħ Ħ ĦĂ Ħ .Ħ U Ħ .Ħ Ĩ Ħ #ā ĦĦ Ħ Ħ@ Ħ Ħ Ħ 5Ħ EĦĦ 1Ħ  Ħ ' Ħ 

 ' ĦĦā ľ ā ľ Tā ā ā U Ħ.Ħ #ā Ħ Ħ@ Ħ ĦEĦĦ 1Ħ ' ĦĦ

 ' ĦĦ Tā ā ā U Ħ .Ħ #ā  ĦĦ Ħ Ħ @ Ħ ĦĦ Ĩ Ħ  Ħ Ħ Ħ EĦĦ  1  ' ĦĦ"ĦĦā ľ Tā ā" ā D  dĦ Ħ ' ĦĦ"Ħ āā ā ā ā"āā ," ā " .Ħ ]

9 Ħ ľ  Ħ Ħ ' ĦĦ Ħ 'ì Ħ 5ĦD Ħ Ħ Ħā ‡ˆä ľ ‡ˆä 0ā  ‘’ä

ā āā āā ā ľ ľ ľ āā ľā? " ā āā  ľ ā? "ā “”ä ā ā ā ā āfā 0Ĩ  0Ĩ ? "ā •–ä ā ā ā ā 8Ĩ  ā 0Ĩ ? "ā Ħ ?  ā “”ä º ā  ā ā ľ Íā ā ľ  ā ľ ā ??āĦ ā?? āEĦ  ā ?0ā ā ā ā ā ľ ā Ħ āā ā>  Ħ ľ ā ? "āāwā ā

K  ĦĦ

1I



 .Ħ 

K  Ħ ā ā  

*ā ā ā2f āā ā ā*ā ā2ā ā *ā *d

2ā f ā

; Ħ *2āL ā 0ā Ħ ĦĦ ā€ 0ā *āÙ ā Ħ2ā€  ā D 1Ħ ā ā*ā ā2ā8Ĩ0ā ĦĦ ĦĦ *ā *d 2ā ā0Ĩ  āD  Ħ ā ā

ā *ā

ā 2ā

*ā ā*dā ā2 *2ā  ā

*ā *d 2ā ā 8Ĩā$ ā

 .Ħ ]

/Ħ 9 Ħ āā gä  Ħ/Ħā  ā>  Ħ Ħ 



ā ā







ā ā  ~ āā/Ħ ā







M

ā  ā



ā E ā āpā  ā

…Ħ  àF

ā ā ā ā ā ÆÇľ  āāā  āāā ā ā ā ÄÅľ ā ā ā ā ā ÈÉľ

~ ā

Nv

Á Û ¤ ä üHMā ā







; Ħ ā Ħ/  Ħ 1ĦĦ ĦĦ ā8Ĩ ā Ħ`Ħ ā ā

āāÕ gä   ā

=âāäā/Ħ=āā āā8 ā=āā  āāāÕäāHMā ā g



  



ā ā HM ā ā HMā ā ā HMā HMā

 

āH ā Mā H{ ā ā ā ā  ā þ

 NĦ %

  .Ħ

ā   ā

āāH { ßä HM ā ā†āā

 pā ā ā ā

- Äāā ā ľ -  ā ā ā ľ ā ā ā ā

.

ā ľ 

Qā ĤĥĦħľ ā ľ 

; Ħ ā ā ā ľ ā Ħ ā ā Ñā8Ĩā Ħ  Ħ Ħ  Ħ ' Ħ `  Ħ`' Ħ ā "Ħ .

ā 0Ĩ 

Qā Ħ ā

 ā

 lĦ  -āāc)}ääā ācāāā ľ āāƒy

1I

,āā c ā

K  ĦĦx ā ľ Ÿäľ  ā  ,āā ā ľ  ā /ľ ,āā cā ľ ,āā ā ľ  ā  ā /ľ ,āā ƒyāľ  x ā

ÒÓľ

  1Ħ Ħ' Ħ  ĦĐ  đĦĦ -āāÔ CĦĦ

 ”Ħ   āā%ā ä ā °±²ľ ñç ā%

1I

 ā

~ Ħ Ħ Ħ - ā-  ā-,  ' ā

ā -   -, ā-ā ā

 Ħ Ħ Ħ ā  Ħ  NĦ 

— Ħ Ħ    Ħ 'Ħā Ħ)ā' Ħ)ā ĦN ā Ħ âNāā ä7 ā   ĦNā  ĦĦmj Ħ;'  "Ħ Nā  ĦĦ) ā Ħ N K āpĦ ãK—āä)Þ ā ~ Ħ Kā )ā ā ā ā —˜ªŽHhä E  E “”G ä 1I

ā

  Ħ

óā

ā

 ā

ªä

1I

ā ā

 ā ľ

 ā

1I

ā

Å ā

 NĦ  ā

ā ā jā ā ā ā ā Ħ CĦiā ÍĦj ¼ā ÎĦiā ÐĦ ‘¸“Ħāā à ¹Ħāā ČÄÉĚñ÷Ħ āā ‘ºĦ āā jµě“Ħ Žä jġÅÏĦā ľ ā ľ ā ľ ā ľ ā Žä j»Ħ, Žäā v Ħ-† , ā ā

1I

1I

ā

, NĦ 

9 Ħā %ā €'Ħ; ĦR#)/

RF/ ā  Ħ Ħ Fā F % ÐÑľ Ni Fā 7Ħ ā F/ā

3Ħ Wā Þßľ



Kv

?I

Mv

#/ā ; Ħ R#)&ā RF&ā  Ħ Ħ F %Fā 78ľ Mi 7Ħ Fā #&ā F&ā  ā āWā 9:ľ ?I

Mv

%- ā ¬ä fä %ā  ā

>  Ħ Ħ Ħ %ā





Kv

%-āWā Wā  %ā 9:ľ C

3Ħ  ā

Kv

āāWā X ā !"ľ Wā 7Ħ

 ā

" .Ħ 

9 

-  āāB- āā I  ā7Ħ.  . ā'āā'ā'āā. āā

;   Ħ ā ā ĦĦ . āò' ' 'ā .āā6ā 7Ħ- āāB- āā ā ā ā Mv

1I

 .Ħ  

p ' ĦĦ ¶Ħ  Ħ ' Ħ7Ħ ā  ,5Ħ K  ĦĦ ĦĦ Ħ@ Ħā 1Ħā ā,ā  Ħ ,Ħ' ĦĦ Ħ@ Ħ ĦĦ ĦĦĦ Ħ  Ħ  ' Ħ 1Ħ  ' 7Ħ ,Ħ ľ -ā8ä Ħāāā āā,ā ľ  ā ,Ħľ -āā Ħāāā āā,ā ľ ā ā ,Ħľ -āā Ħāāā āā,ā 1I

Mv

 NĦ

ā Ħ7ā

  ā

Áā ,āā 7Ħā āĦ,7āĦ7 āwĦ Ħ7ā ā äĦ7ā$ Ħ"7 Ħ7ā āD Ħ-āJŴ  Ħ"7  ā Ħ7ā

ĦĦ"7ā Ħ7ā

Ħ7ā Ħ7ā kv

AI

1I





> Ħ Ħ Ħ  Ħ Ħ ' lĦ Ħ 4 (äŽ Ħ =Ħ-Ħ4(ä Ħ 4Ħ ľ Ħ =$ 5Ħ =Ħä Ħ ã Ħ

Ħ

'I

 lĦ S3 9 ĦÐä ľ ánä D ĦЁä iĦ D  1

(#= =Ħ-Ħ  = =Ħ-Ħ Ħ-ĦÐä-Ħ(#=Ħ-Ħ $  Ħ Ħ-Ħ = $  Ħ(#= (#=Ħ-Ħ $$ Ħ = 0

=(#

Kv



ľ





Mv

ýŴ

ľ

ċ

Kv

ľ

M lĦ H

D 1Ħ

Ё*‚ä

Kv

‚ä ľ S3

Ħ=Ħ-ĦĦ4Ħ Ĩ ÔÕ Ħ=Ħ Ħ$ Ħ4Ħ Ħ=Ħ-ĦĦ4Ħ ā Ö× Ħ=Ħ 2Ħ4Ħ Mv

Mv

Kv

Kv

Ħ= - ĦĦ=Ħ ¦Ħ ĦØÙ z Ħ4 ( Ħ-Ħ Ħz Ħ4&Ħ ľ  Ö×Ħ4Ħ-ĦĦ$ ĦĦ4Ħ-ĦĦĦ4Ħ ľ Ħ Ħ4Ħ ľ  Cv

ÚÛ

Ħ D dĦĦ=Ħ ľ Ĩ$ xä

Mv



xlä D  1

 ĦĦ =Ħ-Ħ3ĦĦ 4Ħ  2 ĦĦ=& -Ħ3 $ ĦĦ4 (ä Kv

HI

Mv

 =7= -Ħ3Ħ ĦŒ Ħ =7=

 Ħ Ħ2 Ħ HĦ



H

G

% ÒĦ % pĦĦ ĦQĦ%Ħ-Ħ6ĦQĦ,Ħ 6Ħ ĦĦ ĦĦĦHĦ0!ĦĦ Ħ ľ 6Ħ H  ĦĦ0ĦĦ Ħ Ħ0!ĦĦĦ Ħ<  ĦĦ He %6,Ħ 6,, Mv

Mv

Mv

K ĦĦ6,,Ħ % Ħľ 6ĦQĦJĦ ĦĦĦĦ Ħ Ħ% Ħ ZĦĦ Ħ0! x %6,ˆĦ Ħ ĦĦ Ħ ĦĦZĦĦx ˆĦ ĦĦ ĦĦ Ħ0ĦĦx,§  ĦĦZĦĦ YĦ  Ħ ľ QĦľŴ % Ħľ 6Ħ-Ħ Ħ % Mv

Kv

6Y .Ħ , I ĦĦ žĦ

6·ÊĦ0Ħ 

 Ħ#Ħb  Ħ  Ħ  ĦĦ

2=#Ħ

 ĦĦĦ

%bĦ

JĦ

#bĦ 6YĦ ?  !ĦĦ0Ħ< ĦĦòĦõ ĦĦ Ħ!Ħ Ŵ

Kv

Kv

#Ħ-Ħ%Ħ

=6=

Fľ ĵľ

J ³ľ #Ì$ J#ĦŴ  Ħ Ħ HI

# $ 6& #2%&ĦŴ Ħ ÉÊ  Ŵ Ħ$ Ħ©9äB©eä 12ľ

!

6Ħ ?Ħ#ĦĦĦ ĀĦ  Ħ #Ħ  aä 0Ħ#Ħ  2fnYĦ PĦ#Ħ Ħ¨Ħ bĦ ±

Ħ ľ Ħ 6fnĦ $ fnĦ  Ħ2 Ħ -Ħ^MĦ ?Ħ $hĦ PĦ #Ħ 2 Ħ bĦ Ħ  Ħ 6Ħ8ĨĦ Ħ  Ħ Ħ 6fnĦ e Ħ  Ħ Ħ 0Ħ ĦĦ Ħ < ĦĦ Ħ-Ħ^8Ħ ĦĦ ĦĦĦ< Ħ Ħ! Ħ   Ħ Ŵ$ e ^&Ħ2 ĦŴ ^&ĦŴ 6 *ā

$ ^& ĦQĦË'äŴ Ħ Ħ * Ŵ , Kv

Mv

Mv

Mv

Kv

Mv



v

Mv

, NĦ  #$ 6& #Ž S&ĦŴ VĦ12ľ #2 ,ĦQĦ& #$ ,$ &Ħ VĦ 34ľ #e ,&Ħ Ħ VĦ 34ľ #$ ,&ĦŴ Ħ-Ħ VĦ Mv

GI

1I

P Ħ ľŴ ā#2 ,ÂĦĦĦĦ ĦŴ ĦQĦ VĦ¾¿ľ VĦ



Kv



Ħľ$ &Ħ Ħ ľ QĦ&ĦhĦ

1I

K ĦĦĦĦ<  "Ħ Ħ "ĦĦľ T ĦĦ Ħ ľ Œ  Ħ0Ħľā Ħ  ĦĦ! Ħ0ĦĦľ$ Ħ0ĦľāĦ Ħ ĦĦ ĦP Ħ ľ † Ħā WĦ Ħ ľāĦā WĦŴ Ħ

X Wg_ĦācĦ Mv

?Ħ ľ ;Ħ W³_ĦāĦĦĦĦ!

0ĦĦĦ ĦĦ ĦĦ ĦU  "Ħ Wg_Ħ Ŵ ĦUā Ħ Ħ Wg_ĦāĦ‚ā Ħ؁‚ā 5ĦEĦ Uā 8ĦĦ Wg_ĦāĦUā Ħ0ĦĦĦ00ĦĦĦĦ Ħ ĦĦĦ  ā Ħ  Ħ ľ āĦ,Ĩ Ħā %Ħ Ħ

êĦ ā %$ c %ĦŴ u&ä Ĩ HĦÕĨ Ŵ ā %i

EĦ ĦĦ #2 ’&Ħ Ħā CĦāYā ā %ĦÖĨ#Ħ ’ĦÛä% ×Ĩ #Ħā HĦ  Ħ  MĦ Mv

Mv

 ĦĦ  !Ħ Ħ ĦĦ#ĦXŴ Ŵ Ħ HĦ ā%Ħ ĦĦ#Ħā ÁHĦ Mv

S¿ .Ħ % š 0 Ħ)āĦ/ā  ĦĦ)/ā E #& āĦ;#&ā0Ħ;)/ā  Ħ !²! 0  ! iĦ oĦĦ& Eā )ā 0Ħ#& Eā )/ ā  ĦĦ ;#&ā

  Ħ /ā ĦĦ

Mv

>I

;)/ ā



#ā ā ä 0Ħ V/ā)ā ā V) ā ?Ħ V)āāV)ā  Y[\ å)āāV /)ā  Y[\ ā

>Ħ æā/ā āY[\ >ā oĦĦ ;āàā /ā /ĨN Y 0Ħ /ā

Mv

)āā)/āā)ā#& ā ‡

>Ħ  #Ħ\ā

/ā /ā

Mv 

Mv



Ħ

» ØĨ #Ħā %M

  Ħ  ÓĦ 9Ħ &ā ā ā 0Ħ #&ā /Ĩ*ā ?Ħ V&#ā āY[\ā Ħ Ħ  Ħš"! Ħ ĦĦ

%Ħ

?Ħ#& ā)ā 7ĦĦĦĦ)ā ā$ /ĦĦm#&ā m)/ ā ĦĦ =I

AĦ /Ħ ‹R W ÔÕAĦĦ7Ħ/ 2/& AĦ $ /Ħ ! ĦĦĦ  Ħ<  "ĦĦĦ Ħ  / 2/&+Ħ/Ħ Ħ 7Ħ? ÎÏľ /ĦRĦ‹ 3Ħ 3Ħ >8Ħ jv

#&ā MY Ħ 7Ħ ä |ä !"ľ #Ħ7Ħ% ā#ā Ki

Ħ#Ħ 7Ħ



J .Ħ , •"ĦĦ"ĦAC2/CĦR A$ /& AĦ+AĦ /Ħ+Ħ/&8ĦĦ Ħ Ħ A +AĦ /Ħ+/Ħ Ħ

A2/Ħ Ħ AĦ™ä/Ħ ACĦ$ /CĦ

9ĦAĦRĦ #Ħ+Ħ ĦĦ/Ħ7Ħ #Ħ$ hĦ Ħ ā Ui !ä

Ħ Ħý&Ħ+Ħ Ħ¿%ä Ħ¼$ä+Ħ Ħ½#ä ¾ A »ä #Ħ+Ħ &$ #$ &Ħ



Ħ   8Ħ † &Ħ+Ħ< -ā+Ħ 55Ħ+Ħ‡ TTJ&Ħ+Ħ‡ TTT& RĦ Ħ²ä$ Ħ+Ħ´ä$ ²ä+MĦ ¼Ħ+Ħ·¸ä@ µ¶+ ä ĦÀä$ ¹º"

Ħ Ħ ĦĦ R® ľ RĦ¯ 7Ħ,Ħ

Ħ

8=

Dv

Ñ .Ħ  ? čĦĦĦ ĦĦ ^KNGā Ħ Sā Ħ?ĦKGāā Gk0ā ĦĦ  ĦĦ ^NGkā 7 ĦĦ ^KNGāRĦSāĦ

&Ħ

? ĦĦ ĦĦ ’KGlā ĦŴ ĦĦĦ  Ħ ! ĦĦĦ  ĦĦ mkGlā 7Ħ  ĦĦ ^KGlā 7ĦŴ MĦ %Ħ



ÚĦ ĦĦ]OĦāðOĦĦĦ ā



 ĦĦ [OrsĦāxä ā  ĦĦ []OsĦāËSā%ā 

%&Ħ

9ZĦ ĦĦ  ĦĦ [\]OĦāā ā  ĦĦ [\ĦrOĦ ĦĦ

=

S% Sāā %ā



 Ħ

3cĦ

Sā1ā % ā

 Ħ ŏèòŴœŔŴ PQĴľūŴ

0FIiñ

4

=

VŗŴ‚Ŵ

Ŵ ŎŴ

%

=îÿ:

ā ā)MĦ

 ĦĦĦ

Sā%ā āā >ā

 ā

I Ħā3cĦĦ 0ā ĦĦ %āā ā;ĦĦ ĦĦ[\ĦOsĦĦ ā )½Ħ  .Ħ %

9Ħ$ā Ħ3ā ĦĦ ĦĦ ĦD ā āā āĦ$āā3āāā Ħ$3āI ā ā KĦ  ĦĦ $ā Ħ3ā Ħ Ħ ĔĦĦ  Pā*ā Bāāāā ā Ħ $Pā*ā $Bāāā1āā 3Pāā*3Bāāāā āI(ĦĦĦ< ĒĦĦ $3āāk¨`Ŵ ĦĦ $āā*ā3B ā Ħ 3āā*ā$B+ā ā    $B3Bā āā ā$ :āā3Ā˜¢āā$āā3 $3ā āž$āā3:L$3$āā3ā :õā ā%Ħ

  Ħ .Ħ ?)Ħ Ħ± ā ā ĦĦ) ĦĦ  Pāā* āāā Ħ"Ħ Ħ  ĦĦ  Ħ  Ħ2ā ĦX ā•"Ħ) !ĦĦ))ĦĦĦ Õā XÚX 1ðwā ā ā .Q 2ā Ûā 2āā :āÖā āā ā ā ā ×ā āLā ā ā% ā  2āXāÜā ā ā2ā ā Xā ā ā

%ā

 NĦ 

CĦF$ CĦFĦ XùF$ ĦF&ĦXĦF+FĦ Ħ ĦċŴ+ĦĦ–&Ħ £M ä jä ā+ĦwĦFĦĦF&ĦM Kv

/I

oĦĦĦ XF2{&Ħ

Mv









£ä= Êä Ħ 

!"ľ

 –FĦ

ÜÝľ

ĦFĦĦFĦ

Lv

Kv

>ĦĦĦ CĦF$ CĦFĦ

/I

Kv

Éäā

Lv

 qä

… ā



 Ħ 

+  +Ħ 

$

¢>Ê ā

ā+Ħā éÌā ā

…  ā



ā Ëä  

áā Êä ā ā

  

((8Ħ



wä Æ3 CĦ{° CĦ{&Ħ  ā Êä

Mv

Mv

ā  ā ľ xä

Kv



z ”Ħ " ûā+Ħ%Ãāā

ā ā /I

ĤĥĦęĠĦ+ĦĜ+ Ħ Ħ¦§äāÌ2 ä ¦§ä [ ä Ħ¨U ä ā ÙÚÑ= ¨ä ā XĦ èĦ+ĦÓ+ Ŝŝ' þŴ ä ā @ā ľ ā ľ 

`ĦĦ `Ħ(Ħ Kv

/I

Mv

āāāā āāāā ā ā ā ā@ā"ā"ā@ ā z NĦ @

9Ħ XĨ ā ľ .Ĩ ľ ľ ľ ľ ľ  ā PĦĦĦ @ĦĦ((Ħ Ħ ëŴ Ħ X "Ħ  ā K(ĦĦ ā "ā ľ  ā 9Ħ @Ĩ

ā PĦ(Ħ!!ĦĦ@Ħ  ((ĦĦ ìãŴ ĦĦ"Ħ " āK(ĦĦ      ā+Ħā āā  :  :ā Îā :ā+Ħā ľ ā XĦ" ā Lv

Mv

Mv

Jv

Lv

ā

 Ħ Ħ   Ħ ˆā Ħ L:"Ħ ā : Ħ Ħ: Ħ L@Ħ ˆā ľ ā  Ħ "

P Ħ @Ĩ

Mv

ā ľ ā ľ ľ ľ Įľ ľ ā ľ @ā ľ ā ľ

 ĦL Ħā L Ħ:ĦĦ£ :Ħā ľ %Ħ ľ 









ľ ľ ľ ľ ľ

BI

ľ ľ ľ

BI

ľ ľ ľ

  >ā

+ā  8Ħ

ā Ŵā ľ ā ľ  ā ľ ľ ľ ľ ľ ā . ľ %Ħ ľ  Pā  Ħ" ´ %Ħ  Ħ"

? L "8Ħ ĦL Ħ ā L Ħv:ĦĦ£ :Ħ @ā ľ ľ ľ įľ ľ  >ā  8Ħ @ā ľ ā ľ ľ ľ ľ ľ  ā :Pā  Ħ" ² ā  Ħ" > 6I

>:8Ħ ā  Ħ"

@Ĩ ľ ā ľ ā

E:Ħ Ħ 8Ħ @Ĩ

Mv

"ā

 ĦĦĦ: Ħ  āo8ĦĦLĦ X® /Ĩ ā ľ @ Ŵ ā ľ "āāā Mv

āā@ ā

  8ĦĦ L: ĦĦ@ ā  :.Ħ 

9:ĦŴ /Ĩ0ā :Ħ < āāā 

$=ā Êā

$= 

íŸ £āā ā ]āā :āā > ā ā “ā

:Ħ L

L:ú:8Ħ:ĦĦ ā ā

:ĦĦLĦ

< ā ā

ā ā

<ā @ľ <³ ā u ā āāÝä ™ā ā ā  ā  ā  < < u  ]ā šā ā  ā ´ ā 9I

8I

7I

‹ ā ‹ā /ĨāāÞ< ā ā ā  ā

!ĦĦ<  Ħ! ĦĦ!Ħ <ā <ā ā ā

'



Mv

1I

ā āāā'āā'ā āā āāgā  ā āLā ] Œ ā ā 

ā ā @I

 Ħ

-=

 ā ] ā ā  āā… Ħ] ā ā ā ā

<ā ā ā 1I

1I

ā Cā

ā

>)8Ħ ā

Mv

  ā ā

Mv



 4 .Ħ  

K aĦĦĦ ā 'i   5ā h  59Zā

1=

1I

 6āā.āā'ā'ā'āā5sZā  6āāoāā'ā'ā'ā ā 59ý

Kv

 ĦĦaĦ 5 h 59oā 59oā ÂÃľ 5ā Mv

1 

=ÉāLāāā ā 59Z'ā

EĦĦ"ĦĦ))ZĦĦĦaĦ  Ħ ĦaĦĦ ā    8Ħ 

1I

ā|āā|āā ĦaĦ 5ā

 › ā 59Zā ā  ā >ā  ā ā 59 ā  ā

K 

1I

1I

1I

œLā4ā ā 44āā 6 āŒôā ā Lā ā 5   64 

5

 ā

>ĦĦĦ ā ā #Ŵ Ŵ 9 .ā Ŵ  Cā ľ ľ İľ Ŵ ā .

­Oā ā ā :ā ā ā äāOāCāā.āCāāāŽľ ľ ľ ľCā.

ā.

ā Bv

=.

ā Oā=  =  =  =  = ā =  ' =  =  = 3444=.



 ā   t ā 9 OāD ā Oā Ŵ  Oā Ŵ ľ ľ ľ {Ĩ :ā OāD ā ā ā ā    ā 5 > > ā  :  Ŵ {Ĩ āŴ 1ā    ā ā āD .

ā ā ā   ā

 > NĦ JS

9Ħ āā ĦĦĦĦ!  Ħ vĦ‚ĦĦ!Ħ< 5Ħ



C ā 1$rā $ā I Ĩ

ā1ā ā

Ħ !Ħ $ā ĦĦ ā1āāC ā āā ā 78ľ   D ā1āā8Ĩ ā ÌÍľ _ā 8Ĩ §ā Ħ ā8Ĩ ā

?  "ªĦĦĦ  D ā8Ĩ ā™  «ĦĦĦ ā āāCāā     D āā  _āC $ ®  _$āCāā 1ā ā =ľ 1āD ā{Ĩāā  ā1āā =ľ ¿ā

=

=

P Ħ Ħ‚Ħ!  "Ħ Ħ"Ħ ĦĦ ā 34i  ā ?Ħ¢ŸĦ ;Ŵ (ĨEŴ 66;Ŵ (ĿĦ  8ĦĦ ĦĦ

  D ā

1āā 

ā 1āā

 āD ā

1ā  j  ā_ā  ā

Ħ @ ĦĦ ‚Ħ< Ħ  "Ħ ā ā 8Ħ ā 1ā, ā Ħ uĦ ňčőŴ ıĔŴ ĎļŖ(QĠIJĩʼn 0ĦĦĦ!  Ħ 5Ħ >dĦ Ħ ĦĦ ā1ā ā ąĪEŴ ùEľ €ŒŴ åŸėē8Ħ .I

$ā }ā¥āā }ā , ā

ā

2;-#@>B)Y #E/)8#E2Y >&2)EHY   & # # & % &  &  ¸Ŵ   & #&  & $&

? "Ħ  ā Û Ħ ā

s ā

 # #& II1?CIQ +GQ ?+@OQ DK1GI6BAGQ +GQ OBKQ -+@ Q &BQ -+:-K:+IBEGQ +E1Q +::BM1 . Q )5BMQ +::Q I51Q GI1CGQ 6@Q OBKEQ MBE96@4 Q +-5Q DK1GI6B@Q -+EE61GQ 5Ŵ ?+E9G Q

4 9ĦbŴ .Q Ħ*!Ħ )GĦGĦaŴ Ŵ.Q -Ŵ .Q Ħ Ħ*Ħ Ħ !ĦĦ4 qĦĦ .Q *ĦĦ  ĦĦ4 4 9Ħ=4m|ĦĦĦ)") )Ħ<   Ħ ĦĦ! Ħ)ĦĦ Q Ħ4ĦĦĦ& Ħ ĦĦ *Ħ !Ħ &ā   ĦĦ*Ħm|Ħ*)Ħ =|ĦĦ}Ħ4mĦĦ @+

qĦĦ}=Ħ-Ŵ @Ĩ4Ħ }âÜ 1I

%5 Ħ)*) ĦĦ!ĦĦ)ĦGĦ* "ĦĦĦĦ G ā9ĦGĦ)GĦ =4Ħ GĦ !Ħ )*) ĦĦ!ĦĦĦą í  Ħ )*) ĦĦĦ*Ħ ‘%Ĩ qĦGĦGĦ * GāKā *)ĦÞ=æĦ4M 4 é ĦĦĦ !Ħ ĦĦ )! Ħ< *  Ħ*! Ħ ĦĦĦ Ħ

" ĦĦĦ@Ħľ ĦĦ!ĦĦZĦĦ)¤Ħ< ! ĦG¤!Ħ Ħ GĦ*Ħ! ĦĦ 7 ‰ 4ā K  "Ħ ľ )ĦĦ"Ħ Ħ*!Ħ  @ĦGĦ Ħ ľ Ħāā Ħ4ā—*Ħ*Ħā Ħ āĦ* 

4 EĦ*Ħ) *ĦĦĦ  XĨ

Mv

Ï@" @" ā

*Ħ Ħ C2EG7HI1@IQ  Ħ Ħ *Ħ *Ħ  *ÿĦ "Ħ "Ħ Ħ ûć!Ħ Ħ  ĕĢ *!ĦĘĆïĎĦ î "Ħ )ĦĦĦ*ö*Ħ 0ā0ā+ 44āāā ĦĉăĦĊœĦ GĦ*þĦ *üĈĦ &Ħ qĦĦœ*ĦGĦ) Ħ ‰Ħ ĦĦ"ĦĦ ĄĦ ĦGĦĦĦ ØĦ @ā

2;,#@>B)Y #E/)8#E2Y >&2)EHY 3=.$A?C*Y$F0*:$F3'$7Y7I:A3$(Y  Y yssĊ   & #&  & $& $#"&

1Ĩ Ĩ Ĩ .Q Ĩ šĨ Ĩ Ĩ Ĩ ā Ĩ Ĩ Ĩ Ĩ ā,Ĩā Ĩ ā,Ĩ

#)Ĩ Ĩ Ĩ Ĩ Ĩ Ĩ Ĩ Ĩ Ĩ Ĩ +Ĩ #Ĩ ”Ĩ Ĩ Ĩ .Q Ĩ òĨ  ĨĨ Ĩ !Ĩ  ā#)Ĩ .Q Ĩ Ĩ!Ĩ ā Ĩ¨Ĩ Ĩ!Ĩ  ā «”

— 6EGIQ GB:KI6B@Q B)Ĩ Ĩ"Ĩ82 Ĩ  5Ĩ1+ +ĨJĨĨ Ĩ O ĨĨ G@Ĩ Ĩ Ĩ GOĨ WĨGÂĨ J!Ĩ gOGĨ WĨNGOĨ /Ĩ2+Ĩ #Ĩ O& Ĩ Ĩ !\Ĩ BĨ NGOĨWĨ2Ĩ ľ "ĨĨgG&ĨĨĨĨ O ĨĨG  Ĩ&+Ĩ

Ĩ Ĩ Ĩ  Ĩ NO @Ĩ Ĩ gO &Ĩ,ĨN& @Ĩ /ĨNê&Ĩ,ĨN& @Ĩ Ĩ NäÙ&ħ NGOĨ,2 Ĩ Ĩ /Ĩ" 3 2Ĩ,2 Ĩ Ĩ /Ĩ"Ĩ /ĨN @+Ĩ ¢Ĩ @ OĨ Ĩ Ĩ  Ĩ TĚóŴ Ĩ ãĥùÛ+Ĩ #Ĩ G@Ĩ/ĨGOĨ,ĨO@ĨWG,Ĩ@ +Ĩ ăľ

 ä

 ä

 )Ĩ <Ĩ ĨC ĨĨĨĨsĨ  5Ĩ ÞGĵŴ ŤťŴ Ô½ŴēĕĔćľ ¡ĨeĨĨ Ĩ  +Ĩ  Ĩ s&Ĩ Ĩ ¡Ĩ Ĩ <Ĩ  Ĩ eĨ Ĩ C+Ĩ #wĨ ÖŴ ěOŴ CDIJľ ıľ úÅŢšMŐŁČŴ :Ĩ Š¥ Ĩ  Ĩ Ĩ ¥&Ĩ ¨Ĩ Ns +Ĩ Š@GsĨ ¹ Še^sĨ Ĩ ŌFĊŚŨũŪşŴ ŠĸŕăIŴ ¦wĎĨ w Ĩ Ĩ Ĩ Ĩ +Ĩ ¢Ĩ @& /Ĩ^&´Ĩ e& Ĩ G&Ĩ íwðĨ à…ÿæŴ Æâ܆Ŵ 5w“õĨ àeĨ Ĩ Ĩ Ĩ  Ĩ  Ĩ &Ĩ Ĩ g<Ú Ĩ N^Ĩ&Ĩ /ĨNÞáÃĨ öÇŞÄŧŴ ĀÝāxˆØŴ Ĝ+Ŵ G+ĮŅDIņ_Ŵ  Ĩ^Ĩ W^&+Ĩ5ę!Ĩ eĨWĨ&e+Ĩ#ĨĨ Ĩ ĄOĝĞüŴ )1-B@ .Q GB:KI6B@Q

34v

*56E.QGB:KI6B@Q

aĨĨ Ĩ"Ĩ,Ĩ2Ĩ WĨĨ  23 "äĨ", Ĩ Ĩ7ŴŇğMŴ Uŀ



Šľ

_Ĩ Ĩ U) Ĩ  @GĨ Ĩ "Ĩ/ĨNĨ 2ĨĨ g \Ĩ ÷F)Ľ)ēĭľ)Ŵ ¤éŴ ×]Ŧ^ÇŴ ËĦĨ  Ĩ gĐøĨĨ H¥$ "+Ĩ#)ĨgG&Ĩ/Ĩ2V"Ĩ N &@ĨWĨ"3 ř„Ŵ áĒŴ ĉ +Ŵ ōĘŴ DįLLİħŴ ñĀĝ 0I

"ā



 ĨQ  Ĩ!Q Q  Ĩ Q%Q ĨĨ)Ĩ č Ĩ Ĩ !Q % ! %Q



Ĩ 23 "=Ĩ " V 2=Ĩ Ĩ I ¸ ā 2Ĩ Ĩ"Ĩ Ĩ"ĨĨ2Ĩ Ĩ2ĨĨ"Ĩ  Ĩ"Ĩā Ĩ"Ĩ{ Ĩ2Ĩ — Ĩ Ĩ Ĩ"ĨĨ2Ĩ Ĩ"ĨĨ2Ĩ /ÉĨ /Ĩ

`Ĩ Ĩ Ĩ Ĩ *Q Ĩ Ĩ H Ĩ ĨğĨ   Ĩ Q Ĩ 4Ŵ #Ĩ "*'Q / # * Q #)Ĩ ;<' *Q ĨĨ Ĩ =>' *Q  Ĩ W Ĩ +KY 5Ĩ' * Q /Ĩ'(Q Ĩ WY /Ĩ+LY ĨĨ Ĩ -Ĩ '(Q /Ĩ'Q ľ ' ®ƒ

ľ ü`$ľ ľ $ľ /Ĩ$aľ #Ĩÿ<ľ Kľ /Ĩľ Ĩ #ĨĨĔĨ Ĩ ľ ĜĞīľ  đZĨ ĨĨĨ *Q(Q Ĩ #*Q Q

XY4Ŵ

ìĨ Ĩ Ĩ  

5Ĩ £) Ĩ 5 Ĩ 5Ĩ é ) Ĩ  Ĩ +ā ’ )Ĩĉ)Ĩ  Ĩ  Ĩ•Ĩ )Ĩ  Ĩ Ĩ ĖŴ Ĩ 0ā +ā

93

ÐÀ

#Ĩ Ĩ ĨĨ Ĩ Ĩ Ĩ )ÁĨ ß Ĩ !Ĩ  Ĩ Ĩ XĨ    Ĩ ĨĨ ĕĨ XĨ)ĨĨ Ĩ!Ĩ XÏĨ Ĩ Ĩ ²Ĩ+++Ĩ­ Ĩ+++Ĩ Ĩ LMNOľ ûľ

_Ĩ Ĩ ĨĨ ĨĨĨ XĨĨ ĨĠ$Ĩ_Ĩ ĨĨ Ĩ ĨĨ ĨĨ Ĩ$Ĩ #)ĨĨHĨ Ĩ !Ĩ QQ )Ĩ Ĩ Q Ĩ Ĩ$$+Ĩ Ĩ$$$ĨĨĨ Ĩ !ĨX³Ĩ ĨXĨ Ĩ Ĩ+Ĩ

ā

'."305 ; 7$ ,7'); 0' 79; Udm`Ypoq\}RYtb\kYtdZYi}SiwkpeY[} URS} 2,,8} Sp\m} U\Ztdom} Tovm[} /} }+a+a :Ħ Ħ¬ Ħ %Ħ

g G Ħ

PkpoqtYmt@}

EKO7JS ''S : IM7KLMKL<8QSQGMJS3EKO7JKS #35;S IM7KL
1Ħ r!Ħ = Ħ7=++Ħ+Ħ ++Ħ /+!++Ħ Ħ=aĦ  `Ħ "7Ħ+Ħa77ĦĦ+uĦĦA¦Ħ 1Ħ ¨7"+Ħ Ħ 7: aĦ IS +Ħ 7++Ħ Ħ 7ēĦ +Ħ +u7Ħ PS HS4IS4 )<%Ħ Ħ ˜=aĦ+Ħ"+Ħ\ĦHS : I S

HS :CCPS :



1Ħ

: \ 7ĦjĦ7Ħ ĦĦ

8S$:



$: !++Ħ

ĦPSQS !: ': H7aĦ  j %Ħ4 8. S

8PQS :49 8P 8Q 8Q  PS4 Ħ\Ħ

S 2++Ħ++Ħ  S. =aĦS  ĦĦ+Ħ\Ħ7ĦĦ/œ+/Ħ\Ħ=+ĦĦ=+1Ħ H7=aĦ +Ħ/+Ħ\Ħ ! Ħ\Ħ7Ħ+Ħ /Ħ7Ħ!7ĦS aĦ+ 7ĦS 7Ħ Ħ+Ħ Ħ 1Ħ HĦ+e+:Ħ +Ħ7/+Ħ+u+ +Ħ\Ħ7Ħ7Ħ

S



S



S



S







)oĦ 2++Ħ +Ħ  ÐĦ +7Ħ 2 29 1 1 -9 ? Ħ\Ħ?S :ĦĦo o 1 ĦĦg¦Ħ



S



S



S



S



S



S



S S

KS \Ħ :Ħ:Ħ1 1 1 ĦĦg1Ħ r!Ħ  Ħ \Ħ +Ħ 7\ Ħ



%1Ħ

S Ħ7Ħ"+7 +Ħ 7=Ħ=ĦĦ 7 #+Ħ : \Ħa7ĦĦ !Ħ (S 9 % ;oĦ : 7 +Ħ !7Ħ ++Ħ •Ħ7Ħ7 7/+aĦ 7Ħ  S 2+Ħ7+Ħ&S ++Ħ 7 +Ħ : 7ĦĦ : 7=+Ħ

4Ħ H7aĦ+Ħ+Ħ\Ħ +Ħ++Ħ•Ħ4Ħ

 Ħ

<1Ħ H Ħ Ħ(3ĦĦ  ("ĦyžĦĦ Ħ ĦĦ Ħ3Ħ ĦĦĦ %†Ħ AĦ HĦ ĦG Ħ™ĦĦ*ĦĦĦ(3Ħ#Ħz*Ħ3Ħ Ħ(ĦĦ Ħ#Ħ Ħ*@Ħ2 ĦĦe  Ħ%ĦG#Ħ(3Ħ*ĦĦ04@@@ĦĦ fĦ ! Ħz*Ħ %@ĦH Ħ  Ħ(3Ħ ĦG#Ħ(3ĦĦ m Ħ@ Ħ ĠĦ ! Ħ z*Ħ 9 %@Ħ 







g1Ħ D(ĦjĦ ĦĦ( Ħ Ħ #&: # & 9 #&4#& ĦĦ 89 H} MĦ ’Ħ Î@Ħ  @Ħ—Ħ*Ħ.Ħ'ĦĦĦĦ  #Ħ  ĦĦ: ! Ħ (Ħ<MĦ 2 Ħ  (#Ħ3 Ħ Ħ*.ĦĦ  ĦĦ.'ĦĦQĦ  Ħ  Ħ eĦ Ħ*'Ħ Ħ …@Ħ’Ħ Ħ "#(ĦĦJQĦ J…MĦ 

&:

4 [Ħ  Ħ #Ħ*.'²Ħ *.Ħ *'Ħ 9 Ħ 4Ħ  ĦCĦ Ħ  Ħ  Ħ Ħ*.Ħ  Ħ *'Ħ  "# ^Ħ IĦ QĦ 3Ħ Ħ ĦĦ4CĦ  ĦĦ  Ħ eĦ Ħ.QĦ  Ħ'QĦ Ħ  Ħ Ħ*'Ħ Ħ*.ĦĦ #:  Ħ ":  " @ĦH Ħ Ħ"(ĦĦ  &4  & 

9

Ħ

 MĦ [Ħ  Ħ #Ħ *.'Ħ *.Ħ$ĦĦ.'Ħ$Ħ%Ħ*'Ħ$Ħ %Ħ TĦ Ħ  Ħ  Ħ Ħ .'Ħ  Ħ4Ħ Ħ  Ħ Ħ Ħ.'Ħ( Ħ  Ħ*4Ħ 3 Ħ «.*'Ħ IĦQĦ 3Ħ ĦĦĦ Ħ   (ĦĦ.ĦĦ*4Ħ ’Ħ Ħ  ĦĦQ TMĦ 9

 :

[Ħ Ê sĦ Ħ *.'ŸĦ ?*ĦBĦ ; Ħ ĉ'Ħ$Ħš ĘĦ : Ħ Ħ Ħ Ħ *.Ħ  Ħ 4Ħ Ħ  Ħ 7 ÊĦĦJ.  Ħ D(Ħ Ħ  #Ħ  ĦĦ : ! Ħ (ĦJ4Ħ( Ħ Ħ Ħ  : %59 * : ’sÉssĦ  Ħ ğĦ Ħ «.'Ħ4ĦĦ ^Ħ '9

MĦ [Ħ ö*.':Ħ 4Ħ  ĦCĦ Ħ Ħ  Ħ Ħ  Ħ  Ħ *.Ħ  Ħ*'Ħ  "# Ħ '4Ħ  Ħ .CĦ  Ħ ĦQĦ ! Ħ «.E'ĦX š ęĦ D(Ħ¿4$ĦA šĦ Ħ 'CĦ$Ħ šA^Ħ H Ħ.'Ħ 9

9

RMĦ IĦ+:$Ħm Ħ ĦĦ   ĦĦ
<1Ħ H Ħ Ħ (3Ħ Ħ#Ħ#(ĦĦ ĦŠ(Ħ -Ħ4

79

Ħ Ħ $Ħ %Ħ} @Ħ %Ħ 9-Ħ

AĦ H Ħ Ħ #ĦĦ  ( Ħ  Ħ  ĦĦ "ĦĦ5`ĦL 5 ĦĦ#ĦĦ $ 9 S1Ħ ¨"Ħ !Ħ Ħ *BĦ mĦ ĦĦ^  ^ ŸĦĦRfĦ  Ħ.Ħ$Ħ0 ĦfĦ ™ Ħ  Ħ (3Ħ Ħ Ħ 8S 9 *Ħ .Ħ! Ħ Ħ3Ħ  ĦĦ ĦĦĦ !Ħ#ĦĦ*^Ħ 

@ĦIĦ5_ Ħ5-ŸĦ@@@Ħ3 ĦĦŠ( Ħ Ħ Ħ  Ħ Ħ5ðĦOĦĦ Ħ5Y$Ħ 5YĐ_6Ħ Ħ##Ħ  @ĦH Ħ Ħ(Ħ  ( Ħ Ħ5YĦ 9 ‡¶¶1Ħ @ĦIĦ QĦ 3Ħ Ħ  G  Ħ # Ħ  3 Ħ Ħ Ħ  #MĦ H Ħ  Ħ (3Ħ Ħ #Ħ ! Ħ" ĦĦ Ħ " ĦĦQĦ( Ħ Ħ Ħ !Ħ" ĦĦ Ħ#Ħ Ħ  Ħ 3 Ħ Ħ #Ħ  Ħ  Ħ " ĦĦQ@Ħ 1Ħ 8Ħ Ħ Ħ  Ħ Ħ Ħ Ħ Ħ Ħ Ħ  Ħ "3Ħ 3 Ħ Ħ 3(Ħ Ħ "3Ħ 3 Ħ Ħ Ħ "3Ħ 3 Ħ  1Ħ r Ħ  Ħ Ħ Ħ Ħ Ħ  ĦĦ#Ħ Ħ ! Ħ  Ħ Ħ %Ħ  Ħ  Ħ Ħ Ħ @Ħ H Ħ  Ħ (3Ħ Ħ #Ħ Ħ 3!Ħ Ħ  Ħ Ħ #("1Ħ @Ħ2 Ħ 3 Ħ Ħ Ħ 83Ħ ƒĦ ĦĦ¬ ĦA<S@Ħ ”Ħ !Ħ Ħ ¬  Ħ 3 Ħ Ħ č  Ħ 3 Ħ Ħ }   Ħ 3 Ħ Ħ 2 (  Ħ 3 Ħ Ħ H  Ħ 3 Ħ RĦ D(  Ħ 3 Ħ %Ħ  Ħ D(  Ħ 3 Ħ <Ħ !  Ħ  Ħ Ħ  Ħ !œĦ !Ħ 8#3Ħ ƒĦ 3¦Ħ ¨"Ħ (Ħ !Ħ ĦĦĦĦ ĦĦ <@Ħ 1Ħ H Ħ Ħ(3ĦĦ
ƒ"#(Ħ (:   Ħ 69 AĦ 69 Ħ 69 Ħ 69 <} ! Ħ  $ @Ħ Ħ

ôĦ

'."305 ; 7$ ,7'*; 0' 79; &-!2/4; 6#+6&(; (8+2&;  ; 3--9} Sp\n}U\Ztdon} Tovn[} /}Uoivtdonr}

/)} 8WĦg)Ħ ƒ ĦĦ Ħ3Ħ!ĦĦ # # ###& ! Ħ Ħ "&$ Ħ :Ħ:Ħ :Ħ1 1 1 Ħ:ĦgĦ! Ħ# &>Ħ# &>Ħ &>Ħ&>Ħ#& OĦ A1Ħ 2 Ħ(3Ħ ĦG"ĦĦ (Ħ Ħ Ħ3"ĦŠ(Ħ Ħ g)1Ħ DĦ Ħ Ħ g)Ħ( Ħ@Ħ

2(} 8qĦ Ħ PĦ>ĦdĦ



P- > Ħ )P > d > )<%Ħ )P > dĦ Ħ Ħ PdĦOĦ $Ħ >ĦĦ%Ħ1Ħ gĦ %Ħ Ħ

1Ħ DĦ P9 >Ħ ĦĦ Ħ(Ħ Ħ ­Ħ 6: >ĦdĦ 9 Ħ ĦĦ Ħ(Ħ Ħ MĦ 8Ħ PĦ>ĦdĦOĦ Ħ>Ħ g1Ħ D P O Ħ):Ħ:Ħ:Ħo ^ o1Ħ [ĦPĦ$Ħ):Ħ PĦ>ĦdĦOĦ Ħ ĦPdĦ 1Ħ [ĦPĦOĦ :Ħ PĦ>ĦdĦOĦ :Ħ PdĦ g:Ħ dĦOĦ  : 2 (Ħ PĦ>ĦdĦOĦ oĦ 



4(} 8£Ħ < Ħ Ħ #!$ $ #&>Ħ :Ħ Ħ!Ħ "Ħ !$#&>ĦĦOĦ $#  #  $&: 1Ħ #$&0ĦĦ$ ­(3 Ħ !Ħ "Ħ ĦOĦ !#  $& >Ħ$ #& Ħ t9Ħ>Ħ$& OĦ tZĦ>Ħ# & IĦ $& OĦ oĦ 2 Ħ t9ĦOĦ jyĦ>Ħ91Ħ D(3(ĦĦ Ħ"Ħ  Ħ Ħ(Ħ #& $&$Ħ :Ħ !ĦĦ 

tyĦ>ĦĦOĦ tytyĦ6 tyĦ>Ħ :Ħ Ħ tyĦ6 -Ħ 2 (Ħ tyĦ



Ħ Ħ  t %Ħ>Ħ&







< 1Ħ

5%} 8WĦ%A Ħ IĦ $.9 Ħ  Ħ (3Ħ Ħ ! Ħ  Ħ  Ħ fsa} Ħ  Ħ *^Ħ }Ħ "Ħ  & 1Ħ 8Ħ  Ħ Ħ  Ħ Ħ  Ħ  Ħ 3Ħ Ħ Ħ ! Ħ ĦĦ  Ħ 3Ħ @ĦDĦĦ(3Ħ Ħ! Ħ Ħ 3Ħ Ħ 3Ħ ĦĦ Ħfsa} ĦĦ ~Ħ 2 Ħ (3ĦĦ! Ħ ĦĦ Ħ f}>Ħ gsa} Ħ *Ħ  "Ħ  Ħ 3ĦĦ đ~Ħ>Ħ 1Ħ DĦ $.19 ~Ħ $.9 2 (Ħ $ 9$Ħ :Ħ $9$Ħ :Ħ $9 :Ħ

&&  & OĦ %A 1Ħ 





7+} 8qĦ Ħ §( Ħ  Ħ(Ħ Ħ  Ħ ^Ħ 2 Ħ ĦĦ  Ħ  Ħ Ħ 29 239 1 1 ^ ĦĦ z-Ħ  Ħ Ħ  ĦĦ zñĦ  Ħ Ħ  ĦĦ 2 9 DĦ Ħ!ĦĦ 1÷Ħ OĦ  1Ħ Ħ

8%} 8WĦĦ *4Ħ/ ĦĦĦ?*Ħ ĦÀ'Ħ/ ĦĦĦ?'Ħ „!Ħ?.'4ĦX?¿*4BĦ ?*| 1Ħ ?*4'ĦB ?.1Ħ r Ħ?Ï'4ĦBĦ?*0?'| Ħ Ħ?4À'ĦXĦA ‹6?.6?* 0?'| Ħ$Ħ ?* 0Ħ?'| Ħ 2Ħ •4Ħ$Ħ'41Ħ 2Ħ  Ħ '4Ħ / Ħ Ħ Ħ &Ħ  ‹Ħ Ħ Ħ *Ħ &Ħ  Ħ : r Ħ Ħ / Ħ Ħ Ħ &Ħ % ‹Ħ Ħ Ħ Ħ Ħ  #Ħ : 2Ħ •4ĦX'4BĦ % ‹Ħ· Ħ$Ħ Ħ

:&} 8qĦ R Ħ ”&Ħ1;Ħ Ħ:Ħ ĦĦĦ!Ħ&Ħ1;Ħ " Ħ %†Ħ Ħ jĦ1;XĦė %Ħ|/S Ħ ©Ħ 0ĦÇ %Ħ|1Ħ ;©Ħ 0ĦÇ %Ħ·1 Ħ ;©Ħ 0Ħ1   Ħ „Ħ Ħ Ħ Ħ Ħ Ħ Ħ Ħ " Ħ Ħ 2Ħ /Ħ &Ħ  "Ħ yžĦ Ħ Ħ  Ħ Ħ Ħ /Ħ ĦĦ Ħ %†Ħ Ħ Ħ ®Ħ !Ħ &Ħ ĦĦ " Ħ %† nĦ ! ĦĦmj ĦjRfĦ$ĦjRĦ$Ħ Ħ0ĦA Ħ0Ħ%Ħ0ĦXĦ Ħ R 1Ħ

<&} 8WĦARRĦ 2ĦĦm5ĦÅĦÆf äåæĦ m 65Ħ 6ÅĦ 6ÆfĦĦĦ/Ĕ Ħ&ĦĦĦ&ĦG#Ħ/Ħ Ħ mĦ M M Ħ²Ħ fĦ !Ħ z*Ħ 9 %Ħ Ħ Ħ Ħ Ħ GĦ /Ħ &Ħ mĦ1 1 1 ĦĦ fĦ !Ħ z*Ħ 9 %1Ħ 2Ħ /Ħ Ħ ÝĦ/Ħ Ħ mnĦ   ĦĦ fĦ Ħ -;X Ħ <<Ħ 2Ħ  Ħ /ĦĦ GĦ/ĦĦ mĦ   ĦĦ fĦ !Ħ z*Ħ 9 %ĦĦ :

:

1} Ď<<Ħ6 /Ħ&Ħ Þ#Ħ/ĦĦ mĦ   ĦĦ fĦ !òĦz*ĦXĦ%ďĦ$ĦARRĦ

?%} 8¼Ħ  Ħ „ĦĦ9¡Ħ 0Ħ9v¡ĦXĦ9-Ħ0Ħ9ß-9ˆ6Ħ0Ħ9àõĦBĦ9Ħ0Ħ9v‡Ħ-6 9-Ħ0Ħ9v--L Ħ$Ħ 9Ħ0Ħ9v‡- 6 9Ħ0Ħ9v‡- 6 -á Ħ$Ħj9Ħ0Ħ9v‡Ħ 2Ħ Ħ ĦBĦ 9Ħ09vĀĦ !Ħ "ĦjĦXĦ-6 -L -â Ħ 2Ħ jĦBĦ-ã -6 -6 ĦBĦ Ħ

/,&} 8qĦ

:

IĦÃĦ/ĦĦ&ĦĦĦ  ĦĦ : Ħ.'Ħ D Ħ ?.*'ĦBĦ ?'JÃĦ !Ħ"Ħ ?*…JĦ$Ħ?J'Q1Ħ 2Ħ»þJQĦĦĦĦ»…J'1Ħ 2&ĦJ'|J…ĦXĦJQ|J'ĦĦ  Ħ 5:QĦĦ J…Ħ$J Ħ '-Ħ$Ħ<-Ħ$Ħ :





//)} 8WĦĦ IĦ4EĦ$9 Ħ ĦEĦCĦBĦZĦ Ħ.'Ħ$5Ħ 8ĦwxĦ4EĦĦ#ĦĦwxĦ.'nĦ!Ħ "Ħ4xĦNĦ.xĦ$Ħ 9›5Ħ 2 (Ħ .x6N NĦ.xĦ$9 Ħ ›5oĦ D#"ĦĦ.x:Ħ !Ħ "Ħ NĦ.xĦ$Ħ 56 9›5Ħ D Ħ Ħ  Ħ  Ħ  Ħ w“ECĦ Ħ Ħ Ħ w“.':Ħ !Ħ 3Ħ N'“Ħ$Ħ 5L Z›5Ħ §Š( Ħ Ħ Ħ BĦ5L 9Ħ }ZN5ĦBĦ5L 5N5BĦĦ } .xĦ '“Ħ

/2)} 8WĦ%Ħ ƒe Ħ.EĦ Ħ*'ĦĦCĦ 2 Ħ *.CĦ Ħ Ħ #Ħ Ħ ! Ħ *.Ħ$Ħ*CĦ  Ħ .EĦ$ĦEC Ħ Ħ 8ĦEĦ  Ħ TĦĦ  Ħ  Ħ Ħ .CĦ  Ħ .'Ħ  "# Ħ !Ħ "ĦE T Ħ Ħ##ĦĦC'Ħ ĦE T$ĦC'N ĦBĦ %Ħ6 N ĦBĦ%Ħ

/4)} 8WĦ)Ħ ăĦ4iĦ Ħ4'Ħ IĦ TĦ3Ħ ĦĦĦ Ħ  (#ĦĦ4ĦĦ*'Ħ 2 ĦJi:Ħ 4 T Ħ Ħ.'ĦĦĦ###@ĦD Ħ4 Ħ Ħ ĦĦJ.Ħ TĦĦ Ħ ĦĦi'Ħ 2 (Ħ4i'ĦĦĦ #ĦĦ ! Ħ4iĦ$4'Ħ Ħ ?i4 TĦ$ĦĆ T4'1Ħ 8Ħ ?iJ*Ħ$Ħ< ;Ħ  Ħ wîi4Ħ Ħ Ħ !Ħ "Ħ ?Ji4ĦBĦ )ġĦ 2 (Ħ ?.'4Ħ$Ħ ? T4'ĦBĦ ?i4 TĦ$Ħ?Ji4ĦBĦ)‹ pĦ

/5)} 8qĦ %Ħ D Ħ4Ħ  ĦCĦ Ħ Ħ  ĦĦ Ħ Ħ*.Ħ  Ħ*'Ħ " Ħ !Ħ "Ħ4CĦ Ħ ###Ħ Ħ .'Ħ  Ħ wE4CĦ Ħ Ħ Ħ wE'.Ħ ! Ħ E4ĦWĦE'$ĦECĦ qĦ E.Ħ$ 4CĦWĦ '.Ħ$Ħ W pĦ IĦECĦ$Ħ9Ħ ĦE4ĦBĦZĦ 2 ĦE.ĦBĦ 9Ħ  ĦE'$Ħ ZĦ ü Ħ—  žĦ Ħ  Ħ Ħ wE.4Ħ  Ħ wE'CĦ !Ħ Ħ  9-Ħ } Z-ĦBĦ A S-Ħ  Ħ  Z-Ħ } 9-ĦBĦ  SA-pĦ 8 Ħ  Ħ !Ħ Š(Ħ !Ħ "Ħ 9-Ħ4 Z-ĦBĦ A S-Ħ }  SA-N)Ħ 2 (Ħ .'-Ħ $Ħ %Ħ  9-Ħ} Z-ĦBĦ A S-Ħ} SA-N)Ħ$Ħ  %Ħ 2 Ħ.'ĦBĦ

/7)} 8WĦ%))%Ħ 2 Ħ!ĦĦ Œ <ŽĦ Œ <ŽĦ4 Œ <ŽĦ ĦĦ^Ħ Œ<ŽĦBĦ _¡Ħ$%Ħ ))%Ħ Ħ } Ħ )Ħ } } <Ħ

%Ħ

/8&} 8¢Ħ <)Ħ  U `);Ħ0Ħ`);VĦ U Ħ);Ħ 0Ħ );VĦ $Ħ U ˆ); L ¹Ħ );ĦĦ);Ħ0Ħ -);Ħ-); L Ħ);Ħ¹);Ħ0Ħˆ);VĦ hĦ  U ˆ);Ħ0ĦˆĦ); 6 Ħ );ĦĦ);Ħ0Ħ -Ħ );Ħ-Ħ);VĦ 9  UĦUĦ -Ħ);Ħ >Ħ -Ħ);V -Ħ6 -Ħ);Ħ-Ħ);Ħ6 Ħ);ĦĦ );VĦ ĥ Ĥ $Ħ UĦL U Ħ ;V-Ħ6 U Ħ ;VVĦ Ħ Ħ $Ħ U6N%6NVĦ $Ħ < )Ħ

/:&} 8¢Ħ Ħ § Ħ ĦuĦZĦ0ĦNZĦX5Ħ ! Ħ5¾Ħ Ħ [Ħ Ħ/Ħ  ĦĦ Z-65ZĦ0ĦĦ 9 Ħ Â/"Ħ ĦĦ Ħ!Ħ"ĦĦqĦ ZĦ 9

5ĢĦ Ħ

Ħ ¾Ħ ĦĦ

2 Ħ ĦuĦ 9-Ħ>Ħ WX} hĦ %Ħ0Ħ %Ħ 9-Ħ

ĦĦĦĦU  : ĦÌĄºĦÌNąºVĦ

/<&} 8¢Ħ  Ħ „Ħ  Ħ 5`65$Ħ5UĦ56VU 5Ħ0ĦVU 5-Ħ0ĦVĦ ”ĦĦ Ħ Ħ Ħ $S5`65Ħ Ħ Ħ $S5`65Ħ }Ħ Ħ !Ħ Ħ )Ħ $S5`L 5Ħ/ Ħ  Ħ Ħ ˜"Ħ qĦ 5ûĦc} U  Ħ )VĦ c}$Ħ ĦĦ ĦĦMĦ 2 Ħ Ħ N}5`³ 5Ħ } Ħ5ĦhĦ Ħ !Ħ "Ħ5`Ħ6 5ĦXĦ Ħ 2 Ħ Ħ eĦ  Ħ MĦ

/?'} 8WĦ < ) Ħ 2 Ħ Ħ _`Ħ Ħ Ħ I} Ħ .@Ħ 2 Ħ Ħ  ĦĦ Ħ _} qĦ I} .Ħ ! Ħ _ c} 9 Ħ Ħ #Ħ c} !: IB}  Ħ  Ħ Ħ)Ħ Ħ _} qĦ I} .Ħ ! Ħ _ c}$Ħ Ħ Ħ ##Ħ c} !: IVxfy}  Ħ _ f} Ħ ‰XĦ Ħ   Ħ)Ħ 2 Ħ Ħ!ĦĦ  <) MĦ 





<Ħ

2,&} 8WĦ Ħ ĊĦĦ&Ħ5Y$Ħ 5Y×ĕĦ Ø Ħ ]Ħ"Ħ





5ĦYĦ  {}$Ħ Ħ 5ĦYĦ´_Ħ  | SĦ SĦ &Ħ#Ħ   Ħ 2Ħ 5YĦ 

 

Ħ  ´ AĦ $Ħ ĦYv_Ħ 5ïĦ µ >} $Ħ ĦY _Ħ

=}

=}

&ĦĦ  øùĦú Ħ 2&Ħ

:/"ĦĦ&Ħ  CDE} Ħ

2Ħ

5_‘_Ħ 9  _‘‘Ħ 9 5_‘-ÍĦ

ċĦĦ ] Ħ Ħ]ĦĦ  pĦ

2/%} 8¼Ħ {S Ħ ÁsěĦ *Ħ /ĦĦ"eĦ&Ħ Q^Ħ HĦ ]Ħ Ħ ĦĦ/Ħ&Ħ ĦĦ"Ħ *ĦĦ #9 ÓģeĦ 8&ĦœĦ] Ħ*Ħ ĦĦ  "Ħ" Ħ&ĦQĦĦ Ħ Ħ&Ħ*Ħ ]ĦĦ s&ÈĦ ]Ħ Ħ" Ħ&Ħ] Ħ]Ħ Ħ Ħ]Ħ" ĦĦ ĦĦ] ĦĦ Ħ ĞÉĦ*ÔĦ Ħ  Ħ #Ħ Ħ/Ħ & pĦ 2ĦĦ      ;$Ħ : ; ] ĦĦ ĦĦ  rs ĦĦĦ  Ħ : ;  ĦhĦ S Ħ Ħ #çĦ



22*} 8£Ħ A%Ħ ÁsÈĦ ): x}79 !: .: N} Ħ A} 79 A}  z Ħ *Ħ hĦ x}79 !: (: N} 79 Ħ "/Ħ / Ħ  z Ħ .Ħ$Ħx}79 !: (: N} 9 ĚĦ ®"/Ħ / Ħ  z}S$Ħx}79 !: (: $S 79 Ħ "/Ħ/ Ħ  z Ħ ČĦu Ħ]ĦĦ 

Ħ Ħ  Ħ Ħ  Ħ Ħ $ĦA%pĦ µ } ³  6 6    Ħ  Ħ } Ħ Ħ }  Ħ  Ħ }

Jh N K N}}% $ $Ħ S

24%} 8WĦ)Ħ ÂĦ& Ħ  Ħ Ħ   Ħ G)G %Ħ }   pĦ }Ħ #Ħ ™Ħ Ħ Ħ /Ħ &Ħ  Ħ4Ħ&Ħ )G)GA<SĦĦ {G)G %pĦ 2Ħ/Ħ&Ħ#Ħ Ħ/]Ħ A<SĦ Ħ )Ħ Ħ

  )Ħ

 

}   AĦ

hĦ Ħ

:73;L   00? C7L ' $ $*?D.$:$L :$L $* 0.1? J%:?L .F@L 





%+? J$:>L 7*L I.0!.L '? :%L 1$8L

1*>'+*' >2(++'+*'$0 )' %0'''?

04!&L )+),)? 1$H$?L L :&204#%:L 7+L *?I.$4L #0H0#$#L



JL .? 1 $:CL 04>C$04L I>L

)+)+)? 7:4L 74L

:0#J L .$L 4>I$:L 0>L 

  4>L $1..+? 'CL 37? #$47C$L C.$L 4F<5 (:L 7*L AG!.L 6#0,0CL 04C$,$:> L 374,L C.$>$L 47? 04C$,$:@L 1$CL 57? #$47C$L C.$L 4F<5 $:L 7*LC.7?%LI.0".L$4#LI0C.L$? JL >J22$C:JL C.$L 4F=4 $:L 7*LC.7>$LI.0!.L



$4#L I0C.L '? 7:L )? 0?L 1?7L $9F1L E7L 57? $4!$L %37?

?

'57?

?





%4#L I0C.L ? 7:L 

$4#L I0C.L %'? 7:L 

%47&?

%-7?

?

%?





$4#L I0C.L '%? 7:L )%?

)4#L I0C.L $? 7:L $? /FAL 47? $48 ?



$L .H$L 3?



)? 3&?



)? >?* 



'57?



+47&??+576?



+47&??$47  $47&? !



)47? ?%49&?

.? JL 0C$:C04,L I$L ,&CL 3/?





$1..+?

  4@L $.)? F@04-KL C.$L *!CL C.CL

 



      ' ?



=? ? 

L

?

':? ?

? 



?

' 6L ? 6







6L  ?

?



 "#!  L I$L .H$L 

     



' 6L ?

' 6L ?



6L

' 6L 

) 6L ?





?

  ?

';?  

?

6L

.FBL I.%4L   %?

 $'?  '0?  )?  )'?  ).?

)1?



%.)?

?



?

?

?



'


'."305 ; 7$ ,7'); 0' 79; Uem`Ypoq]}RYtb]kYteZYj}SiwkpeY[} URS} 2,,8} Sp]m} U]Zteom} Up]ZeYj} Tovm[} D  Ħ ĦĂ Ħ %Ħ

S Ù  Ħ

PkpoqtYmt@}

LL7DHLS3KSD3EQS IM7KL


:

­!Ħ Ħ  Ħ Ħ &Ħ Ħ Ħ Ħ Ħ &Ħ  Ħ   #Ħ &Ħ /Ħ }  ĦĦ"ĦĦĦ&!Ħ !Ħ&ĦĦ ĦĦ %   )S ĦAĦĦ" " " y} seĦĦ"ĦĦĦĦWĦ Ħ G}









Ħ Ħ Ħ Ħ Ħ Ħ 0} : 6} : ;} : u} : Ħ : Ħ :  Ħ : %

èĦ ý Ħ Ħ u ĦH H S Ħ &Ħ Ħ  Ħ Ħ &Ħ  Ħ<S  nĦ Ħ/=S %02:6ĦĦ/=S 9 / =BS4Ħ 8Ħe#ĦĦĦu Ħ Ħ)ĦĦ Ħ<Ħ D!Ħ Ħ Ä Ħ Ħu Ħ ĦĦ˜Ħ/Ħ&ĦpĦ 



:

IsĜĦ ,Ħ/ĦĦ"ĦĦ IĦ 1 S1 S 1@S /ĦĦ  Ħ&Ħ ,GĦ/Ħ &Ħ %nĦ nĦnĦĦ 1  ĦÑ,Ħ6 nĦ,y} Ħ Ħ 8/: 3: 81: Ħ Ħ Ħ ,Ħ #Ħ &Ħ #Ħ Ħ &c} 9 >S &f"} D!Ħ Ħ 







 4 IĦ3 4SS  Ħ ,Ħ/Ħ "ĦĦ —"ĦĦ ,†Ħ " Ħ

4FB3 3S44 3S 0Ħ4 }



) Ħ



  }  ,L 4 Ħ

'."305 ; 7$ ,7'*; 0' 79; &-!2/4; 6#+6&(; (8+2&;  ; 3--9} Sp\m} U\Ztdom} Up\ZeYi} Tovm[} Uoivtdomr}

4 DĦ Ħ Ħ b =Ħ F FĦ FĦ Ħ S FĦ Ħ =Ħ = Ħ S FĦ Ħ 9GeFĦ IĦ Ħ b =Ħ &Ħ S /Ħ &4 Ħ = Ħ Ħ b =Ħ &Ħ S /Ħ %4% 4 2=Ħ Ħ b =FĦ&Ħ4:Ħ C:Ħ – = Ħ T Ħ &"4%% 4&& 4= Ħ %"4Fb" Ħ 9

(9

9

2FĦ 8FĦ

S$Ħ% &4} %!4$Ħ &4 %&4}%4

2&Ħ T – ĦhĦ  S [=Ħ Ħ G= Ħ =Ħ C T –:Ħ CªĦ$ĦR T –Ħ 2FĦ CªĦhĦ RS h C4Ħ 2ĦFĦ ?C4ćĦ$Ħ?Cª4Ħ 4 DFĦĦĦF==ĦFFĦĦbbFĦ&Ħ‰Ħ F=bĦF"Ħ=F:Ħ !ĦĦ 9_ :Ħ 99 !,9 !Ħ !+"9 4  Ħ@ĦD=bĦ4$ĦĈĦ

*4 ‰ 

,

09 

 ĦĦ

&Ħ !bĦ !ĦĦ‰Ħ 9 4  ĦMĦ r=bĦ‰Ħ$Ħ444  Ħ D=bĦ Ħ 4 4 4 4          F.} 4 4 4 Ħ Ħ !Ħ FĦĦ!Ħ = ĦĦFĦAĦFoĦ 4

Ħ ƒË Ħ&Ħ ĦĦĦ  Ħ Ħ Ħ &Ħ Ħ &Ħ%lĦ9: {Ħ [&ĦĦĦ PĦ %lĦ0ĦĦĦĦ Ħ

Ħ Ħ 

Ħ &Ħ :Ħ Ħ/Ħ &Ħ  Ħ &Ħ PL Ħ %lĦ0ĦĦ Ħ PĦ0ĦĦĦ €/#Ħ/ Ħ Ħ r Ħ Ħ

Ħ £Ħ éĦ %lĦ0ĦĦ Ħ-Ħ%lĦ0Ħ{Ħ   ĦnĦ ó%lĦ0ĦĦ   Ħ 



2 Ħu Ħ ĦĦĦ&€Ħ€ĦĦ 2Ħ €Ħ Ħ Ħ !Ħ˜ĦĦ  Ħ / Ħ HÒĦI#Ħ 2 :Ħ  Ħ ËĦ?S  Ħ  Ħ ~ĦL Ħ   Ħ%lĦ0ĦĦ 2 Ħ ~%lĦ0ĦĦ 9 Ħ  Ħ %lĦ0Ħ Ħ r Ħ ~ĦĦ%lĦ0ĦĦ Ħ Ħ Ħ 8Ħ # Ħ Ħ Ħ /Ħ €Ħ &Ħ  Ħ Ħ %lĦ0Ħ{Ħ Ħ IĦ *Ħ 9 % Ħ Ħ 2 Ħ







},Ħz #} IĦ L} /Ħ  Ħ & Ħ &Ħ /Ħ Ħ *Ħ ! Ħ ,Ħ0ĦGĦ

,Ħ 

NLO} hĦ , 0Ħ



„Ħ Ħ€ Ħ,Ħ0ĦÚĦĦ (/: ĦĦĦ/ĦĦL$} D Ħ (/4:(1: ĦĦ Ħ,ĦĦĦ*Ħ Ħ Ħ,Ħ0ĦG#ĦĦ (/: Ħ ĒĦ&Ħ Ħ,Ħ0Ħ G#Ħ Ħ (1: &Ħ##Ħ Ħ¤ĦcĦ¥Ħ>S½Ħ? S 2 Ħ ~Ħ

QLQ}  ; ëĦ

4/9

r Ħ

, ,Ħ0Ħ ?S , ,Ħ0 ĦĦ 

,,Ħ0 Ħ , ,Ħ0 Ħ

¥Ħ ?S6





,ĦKĦĦ,ĦL  }KĦ KĦ,Ħ ,ĦKĦ Ħ ,ĦL  }KĦ KĦ,Ħ ÍĦ













[Ħ Ħ/Ħ !Ħ Ħ ,L c, 0cĦ ,K, ¥ X%Ħ  ,Ü c,Ħ0ĦcĦ 6 ,ĦKĦ,Ħ &Ħ#Ħ ¤Ħc½Ħ ,6 {N Ħ [&Ħ,ĦĦ Ħ  Ħ ,ĦKĦĦ,Ħ6 {ĦKĦ KĦ,Ħ ,ĦKĦ ,ĦL  }KĦ   ĦKĦ,Ħ







YÛ¯‚-Ħ

;

)9

,L c,Ħ0ĦcĦ  Y±¯‚ pĦ %  ,6 c,Ħ0ĦcĦ

[&Ħ,ĦĦ €Ħ  Ħ Y ‚-Ħ êĦ } ,6 ¸,Ħ0Ħ¸Ħ ; ,ĦKĦ,6  }KĦ KĦ,Ħ ,6 ,N Ħ vhĦ ¤Ħ %_‚-%Y‚-Ħ 9 %Y±¯‚-Ħ ,ĦKĦ ,6  }KĦ KĦ,Ħ ,6 ,N Ħ *  ,6 c,Ħ0ĦcĦ 











9

) Ħ

4 }ĦĖĦk"ĦĦ&kĦ°Ħ kĦ /4!Ħ4 F}/4  l} &Ħ/#4 Ml} Ħ/#4 4 %. +$$44 %$44%4  4$4 l % 4 [&Ħ/44 %4 ĦĦ &9



l} F}  *4 G

 *+4  

/4 * ,4

l}

Fl} / 4





!Ħ "Ħ &9  l 4ĦĦ/#-%. ,4 2Ħ !Ħ Ħ/#4 4 %. $$44%$44%4111$4 l %4 [&Ħ/4^} %4 Ħ kĦ eĦ Ħ "Ħ kĦ %4  Ħ Ħ %%4 4   Ħ/#4 „Ħ Ħ/4^}%4 2Ħ %'$$44%$44%4







$4 l %4

$%$)44.}14114 4 $%44l 4   Ħ/#4

8Ħ Ħ kĦ Ħ Ħ&ĦĦ /"Ħ k Ħ Ħ Ħ k  Ħ &Ħ l}  "Ħ kĦ ĦĦ "/Ħ/°Ħl!} ÿĦ®Ħk&kĦ "/#Ħ/°Ħ/#4Ħ 2ĦĦ /#4 4 %'$$44%$44%4123$4 l %4 D Ħ /4 ^} )4 !Ħ "Ħ /#4%(4 9 4 Ħ Ħ /#4 4 $$44%$44%4 Ħ /#4 4 %.$$44%$44%4111$4 l %4



4





$4 l %4  Ħ

        ',% /.0$; 2&$+ 2'! *;*6+/' #;;      #  y

ApJiIEvy  y>Evy y

  .48%2; ; + 16%45(/.4 ; .5%2;9/62; .48%24;/.;5'%; .48%2;4'%%5;037($%$ ; /2;5'%;-6,5(0,%; #'/(#%;16%45(/.4; %.5%2; /.,9;5'%;,%55%24; ; "; #; $; /2 %; #/22%40/.$(.&; 5/;5'%;#/2: 2%#5; .48%24;(.;5'%; .48%2;4'%%5 ; /2; 5'%;/5'%2; 4'/25;16%45(/.4; 82(5%;9/62; .48%24;(.; .48%2;4'%%5; .$;4' $%;5'%; 00302( 5%;"6": ",%4;"%,/8;9/62; .48%24 ; /;45%04; 2%;.%%$%$;5/;*645);9/62; .48%24 ;  #';16%45(/.;# 22(%4; + - 2+ ; /;# ,#6, 5/24; 2%; ,,/8%$;

 CREoyZiyoRJyp`ZoiyIZQZoyaKy ! ' Ey $y

Fy $y

Hy$y

Iy!$y

Jy#

 CREoyZiyoRJyqE\pJyaKy  ;S y PS )y PS y y S y PS y PS y  y S y *'y Ey $y

Hy$y

Fy $y

Iy$y

Jy 

y CREoyZiyoRJy\EioyorayIZQZoiyaKy y S y S y S y S Ey$y

Fy %y

Hy$y

Iy$y

37 37 37

S y S y S 'y

Jy!

 9ary^E`vy wJhaJiyIaJiyoRJy`p^FJhyy QS #y PS '/BOS Ey /2B Fy#$y

Hy $y

Iy  $y

17 17 27

QS y QS y OS

yJ`IyrZoR'

Jy 

 :`y Ey iepEhJy ; \Joy ;FJyEy caZ`oy a`yoRJy iZIJy ;ipHRy oREoy ; + ; E`Iy ;FJy oRJ ^ZIcaZ`oyaKy ; :KyoRJyEhJEyaKyoRJyohZE`Q\Jy ;Ziy  yrREoyZiyoRJyEhJEyaKyohZE`Q\Jy 

MS

Dy *2B Fy "2B Hy "*2B Iy #2B Jy #*B

,

ARJyqD\pJiyaKy byrRTHRyiDlTiKvylRJyJepDlTa`i b$BS 0(;S %Bby bfy + b ?f

/B

+

% B

^DvyFJyKap`IyFvyia\qT`Qy

> ,B+ 2B Fy !%>$ "#"> '#,B+ 2B Hy #0>$ !!> !.'B+ 2B Iy #0>$BS !!*> !%#,B+ 2B Jy %0>$ !'> !.'B+ B Dy >$

.

BiT`QylRJyITQTliy "B#B%Bya`\vya`HJylayKag^yDy  ITQTly`p^FJgyRary^D`vyaKylRJ^yDgJyITqTiTF\J Fvy ""5 Dy '3B Fy *2B Hy ,2B Iy .3B Jy /

/

:KyDyca\vQa`yRDiyTliyip^yaKyT`lJgTagyD`Q\Jiyi^D\\JgylRD`y #*?BrRDlyTiylRJy^DsT^p^y`p^FJg aKyiTIJiyaKylRJyca\vQa`' Dy ""2B Fy "#2B Hy !%3B Iy "'2B Jy "*

0

ARJy\Dily #BITQTliyaK

#*BS #*$BS #*&BS )*+S S #*$+B

AB7S

Dy 2B Fy *2B Hy #*2B Iy *2B Jy .*B

"

@pccaiJy yITilT`Hly`p^FJgiyDgJyHRaiJ`yKga^y " B#B lRJy\DgQJilycaiiTF\JycgaIpHlyaKylRaiJy %B`p^FJgi' D . &

<2B

F

<&2B

H

""

@pccaiJy ; B";; BD`I

"#

4T`IylRJyJsDHlyqD\pJyaKy

<  <8S

I

  BB%
<& .<  ,2B

J

<& .. ,

8; + :; + #*B 4T`IylRJyqD\pJyaK 

rRJ`y >B+ #,&#'&

 y @pccaiJy > *+ + "B 4T`IylRJyqD\pJyaKy

#B

"'

@T^c\TKv

B"B 3;B"B 8B"B ; B"B 7;  "B ; @" AB " 

7 67 7

7 07

 ARJy OQphJy ; TjyDyhJQp\DhyRJtDQa`y 3qD\pDlJylRJyepalTJ`l ,hJDyaKyRJtDQa`y ; ,hJDyaKylhTD`Q\Jy ;

",

@pccajJy 4;D`Iy 5;DhJylrayhJD\y`p^FJhjyjpHRylRDly 4;;5;  "BD`Iy 4; ;5;  aKy 4;;)

".

4T`IylRJyqD\pJy T`ylRJyjT^c\JjlyKah^yaKy

"/

:Ky 4;;yD`Iy 4; ; ; + yLV`IylRJyqD\pJyaKy 4; ;9  

"0

ARJyITDQhD_yjRarjyDyjJQ^J`lyaKyDyHThH\JyjpHRylRDly ;TjycJhcJ`ITHp\DhyFTjJHlahyaKylRJyHRahI ; 8TqJ`ylRDly ;  ",BD`Iy ;  y O`IylRJyITD^JlJhyaKylRJyHThH\J

;y 

",.B 4T`IylRJyqD\pJ

y

=S

;S

GS

<S

 ,ycD\T`Iha^Jy`p^FJhyTjylRJyjD^JyJTlRJhyhJDIyKha^y\JMlylayhTQRlyahyhTQRlylay\JKlyKahyJtD^d]J "#"BTjyDycD\T`Iha^Jy`p^FJhy 9ary^D`vy ITQTlycD\T`Iha^Jy`p^FJhjyDhJylRJhJylaQJnRJh(  y =Jly (+FJyDychT^Jy`p^FJhyjpHRylRDlylRJy`Jtly\DhQJhy`p^FJhyTjyDycJhKJHlyjepDhJy 4T`IylRJyjp^y aKy D\\y jpHRy chT^Jy `p^FJhjy  4ahyJtD^c\JyTKy vapy lRT`[ylRDly ""BD`Iy "%BDhJy lrayjpHRychT^Jy `p^FJhjylRJ`ylRJyjp^yTjy  y

 y :`ylRJy OQphJyFJ\aryTKy ; S ;-y ;; ;; ;;1y ;; ;;7y

lSJ`y rRDly Uky=4B

y

7

=BIJQhJJk

+

!+

#%

8WqJ`ylRDly =B

7

#6)B %7$BrRJgJy 6BD`Iy GDgJycaiWlWqJyW`lJQJgiy P`IylRJy\JDilycaiiWF\JyqD\pJyaK 7

D(S

#'

:Ky =$By= #+

#*

:`ylRJyIWDQgD^y+-y  25y /.y 8WqJ`ylRDl +.-/y  #* B-.y P`Iy .5

7

BP`IylRJy qD\pJy aKy =(B %=$By%

<S

#,

7

BD`Iy 2.2/y *

HS

@S

4W`IylRJyip^yaKyD\\ycaiiWF\JyqD\pJiyaKy 6BipHRylRDlylRJyKa\\arW`QyJepDlWa`yRDiygJD\ygaalyW`y =1

= 69By=$ %=By#$B B 7

#-

:Ky lray caiWlWqJyW`lJQJgiy :BD`Iy ; BFalRyFWQQJgylRD`y +iDlWiKvylRJyJepDlWa`

#*$BB:$B #'$BB;$ 7

P`IylRJyqD\pJyaNy:BB

#/

7

B

@pccaiJy 6B  B7B  BFS %& BD`Iy

2S 

7

S 

7





B 4 U`IylRJyqD\pJyaK 'B

 LS !.31; 0"; !06-; .-; 3&"; '0 4,#"0"- "; .#; ; '0 +" ; 9; ).'-'-%; ++; 3&"; !.31; 3.; .-"; -.3&"0; 9; 130'%&3; +'-"1; ;3&"; ,7',4,; -4,"0; .#; 0"%'.-1; 3&3; -; "; #.0,"!; '3&"; '0 +"; '2; .4-3"! ; .0; "7,/+";6&"-; LS :S ; 3&"; ,7',4,; -4,"0; .#; 0"%'.-1; '1;

 +

&3; '1; 3&"; ,7',4,; -4,"0; .#; 0"%'.-1; 3&3; -; "; #.0,"!; 6&"-; KS 9S ;

 .4; 6+*; ; 1/'0+'-%; ,:"; .-; 3&"; 03"1'-; /+-"; 1; #.++.61; 1303'-%; 3;  ; ; -! 3&"; $013; $5"; 13./1; 0"; 3; +  ;+ ;;+  ;;+ ;; -!; +   ; .40 -'-3&; 13./; '1; 3; 3&"; /.'-3; ;; -!; 1.; .-; 1""; 3&"; !(%0,; "+.6 ; &3; '1; 3&" 8 ..0!'-3"; .#; 3&"; /.'-3; 6&' &; 9.4; 6.4+!; 00'5"; 3; .-; 9.40; 

3&; 13./















6



 0 

N 5



1 -7 7 7 7 7 7 7

7 7 7 7 7 7 7

7 7 7 7 7 7 7

7 7 7 7 7 7 7





7 7 7 7 !7 !7 7 /7

"##$%7 .7

1 

%+$%%%,7



;

/7

 y :`ylRJyKa\\arT`Qy PQpgJy+/&y +-y
+

%+

+

%#

+

@pccaiJylRDly ES !;#;DgJyITilT`Hly`p^FJgiyipHRylRDl ! E

L` X IylRJy qD\pJyaKy

%%

3



! ## Ey&y

JI'S  

6T`IylRJy`p^FJgyaKyJqJ`yITQTliyT`ylRJycgaIpHlyaKylRJylray  ITQTly`p^FJgi

##########BCS """"""""""y

 6T`IyD`y T`lJQJgy uylRDlyiDlTi LJY iylRJyJepDlTa` u \ u """uy S "y&yy

%*

0JlJg^T`JylRJyiJHa`Iyi^D\\JilycgT^JyKDHlagyaK yS +

BS y

#yS y #BS y

%yS y %BS '

,B

# *4S S  # *BS y

          HUZRJ^\`Ph AJcSPXJcUMJWh CWeX^UJNh 

@dYT[_h GOLbT[Yh G[VdbT[Yah %Š @py8Š [Š

/-h 1h /,h ƒŠ /h /hns[Š%"ŠygpW\Š /,h # %Šns[Š%" 

(Š @py8Š UŠ I\{Š Q IhK2h IhK*h1hI*hh+I)Kh-h K& hK*'h Me\pŠ _"0 Š%0Š 1h "0Š(*h # % -Š @py;Š UŠ %Šh(Šh%&$hh(""0Š # gh ƒŠ (""0ŠƒŠ (""2Š E\pW\Š{e\ŠlTy{Š{€sŠ[gdg{yŠgyŠ%0 (Š /Š @py<Š \Š A…Š Ws}p{gpdŠ{e\Šp}nU\xŠ s^Š ^TW{sxŠ 0ŠgpŠ{e\Švxs[}W{ Š €\Š\Tygl…Š y\\Š{eT{Š{e\Š p}nU\xŠ 0"Š 0"Š 0"Š \p[yŠ €g{eŠ  # %(Š†\xsyŠ 0Š (0Š 0Š @py8Š [Š

D9h

Me\Š{€sŠ{xgTpdl\yŠTx\ŠygnglTxŠTp[Š ;h 1h (Š E\pW\Š@x\TŠE;62h/ƒŠ @x\TŠD9Eh3h (" E 2Š @py;Š TŠ %Š L}Uy{g{}{gpdŠ 2h ] 5Šgp{sŠ{e\Š bxy{Š \w}T{gsp Š €\Š d\{Š T

F

/h @py;Š \ H^Š{e\Šaxy{Š[gdg{ŠgyŠ% Š {e\pŠ{e\Šp}nU\xŠgyŠ[g~gygUl\ŠU…Š %%Šgnvlg\yŠ{eT{Š{e\Š-x[Š[gdg{ŠgyŠ /ŠTp[Š{e\x\ŠTx\Š{€sŠ €T…yŠ{sŠv}{Š(ŠTp[Š-Š Me\ŠWTy\yŠ €e\pŠ{e\Šcxy{Š[gdg{ŠTx\Š(Š -Š /Š x\yv\W{xg~\l…Š WTpŠ U\Š[\Tl{Š €g{eŠ ygnglTxl…Š E\pW\Š{e\Š {s{TlŠp}nU\xŠgyŠ/ŠƒŠ (Š # 5Š 5Š @py;Š WŠ Me\Šy}nŠs^Šgp{]xgsxŠTpdl\yŠgyŠ   (ŠƒŠ %5"ˆŠ # (""0‰Š E\pW\Š{e\ŠnT„gn}nŠ # gyŠ%- 6Š @py8Š UŠ Me\ŠlTy{Š (Š [gdg{yŠs^Š (""0rŠ gyŠ(0Š\‚W\v{Š ## %Š€egWeŠdg~\yŠ0Š E\pW\Š{e\ŠlTy{Š (Š[gdg{yŠ gyŠ0Šh (""/ŠƒŠ (0Šns[Š %""Š # "0

/h

%"Š @py;Š  T  DxsnŠ{e\Šg[\p{g{…Š .SVŠ>Š SŠVŠ&  S VŠ& sp\Šy\\yŠ{eT{Š €e\pŠ SŠVŠ gyŠb„\[ Š {e\ vxs[}W{ŠgyŠ{e\ŠmTxd\y{Š €e\pŠ hS ViŠ gyŠ{e\ŠynTmm\y{Š Js€Š m\{Š SŠ # VŠ # ZŠ U\Š & [gy{gpW{Švsyg{g~\Šgp{\d\xyŠy}WeŠ{eT{Š SŠVŠ  ZŠ -qŠ Tp[Š{e\Š vxs[}W{ŠSVZŠgyŠ{e\ŠUgdd\y{ŠvsyygUm\Š H^ŠSŠTp[ŠVŠ[g`\xŠnsx\Š{eTpŠ( Š{e\pŠU…ŠgpWx\TygpdŠ SŠ U…Š %Š Tp[Š[\Wx\TygpdŠ VŠ U…Š % Š {e\Š vxs[}W{Š SVZŠ €gmmŠ U\Wsn\Š Ugdd\xŠ E\pW\Š SŠ Tp[ŠVŠ [g`\xŠ U…Š T{Šnsy{Š (Š LgngmTxm…Š VŠ Tp[ŠZŠ [g`\xŠ U…Š T{Šnsy{Š (Š KpŠ{e\Š s{e\xŠ eTp[Š g^ŠZŠ>ŠVŠŠ(Š>Š SŠ. Š {e\pŠsp\ŠWTpŠgpWx\Ty\Š{e\Š~Tm}\Š s^Š SVZŠ U…Š [\Wx\TygpdŠZŠ U…Š %Š Tp[ŠgpWx\TygpdŠ SŠ U…Š %Š LsŠ {egyŠ WTy\ŠgyŠ x}m\[Š s}{Š LgpW\Š SŠŠVŠŠZŠ gyŠ [g~gygUm\Š U…Š & g{Š gyŠ ps{Š vsyygUm\Š {eT{Š ZŠ>Š VŠŠ%Š>Š SŠŠ & sxŠ ZŠ VŠŠ(Š> SŠ  & LsŠ€\Šn}y{ŠeT~\ŠZŠ=ŠVŠ%Š Š>Š SŠ(Š E\pW\Š{e\Š{ex\\Šp}nU\xyŠTx\ŠqŠ % Š qŠTp[ŠqŠ%Š Š Me}y Š {e\gxŠ vxs[}W{Š gyŠq %qqŠŠ%Š=Šq qŠ 



%%Š @py;Š (""0Š VŠ=Š(""0SŠ Tp[ŠZŠ=Š(""0VŠ # VŠŠZŠ=Š(""0SŠ VŠ E\pW\Š

VZŠ >Š(""0 SVŠ

%(Š @py;Š (""0Š I\{ŠqŠ>Š(""0Š Me\pŠ #&>ŠqŠŠ  q &>Š2q*ŠŠ(Š E\pW\Š

fh=Š(""0

%-Š @py;Š %Š Me\Š\„vx\yygsp>Š # $# $#&# h $&$h  #$& L}Uy{g{}{gpdŠ # $ y\\yŠ{eT{Š{e\Š \„vx\yygspŠ=Š &#& $&&=Š%Š



% Š sp\Š

%.Š @py;Š %""-Š

 Š%Š  %Š >Š%""- Me\Š \„vx\yygspŠ>Š %Š   &=Š  "# "#   #

%0Š @py8Š & %Š I\{Š  U\Š{e\ŠW\p{\xŠs^Š{e\Šx\d}mTxŠe\„TdspŠ Me\pŠ@x\TŠ B8h:h ?Š @x\TŠ  48h>Š .h!# @x\TŠ 478:<>#h Me\Šx\y}m{Š ^smms€y %2Š @py;Š (""0!Š %24Š>Š  #h% #$& # #$





24 Š &=Š(""0 Š E\pW\Š #& $&=Š #&h$# #$&$ (

%4Š @py8Š /Š Bsnvm\{gpdŠ{e\Šyw}Tx\y Šsp\Šy\\yŠ{eT{Š{e\Š\„vx\yygsp>Š 



h





%5Š @py7Š %%"Š %Š @yŠ #& # " Š €\ŠeR~\Š #&



#

# %Š %Š %Š 0 Me}y Š #& h # #& #&h& +Š # %%" #

#

%6Š @py8Š ("Š I\{Š !& U\Š{e\ŠxR[f}yŠ Me\pŠ "&



! &h5& # !



#

%" Me}yŠ{e\Š[fRn\{\xŠfyŠ("

("Š @py9Š 6""Š L}WeŠRŠtRlfp[xsn\Šp}nU\xŠfyŠWsntl\{\l…Š [\{\xnfp\[ŠU…Š f{yŠaxy{Š{ex\\Š[fdf{yŠ Me\x\Š Rx\Š6 Š%" Š%"ŠWesfW\yŠs^Š{e\Š%y{ Š (p[ŠRp[Š+x[Š[fdf{yŠ x\yt\W{f~\l… Š e\pW\Š{s{RlŠ # 6""Š (%Š @py7Š +Š H^ŠuŠ h%Š # & uŠ  #h %Š LfpW\ŠufyŠtxfn\ Š # # (Š E\pW\ŠuŠ# +ŠfyŠ}pfw}\ 

((Š @py8Š 0."Š A…Š [f~f[fpdŠ }tŠ fp{sŠ{xfRpdl\yŠ €f{eŠ x\yt\W{Š {sŠ RŠW\p{\xŠ # Š sp\Š y\\yŠ {eR{Š {e\x\Š Rx\Š y\~\pŠ{xfRpdl\yŠ^sxn\[Š\dŠ B58h B7:h \{WŠ Me\Š nf[[l\Š Rpdl\ŠR{Š#ŠfyŠR[[\[Š}t {€fW\Š E\pW\Š #& # 4Š # %5" 4("Š # 0."Š (+Š @py8Š %000(Š KUy\x~\Š{eR{Š+Š FŠ# Rp[Š(Š GŠ VŠ L\{ŠVŠ (V&Š Ne\pŠ€\Šy\\Š{eR{Š(Š GŠ# Me}yŠ{e\ŠynRll\y{ tsyyfUl\Š ~Rl}\Šs^Š# fyŠ2Š E\pW\Š #& # (2 1Š %000(Š 



(.Š @py8Š (Š PyfpdŠlspdŠ[f~fyfsp Š €\Šd\{Š # #&h+Š # #&h#  # # %Šh( (0Š @py9Š 2"Š CxsnŠ{e\Š\w}Rlf{f\yŠ

8h



5h 7>h 78h 7>h 78h

:<8
%0"Š

#

8>h

7<5h

587:h 78h

(0"Š Rp[Š €\ŠeR~\Š 8>h # 2" &

(2Š @py9Š +Š # +ŠŠ(Š # #  # "Š  # %ŠsxŠ # (Š (4Š @py8Š (%%Š LfpW\Š  #     # h# (%%Š

h

 #

.""6Š # (%%Š





#

%6Š €efWeŠ fyŠ RŠ txs[}W{Š s^Š txfn\zŠ E\pW\

(5Š @py7Š 'Š I\||gpdŠ =Š j Š €\Š d\|Š SŠ=Š Vj ŠVŠ=Š XjŠ Rp[ŠXŠ # SjŠ Me}yŠ SŠ=Š Sj,Š Me}yŠ jŠ=Š 'Š Rp[



SŠ 0h VŠ=ŠXŠ Me\Š x\y}l|Š ^slls€y (6Š @py7Š 04Š Kp\Š YRpŠ Ys}p|Š [gx\Y|l…Š |eR|Š |e\ŠoR‚go}oŠp}oU\xŠs^Š x\dgspyŠgyŠ04Š +"Š @py7Š +Š KUy\x\Š |eR|Š |e\Š y|stŠ p}oU\xyŠ ' Š6 Š(0 Š.6 Š  Š Rx\Š R|Š |e\Šls€\xŠ xgde|Š Ysxp\xyŠ Me\Š tsgp|Š" ŠqŠgyŠR|Š|e\Šy|stŠp}oU\xŠ(qŠŠ') qŠ' Š Š # .q)ŠŠ+qŠ Qe\pŠqŠ # (( Š €\ŠeR\Š.(()Š+Š ((Š 0h(""(Š LsŠ|e\Štsgp|Š" Š((ŠgyŠ|e\Š(""(|eŠy|stŠ Me}yŠ|e\ tsgp|Š+ Š((Š gyŠ|e\Š (""0|eŠy|stŠ +'Š @py7Š (0Š PtspŠ xs|R|gpdŠ 4=:h Rp|gYlsYk€gy\Š 6"‡Š |exs}deŠ 4h €\Š d\|Š RŠyw}Rx\Š Me}yŠ |e\Š Rx\R gyŠ0*Š=Š(0Š +(Š @py7Š 'Š KUy\x\Š |eR|Š V XŠX S) .V XX S=Š "Š E\pY\Š V X X S)Š=Š"Š Me}yŠVŠ XŠ # XŠ SŠ ++Š @py:Š '"Š ((((((((((Š # '"&$Š 'Š=Š(((((((((("""""""""" (((((((((( #

((((((((('44444444445Š

+.Š @py7Š '"Š PtspŠx\€xg|gpdŠ |e\Š \w}R|gspŠ RyŠ * '"', 666ŠŠ'Š=Š" Š sp\Šy\\yŠ |eR|Š Š=Š gyŠ|e\Šspl…Š gp|\dxRlŠysl}|gspŠ +0Š @py7Š ''Š

*$$1 'Š 'Š Me\Šy}o=Š O *Š OŠ'Š 0h .h # (""0Š# (""3Š !# ."'' h # (""0Š # (""3(""0Š Ptsp  #

ygotlg^…gpd Š €\Š y\\Š |eR|Š |e\Š y}oŠ=Š 0Š # ."'Š # '+.""6Š=Š 0Š # ''Š # ."'Š !# '('5'6Š!  LgpY\Š( Š+ Š4Š[s\yŠps|Š[gg[\Š."'ŠRp[Š'('5'6 Š |e\Šy\Ysp[ŠyoRml\y|Štxgo\Š^RY|sxŠgyŠ''Š

'"Š

       ,?I:/OLR8] &/U<8F/U?2/C](DXFO?/5] ,&(]  ,8I?LR],82U?LI

/K8H72O Q  & -2OQ  &

  &

$FOLRU/IU +27; ; Q /524$-+2; +4; 9-5; +272; -+; 4"; +27; 2"4; .*6$ ; ; 4"; )5(4$.(; "-$; /524$-+2; +4; 9-5; +272; $+; 4"; +27; 2"4; 9; 2"$+!; 4"; 5(2; -+4$+$+!; 4"; (442; UÄ ]

] YÄ -; [ Ä -2.-+$+!; 4-; 4"; -4; +272 ;

; 4"; -4"; 2"-4; /524$-+2; $4; 9-5; +272; $+; +27; 2"4; +; 2"; 4"; ..*: .$4; 5(2; (-7; 9-5; +272 ; -; 24.2; ; +; 4-;%524$ 9; 9-5; +272 ; "; /524$-+; $2; Q )& ; -; (5(4-2; ; ((-7 ;

 0<2JQ =HQ J<8Q H?2>>8HJQ DCH=J=L8Q DF=?8Q :25JCFQ C:Q J<8Q =BJ8;8FQ   & Q   &

U Ä &

]

&

]

&

YÄ

"]

&

&

 0<2JQ =HQ J<8Q L2>K8Q C:  & Q &   &  & Q

&

 &

U Ä  &

]

]

 &

 &

!]

 &

#

.CB8Q C:Q J<9Q 23CL8Q

 ,8JQ .; 38Q 2Q F82>Q BK?38FQ HK6OQ CB8Q HC>KJ=CB

/<8B

U Ä .;  LÄ



.;



&



.;  &

YÄ

.;

[ Ä

.;  &

 0<2JQ =HQ J<8Q IK?Q C:Q J<8Q >2HJQ JMCQ 7=;=JHQ C:Q J<8Q =BJ8;8FQ & Q & Q & Q

U Ä &



Q

]

&

YÄ GÄ







Q  &

[ Ä J

 !$"#&  & =HQ 8EK2>Q JCQ &   

U Ä &

]

  &

]

  &

&

!]

   &

#]

   

E !Ä c¢Ä ­‘„Ä ƒ”~¨~žÄ ; ; ~¢ƒÄ ; ~¨„Ä ­‘¨„„Ä ¦¥”¢­¬Ä ¥¢Ä ­‘„Ä ”¨š„Ä ±‘¥¬„Ä „¢­¨„Ä ”¬Ä  v‘„Ä š”¢„¬Ä ; ~¢ƒÄ ; ~¨„Ä ¦¨¥ƒ®„ƒÄ ­¥Ä ž„„­Ä ~­Ä ; t®¦¦¥¬„Ä ­‘~­Ä ;PÄ ; ~¢ƒÄ ;PÄ25¾Ä~¢ƒÄ ;   \”¢ƒÄ­‘„į~š®„Ä¥‰Äµ! 

UÄ :EÄ

&

] ?5Ä WÄ ALÄ YÄ B?Ä "] E+

l„­Ä µÄ ~¢ƒÄ ·Ä „Ä ¦¥¬”­”¯„Ä ¨„~šÄ ¢®ž„¨¬!Ä y‘~­Ä ”¬Ä ­‘„Ä ¬ž~šš„¬­Ä ¦¥¬¬”š„Ä ¯~š®„Ä ¥‰Ä 2EÄ 2+LÄ   µ·R ·Ä  U Ä 2EÄ

] 2LÄ  WÄ 5?Ä YÄ :+Ä

"] :E

5µÄ 2Ä ”¬Ä¨„Ž„­„ƒÄ~¥®­Ä­‘„Ä µ 2Ä š”¢„Ä·ÄPĵ!Ä y‘~­Ä”¬Ä­‘„Ä„§®~­”¥¢Ä ‰¥¨Ä­‘„Ш~¦‘Ä¥‰Ä­‘„Ä”ž~„Ä¥‰Ä­‘„Ĩ„Ž„­”¥¢RÄ

L!Ä c¢Ä­‘„ÄW~¨­„¬”~¢Ä¦š~¢„Ä ­‘„Ш~¦‘Ä¥‰Ä­‘„Ä ‰®¢­”¥¢Ä ·ÄPÄ  2Ä 5µÄ 2 ÄÄ 2  "] ·PÄ 5 µÄ

U Ä ·

Q





t”ž¦š”Ä

 ·





5Ä 2 Ä Ä 2Ä Ä

]





2Ä  Ä5Ä 



 WÄ ·ÄPÄ

5µÄ 2  2 ÄÄ

YÄ ·ÄPÄ



2 ÄÄ  5Ä 

Ä ±‘„¨„ÄµÄ ”¬Ä~¢¸Ä ¦¥¬”­”¯„Ĩ„~šÄ¢®ž„¨#

2 7

ÄÄ YÄ µ7Ä 2

WÄ

[ Ä

2+&Ä l„­ÄµÄ ~¢ƒÄ ·Ä „Ĩ„~šÄ¢®ž„¨¬Ä¬®‘Ä­‘~­Ä µ7Ä Ä·7ÄQÄ5µÄ 5·ÄQ 5!Ä y‘~­Ä”¬Ä­‘„Äš~¨„¬­Ä ¦¥¬¬”š„į~š®„Ä¥‰Äµ7Ä Ä·7RÄ Ä xÄ U Ä 2+Ä ÄLw8Ä ] LÄ E [ Ä p¥¢„Ä¥‰Ä­‘„Ä~¥¯„Ä

 WÄ EÄ Ä?xÄ

25Ä

YÄ AÄ 5Ä ¹

22" ]”¢ƒÄ­‘„Ш„~­„¬­Ä”¢­„„¨Äš„¬¬Ä­‘~¢Ä 5Ä Ä 25

 !"Q ”¬Ä ~Ä ­¨”~¢š„Ä ¬®‘Ä ­‘~­Ä "QQ M+¾ Ä t®¦¦¥¬„Ä "QQ 2BEÄ …žÄ !QQ 2EMÄ …ž ~¢ƒÄ ­‘„Ħ„¨¦„¢ƒ”®š~¨Äƒ”¬­~¢„ĉ¨¥žÄ "Q ­¥Ä !Q ”¬ÄµÄ …ž!Ä ]”¢ƒÄ­‘„į~š®„Ä¥‰Äµ&

2: ]”¢ƒÄ ­‘„Ĭ®žÄ¥‰Ä~ššÄ­‘„Ä ¨„~šÄ ¢®ž„¨¬ÄµÄ ­‘~­Ä ¬~­”¬‰¸Ä ­‘„Ä „§®~­”¥¢

2? v‘¨„„Ħ¥¬”­”¯„Ä”¢­„„¨¬Ä~¨„Ĭ®‘Ä­‘~­Ä­‘„¸Ä ƒ”‹„¨Ä‰¨¥žÄ„~‘Ä¥­‘„¨Ä¸Ä ~­Äž¥¬­ÄE Ä c­Ä”¬ ~š¬¥Ä –¢¥±¢Ä ­‘~­Ä ­‘„Ä ¦¨¥ƒ®­Ä ¥‰Ä ­‘„¬„Ä ­‘¨„„Ä ”¢­„„¨¬Ä”¬Ä5K+K#Ä y‘~­Ä ”¬Ä ­‘„Ä ¬ž~š„¬­ ”¢­„„¨Ä~ž¥¢Ä­‘„žR 2BÄ!Ä GPQdž¦œ¸Ä AQ

5++:65++?6Ä Ä5++BÄ 5++B65++?6 5++:Ä !Ä 5++?; 2Ä 5++?6 25++?;Ä Ä2Ä 

2E W¥¢¬”ƒ„¨Ä­‘„Ä ‰®¢­”¥¢ †³Ä 



Y[] ]”¢ƒÄ­‘„į~š®„Ä ¥‰

:²Ä Ä :Ä

g
Q  !Q Q 2ECÄ Ä53,"Ä]”¢ƒ

2K" UÄ ­¨”~¢š„Ä  !"Q ”¬Ä ”¢¬¨”„ƒÄ ”¢Ä~Ä ”¨š„Ä¥‰Ä¨~ƒ”®¬Ä Q …ž!Ä t®¦¦¥¬„Ä­‘~­Ä  "Q !QQ ?Ä …žÄ ~¢ƒÄ­‘„Ä~¨„~Ä ¥‰Ä  !"Q ”¬ÄµÄž9!Ä ]”¢ƒÄ­‘„į~š®„Ä¥‰Äµ7) 2M" l„­Ä 1Q €~¢ƒÄ  „Ĩ„~šÄ¢®ž„¨¬Ä¬®‘Ä­‘~­ 1Q K

€Ä~¢ƒÄ 6ÄQ 1 € 2E"Ä 

]”¢ƒÄ­‘„į~š®„Ä¥‰Ä 1Q Ä

52# ]”¢ƒÄ­’„į~š®„Ä ¥‰Ä­‘„Ħ¥¬”­”¯„Ä”¢­„„¨Ä  ”‰ 2Ä 2Ä

Ä"""Ä Ä

Ä ‡£ ÃÄ S ÀÄ À ÁÄ Á ÂÄ 2Ä

Q



2:Ä



2+"Ä

Q E+¾

55 l„­ÄTÄ~¢ƒÄVÄ „Ä­±¥Ä¦¥¬”­”¯„Ħ©”ž„Ä”¢­„„©¬Ä ¬®‘Ä­‘~­ 2Ä 2Ä  TÄ VÄ





2M5Ä 5++B6Ä 5++?6Ä

\”¢ƒÄ­‘„į~š®„Ä¥ŠÄV Ä 5:$ e¢Ä TVXÄTVÄNÄ TXÄ ? N Ä:Ä ~¢ƒÄ mÄ ”¬Ä ­‘„Ä ž”ƒ¦¥”¢­Ä ¥ŠÄVX&Ä Z”¬ ~Ä ¦¥”¢­Ä ¥¢ÄTV ~¢ƒÄ ^Ä ”¬Ä ~Ħ¥”¢­Ä¥¢Ä TXĬ®‘Ä­‘~­ÄTZÄ NÄ T^Ä 5ÄNÄ2%Ä e­Ä”¬Ä~š¬¥Ä”¯„¢Ä­‘~­ÄZÄ ^Ä ~¢ƒ TmÄ ”¢­„©¬„­Ä~­Ä _ı”­‘Ä_^Ä H5Ä …žÄ~¢ƒÄ _ZÄ ³Ä…ž&Ä \”¢ƒÄ ­‘„į~š®„Ä¥ŠÄ³% 





5? e­Ä”¬Ä”¯„¢Ä ­‘~­Ä³Ä



2



=% \”¢ƒÄ­‘„į~š®„Ä¥Š 5 h :Ä

5B l„­Ä  ; ~¢ƒÄ  „Ä­‘„Äš„¢­‘¬Ä ¥ŠÄ­‘„Ä­‘©„„Ä ¬”ƒ„¬Ä ¥ŠÄ~Ä ­©”~¢š„&Ä t®¦¦¥¬„Ä  ~¢ƒÄ ; ~©„ ­‘„Ä©¥¥­¬Ä¥ŠÄ­‘„Ä „§®~­”¥¢

~¢ƒÄ­‘„Äš~©„¬­Ä~¢š„Ä¥ŠÄ­‘„Ä­©”~¢š„Ä ”¬Ä  \”¢ƒÄ­‘„į~š®„Ä¥ŠÄ³%Ä 5E \”¢ƒÄ ­‘„Äš~©„¬­Ä©„~šÄ¢®ž„©Ä³Ä¬®‘Ä­‘~­ ³6Ä Q ³ 2Ä Q ˆ³6 ³ 2{Ä 5Ä



:B³ 5B+ÄÄ

5H `¥±Ä ž~¢¸Ä ±~¸¬Ä ~¢Ä ­‘„Ä ±¥©ƒÄ nUv`[oUveWtÄ „Ä ¦~©­”­”¥¢„ƒÄ ¬¥Ä ­‘~­Ä „~‘Ä ¦~©­ ¥¢­~”¢¬Ä ~­Ä š„~¬­Ä ¥¢„Ä ¯¥±„šRÄ \¥©Ä„¶~ž¦š„Ä nUv`[nUveWtÄ nUv`[nUveWt nUv`[nUveWtÄ~¢ƒÄnUv`[nUvcWtÄ~©„Ĭ®‘Ħ~©­”­”¥¢¬&

2?Ä

5K-Ä W¥¢¬“ƒ„¨Ä~Ĭ„§®„¢‚„Ä ¥ŠÄ¨„~šÄ¢®ž„¨¬Ä »}¡¼Ä ƒ„¢„ƒÄ¸Ä

2Ä \ŸƒÄ ­‘„į~š®„Ä¥Š   5....../Ä }6¤¤ª 

5M-Ä \¥¨Ä ~Ä ¦¥¬“­“¯„Ä“¢­„„¨Ä —Ä ±„ı¨“­„Ä

±‘„¨„Ä},Ä}4Ä -ÄÄ}•Ä~¨„Ä­‘„Ä‚¥„Œ‚“„¢­¬Ä¥ŠÄ­‘„Ħ¥š¸¢¥ž“~š Ä \“¢ƒÄ­‘„Ĭž~šš„¬­Ä¦¥¬¬“š„ ¯~š®„Ä¥ŠÄ—“ŠÄ},ÄQ }4ÄQ }6ÄQ /Ä/Ä/Ä Q }•˜Ä “¬Äƒ“¯“¬“š„Ä¸Ä 5..D0 



:.1Ä \“¢ƒÄ ­‘„Äš~¨„¬­Ä¦¥¬“­“¯„Ä¢®ž„¨Ä³Ä¬®‚‘Ä­‘~­Ä 4 5³; ³6 ³ÄQ2 Ä



2"

:2% `¥±Ä ž~¢¸Ä ¥¨ƒ„¨„ƒÄ ¦~“¨¬Ä¥ŠÄ“¢­„„¨¬Ä ³Ä·Ä ¬~­“¬Š¸Ä ­‘„Ä„§®~­“¥¢ ³6ÄQ ·6Ä



5³ÄQ ·ÄQ ³·RÄ

:5-Ä \“¢ƒÄ ­‘„Ä ¢®ž„¨Ä ¥ŠÄ ¥¨ƒ„¨„ƒÄ J­®¦š„¬Ä ¥ŠÄ ¦¥¬“­“¯„Ä “¢­„„¨¬Ä }4}6 }; }@ }« }F }I ­‘~­Ä‘~¯„Ä¥­‘Ä¥ŠÄ­‘„ÄŠ¥šš¥±“¢Ä¦¨¥¦„¨­“„¬NÄ  “ ] }¡ÄQ }¡ ™Ä }¡ 6Ä Š¥¨Ä2Ä    DÄ~¢ƒ  ““ ] }FÄ 5..D-Ä 



::0Ä l„­Ä  „Ä~Ħ¥¬“­“¯„Ä“¢­„„¨Ä¬®‚‘Ä­‘~­Ä ¥¢„Ä¥ŠÄ­‘„Ĩ¥¥­¬Ä¥ŠÄ­‘„ħ®~ƒ¨~­“‚Ä „§®~­“¥¢Ä ?³6 ?°>ÄQ ?³ÄQ °>  5?Ä





“¬Ä~¢Ä “¢­„„¨#Ä \“¢ƒÄ­‘„į~š®„Ä¥ŠÄ    :?& W¥¢¬“ƒ„¨Ä­‘„Ĭ“ž®š­~¢„¥®¬Ä „§®~­“¥¢¬

³·Ä ³ºÄ ³º ·ºÄ Q





5DDÄ 55?&Ä

\“¢ƒÄ­‘„Ä¢®ž„¨Ä¥ŠÄ¥¨ƒ„¨„ƒÄ­¨“¦š„¬Ä¥ŠÄ¦¥¬“­“¯„Ä“¢­„„¨¬Ä ]³Ä·Ä ] ­‘~­Ä¬~­“¬Š¸Ä­‘„Ä~¥¯„ ¬¸¬­„žÄ¥ŠÄ„§®~­“¥¢¬"Ä :D0Ä \“¢ƒÄ ­‘„Ä­¥­~šÄ ¢®ž„¨Ä ¥ŠÄ¦¥¬“­“¯„Ä Š¥®¨ƒ““­Ä “¢­„„¨¬Ä ; ¬~­“¬Š¸“¢Ä ¥­‘Ä ¥ŠÄ­‘„ÄŠ¥š½ š¥±“¢Ä ¦¨¥¦„¨­“„¬OÄ  “] ; “¬Äƒ“¯“¬“š„Ä ¸Ä JÄ ~¢ƒ  ““ ] ±‘„¢Ä ­‘„Ä ¨¬­Ä ~¢ƒÄ š~¬­Ä ƒ““­¬Ä ¥ŠÄ ; ~¨„Ä “¢­„¨‚‘~¢„ƒÄ ­‘„Ä ¨„¬®š­“¢Ä ¦¥¬“­“¯„ “¢­„„¨Ä “¬Ä ~š¬¥Ä ƒ“¯“¬“š„Ä ¸Ä J¿Ä  q¥­„Ä ­‘~­Ä ­‘„Ä ¨„¬®š­“¢Ä “¢­„„¨Ä ¢„„ƒÄ ¢¥­Ä „Ä ~ Š¥®¨ƒ““­Ä¢® „¨" 2DÄ

        +>H9.NKQ7] %.T;7E.T>1.B] 'BWEN>.4] +%']  *6G=JP] *60S=JG] *M60=-A] )JVG3 +M:+ 25:+Ä

t~­®¨ƒ~¸Ä MÄi®š¸Ä 5++BÄ 33).3; 1; )+8; /513#,+1; 1; 8,5; +; ,; '5'3,01; 0; '',7; ",7; 3"; 13.1; #+; 8,50; '5'3#,+1;

"; /513#,+; 00#1; Q )0&1;

2( v‘„Ä ƒ““­¬Ä ¥‰Ä ~Ä :ƒ““­Ä ¢®ž„¨Ä ~¨„Ä“¢­„¨‘~¢„ƒÄ ¬¥Ä ­‘~­Ä ¢¥¢„Ä ¥‰Ä ­‘„Ä ƒ““­¬Ä ¨„­~“¢ ­‘„“¨Ä¥¨““¢~šÄ¦¥¬“­“¥¢¬$Ä v‘„ă“‹„¨„¢„Ä¥‰Ä­‘„Ä­±¥Ä¢®ž„¨¬Ä“¬Ä~Ä5ƒ““­Ä¢®ž„¨Ä~¢ƒ “¬Ä~Ħ„¨‰„­Ä¬§®~¨„$ 5% W¥¢¬“ƒ„¨Ä ­‘„Ä ¢¥¢¥¢¯„´Ä§®~ƒ¨“š~­„¨~šÄ !"#Q ±“­‘Ä +"Q  2K+¾$Ä j„­Ä ­‘„Ä ¬“ƒ„Ä #" „´­„¢ƒ„ƒÄž„„­Ä !Q ~­Ä'Q ~¢ƒÄ­‘„Ĭ“ƒ„Ä!"Q „´­„¢ƒ„ƒÄž„„­Ä #Q ~­Ä % Q U̓¢„Ä“¢­„¨¬„­¬ ­‘„Ä“¢­„¨“¥¨¬Ä¥‰Ä­‘„Ĭ“ƒ„¬Ä ! Q $Q !"Q "#Q ~­Ä¦¥“¢­¬Ä *QkÄ) Q aÄ ¨„¬¦„­“¯„š¸#Ä s¨¥¯„ ­‘~­Ä“‰Ä #(Q "'Q ~¢ƒÄ !)Q "&Q ­‘„¢Ä *)Q ak$







:$ j„­uÄ„Ä~Ĭ®¬„­Ä¥‰Ä »2Ä5Ä:Ä% $ ÄÄ5?¼Ä ±“­‘Ä›ubÄ 2+$Ä t‘¥±Ä­‘~­Äuđ~¬Ä­±¥Ä5„š„ž„¢­ ¬®¬„­¬Ä »Ä 8 Z] ~¢ƒÄ »Ä  Z] ¬®‘Ä­‘~­Ä  Q NQ  Q 





?% Y„­„¨ž“¢„ı‘„­‘„¨Ä­‘„¨„Ä„´“¬­¬Ä~Ä ¦¥¬“­“¯„Ä“¢­„„¨Ä  ¬®‘Ä­‘~­Ä @Q „“¢¬Ä±“­‘Ä5++B' i®¬­“‰¸Ä ¸¥®¨Ä ~¢¬±„¨

2EÄ



        Icm]Rtpw[ CR|_[hR|cURf Ef‰htcRX 



HZlbovHZT{bolHoe}{boly ' 6nw*† 9 IcnRV†(""/…NnT†(""7…NtV†pTT†n{mPVtw†wp†NtV†(""/)!!9…NnT†(""7)!!1…K`{w†(""/)!!9… (""7)!!1… cw† V|Vn† NnT†cy†cw†Tc|cwcPiV†P€†y`V†wmNiiVwy†qtcmV†n{mPVt†( ( 6nw)† 6 (""/)… …(… +; (""/… …'<


:""…

:""…

D…(""""

, 6nw)† : IcnRV†y`V†Vr{Nycpn†(})… :}… rD…"… `Nw† pni€†pnV†wpi{ycpn† cyw†TcwRtcmcnNny …:)… -… +; (… *; …+rD…" K`{w†rD…: - 6nw*† 8 FnV†VNwci€†R`VRgw†y`Ny†Ypt†w… ( '" … w… }cii†VnT† }cy`† y}p†„Vtpw† 6iwp '… …(… …,… …-… …/… …6… …7… …:… …<… D…'… …(… …6… …(-… …'("… …7("… …/"-"… …-",("… …,6(::"… 

-"<'',…

K`{w†y`V†w{m†pY†y`V†iNwy†y}p†Tc_cyw†cw†'… ,D-… / 6nw*† 8

6 6nw+† 6 IcnRV† (

## /""()!!1sxm2##*…)!!1…D…/"ynzm3##+…)!!1* 3





(""/)!!1

( }V†`N|V†( ( >VnRV† (D… (



(D… (D…(… 2; '(€…D…(-€ 

K`{w† (D…':"€…



 (

D…':"€… ':"€…  (-€… (-€… '(€… ,6€… 

'7…

'(€…K`Vn

8 6ow0† ; Lwbo_†6tby`mVybR†DVNo† …† =VpmVytbR†DVNo† }V†`N|V Aš AWš A8[š % & š šŽƒ š …† & Gš ( ( ƒ† A8[š AWš  & š šƒ†…† GW( ƒ†



A8[š * 2; Žƒ†2†AC ƒ†

GWš A8[š Žš2† 2† ACš }`bR`† bw† wNybw]VT† }`Vo Gš ( š  NoTƒ2†;…K`{w†y`V†wmNiiVwy†qpwwbPiV†|Ni{V†bw† GW/š

;r{Niby€† bw† NR`bV|VT† }`Vo† Žš

2;

AWš



[, 6ow*† ; MV† wbmqi€† tVqiNRV†Žš P€† ƒ†NoT†ƒ†P€†Žš yp†_Vy Žš



C  š' Aš ƒ† šŽš C Aš  Ž ƒ ( A š AŽš  ;ƒ2 † † C Žš 

; 6ow)† 8 CAš š

RŽPš Ž š \š( CŽPš A š  RŽšPš

CŽEš šŽPš šAš Cš CŽEš š E ŽEš A ŽEš ŽEš A šš wboRV†Žš $; 8,š (

A8- 6ow)† 9 š š2† R š š šƒ † šCš A ŽEš  š E CŽš šCš2† Cš 678; ŽE CŽš A 345;

Ž $A Eš š ƒ † šA E



CE-š

K`VtVZptV† y`V† qpboyw† šŽš š bo† y`V† 9NtyVwbNo† qiNoV† wNybwZ€bo_† y`V† Vr{Nybpo† Rpowbwy pZ†qpboyw†po†y`V†RbtRiV†RVoytVT†Ny† šAšA š NoT†pZ†tNTb{w† C,š
}`Vn†cy†cw†y`V†qpcny†cn†y`V†Yp{ty`†s{NUtNny†k€en_†pn†y`V†icnV†qNwwcn_†y`tp{_`†  88 š NnU† BšA /š @n†py`Vt† }ptUw $%(



3 J 'A J NnU†  Eš% ( aš Bš ) w Eš B w EšašWšRw,

BB, 6nw)† B]G

Cš kI 8 3† v CškI ExEš3† RšR kIšG E 3† YšR kI Eš 3† R]šTW kMšR[š  B]G3]/

K`{w† y`V†_tVNyVwy†cnyV_Vt†iVww†y`Nn† Cš 08 cw†A]GBC, 6nw)† W8 8€† H€y`N_ptNw† K`VptVm† JL… aš ATWEš 3† WT3š BVy† y`V† qVtqVnUcR{iNt† Ytpm L… yp†IJ… mVVy†IJ… Ny†N… K`Vn† 4IJL…… 4IL…N… K`{w

LN… JL… ( WTš ) aš ( 45† ) 3† ) ( ( ašW8/ BTWš BW]š IL… IJ… AG/ 6nw)† Y BVy†{ a šGš CYš NnU† }š 3† Tš WCTš wp†y`Ny†y`V†Vs{Nycpn†PVRpmVw {šEš}Eš aš {š } šEš ; C{}šaš8š ; {š 3† 8š pt† }š 3† 8/

M`Vn† {š 3† 8š }V†`N|V† Gš CYš 3† 8š ; $( ašG-š M`Vn† }šaš8š }V†`N|V† T WCTšaš8š 45† † 3† R/š >VnRV† y`V†w{m†cw† GšRš 3† Y/ BR, 6nw)† BC BVy†† PV†y`V†wmNiiVwy†cnyV_Vt† K`Vn†}V†`N|V

"† ( C[8[š (   † K`V†^twy† cnVs{Nicy€† cmqicVw† y`Ny† †

)

) W /

BRš }`ciV† y`V† wVRpnU† cnVs{Nicy€† cmqicVw† y`Ny

$( () BB,š AY† $( ( AGš y`Vn† y`V†qtpU{Ry†pY†y`V†py`Vt†cnyV_Vtw†cw† CFš 1; GNš 3† CAW3š K`cw†cw cmqpwwcPiV† wcnRV†y`V† wmNiiVwy† YNRypt†pY† CBWš _tVNyVt†y`Nn†AGš cw† B[š P{y†B[Eš ( GCRš #; CBW,šK`VtVYptV† }V† m{wy†`N|V†$( aš BC,š K`V†py`Vt†y}p†cnyV_Vtw†NtV†VNwci€†wVVn†yp†PV

BGš NnU† B[1š BT, 6nw*† A BVy† !) 3† C88R/š K`Vn†y`V†V~qtVwwcpn†cw†Vs{Ni†yp

A]š

AW/ 7nw+† W l ‡*Ž š aš l Ž š

’Jš Gš š’Jš Aš Gš Aš Aš aš aš aš š š >  Gš ’ š J G š ’ š Jš G’Jš šGš Gš šG“š Gš šG’Jš ’J

K`VtVYptV† rI šT š š W šrI š*R š š  T šrM šG š š R  x v vl  xš  yš v k kI I šn vl šC š š  G zš š vl š!A š š C xš š vl  o š š A z Aš Aš Aš Aš Aš A ašW, š’ š’ š’ š’ aš rIš š ’Jš “š Jš Jš Jš Jš





AY/ 7nw(† C8R]

Mcy`p{y† ipww† pY† _VnVtNicy† }V† mN† Nww{mV† y`Ny† eš ^š f,š Ay† Ypiip}w†y`Ny† A888š _š eš `š A8CRš NnT† N† TctVRy† R`VRg† w`p}w† y`Ny† A8CRš cw† y`V† pni† n{mPVt† PVy}VVn† A888š NnT† A8CRš y`Ny†Tc|cTVw† RAš 0; TEš 0; C ? :/š K`{w† ešašA8CRš NnT† fšašA8CT/š K`VtVYptV e šfšašA8CRš A š 8CTšaš C8R]/š

A[, 7nw)† A]C BVy† efšaš €š Wm† fgšaš{š Wm†NnT† gšešaš Q†Wm† 8† y`V†RpwcnV†t{iV

BVy† ) PV† y`V† RVnytV† pY† y`V† RctRiV† K`Vn† fpi aš Ceš ašAC8—1š DptVp|Vt† pfš  pgšašRš Wm† K`VtVYptV† P† y`V†RpwcnV†t{iV {Ešaš REš RE Cš 0; Rš 1; RŠ‹AC8—šaš R[šd{ aš RmO6š

K`Vn†QX† aš{Eš RšašR[š AWšaš GC3š K`{w ’Jš Aš Aš NtVN† hefg š aš 5 QSwcn†W8˜š aš 5 0; GCš 0; 6 aš [ kI7

K`VtVYptV† Žšaš[ Jš NnT†ŽEšaš [ J EšašA]C6š A]/ 7nw+† R €Eš aš OQ AWšaš [š & Q† Q†' AWšaš}Eš š[}AW ";

š Wšaš 8šdš€Eš š Q R Ešaš8š "; €šaš8š NnT† Q†ašR, €Eš š}Eš[}š A C8š

IcnRV†e…9. …cy†Ypiip}w†y`Ny† P9:… .…9.…K`{w† P,;9.…"…9.…@n†y`cw†wpi{ycpn }V†`N|V†{wVT†y`V†YNRy†y`Ny†y`V†w{m†pY†y}p†npn nV_Nyc|V†n{mPVtw†cw†„Vtp†pni€†}`Vn† Ppy`†n{mPVtw†NtV†„Vtp† ("… 6nw,† '† FPwVt|V†y`Ny†

Ip†y`V†pni€†qpwwcPicicy€†cw†y`Ny†Pa9%…Ypt†VNR`†)9% …( …… …:…>VnRV† P2ŽŒPx9: (%… 6nw)† %.,… FPwVt|V†y`Ny†Ypt†Nn€†qpwcyc|V†n{mPVt†| … %…

N"‡

šA4# šŠ4

|… ‹.



K`{w†}V†`N|V† %…







///

9š… Kš  †///šš ‚ 9ƒ%1/ >VnRV†„$ L9%"… :;„9%(… :; j%…9%-.… :; j9%., ((… 6nw+† (%%… %… %…  I… J…





%;(… J I… %;(… 9 ."";… +; J I š9%;(… '; I… ); (; ;   (""0)(""-)…  I… J… -"";… ;

K`V†qtcmV† YNRyptc„Nycpn† pY†-"";… cw† _c|Vn† P€†."";…9%;… 0; (%%… IcnRV†%;( NnT†J… #; I …cy†Ypiip}w†y`Ny†I 9%;… NnT†J…9(%%



(5… 1; ,…

(,… 6nw)† %":… BVy†y`V†NtVN†pY†OITS…PV†]&… NnT†y`V†NtVN†pY†OIMR…PV†])…BVy†y`V†NtVN†pY†OItlL PV†] …wp†y`Ny†y`V†NtVN†pY† OIJZ… cw†Niwp†] …wcnRV†Z… cw† y`V†mcTqpcny†pY†JL… Ep}† IT‚IS… ] NnT )… ]… IZ IL… K`VtVYptV†

])… ]r…



IT… IR…ƒ…Ik… IL… IZ‚IJƒ ITƒ IS… 



,… ( '( TR… 9 … K`VtVYptV† EpyV†y`Ny† ) 9 T…S… 8(… (… ; 



IT IR… IZ IJ…

(

 

(%…

, ' C .…



8(…

;

,… („…



|…9%":

CQ, 6nw)† C ( aš

;š -š Cš šqJ Cš štš L aš bš aš Cš šr , Q Gš CqJš CqKš C qKš ;š

KaVn† Ž Cš aš qJš "; Ž qJš bšCš !; ŽDQŽš š;š bš8š NnT† ŽD CqJŽ ;š aš 8,š Ka{w† š Fš QŽDš šCŽš qIš ŽVš  CsŽSšŽPš Ž š ŽDQŽš š;š šŽqIš 1†ŽP ŽD  CsŽ ;š Ž 1†8š š8š šCš šqI  sšaš C,š CT. 6nw-† ]8 IcnRV†{š}š NvV†yaV†vppyw†pY†yaV†Vs{Nycpn†ŽDš š~š Qš Žš šQ š~š šC šcš8š cy†Ypiip}w†yaNy { š}šaš ~š šQš NnT† NP†cšQš šC /š

KaVn†{Dš š} Dšb{ š}D C{}šaš  ~š šQ D Z~š šC š yvcNn_iV†cw† vc_ayNn_iVT† NnT†Žš 1†]80š

";

{Dš }š Dšbš~D-š >VnRV†yaV

CW- 6nw+† CT FPwVv|V†yaNy† ™{ š}š šj {}…š 1mN~ †{ š},š 6iwp † ŽDš #; Ž ;š wcnRV† ŽD Žš š;š 1 Ž ™Dš š #; "…Ka{w † }V†aN|V

Ka{w†yaV†Vs{Nycpn†PVRpmVw†ŽDšcš GTŽCT8š G… ŽCT Ž;8š aš8š KaVvVYpvV † yaV†iNv_Vwy† |Ni{V†pY†Žš cw† CT,š

;

ŽšbšCTš ;8,š

CY- 6nw+† GW Kp†TVyVvmcnV† yaV†n{mPVv†pY†qNvycycpnw † }V†f{wy†aN|V†yp†TVRcTV†yaV†n{mPVv†pY† }N‚w pY† cnwVvycn_† N† qNvycycpn† PVy}VVn† y}p† RpnwVR{yc|V† |p}Viw† cY† }V† cnwVvy† N† qNvycycpn† Ny Nii† Ka{w†yaV†ypyNi†n{mPVv†pY† qNvycycpnw† cw† Qš 0; Gš 0; GšbšGW1 CZ/ 6nw+† ]8;;

;š ;š ;š + bš šC88Qšaš šC88Gš šC88Q { ‰‰Hš { 99Sš {D99P D D ;š C88Qš *; C88Tš cš  š;š šCš ) Gš š,,,š šC88Qš cš;š š aš C88]8;;2š {<š Cš

>VnRV†    C888888šcš]8;;,š

CCš

C]/ 6nw.† Q8; 8€†RpmqNtcn_†y`V†RpV\RcVnyw†pY†Žƒš cy†cw†VNw€†yp† wVV† y`Ny† {ƒš 1†„,š BVyycn_†Ž }V†_Vy { ‰š š{<š š š



 

) {šƒš



š;š š; šš;š šCšš;š) Gš

 



;š š„š





;š

š„š š; š

K`VtVYptV† { 9š š{<š š š{ƒ†š „š š; „š „ „-š EpyV†y`Ny† C88T Tš /; Q8;š }`VtV† Q8;š cw† N†qtcmV†n{mPVt† Ay† cw† VNw€† yp† wVV†y`Ny† N† qtcmV†n{mPVt†) Tc|cTVw† „š pni€†cY† „š ) ) K`VtVYptV† y`V†wmNiiVwy†qpwwcPiV†|Ni{V†pY† „š cw† Q8;,š 



 







G83 6nw.† ; IcnRV† y`V† V~qpnVny† pn† y`V† iVYy† wcTV† pY† y`V† _c|Vn† Vs{Nycpn† cw† npn„Vtp† cY† Žš $; 8š cy Ypiip}w†y`Ny†y`V†Vs{Nycpn†cw†qpwwcPiV†pni€† }`Vn

IcnRV†Žš $; 8š cy† Ypiip}w†y`Ny†Žš



;/š

G;3 6nw/† W MV†RNn†tVNttNn_V†y`V†Vs{Nycpn†yp†_Vy ŽDŽCš  š š šD Cš



8/š

6w† N† s{NTtNycR† Vs{Nycpn† cn†yaV†|NtcNPiV†Žš y`V†TcwRtcmcnNny† pY†y`V†NPp|V†Vs{Nycpn cw†_c|Vn†P€† :cwRtcmcnNny 1†Cš š  D Q ; D Cš



Qš š;C GDš 1†;W G  C D-š

IcnRV† Žš cw† Nn† cnyV_Vt† y`V† TcwRtcmcnNny† m{wy† PV† N† qVtYVRy† ws{NtV† NnT† y`{w†  npn nV_Nyc|V† >VnRV † ;W G  (  Dš ) 8š :; 

;Wš C Dš ') 7 :; ju

) )

Qš C =šC… ,C… G3

IcnRV† š cw† Nn† cnyV_Vt† cy† Ypiip}w† y`Ny† cyw† pni€† qpwwcPiV† |Ni{Vw† NtV† _c|Vn† P€† š 8š;šCšGšQ-š M`Vn† š ;šGš y`V† TcwRtcmNny† cw† ;Cš }`cR`† cw† npy† N† qVtYVRy† ws{NtV K`{w† š 8š( …Q3š M`Vn†š ( 8š }V†`N|V†ŽDš CŽš 1†8š <= &; 8š( M`Vn†š Cš }V†`N|V†ŽDQŽš( 8š :;Žš 8šQ3š M`Vn†š Qš }V†`N|V†ŽDš WŽš šZš 8š >?Žš ( ( …Q, K`{w† y`VtV†NtV† Wš wpi{ycpnw† nNmVi€† 8š8š ( …8š8šCšQšCšCšQšQšQ,š

















! 6nw)† ;GG <tpm†  c † cy†cw† VNw€† yp†wVV† y`Ny†yaV†}`piV†ptTVtVT†&y{qiV† cw†TVyVtmcnVT†P€† {>š NnT { 3š Ay†cw†VNw€†yp†R`VRh†y`Ny† {V G{>š šX{ ,š K`{w†P€†  cc † }V†`N|V†G{>š šT{ C88U,š D D Dš Ay† Ypiip}w† y`Ny† {>š cw† Tc|cwcPiV† P€† T3š MtcyV† {>š T„/š K`Vn† }V† `N|V† ;T„š šT{ Dš C88Tš >? G„š š{ ( Q8;,š K`V† qpwwcPiV† |Ni{Vw† pY† { NtV† •Cš/ …Zš  G]Z–š cY† Ppy`† ‚š Dš Dš 







 

CGš



NnU† { NtV†qpwcyc|V† EpyV†y`Ny† VNR`† pY† y`V†NPp|V†|Ni{Vw†pY† { UVyVtmcnVw† N† {ncs{V Eš Eš qpwcyc|V† cnyV_tNi† |Ni{V† pY† {?š wNycwYcn_† y`V† Vs{Nycpn† G{@š ) T{ aš (""0… K`VtVYptV Eš y`VtV†NtV† ;GGš w{R`†wVs{VnRVw† GG/ 6nw)† ;C BVy†.; PV†Nn†cnyV_Vt†wNycwYcn_†y`V†_c|Vn†Vs{Nycpn† wp†y`Ny R|E! RnšR|škMˆ CRšbš8 ;

R|Eš R| CRšbšn  R| ˆ/š

IcnRV† ~cw†cttNycpnNi† cy† Ypiip}w† y`Ny R|E R| CRšaš "… NnU† R|ˆšaš "

I{Pwycy{ycn_† |šbš cnyp†y`V†^twy†Vs{Nycpn† }V†`N|V





R  E R   CRšbš" ;

ˆEš Rˆ]Wšaš "

;

ˆ" ;C ˆš'† aš "

;

ˆšaš ;Cš '

IcnRV† ˆš cw†N†qpwcyc|V†cnyV_Vt† }V†`N|V† ˆšbš; C3š GR3 6nw)† C I{PytNRycn_†y`V†wVRpnU†Vs{Nycpn†Ytpm†y`V†]twy†pnV† }V†`N|V Žš”šaš CTTš# CCRšaš G; ;

Žš”šbšG;3

IcnRV†G;š cw†N†qtcmV†n{mPVt† }V†`N|V†šcš;š NnU†Žš”šbšG;5š Kp_Vy`Vt†}cy`†y`V†]twy _c|Vn† Vs{Nycpn† }V†`N|V† 

;šŽG;š!

 ( CTT

;

Žš$;T Ž;Yš( "

;

Žšaš ;Tš pt† ;Y4š

M`Vn† Žšcš;Tš šaš;š }V†`N|V† ”šaš ;W,š M`Vn† Žšbš;Yš šcš;š }V†`N|V† ”šaš ;R,š K`VtVYptV† y`VtV†NtV†y}p†w{R`†wpi{ycpnw† nNmVl† ;Tš;š;Wš NnU† ;Yš;š;R,š

CRš

,/ 6nw*† (%" BVy†dfihE… p\\\d …q\\f …p\g h…PV†N†Yp{t Uc_cy†n{mPVt†Uc|cwcPiV†P€†8…An†qNtycR{iNt %… =>… d?@… ;… MV†NtV†_c|Vn† y`Ny†hfgd…F… q\\\h… … q\\f… …q\g… d …cx† Niwp†Uc|cwcPiV† P€† 8 FPwVt|V† y`Ny dfih hfgd…D…;;;d;;;h… ;;; d h… 

IcnRV† 8… UpVw† npy† Uc|cUV†;;; …cy† Ypiip}w†y`Ny†8… m{wy†Uc|cUV†d… h… K`{w† d… ; h… mpU 8 IcnRV†8… Uc|cUVw†%""% …}V†`N|V %…\\\d… …%…\\f… …%…\g… …h… ; "… mpU† 8… H…  d …p\\f… …q\g… …h… ; "… mpU† 8… G… q\\f… …q\g… ; "… mpU†



G… %"%"f… g… ; "… mpU† 8 H… %…\f… …g… ; "… mpU† 8… K`VtVYptV† }V†m{wy†`N|V†dF… h… mpU†8… NnU† p\f …g… ; "… mpU† 8 9pn|VtwVi€† P€†tV|Vtwcn_†y`V† NPp|V† Nt_{mVnyw † pnV†VNwci€†wVVw†y`Ny†cY† d… ; h… mpU†8… NnU† q\f …g… ; "… mpU† 8 …y`Vn†Ppy`†dfih… NnU†hfgd… NtV† Uc|cwcPiV† P€†8… Ep}† y`VtV†NtV†%.… qNctw†pY†  d …f… wNycwY€cn_†d… ; h… mpU†8 … }`cR`†NtV % …% …( …( …, …, …   … ; …; …% …'  †( …; …8 …" …'†% …; …(… IcnRV†"… =>… q\f …g… AB… ;; …y`VtV† NtV†%/… qNctw† pY† f …g… wNycwY€cn_† p\f …g… ; "… mpU† 8 cnRi{Ucn_† " …"… An†YNRy† y`cw†cw† Vr{Ni†yp†y`V† n{mPVt†pY†npn nV_Nyc|V†cnyV_Vtw†iVww† y`Ny†%""… y`Ny†NtV†Uc|cwcPiV† P€† 8…K`VtVYptV † y`VtV†NtV†%-… 0; %/…D…(%"… n{mPVtw† }cy` y`V†tVr{ctVU†qtpqVtycVw†

(/…



        G`k\OqnuY BOz^YgOz`SOd Ddˆgq`OV GBD  

HZlbovHZT{bol HsZTbQeFo}lWHoe}{boly  IbnRV†y`V†w{mw†pY†y`V†Tb_byw†pY†y`V†y}p†n{mPVtw†NtV†y`V†wNmV† y`V€†iVN|V†y`V†wNmV tVmNbnTVt† }`Vn†y`V€†NtV† Tb|bTVT† P€†;… K`{w† y`Vbt†Tb[VtVnRV†bw†Tb|bwbPiV† P€†;… Ip y`V†qpwwbPiV†Nnw}Vtw†NtV†!%† pt†'† ?y†bw†VNw€†yp†wVV†y`Ny†Ppy`†RNwVw†NtV†qpwwbPiV(†


K`{w†



PXWJ… 9; PXUZ

PYW…[… 9; PYU…N…

P[WR… 9; PNLQ

PKLS… 9; PZUS…

XW… XW WU… ZU… f… [W… d UL… f LW… f…

WU… UY… [W… f LW… d…

VY… d… ZU… d UL…

D{iybqi€bn_†y`V†Yp{t†Vr{Nybpnw† }V†_Vy† U…Y……W…U… … …X…W…U…Y… df…

…X…W……W…U… … …X…W…U…Y… df… }`bR`†€bViTw†XW…D…UY…



Z… [… &

! I{qqpwV†y`Ny†y`bw†tVw{iy†bw†npy†yt{V† FPwVt|V†y`Ny†]… `Nw†ypyNi†pY†  D…#$† VjVmVnz w{PwVyw† CVy† ) ~… !"( !  ( )D… †† #$† PV†y`V†#$† ViVmVny† w{PwVyw†pY† ) NnT )F… !"( … $ ) K`Vn† P€† y`V† Nww{mqybpn† y`V†#$† |Ni{Vw† ) NtV† m{y{Nii€† TbwybnRy IbnRV† !† ( ) =>… #†&† NnT† y`VtV† NtV† V~NRyi€†#$† n{mPVtw† Ytpm†!† y`tp{_`†#†& † }V† `N|V 

 



%†



~…#!(  ) D…  †… …- /…D… ~, …- …… -… 7… JcnRV† y`VuV† cw† qNcu† w{mmcn_† yp†,… NnT† N† qNcu w{mmcn_†yp†-7 …y`V†n{mPVuw† †( …(, (-… %; ) 8{y†y`Vn†† …(-…D…(… …(,… _c|Vw†ucwV†yp N†RpnyuNTcRycpn† - BVy† u…D… † BVy† Zv…D… u… … i  }`VuV† i cw† Nn† cnyV_Vu† w{R`† y`Ny † ( i ) † GPwVu|V† y`Ny† cY† Zv…D… deihj… … † }`VuV† d …e …i …h …† NuV† y`V†Tc_cyw pY†Zv …y`Vn† y`V†]uwy†-… Tc_cyw†pY†Zv o… NuV†  c  deih …cn† }`cR`†RNwV†y`V†]Yy`†Tc_cy†cw d …j… #; -;  cc y`V†]uwy†Yp{u†Tc_cyw†pY†deih …†pu† ccc  y`V†]uwy†Yp{u†Tc_cyw†pY†deih …( K`V†iNwy†RNwV†RNn†`NqqVn†pni† }`Vn†d D …;… K`VuVYpuV† }V†wVV†y`Ny†cY†dC…; …Nmpn_ y`V†n{mPVuw† Z! …Z$ …… …Z$! …y`VuV†cw† Ny†iVNwy†pnV† Ypu† }`cR`†y`V†]uwy† Yp{u†Tc_cyw NuV† y`V† ]uwy† Yp{u† Tc_dyw† pY† #%&") … † }`VuV† #%&") NuV† y`V† ]uwy† Yp{u† Tc_cyw† pY† Z! ?y† Ypiip}w† y`Ny† y`V† ]uwy† Yp{u† Tc_cyw† pY† y`V† n{mPVuw† n{mPVuw† Z! …Z$ …… … Z$!!!! cnRi{TVw†Nii†pY† † † - …:;;;… >VnRV†y`V†Nnw}Vu†cw† Vw

(7…

        !4=/%B?C.J %F0.8%F4)%7J 7H8B4%+J !J  @-<J -(D2>< J >G<*J J "7."%)""Ž

U\[s\{[V‡ Ž %ŽH€s\Ž )""2Ž AGM5FQ $$Q % DJ5GI:BAG

.F:I5Q OBJFQ /AGM5FGQ :AQ I95Q /AGM5FQ G955IQ CFBL:454Q /A4Q G9/45Q I95Q /CCFBCF:/I5Q 1J11>5GQ 15>BMQ OBJFQ /AGM5FG Q (BQ GI5CGQ /F5Q A55454Q IBQ ;JGI:7OQ OBJFQ /AGM5FGQ /29Q DJ5GI:BAQ 2/FF:5GQ % @/F< (BQ 2/>2J>/IBFGQ /F5Q />>BM54 Q

% Bfs[Ž ~d\ŽmV{~Ž ~dz\\Ž[fcf~{Žt]Ž 7&$$Ž % ) >fzZm\{Ž?'Ž Vs[Ž ?*Ž dV‚\Ž zV[ffŽ .Ž Vs[Ž 5Ž z\{u\Z~f‚\m‡Ž Sd\Ž ZfzZm\{Ž fs~\z{\Z~Ž V~Ž[f{~fsZ~ utfs~{Ž Q Vs[Ž=Ž<Žutfs~ŽNŽt€~{f[\Ž?*Ž mf\{ŽtsŽ~d\Žmfs\Ž[\~\zofs\[ŽX‡Ž Q Vs[Ž =ŽV~ŽV [f{~VsZ\Žt]Ž2Ž]ztoŽ~d\ŽZ\s~\zŽ t]Ž ?(Ž Mtfs~ŽPŽf{ŽZdt{\sŽtsŽ?-Ž {tŽ~dV~Ž OPŽf{Ž~Vsc\s~ ~tŽ ?*Ž V~ŽPŽBfs[Ž~d\Žm\sc~dŽt]Ž ~d\Ž{\co\s~Ž NP . Bfs[Ž ~d\ŽmVzc\{~Ž ut{f~f‚\Ž fs~\c\zŽ rŽ{€ZdŽ~dV~ŽrŽ \s[{Ž ƒf~dŽ \…VZ~m‡Ž %""Ž‰\zt{ 0 Bfs[Ž ~d\ŽmVzc\{~Ž‚Vm€\Žt]Ž 
2! Bfs[Ž ~d\ŽmV{~Ž~ƒtŽ[fcf~{Ž fsŽtz[\z Ž t]Ž

 

4 BztoŽ~d\Žbz{~Ž )""2ŽsV~€zVmŽ s€oX\z{ Ž % t]Ž~d\oŽVz\ŽVzXf~zVzfm‡Ž Zdt{\sŽUdV~Žf{Ž~d\ m\V{~Ž ‚Vm€\Ž t]Ž % ~tŽ \s{€z\Ž ~dV~Ž ~d\z\Ž f{Ž V~Ž m\V{~Ž ts\ŽuVfzŽ t]Ž s€oX\z{Ž {€ZdŽ ~dV~Ž ts\ t]Ž ~d\oŽf{Ž [f‚f{fXm\Ž X‡Ž ~d\Žt~d\z; 5 J\~Ž _Ž$ Ž  #% % %$% % $% % %% !$"% ƒd\z\Ž  Vz\Ž stss\cV~f‚\Ž fs~\c\z{Ž ]tz  " Ž% Ž)Ž ŽŽr!Ž G]Ž k%Ž )%ŽVs[Ž _)2Ž 56.25 Ž bs[Ž~d\Ž‚Vm€\Žt]Ž _%" !Ž 







6 G]Ž $% f{ŽVŽz\VmŽs€oX\zŽ ~dV~Ž{V~f{`i\{

     %""Ž %""Ž $% %

% $% %



)6Ž









% $%%



77Ž %""Ž



542ŽŽ

aq[Ž ~d\Ž‚Vm\Žt]Ž J"& J E\w\Ž #JJ [\qt~\{Ž~d\ŽmVwc\{~Žeq~\c\wŽ  #J & Td\Ž ]qY~etqŽ  & e{Ž [\aq\[Ž ]twŽ VmmŽ ut{e~e‚\Ž eq~\c\wŽ & Vq[Ž ~Vl\Ž tqŽ qtqq\cV~e‚\Ž

eq~\c\wŽ‚Vm\{Ž{YdŽ~dV~Ž &



& &  & Vq[Ž &

&& J & & & & && &



&



tw

....Ž <m{t Ž ]twŽVmmŽ & &  &

A\~\woeq\Ž 

 &  & F~Že{Žlqt„qŽ ~dV~Ž ~d\Ž.Ž{e[\{Žt]Ž VŽ ~weVqcm\Ž Vw\ŽYtq{\Y~e‚\Žut{e~e‚\Žeq~\c\w{ŽVq[Ž ~d\Ž

mVwc\{~ŽVqcm\Že{Ž~„eY\Ž~d\Ž{oVmm\{~ŽVqcm\ŽBeq[Ž ~d\Žu\weo\~\wŽt]Ž~de{Ž~weVqcm\Ž & <Ž~weVqcm\Ž QQ e{Žeq{YweX\[ŽeqŽVŽYewYm\Žt]ŽwV[e{Ž& „e~dŽ #QQ



 % & <m~e~[\{Ž

Vq[Ž Q t]Ž Q eq~\w{\Y~Ž V~Ž " Q Beq[Ž ~d\Ž {oVmm\{~Ž ut{{eXm\Ž ‚Vm\Ž t]Ž ~d\Ž m\qc~dŽt]Ž ~d\Ž{\co\q~Ž " Q

Q

 & J\~Ž % X\Ž ~d\Ž {\~Ž t]Ž VmmŽ ut{e~e‚\Ž eq~\c\w{ Ž <Ž ]qY~etqŽ &  %  % {V~e{a\{Ž & J

Vq[Ž & & <m{t Ž qtŽ ~„tŽ t]Ž ~d\Ž ‚Vm\{ &&& Vq[Ž & YteqYe[\ Ž Et„Ž oVq‡Ž ~dw\\[ece~Ž ut{e~e‚\Ž eq~\c\w{Ž & {V~e{]‡Ž  & 

& &



& J & ]twŽ VmmŽ & &



%





& J\~Ž & X\ŽVŽw\Vm‚Vm\[Ž]qY~etqŽ{tŽ ~dV~Ž "&#&



"&$ #& $ $&

]twŽVmmŽw\VmŽ qoX\w{Ž "&#& Vq[Ž $ & Beq[Ž 

&

& & J\~Ž#J





& Vq[Ž]twŽ  % )

UdV~Ž e{Ž ~d\Ž‚Vm\Žt]Ž 

&!!&  & Beq[Ž ~d\Ž{oVmm\{~Ž~dw\\[ece~Ž qoX\wŽ  {YdŽ ~dV~Že]Ž~d\Ž ~dw\\Ž[ece~{ŽVw\Ž# J &JVq[Ž' J

~d\qŽ 

 & J\~Ž#J



& Vq[Ž#J



#JJ&JJ'JJ#&JJ&,JJ# 'JJ#&'J

 & Vq[Ž]twŽ  9Ž . Ž m\~Ž#9JX\Ž~d\ŽmV{~Ž~„tŽ[ece~{Žt]Ž#: 5J#;"J

UdV~Ž e{Ž~d\Žw\oVeq[\wŽt^Ž#1J #IJJ





Q #IJ „d\qŽ e~Že{Ž [e‚e[\[Ž X‡Ž 6

 & Beq[Ž~d\Ž{oVmm\{~Ž~„t[ece~ŽqoX\wŽ (Q {YdŽ ~dV~Ž~d\Ž {oŽ t]Ž [ece~{Ž t]Ž  & [e‚e{eXm\ŽX‡Ž  &

,7Ž



(Q

e{Ž

%6 Beq[Ž~d\Žm\V{~ŽrŽ{YdŽ~dV~Žƒd\q\\wŽ~d\Ž\m\o\q~{Žt]Ž~d\Ž{\~ŽŠ% Ž) ŽŽ Žr‹ŽVw\ŽYtmtw\[ w\[ŽtwŽXm\ Ž~d\w\ŽVmƒV‡{Ž\…e{~Ž ˆ Ž  qt~Žq\Y\{{Vwfm‡Ž[f{~eqY~Ž t]Ž~d\Ž{Vo\ŽYtmtw {YdŽ~dV~Ž  %ˆŽ% Q  %""Ž ˆŽ )""Ž Ž Ž UdV~Ž e{Ž~d\Žoeqeoo %7 J\~Ž  Vq[ˆŽX\Žut{e~e\Žp~\c\w{Ž{YdŽ~dV~ %2%Ž J J J )2% Vm\Žt]Ž †: )" Bfq[Ž~d\ŽoV…eooŽut{e~e\Žfq~\c\wŽ rŽ{YdŽ~dV~ r*Ž % %4"Ž  %5"Ž  %6"Ž  %7"

)% Beq[Ž~d\ŽqoX\wŽt]Ž ut{e~e\Žeq~\c\w{Ž  {YdŽ~dV~ rŽ% )r*Ž % .r/Ž 

 



% )""2r*##3

f{Ž[ee{eXm\ŽX‡Ž rŽ %Ž )) Df\qŽ~\qŽL{ŽVq[Ž~\qŽ%{ Ž dtƒŽoVq‡Ž "%ŽXeqVw‡Ž{\v\qY\{ŽYVqŽX\Ž]two\[Ž{YdŽ ~dV~ qtŽ~dw\\ŽtwŽotw\Ž~dVqŽ~dw\\ŽL{ŽVw\Ž~tc\~d\w:Ž BtwŽ\…Voum\ Ž "%""%""%"%""%%%"%"%% f{Ž {YdŽ VŽ {\v\qY\ Ž X~Ž ~d\Ž {\v\qY\Ž "%""%"""%"%""%%%"%%%Ž [t\{Ž qt~Ž {V~f{]‡Ž ~df{ Ytq[e~ftq ). FqŽ ~weVqcm\Ž  Q Q )6 Ž QQ )%ŽVq[Ž QQ %0Ž Mtfq~{Ž Q Vq[Ž Q Vw\Ž tqŽ  ƒf~dŽ Q 5ŽVq[Ž %QQ & Q Bfq[Ž ~d\Žm\qc~dŽ t]Ž  

)0 BtwŽutfq~{ŽeqŽ~d\Žtw[\wŽQQ Q Q me\ŽtqŽVŽYfwYm\Žƒf~dŽ~d\Ž\…~\q{ftqŽt]Ž Q o\\~fqc ~d\Ž \…~\q{etqŽ t]Ž Q V~Ž Q Vq[Ž ~d\Ž \…~\q{etqŽ t]Ž Q o\\~fqcŽ ~d\Ž\…~\q{ftqŽ t]Ž Q V~Ž  Q K\~Ž *Q Vq[Ž ,Q X\Ž ~Vqc\q~{Ž ~tŽ~df{ŽYfwYm\Žƒf~dŽ uteq~{Ž t]Ž ~Vqc\qY‡Ž *Q Vq[Ž , w\{u\Y~f\m‡ŽQuut{\Ž *Q Q 4"ŽVq[Ž ,QQ 4.Ž@\~\woeq\Ž~d\Žm\qc~dŽt]Ž  )2 <Ž u\q~VctqŽ Q Q e{Ž fq{YweX\[Ž fqŽVŽYewYm\Ž t]ŽwV[e{Ž %"Ž{YdŽ~dV~Ž Q f{Ž uVwVmm\m ~tŽ Q Vq[Ž Q fq~\w{\Y~{Ž QQ V~Ž % Rd\Ž ~Vqc\q~{Ž ~tŽ ~df{Ž YfwYm\Ž V~Ž Q Vq[Ž  o\\~Ž ~d\Ž \…~\q{etqŽ t]Ž Q V~Ž VŽ YtootqŽ uteq~Ž *Q Quut{\Ž % *QQ )0Ž Vq[ &*Q ."ŒŽ Bfq[Ž % 



."Ž



        !4=/%B?C.J %F0.8%F4)%7J 7H8B4%+J !J  A-<J -(E2>< J A-(3$6J >G<*J QW~x[W‡ Ž +ŽHm‡Ž +""2Ž

"7"" %.."Ž

HH5?CHQ /GQ ?/AOQ DJ5GH:BAGQ /GQ OBJQ 2/A Q 'BQ 2/=2J=/HBEGQ /E5Q /==BM54 Q -9BNQ H95Q GH5CGQ :AQ OBJEQ 2/=2J=/H:BAG Q 029Q DPK5GH:BAQ 2/EE:5GQ % ?/E
% <sŽ gs~\c\xŽ g{Ž{vWx\]x\\Žg]Ž g~Žg{Žst~Ž[g‚g{gXm\Ž X‡Ž /Q ]txŽWs‡Ž gs~\c\xŽ /Q  %ŽJ\~Ž % X\ ~d\Ž{\~Žt]Ž ut{g~g‚\Ž {vWx\]x\\Ž gs~\c\x{ŽA\~\xogs\ Ž ƒg~dŽj{~gaYW~gts Ž ~d\Ž‚Wm\Žt]







ƒd\x\Ž nŽI Ž [\st~\Ž~d\Žcx\W~\{~Žgs~\c\xŽm\{{Ž~dWsŽtxŽ\vWmŽ~tŽ  BtxŽ\…Woum\Ž n+2ŽIŽ +Ž



+ J\~Ž !Q X\Ž ~d\Ž Y\s~xtg[Ž t]Ž  Q RdxtcdŽ !Q [xWƒŽ WŽ mgs\Ž uWxWmm\mŽ ~tŽ Q Ws[ gs~\x{\Y~gscŽ ~d\Ž {g[\{Ž Q Ws[Ž Q W~Ž )Q Ws[Ž +Q x\{u\Y~g‚\m‡Ž J\~Ž +Q gs~\x{\Y~ !Q W~Ž Q Ws[Ž )Q gs~\x{\Y~Ž !Q W~Ž Q Mxt‚\Ž ~dW~Ž ~xgWscm\{Ž Q Ws[Ž  Q Wx\ {gogmWx 

J\~Ž 0Q1Q2Q X\Žx\WmŽsoX\x{Ž{W~g{]‡gscŽ 0Q1QQ3Q /Q 1QQ 2Q Mxt‚\Ž ~dW~Ž "Ž0Q%Ž1Q  Q Q.Ž2 Q QQ0



4ŽWs[Ž 01Q16 Q 03Q



7

1 MmWY\Ž +""2Ž uths~{Ž tsŽ ~d\Ž YgxYo]\x\sY\Ž t]Ž WŽ YgxYm\Ž SƒtŽ utgs~{Ž )Q+Q Wx\Ž {Wg[Ž ~t ]txoŽ WŽuWgxŽ t]Ž A5:81BJEGQ g]Ž ~d\ŽYdtx[Ž )+Q {X~\s[{Ž WsŽWscm\Ž t]ŽW~Ž ot{~Ž %"Ž W~Ž ~d\ Y\s~x\Ž Cgs[Ž ~d\Ž |oWmm\{~Ž soX\xŽ t]Ž uWgx}Ž t]Ž s\gcdXty{

.%Ž

!#"$* &&* "&'* `zud“”p£ Vdšvp‰dšzhdˆ£ [ˆŸ‰“zdl£ `V[#£ Z’oŒ£_og™yŒ)£ ]œŒk£ G _‡œ™yŒ—"£ %`

C™©6Ä



###`

xž£Ä ©ž˜vÄ ‰™®vƒv£Ä R` f…²© Ä ®…vÄ ”j©®Ä ®…£vvÄ t‰ƒ‰®©Ä j£vÄ ###`

+`

C™©7Ä

0

AG ”‰v©Ä ž™Ä ®…vÄ £jt‰sj”Ä j»‰©Ä žxÄ KS` j™tÄ K nž®…Ä s‰£s”v©Ä [ž¹Ä

,` j™tÄ …v™svÄ …j©Ä v¢²j”Ä ¡ž¹v£Ä ¹‰®…Ä £v©¡vs®Ä ®ž

EF,` ?``ž¹v£Ä žxÄ E` ¹‰®…Ä £v©¡vs®Ä ®žÄ O? `ž¹v£Ä žxÄ E` ¹‰®…Ä £v©¡vs®Ä ®žÄ O= ?2, 1,` %4` 



1

C™©8 Sx

0#8`

W` v™t©Ä ¹‰®…Ä v»js®”¾Ä %##` ¿v£ž© Ä ®…v™Ä ‰™Ä ®…vÄ ¡£‰˜vÄ xjs®ž£‰¿j®‰ž™Ä žxÄ W ` ®…vÄ ¡£‰˜v

xjs®ž£Ä 2` žss²£©Äv»js®”¾Ä %##` ®‰˜v©Ä  ¹vęvvtÄ ™ž®Ä ¹ž££¾Ä jnž²®Ä®…vÄ¡£‰˜vÄxjs®ž£Ä C’ ©‰™svÄ ‰®Ä ¹‰””Äžss²£Ä˜ž£vÄ®…j™

%##`®‰˜v©

Ä f…vę²˜nv£ÄžxÄ ®‰˜v©Ä®…j®Ä 2` žss²£©Ä‰™›Ä ‰©Ä ƒ‰¶v™

n¾Ä

C”©žÄ ®…vÄ xjs®ž£Ä 2 žss²£©Ä +0` ®‰˜v©Ä ‰™Ä %##` f…²©Ä ®…vÄ j™©¹v£Ä ‰©Ä jnž²®Ä 0##` [ž¹

T0##N` / qT0##N`  T0##N` 0 78 Ä A Q7’ G %4` G 1Ä A 88 2,` 2` 2.` S®Ä xž””ž¹©Ä ®…j®Ä 0##` v™t©Ä ¹‰®…Ä v»js®”¾Ä 88`¿v£ž©Ä f…²©Ä ®…vÄj™©¹v£Ä ‰©Ä 0#8`

0`

C™©9Ä

0

Nž£Ä ®…vÄ v¢²j®‰ž™Ä ®žÄ …j¶vÄ jÄ ©ž”²®‰ž™ Ä

R

˜²©®Ä nvÄ ™ž™™vƒj®‰¶v Ä xž£Ä ‰xÄ R

Z,` R`+` Z,` %` 

# ` ®…v™

Z,` R` A Z`

[,` %` @` Z` Z,`. `q G¾Ä ©¢²j£‰™ƒÄ ¹vÄ žn®j‰™ R` e¢²j£‰™ƒÄ jƒj‰™Ä j™tÄ ©ž”¶‰™ƒÄ xž£Ä Z ` ¹vÄ žo®j‰™Ä

bv¹£‰®vÄ ®…vÄ ƒ‰¶v™Ä v¢²j®‰ž™Ä j©Ä

+Z,`Rq  0`M£

A

,` A

,A

QC) R`

FC’

+` jœtÄ ®…vÄ žœ”¾Ä ¡ž©©‰n”vÄ ©ž”²®‰žœÄ Œ©Ä ž

f…²©Ä #` :` R` A

B1|£



e²n©®‰®²®vÄ ®…‰©Ä 

¶j”²vÄ žxÄ ž£ ‰œ®žÄ ®…vÄ ƒ‰¶vÄ v¢²j®‰žœÄ jœtÄ ˜²”®‰¡”¾‰œƒÄ ®…£ž²ƒ…ž²®Ä n¾Ä žn®j‰œÄ

0- 7+ R

0- 7R+ R` G



R` ¹v

0 <

Pvœsv Ä U/R 0&`+ q R` 0 R` jœtÄ ©žÄ U/R 0'` 7q  1` 0` ¹…‰s…Ä …ž”t©Ä ‰xÄ jœtÄ žœ”¾Ä ‰xÄ /R` 0`:`#<` ‰"vÄ /R` 6q -q f…v£vxž£v Ä ®…vÄ ˜j»‰˜²˜Ä ¶j”²vÄ žyÄ /R`‰©Ä 0 ` 

2`

Cœ©6Ä





0/`

B

[ž®vÄ ®…j®Ä 6



%` ˜žtÄ %##` [ž¹ /-$$3 `%-$$3 



%` /` ˜žtÄ -  

$$3` 0R` G /` xž£Ä ©ž˜vÄ ¡ž©‰®‰¶vÄ ‰œ®vƒv£Ä R `f…²©Ä

PvœsvÄ / -



0

4`

Cœ©:Ä

A 

6B|£A A



6 A



0/` ˜žtÄ %## `

%##0`

fj’vÄ jœ¾Ä ©v®Ä

%##0` œ²˜nv£©Ä

h£‰®vÄ vjs…Ä œ²˜nv£Ä ‰œÄ ®…vÄ xž£˜Ä +H=JL` ¹…v£vÄ IL`  #`

jœtÄJM`‰©ÄžttÄ f…²©Ä žn®j‰œ‰œƒÄ %##0` žttÄ œ²˜nv£©Ä J(`

  ``J($$1` e‰œsvÄ ®…v£vÄ j£vÄ %##/`

žttÄ œ²˜nv£©Ä ‰œÄ ®…vÄ £©®Ä +##2` œj®²£j”Ä œ²˜nv£© Ä j®Ä ”vj©®Ä ®¹žÄ žxÄ ®…vÄ žutÄ œ²˜nv£©

JM` JO` f…vœÄ +H>JL` A` +HBJP` ‰xÄ +H>JL` A +H>JM` f…vÄ œ²˜nv£© %##/`%##0`  ` `+##2` j£vÄ %##/` œ²˜nv£©Ä ¹…v£vÄ®…v£vÄ ‰©Ä œžÄ ¡j‰£Ä ©²s…Ä ®…j®Ä žœvÄ žxÄ ®…v˜Ä Œ©Ä t‰¶‰©‰n”vÄ n¾Ä ®…vÄ ž®…v£!Ä ežÄ ®…vÄ jœ©¹v£Ä ‰©Ä %##0 ` j£vÄ ®…vÄ ©j˜v Ä ©j¾Ä

6`

Cœ©:Ä



2#86`

hvÄ ²©vÄ ®…vÄ xjs®Ä®…j®Ä v¶v£¾Ä ¡ž©‰®‰¶vÄ ‰œ®vƒv£Ä sjœÄ nvÄ v»¡£v©©vtÄ ²œ‰¢²v”¾Ä ‰œÄ jœ¾Ä

+%` ¹vÄ©vvÄ®…j®Ä®…vÄ sžv~sŒvœ®©ÄžxÄ q£j£vĜžœœvƒj®‰¶vÄ ‰œ®vƒv¤© ®…jœÄ +2 `f…v£vxž£vÄ "+2` 67/26` ¹…vœÄ ¹£‰®®vœÄ ‰œÄ nj©vÄ +2` j•”ž¹©Ä ²©Ä ®žÄtv®v£˜‰œv  +` C©Ä !%`





®…vÄ sžvs‰vœ®© Ä e‰œsvÄ

67/26` 6q8 +A +2`G 2`G`+2



¹vĞn®j‰œÄ rž£ 

7`

Cœ©;Ä

6`G I£ G DžA6£ PvœsvÄ "%#` 2#86`



72/`

„£ž£R£ ž£Ä £Q££q%q xž£Ä %'q 4q R`:` 88` e‰œsvÄ ®…v£vÄ j£vÄ 78` ®v£˜©Ä žœÄ ®…vÄ •vx®…jœtÄ ©‰tvÄ žxÄ ®…vÄ v¢²j®‰žœÄ jœtÄ 78` =A Q’ A 642` A 78`=A 8` ¹vÄ tvt²sv 8` xž£Ä 7` [ž¹Ä ©²¡¡ž©vÄ †ž£G 7 xž£Ä %%` 6q R;`V` jœtÄ ckqq ®…j®Ä …žS£

G G %q4q :`88` f…vœÄ N‰£©®Ä žn©v£¶vÄ ®…j®Ä

ƒ£ž£G









7V %# ` G 888 V` 642` 

¹…‰s…Ä ƒ‰¶v©Ä

7 5q ž£ G

V



A

8

04`

f…v£vxž£vÄ

jœtÄ 8

5q ž£q

‚£ G A

//`



Q’ jœtÄ „ž£G

%#` £v©¡vs®‰¶v”¾Ä



8 ` ¹…s…Ä ‰˜¡–¾Ä

f†vÄ ‰œv¢²j”‰®‰v©Ä ”vjtÄ ®žÄ

0.-q jq A 2.-q j™tÄ 2 .+q jq A 3.+q Jž™©v¢²v™®”¾ Ä ¹vÄ ©vvÄ ®…j®Ä 2.+q jq A 2.-q c&$$jUq 7q2.+ q

Pv™svÄ ¹vÄ sž™s”²tvÄ ®…j®Ä

3

C™©8Ä

//2

f…vÄ ƒŽ¶v™Ä ¥v”j®Žž™Ä Ž˜¡”Žv©Ä ®…j®

[dqqfq FG [dqq[ fq `²®®Ž™ƒÄ dq 7q ™vƒj®Ž¶v Ä

fq 7q &q ¹vÄ žn®jŽ™Ä [  )q FG )[  &!q [  &q7q$!q [v½® Ä ©Ž™svÄ

eŽ™svÄ

[  +q7q [  ) q &q q7q [  )qq[  &qqm$q

ž¥Ä

[)q 7q $q j™tÄ [ &q Ž©Ä ™ž™À

&oq7q m$q

ž£Ä

&o

¹…v£vÄ [+q A

$ q Ž®Ä xž””ž¹©Ä ®…j®Ä [+q 7q &q [ž¹ Ä Ž®Ä sj™Ä nvÄ ¡£ž¶vtÄ Ž™t²s®Ž¶v”¾Ä ®…j®Ä [+fq FG fq xž¥Ä j””Ä fq C”©ž Ä ŽxÄ [  +fq A fq xž£Ä ©ž˜vÄ 1A ®…v™Ä [+dq A dq xž¥Ä j””Ä dq FG f"q ežÄ ©Ž™svÄ [  + q;A ++++q7q [3333q7q++++qŽ®Ä xž””ž¹©Ä ®…j®Ä [+fq7q fq xž¥Ä fq ++++q T™Ä ¡j£®Žs²”j£Ä [  + q;A )$$.q7q)$$.q Jž™©v¢²v™®”¾ Ä )$$.q7q^  + q(A )$$.q FG ^  ) q)A )$$.qq^ )$$.q FG +[ )$$.q j™tÄ ©žÄ [ )$$.q )$$.#+q A

//3q ^™Ä ®…vÄ ž®…v¥Ä …j™t Ä

^ )$$.q FG ^ )$$-qq^ &q7q ^ .q*A //2q7q//2q f…v¥vxž£vÄ [ )$$.q7q //2q

&$

C™©7Ä

&.

Wv®Ä Mq ®…vÄ nŽ©vs®ž£Ä žxÄ ,$G Wv®Ä Dq f…²©Ä D>#E 7q j &*# jqq&"q C”©žÄ <>#9q 7 <> 7 jj &#  j  q & q "q 



-A

A <.A

C©Ä 9Dq7

#A

9>qE>q¹vÄ …j¶vÄ jj &# jq& q qq j &*q\ jq& q q7qjq& q q ež”¶Ž™ƒÄ :A 7q .q f…²©Ä ®…vÄ ¡v¥˜v®v¥Ä žzÄ ®…vÄ ®¥Žj™ƒ”vÄ ‘©Ä -qq. q /q q7q &.q ®…Ž© Ä ®…vÄ ž™”¾Ä ¡ž©Ž®Ž¶vÄ ©ž”²®Žž™Ä ©Ä

+-q

: :2’

C™©7Ä



DMÄY’ DHsž©4)Ã!Ä XiDQMÄY’YDKHÄ Pv™sv DQÄY’ DM(©‰™YDQMÄY’ DHsž©4)Ã(©‰™YDKHÄ Y’ .csž©4)Ã

Y’ :’ ‰©Ä ®…vÄ £jt‰²©Ä žxÄ ®…vÄ s‰£s”vÄ f…v£vxž£v Ä DQÄY’ :5 ¹…v£vÄ cÄ %)q

C™©7Ä

)).q

[ /q’fq [  [ /q’fqY’ [  ) q$£ fq xž£Ä j””Ä fq ®…vÄ x²™s®‰ž™Ä [q ‰©Ä ¡v£‰žt‰s ¹‰®…Ä ¡v£‰žtÄ -q ©®j£®‰™ƒÄ x£ž˜Ä +q ž™¹j£t©Ä [ž¹Ä [ / q[ .qY’ [ 3 q[ -qY’ %)q j™t [+qY’ %.q j£vÄ xž²£Ä t‰©®‰™s®Ä ¶j”²v©Ä f…²©Ä ‰™Ä v¶v£¾Ä ƒ£ž²¡Ä žxÄ -qsž™©vs²®‰¶vÄ ¡ž©‰®‰¶v ‰™®vƒv£©Ä  +q ®…v£vÄ ‰©Ä v»js®”¾Ä ž™vÄ ®…j®Ä ‰©Ä ˜j¡¡vtÄ n¾Ä {Ä ®žÄ )$$."q e‰™svÄ ®…vÄ sž””vs®‰ž™Ä žxÄ ®…£vvt‰ƒ‰®Ä ¡ž©‰®‰¶vÄ ‰™®vƒv£©Ä sj™Ä nvÄ t‰¶‰tvtÄ ‰™®žÄ v»js®”¾Ä )).q ƒ£ž²¡©Ä žxÄ -q sž™©vs²®‰¶vÄ ‰™®vƒv£©Ä vjs… Ä ®…v£vÄ j£vÄ )).q ®…£vvt‰ƒ‰®Ä ¡ž©‰®‰¶vÄ ‰™®vƒv£©Ä fq ®…j®Ä ©j®‰©‚v©Ä [fqY’ )$$. q e‰™svÄ

%+q

C™©<Ä



.q

ev®®‰™ƒÄ jqY’ EGY’ l q ¹vÄ ©vvÄ ®…j®

[  jqjqY’ [  jqj  )[ j qj )jq xž£Ä j””Ä j"q Pv™svÄ [  jqj qY’ jq xž£Ä j””Ä jq ev®®‰™ƒÄ EGY’ jq ƒ‰¶v©

[ jqjqY’ [  jql  )[ jql  )lq xž£Ä j””Ä jq j™tÄ ?A Pv™svÄ [qqj qlqY’[qqj qjq )lqY’ jq )lq xž£Ä j””Ä jq j™tÄ lq )$$.q%$$$qY’ ."q

%-q

C™©7Ä

f…v£vxž£v

)q

…’

…?’ v…Y’  v| *v|’Y’ fBv  1A : Bv… |Z’ |Z€’

vB88L’Y’

%.q

)$$- )$$+q  )$$-q vB88I’Y’5 0 3’Y’ )$$/q)$$. )$$/q











)q :£ v @’Y’ )$$.q +q 

:RR’ N£ž˜Ä :77v:7xy’Y’vxyvxxyvyvxy’ ¹vÄ ƒv®Ä RRxyxyv’Y’xyR2 [ž®vÄ ®…j®Ä VPeÄVW’ $q ©‰™svÄ x’$£y’ xzY’:’’W %q y  :’ 0G RR£ j™tÄ dPeÄ 1G $"q f…²© ¹vÄ ˜²©®Ä …j¶vÄ x’Y’y’Y’R0’ e‰™svÄ vVW’ :’ 1AY’ :RR’‰©Ä ®…vÄ j™©¹v£# C™©<Ä

+.q

>O2

C™©<Ä

>

[ž®wÄ ®…k®Ä ®…wÄ ¶k—²wÄ žxÄ ˆD’

˜žtÄ F£ ©Ä qž˜¡—w®w—¾Ä tw®w¦˜™wtÄ o¾Ä ®…wÄ ¶k—²wÄ žxÄ ˆ

p…’ A p…’$£p… D’ ˜žtÄ >77’L£ p…!’$£p…"D’ ˜žtÄ H2 ’ ˜žtÄ H …w ’ w¢²k—©Ä >’C’G’>’7’>’>’C’G’>’7’>’>’C’2 / 2  ¹…q…Ä …k©Ä kÄ q¾q—wÄ žxÄ —w™ƒ®…Ä O2’ C—©žÄ ®…wÄ ©²˜Ä žxÄ ®…wÄ ©¢²k¦w©Ä ™Ä ž™wÄ q¾q—wÄ ©Ä 7’ ˜žtÄ F3£ [ž¹Ä C77K O’ 9A GGH’ ’>2’ f…²©Ä ®…wÄ ¦w¢²¦wtÄ k™©¹w¦Ä ©Ä >/’ ˜žtÄ H2’ N²¤®…w¦˜ž¦wÄ

f…²©Ä ®…wÄ ©w¢²w™qwÄ cE<£ 

>P2

C™©5Ä

C7

€Yc%£e4£ h…w™Ä e£N£7’ gqn’+ \Ä ©Ä kÄ \tƒ®Ä ™²˜ow¦Ä ¹…ž©wÄ €¦©®Ä \ C tƒ®©Ä k¦wÄ H£ k™tÄ ®…wÄ —k©®Ä ®¹žÄ tƒ®©Ä k¦wÄ H, c£k™tÄ >7$ e4£ f…²©Ä ®…wÄ ©²˜Ä žxÄ tƒ®© žxÄ gqn% \Ä w¢²k—©Ä H W, C’$£>R) c $£e£A FHc£$£Fe£$£ G f…²©Ä ¹wÄ ™wwt

Vw®Ä \Ä 

cL£g ˜žtÄ CT’ -c $£Gx’ A &> ˜žtÄ KU’ Hv’ ’Fe£A)> ˜žtÄ >P2’ c£A >’ …k©Ä ™žÄ ©ž—²®ž™Ä o²®Ä c£A G’e£ H£©ÄkÄ ©ž—²®ž™# 

h…w™Ä e£ 

c

>Q2



7’ ®…wÄ ©²˜Ä žxÄ tƒ®©Ä ©Ä H W+ C’q >7$ c£ FHc+ F4£ f…²©Ä c£ ©Ä w¶w™$ C’ ©Ä kÄ ©ž—²®ž™Ä f…²©Ä ®…wÄ k™©¹w¦Ä ©Ä C7’ ©™qwÄ ®Ä ©Ä A GR2’

C™©5Ä



>>

’  ’H(£>7’ k¦wÄ ¦wtÄ k™tÄ G’H’K’O’P’F£ k¦wÄ o—²w Ä ®…w™ S®Ä qk™Ä wk©—¾Ä owÄ q…wq’wtÄ ®…k®Ä xÄ >  C ®…w¦wÄ tžÄ ™ž®Ä w»©®Ä ˆ’‰’>A 5A žxÄ ®…wÄ ©k˜wÄ qž—ž²¦Ä ©²q…Ä®…k®Ä ˆ’ ’‰’$£>A A 5A ež›?@Ä

>>2

> >’ ©²q…Ä ®…k®Ä ®…w¦wÄ tžÄ ™ž®Ä w»©®Ä ˆ’‰’>A 5A f…w™Ä >’H£ ˜²©®Ä owÄ žxÄ kÄ t}w¦w™®Ä qž—ž²¦ x¦ž˜Ä G2’ f…²©Ä ©k¾Ä >’ k™tÄ H£ k¦wÄ ¦wtÄ k™tÄ G’ ©Ä o—²wÄ e™qwÄ P’$£>’ ’>’ H'£ P’ ˜²©®Ä ®…w™Ä ow o—²wÄ [ž¹Ä C’qG’$£G’ A P’ ©žÄ ®…k®Ä C’ ©Ä ¦wtÄ k™tÄ …w™qwÄ H’ ’A >’$£ >’$£C’ ©Ä o—²wÄ I²® >>’M£G’$£H’$£H’ H£$£>’$£>’ kÄ qž™®¦ktq®ž™# e²¡¡ž©wÄ ®…w¦wÄ w»©®©Ä kÄ qž—ž²¦™ƒÄ xž¦Ä›Ä A

žxÄ ®…wÄ ©k˜wÄ qž—ž²¦Ä ©²q…Ä ®…k®Ä ˆ’$£‰’$£ >A



5A





>R2

C™©=Ä

G

f…wÄ ™w¢²k—®¾Ä qk™Ä owÄ ®¦k™©xž¦˜wtÄ ®žÄ

G7C‰’ CK‚‰’ 2’ C77’ 'A C77’ ·¸wÄ €¦©®Ä ™wwtÄ ®žÄ ®…wÄ ˜™˜²˜Ä ¡ž©®¶wÄ ™®wƒw¦Ä ‰’ ©²q…Ä ®…k®Ä ®…wÄ ™®w¦¶k—Ä ’ qž™®k™©Ä k™Ä ™®wƒw¦#Ä f…©Ä ˜™˜²˜Ä ¶k—²wÄ žxÄ ‰’ ©Ä C2’ h…w™Ä qž™®k™©Ä ž™—¾Ä ž™wÄ ™®wƒw¦Ä ®…k®Ä ©Ä G/’ f…²©Ä ®…wÄ k™©¹w¦Ä ©Ä G/’

C7/

‰’ A C’ ®…©Ä ™®w¦¶k—

G7HRR Vw®Ä {Ä M£ >O7’ 7A >P7’ 8A >Q7’ 9A >R72’ ^o©w¦¶wÄ ®…k®Ä {Ä  >PKE’ >KD>PKD’' KD2’ Vw® ˆ’ A >PKD#>CK2’ f…w™ {Ä A ˆ#‚qqˆ ƒqq’ X’ ˆD(>7777’A ˆD2’ e™qw ¼Ä >PKD>CK’ C™©5Ä





GO’

j™tÄ {# ˆ) >B’Y’ˆB’) >777*

ˆ’# >\’8q Cˆ’) >777>’ ºvÄ …j¶vÄ {’ A ˆ’) >B6’ f…²© >’ >PKB’) >CO’8q G7HRR3’

®…vÄ ˜™˜²˜Ä ¡ž©®¶vÄ ™®vƒv¥Ä „’ ©²s…Ä ®…j®Ä „B’ A {’ ©Ä ˆ%

C>2



>O Wv®Ä „’’ ’C77K„B88L’8q { „ „’+>’ v’ º…v¥vÄ {„’ ©Ä jÄ ¡ž”¾™ž˜j”Ä žxÄ „2’ SxÄ „’Y’ >’ ’ ’’ ’C77K’8q C77K’6A >77G0’ ^n©v¥¶vÄ ®…j®Ä „’’ ’C77K„B88L’ © ºvÄ …j¶vÄ vY’>’C t¶©n”vÄ n¾Ä „’) >’ xÄ j™tÄ ž™”¾Ä xÄ v’Y’C77K’ 6A >77G’ ©Ä t¶©n”vÄ n¾Ä „’, >2’ f…vÄ ¡¥˜v xjs®ž¥¿j®ž™Ä žxÄ v’ ©Ä v’Y’K’ 7A H7>’ 6A >P’ 7A CH5£ ežÄ v’ …j©Ä ž™”¾Ä CJ’Y’ >O’ xjs®ž¥©Ä f…²©Ä ®…v¥vÄ j¥vÄ >O’ ¡ž©©n”vÄ ¡ž©®¶vÄ ™®vƒv¥©Ä 1A ©²s…Ä ®…j®Ä „’’ ’C77K„B88L’ ©Ä t¶©n”v n¾Ä „# >2’ C™©5Ä











 



CC0

C™©>Ä



 



CH7OQ

Lž™©tv¥Ä ®…vÄ©v¢³v™svĞxÄ®v™Ä >’© Ä f…v¥vÄj¥vÄ v”v¶v™Ä©¡jsv©Änv®ºvv™Ä®ºžÄ >’©Ä ž¥Änvxž¥vÄ ®…vÄ ”vx®˜ž©®Ä >’ ž¥Ä jx®v¥Ä ®…vÄ ¥ƒ…®˜ž©®Ä > 0’ Nž¥Ä vjs…Ä žxÄ ®…v©vÄ ©¡jsv© Ä ºvÄ sj™Ä ¡²®Ä v®…v¥

®ºžÄ _©Ä  tž²n”vÄ _©£ ž¥Ä ž™”¾Ä ž™vÄ 7’  ©™ƒ”vÄ 7 3’ SxÄ ®…v¥vÄ j¥vÄ v»js®”¾Ä

}’ tž²n”v©Ä _©

®…v™Ä ®…v¥vÄ j¥vÄ ž™”¾Ä >7’#C}’ ©™ƒ”vÄ _© Ä f…vÄ ™²˜nv¥Ä žxÄ ºj¾©Ä xž¥Ä ®…©Ä ®žÄ …j¡¡v™Ä © Ä ‹££;;*{£5£ f…IJ©Ä ®Ä…ÄvÄ j™©ºv¥Ä -©Ä u ~’

~’ @ 8 B

 >>>>’)}’Y’CH7OQ0’



* }’ ~[r’ CG2

C™©5Ä

>†’) C}’

>C

Vv®Ä (G ˆ2’ G¾Ä sž©™vÄ ¥²”v sžªÄGY’

C>B’’CQB# >HB’ M£ P’ G£

’ B#C>H P ’ f‡³«Ä ;>(q M£ >HB’P

j™tÄ sž©Ä]Y’



>HB’G CQB,C>B’ C>H CQ

>>’ >O’

¬žÄ ®ˆj®Ä <>8q

-A

A

A

#A

B@ S’

%A

@A

!A

CpžÄ

=OQL9;>q M£ f‡´­Ä + G



B’ C>H’

P’ Q’

8

+"%GY’ +!#G'G ­ŸÄ ¯‡l®Ä ¯§mµ„”vÄ )*G ±Ä ©ž­sv”v©%Ä f…v¥vxž¥v&



3A

#G

C’sž©Ä

C> Q Y’ -Y’>C0’ G C’ P’

GP’

CH2

C™©;Ä

QP

Vv®Ä ®…vÄ s‰£s²˜s‰£s”vÄ žzÄ Nabd’ ˜vv®Ä ®…vÄ ”‰™vÄ cd’ k®Ä e.’ f…v™Ä e’a’b’d’ k£vÄ s ™Á s¾s”‰s#Ä [ž¹Ä jc_a’ k]ba’ jaed’©žÄ®…k®Äc’_’a’e’k£vÄk”©žÄsž™s¾ s”‰sÄ f…²© 



csB’ c_c]’X’caŽcb’ ceŽcd’k™tÄ dtB’X’dbŽd]’ daŽd_’ deŽdc0’ f…v£vzž£vÄ csB’G dtB’ X’ ce’Ž’cd’d q e’dc X’ ce’G de’Ž’cd’ cdB0’ Lž™©v ¢²v™®”¾ Ä cd’X’ G OFB’X’QP0’ 









&A

2A

$A

CK3

C™©7Ä

:F

Wv®Ä *Ä ovÄ®…vÄsv™®£vÄ žzÄ®…vÄs‰£s”v Ä N‰£©°Ä ¹vÄ ©…ž¹Ä ®…k®Ä s’`’*ÄEUÄ c’”‰vÄ ž™ÄkÄsž˜˜ž™ s‰£s”v Ä L”vk£”¾Ä

s’_’*Ä c’ k£vÄ sž™s¾ s”‰s#Ä

C©Ä j_sh;’



jf_a’X’ j_c1m’



F7‹’

¡ž‰™®©Ä s’_’G c’ k£vÄ k”©žÄ sž™s¾s”‰sÄ f…²©Ä s’_’*Ä ]UÄ c’k””Ä ”‰vÄ ž™Ä ®…vÄ s‰£s²˜s‰£s”v

žzÄ Ns_c0’

2A

"A

e‰™svÄ ls’_o’ X’ R7Œ’ so’ ‰©Ä kÄ t‰k˜v®v£Ä žzÄ ®…‰©Ä s‰£s”vÄ k™tÄ so’ X’

f…‰©Ä s‰£s”vÄ ‰©Ä k”©žÄ ®…vÄ s‰£s²˜s‰£s”vÄ žzÄ Ns’_^
FQ’



CO # G :7B’ CO’©‰™ÄF7‹’X’ >F 0’ 

!#"$* &&* "&'* `zud“”p£ Vdšvp‰dšzidˆ£ [ˆŸ‰“zdl£ `V[#£  X‘mŠ£^mf˜wŽŠ&£ ^‘mfwb£\Ž›Šj£^Ž›˜wŽŠ•£

%q

hwÄ ©…j””Ä ©…ž¹Ä ®…j®Ä ‰™Ä ƒw™w£j”Ä xž£Ä j™¾Ä „’ Aq -

* &/G

;

X „4

`q Aq Rq™ž®wÄ ®…j®Ä cqsP£ ]q ‰xÄ j™tÄ ž™”¾Ä ‰xÄ ]q ‰©Ä ®…wÄ ”j£ƒw©®Ä ‰™®wƒw£Ä ©²s… ®…j®Ä ](`q G „3’ SxÄ i_q X’ m%(` q)(`q   q](`nq¹…w£wÄ ]q ‰©Ä ®…wÄ ”j£ƒw©®Ä ‰™®wƒw£Ä ©²s…Ä ®…j® ](`q G „’ ®…w™Ä ~£tS£ HR_bpq C”©žÄ ‰xÄ `q j™tÄ /A j£wÄ t‰©®‰™s®Ä ©¢²j£wÄ x£wwÄ ‰™®wƒw£©  Pw™swÄ ®…w™Ä R_q0A Rheq

Nž£Ä j™¾Ä







e‰™swÄ w¶w£¾Ä ‰™®wƒw£Ä sj™Ä nwÄ ²™‰¢²w”¾Ä ¹£‰®®w™Ä j©Ä ](`q ¹…w£wÄ aq ‰©Ä ©¢²j£wÄ x£ww Ä 2:&/G 

m%q.Ä #' Ä q„nq

f…‰©Ä sž˜¡”w®w©Ä ®…wÄ ¡£žžx#

Vw®Ä Sq nwÄ ®…wÄ SG#G:q 9q j£wÄ sž””‰™wj£%Ä V‰“w¹‰©wÄ > qd’ Tq j£wÄ sž””‰™wj£ f…w£wxž£wÄ >Cq IHq :9q j™tÄ Cq ‰©Ä ®…wÄ ˜‰t¡ž‰™®Ä žxÄ Fq JKq <9q j™tÄ ‰©Ä ®…wÄ ˜‰t¡ž‰™®Ä žxÄ >Tq f…²©Ä CFq HJq STq IHq <:q e‰™swÄ ®…wÄ sž££w©¡ž™t‰™ƒÄ ©‰tw©Ä j¨wÄ

.%Ä N‰£©®Ä ¹wÄ ™ž®wÄ ®…wÄ sw™®£ž‰tÄ t‰¶‰tw©Ä wjs…Ä ˜wt‰j™Ä ‰™Ä ®…wÄ £j®‰žÄ +>.%Ä ˜‰t¡ž‰™®Ä žxÄ



¡j£j””w” Ä ®…wÄ ®¹žÄ ®£‰j™ƒ”w©Ä j£wÄ ©‰˜‰”j£ Ä

9q

:q

>q

@H£

4G



+

VqWqXq BÄ )Ä ©‰™qvÄ ‰x Ä xž£Ä v»j˜¡”v Ä Xq 7q ) Ä ®…v™Ä VqqWq8q /q j™tÄ VWq 7q 3q ‰˜¡”¾ ®…j®Ä Vq8q Wq8q +qjÄ qž™®£jt‰q®‰ž™Ä C”©ž Ä VqWqXq ©j®‰©€v©Ä j,q /j(qq3 jq VWXq 7q ) Ä ‰v j +(q 7qVWX\jq e‰™qvÄ j +(q /Ä )Ä xž£Ä j””Ä jq¹vÄ ©vvÄ ®…j®Ä VqWqrj£vÄ j””Ä žxÄ ®…vÄ ©j˜v ©‰ƒ™ Ä j™tÄ ®…²©Ä VqWqXq A )Ä j©Ä VqW q qXq q7q/ N‰£©®”¾ Ä

[ž¹Ä ®…vÄ ƒ£j¡…Ä EG 7q j,q

/j(qq3j VWXq …j©Ä ©®j®‰ž™j£¾Ä ¡ž‰™®©Ä j®Ä jq8q %q j™tÄ jq8q + j™tÄ q²®©Ä ®…vÄ ¼j»‰©Ä j®Ä jq8q VWXq©žÄ ®…j®Ä Vq Aq  g% qWBq %+q j™tÄ Xq Aq +g  N‰™j””¾ Ä ‰xÄ Xq A -q®…v™Ä VWq8q X +(q A %q©žÄ ®…j®Ä VWqWYqVXq A %qq Wq%q #W q F²® jq%q #jq ‰©Ä ‰™q£vj©‰™ƒÄ xž£Ä jqAq %qg q f…²©Ä %qq Wqq%#Wq A 3qjÄ qž™®£jt‰q®‰ž™ .63@?5G B@=DC8@?G C©Ä ‰™Ä ®…vÄ €£©®Ä ©ž”²®‰ž™ Ä VqWq j™tÄ Xq ©j®‰©x¾Ä ®…vÄ v¢²j®‰ž™Ä j,q  /j(q 3j VWXq8q ) Ä ž£Ä jj +(q 7q VWXq e²n©®‰®²®vÄ jq 7q Xq ®žÄ ƒv®Ä XX +(q 7q VWXq ž£ X +(q 7q VWq [ž¹ Ä j©©²˜vÄ Xq G -q f…‰©Ä ‰˜¡”‰v©Ä ®…j®Ä VWq 0Ä + Ä ¹…‰q…Ä ‰™Ä ®²£™Ä ‰˜¡”‰v© V  Wq q A )q ¶‰jÄ CZOZÄ f…²©Ä Xq A -q jÄ qž™®£jt‰q®‰ž™Ä Pv™qvÄ Xq A -q hvÄ ƒv® VWX8q XX +(q A -q Xq A )q©‰™qvÄ VqWqqXq7q / Nž£Ä ®…vÄ v¢²j®‰ž™Ä Zjq 7q VWXq¹…v£vÄ VWXq A

-qZNq 7q ) Ä Z%q 7q -q ‰˜¡”‰v©Ä ž™vÄ £žž® -qZ+q 7q )Ä ‰˜¡”‰v©Ä ž™vÄ £žž®Ä nv®¹vv™Ä %q j™tÄ +q Z+q8q ) -q ‰˜¡”‰v©Ä ž™vÄ £žž®Ä nv®¹vv™Ä +q j™tÄ -

nv®¹vv™Ä )Ä j™tÄ %q Z%q

Zq-q

H0





Vv®Äs’aÄ nvÄjÄ¡j‰£Äžxęv‰ƒ…nž²£©Äe²¡¡ž©vÄ s’ …j©Ä`q™v‰ƒ…nž²£©Ä¹…žÄj£vꞮęv‰ƒ…nž²£© žxÄ aÄ j™tÄ aÄ …j©Ä 9G ™v‰ƒ…nž²£©Ä ¹…žÄ j£vÄ ™ž®Ä ™v‰ƒ…®nž²£©Ä žxÄ s’ ¹‰®…Ä aq G 7G f…v™Ä n¾ ˜ž¶‰™ƒÄ s’ ®žÄ jÄ ¡ž‰™®Ä ¶v£¾Ä q”ž©vÄ ®žÄ aÄ ®…vÄ ™²˜nv£Ä žxÄ ™v‰ƒ…nž²£‰™ƒÄ ¡j‰£©Ä tžv©Ä ™ž® ‰™q£vj©vÄ j™tÄ ®…vÄ ©v®Ä žxÄ ™v‰ƒ…nž²£©Ä žxÄ

s’ j™tÄ aÄ qž‰™q‰tvÄ

g…²©Ä n¾Ä £v¡vj®‰™ƒÄ ®…‰©

¡£žqvt²£v Ä ¹vÄ ©vvÄ ®…j®Ä ®…vÄ ˜‰™‰˜²˜Ä ‰©Ä j®®j‰™vtÄ ¹…v™Ä ®…vÄ ¡ž‰™®©Ä j£vÄ t‰¶‰tvtÄ ‰™®ž 4A

q”²©®v£© Ä

4A

G

+. q

©²q…Ä ®…j®Ä ®¹žÄ ¡ž‰™®©Ä j£vÄ ™v‰ƒ…nž²£©Ä ‰|Ä ®…v¾Ä j£vÄ ‰™Ä ®…vÄ ©j˜v

q”²©®v£Ä f…²© Ä ‰xšŠÄ ‰©Ä ®…vÄ ™²˜nv£Ä žxÄ ¡ž‰™®©Ä ‰™Äq”²©®v£Ä 8 G ®…v™

f…vÄ ”j©®Ä ‰™v¢²j”‰®¾Ä xž””ž¹©Ä x£ž˜Ä ®…vÄ xž””ž¹‰™ƒ5Ä RxÄ ®…vÄ ”j£ƒv©®Ä žxÄ ®…v̋Ä© Ä ©j¾Ä A

.2 q

®…v™Ä ®…vÄ ©˜j””v©® Ä ©j¾Ä

Wq

‰©Ä AÄ

Vq‰©

.1q j™tÄ ¡£q(* G aK}£q%* ¹…‰q…Ä qj™

nvÄ ¡£ž¶vtÄ n¾Ä t‰£vq®Ä qž˜¡²®j®‰ž™Ä g…‰©Ä ˜‰™‰˜²˜Ä ‰©Ä j®®j‰™vtÄ ¹…v™Ä ®…v£vÄ j£vÄ 13Ä q”²©®v£© Ä

).q ¹‰®…Ä .1q ¡ž‰™®©Ä j™tÄ ,)Ä ¹‰®…Ä .2q ¡ž‰™®©Ä

2)Ä


Related Documents


More Documents from "SMS_SMO"